21 June 2016 Written Answers.

The following are questions tabled by Members for written response and the ministerial replies as received on the day from the Departments [unrevised].

Questions Nos. 1 to 10, inclusive, answered orally.

Questions Nos. 11 to 13, inclusive, resubmitted.

Questions Nos. 14 to 21, inclusive, answered orally.

Departmental Staff Remuneration

21/06/2016WRA0070022. Deputy David Cullinane asked the Taoiseach and Minister for Defence the cost of ensuring that workers in his Department recruited post-2011 are paid the pre-2011 salary rate; and if he will make a statement on the matter. [16653/16]

21/06/2016WRA00800Minister of State at the Department of Defence (Deputy Paul Kehoe): Policy in relation to pay in the Public Service is, in the first instance, a matter for my colleague, the Minister for Public Expenditure and Reform.

The cost of ensuring that employees in my Department who were recruited post-2011 are paid the pre-2011 salary rate is governed by the manner in which the provisions of the National Recovery Plan and subsequent Haddington Road and Lansdowne Road Agreements have been implemented.

There are various factors which impact on the pay scale under which employees were re- cruited to my Department, including:

- Employees may have had prior public service experience in which case they would enter on pre-2011 pay scales.

- Employees may have been offered and accepted employment in my Department in 2010 but may not have been in a position to undertake duties until post-2011. Such staff were ex- empted from the 10% reduction in pay encompassed by the National Recovery Plan.

Accordingly, no additional costs would arise to ensure that employees in such instances were paid the pre-2011 rate.

Pay scales were further impacted by the merging of pre- and post- 2011 pay scales from 1 November 2013. The resultant level of mitigation would vary depending on the date of entry of the employee, the point on the pay scale the employee had entered at and whether they had subsequently been promoted and placed on a pre-2011 pay scale.

The complexity of calculating these costs is further compounded by the fact that pay costs impact on individual employees differently, depending on the pay band into which they fall. 1 Questions - Written Answers Accordingly, it is not possible to accurately capture the notional costs referred to by the Deputy, given the above complexities.

I can assure the Deputy that the provisions of the Lansdowne Road Agreement, which takes cognisance of the State’s improved finances and which now commences the process of income restoration, will further mitigate any disparity which would have arisen initially under the Na- tional Recovery Plan between pre and post 2011 salaries, and the current implementation of the Haddington Road Agreement.

Questions Nos. 23 and 24 answered orally.

Defence Forces Properties

21/06/2016WRA0100025. Deputy Thomas Byrne asked the Taoiseach and Minister for Defence his plans for the future of Gormanston Camp in . [16861/16]

21/06/2016WRA01100Minister of State at the Department of Defence (Deputy Paul Kehoe): Gormanston Camp is, and will continue to be, an important operational military installation, and there are no plans to change this.

My Department has invested considerably in facilities and infrastructure at Gormanston Camp. Since 2010, just over €673,000 has been spent on a range of capital and non-capital developments and improvements whilst a further €50,000 is to be spent this year. This is in ad- dition to routine ongoing investment in repair and maintenance of the facility.

In addition, in implementing the White Paper on Defence 2015, a number of projects have been identified that will address infrastructure requirements across the . In this regard, a rolling five year infrastructure development plan will be produced based on an assess- ment of needs for all individual barracks. In addition, a review will be carried out of training facilities, which includes Gormanston Camp, in order to ensure that they are developed in accordance with the needs of the Defence Forces. At present, the Camp is currently used for RDF training and is the only site in the Defence property portfolio that is used for Air Defence training and Air Corps firing.

Questions Nos. 26 and 27 answered orally.

Departmental Staff Allowances

21/06/2016WRA0130028. Deputy David Cullinane asked the Taoiseach and Minister for Defence the cost of reintroducing public sector allowances for workers in his Department; and if he will make a statement on the matter. [16652/16]

21/06/2016WRA01400Minister of State at the Department of Defence (Deputy Paul Kehoe): Policy in rela- tion to the payment of allowances in the Public Service is, in the first instance, a matter for the Minister for Public Expenditure and Reform.

I can say however, that following the review of public service allowances and premium pay, a number of changes were made in respect of allowances paid to public servants. This review was completed in 2012.

In that regard, the allowance which was payable to officials who travelled outside of Ireland and the UK to represent Ireland at meetings of the EU or other international organisations, or

2 21 June 2016 who acted as Chairpersons of EU committees, collectively known as Chairpersons and Del- egates allowance, was abolished.

This was the only allowance in respect of civil servants in my Department that was abol- ished which had a cost implication for my Department.

This is a demand-led allowance, and depends on the number of times staff in my Department travel abroad to represent Ireland at meetings of the EU or other international organisations. On the basis that it is not possible to predict the number of such meetings to be attended it is not possible to calculate accurately the cost of restoring this allowance to staff in my Department.

Question No. 29 answered orally.

Defence Forces Operations

21/06/2016WRA0160030. Deputy Clare Daly asked the Taoiseach and Minister for Defence the number of aid-to- the-civil-power duties the Defence Forces carried out at Shannon Airport in County Clare and at Dublin Airport in County Dublin involving military aircraft of the United States of America, by month, in 2016; if he has concerns regarding the frequency of these duties; if he plans to discuss these arrangements with the Department of Justice and Equality; and if he will make a statement on the matter. [16706/16]

21/06/2016WRA01700Minister of State at the Department of Defence (Deputy Paul Kehoe): The Department of Justice and Equality and An Garda Síochána have primary responsibility for the internal security of the State. Among the roles assigned to the Defence Forces in the White Paper on Defence is the provision of Aid to the Civil Power (ATCP) which, in practice, means to assist An Garda Síochána when requested to do so.

Since 5th February 2003, the Gardaí have requested support from the Defence Forces at Shannon Airport on occasion. I have no concerns with the frequency of these duties and I am satisfied that there is ongoing and close liaison between both An Garda Síochána and the Defence Forces, and between my Department and the Department of Justice and Equality re- garding security matters generally, including the Defence Forces ATCP roles. The number of duties incurred in the provision of ATCP support at Shannon Airport in 2016 involving military aircraft of the United States of America is set out in the following table. There have been no such duties incurred in the provision of ATCP support at Dublin Airport in 2016.

Month No. of Duties January 36 February 27 March 39 April 47 May 33 June (to date) 21

Ministerial Responsibilities

21/06/2016WRA0180031. Deputy Richard Boyd Barrett asked the Taoiseach and Minister for Defence the de- tails of the division of responsibilities between him and the Minister for State in his Depart- ment. [15600/16]

3 Questions - Written Answers

21/06/2016WRA01900Minister of State at the Department of Defence (Deputy Paul Kehoe): I have been ap- pointed as a Minister of State attending Government and Minister of State at the Department of the Taoiseach and the Department of Defence , with special responsibility for the Department of Defence.

Following this appointment, the Government has made the Defence (Delegation of Ministe- rial Functions) Order 2016 under the Minister and Secretaries (Amendment) (No. 2) Act 1977 to delegate ministerial functions to me. Under this Order I now have statutory responsibility for a wide range of Defence matters as specified in the Order. The principal powers delegated include the powers to make regulations under the Defence Acts 1954 to 2015 covering a wide range of Defence matters and personnel-related issues affecting members of the Defence Forc- es. In addition, Ministerial powers arising under other legislation relating to the Department of Defence have been delegated to me.

On an overall level, primary responsibility for the Department of Defence will rest with An Taoiseach as Minister for Defence, who will remain responsible to Dáil Éireann and as a member of the Government for the exercise and performance of the statutory powers and du- ties delegated. Every power or duty delegated to me continues to be vested in An Taoiseach as Minister for Defence concurrently with me and may be exercised or performed by either.

Defence Forces Medical Services

21/06/2016WRA0200032. Deputy Lisa Chambers asked the Taoiseach and Minister for Defence his plans to continue prescribing Lariam for members of the Defence Forces; the alternative medications available and used; the number of members of the Defence Forces, in the past or currently serv- ing, who have reported negative health effects which they attribute to the use of Lariam; and if he will make a statement on the matter. [16899/16]

21/06/2016WRA02100Minister of State at the Department of Defence (Deputy Paul Kehoe): As I have made clear, the health and welfare of the Defence Forces are a priority for me and this informs my approach to the issue of Lariam.

Malaria is a serious disease which killed approximately 438,000 people in 2015, with 90% of deaths occurring in sub-Saharan Africa as reported by the World Health Organization. It is a serious threat to any military force operating in the area. Anti-malarial medications, including Lariam, remain in the formulary of medications prescribed by the for Defence Forces personnel on appropriate overseas missions, to ensure that our military personnel can have effective protection from the very serious risks posed by this highly dangerous disease.

There are three anti-malarial drugs in use by the Defence Forces, namely Lariam (meflo- quine), Malarone and Doxycycline. The choice of medication for overseas deployment, includ- ing the use of Lariam, is a medical decision made by Medical Officers in the Defence Forces, having regard to the specific circumstances of the mission and the individual member of the Irish Defence Forces.

Information in relation to illnesses contracted by Defence Forces Personnel including in- stances of malaria is maintained on each individual’s medical file. While there is no way of providing the exact information sought by the Deputy without examining every medical record of each member who served overseas in sub-Saharan Africa, I am advised by the Medical Corps that the number of incidences of malaria among Defence Forces Personnel is low. The Deputy might also note that since deployment of Defence Forces Personnel to sub-Saharan Africa, not a single member has died from the malaria disease.

4 21 June 2016 Defence Forces Personnel

21/06/2016WRA0220033. Deputy Lisa Chambers asked the Taoiseach and Minister for Defence the steps he will take to address the attrition rate and identify the cause of premature departures and the loss of highly trained and skilled persons from the Defence Forces; and his plans to encourage and sup- port the retention of persons. [16902/16]

21/06/2016WRA02300Minister of State at the Department of Defence (Deputy Paul Kehoe): As I have already advised the Deputy in my reply to her ealier question today, there are a number of initiatives in place that address recruitment and retention in the Defence Forces.

I wish to reaffirm to the Deputy that the Government is committed to maintaining the stabi- lised strength of the Permanent Defence Force at 9,500 personnel, comprising 7,519 Army, 887 Air Corps and 1,094 Naval Service as provided for in the current C.S.4 regulations.

The manpower requirement of the Defence Forces is monitored on an ongoing basis in ac- cordance with the operational requirements of each of the three services. Personnel are posted on the basis of operational needs across the organisation both at home and abroad. As there is significant turnover of personnel in the Permanent Defence Force targeted recruitment takes place so as to maintain personnel numbers at or near the agreed strength levels.

I am satisfied that the operational capacity of the Defence Forces is being maintained. With the support of the Chief of Staff, I am confident that we shall retain the capacity of the Defence Forces to operate effectively across all roles and to undertake the tasks assigned by Govern- ment, both at home and overseas.

EU Battle Groups

21/06/2016WRA0240034. Deputy Richard Boyd Barrett asked the Taoiseach and Minister for Defence the de- tails of military exercises involving Irish soldiers and European Union battle groups; and other joint military exercises that have taken place in the past year, the other countries which were involved in the exercises; if he plans such exercises in the future;and if he will make a statement on the matter. [16872/16]

21/06/2016WRA02500Minister of State at the Department of Defence (Deputy Paul Kehoe): Participation in the EU Battlegroups demonstrates Ireland’s commitment to the development of EU capabilities in the area of crisis management and contributes to our reputation as professional and effective peacekeepers and our credibility within the Union.

In January 2014, the Government approved Ireland’s participation in the German led EU Battlegroup 2016, which will be on standby for the second six months of 2016. The other members of the Battlegroup are Germany, Austria, Czech Republic, Croatia, Luxembourg and the Netherlands.

The Defence Forces contribution to the German Battlegroup will involve an ISTAR (Intel- ligence, Surveillance, Target, Acquisition and Reconnaissance) Company together with staff posts at both the Operational and Force headquarters. The total number of Irish Defence Force personnel that will be involved in the German Battlegroup, should it deploy will be about 175.

In July 2015, the Government approved Ireland’s participation in the UK Battlegroup, which will also be on standby for the second six months of 2016. The total number of Irish Defence Force personnel involved in the UK Battlegroup will be five and they will be deployed in the Operational and Force headquarters. Apart from Ireland, the other members of the Battlegroup 5 Questions - Written Answers are UK, Sweden, Finland, Latvia and Lithuania.

For the most part, Battlegroup training takes place in the contributing member States. For example, Irish troops are mainly trained in Ireland, Swedish troops in Sweden etc.

The pre-standby period for a Battlegroup involves a range of intensive planning exercises, training, and desktop and actual exercises for the different components of the Battlegroup and for the Battlegroup as a whole. This is essential so as to prepare Battlegroup personnel for the eventualities they may face if called upon to deploy on operations. As the Deputy will appreci- ate, from a force protection perspective, you would not want to have a situation where the first time a Battlegroup rapid response contingent actually meet and commence operations is on an operational deployment into a conflict zone. Exercises are designed to simulate the situations that may arise at the strategic, operational and tactical level so as the Battlegroup can operate as an effective and coherent military response. In Ireland’s case participation in that response will be on the basis of supporting or reinforcing a mission a UN Mandated mission or a humanitar- ian operation.

Joint exercises in relation to both the German and UK led Battlegroups only commenced in January 2016. There were no exercises conducted in 2015.

In respect of the German led EU Battlegroup, ten Defence Force personnel, who will be deployed to the Force Headquarters, took part in a Command and Staff Certification exercise in Strasbourg, France in early 2016. A further Force Headquarters certification exercise took place in May, 2016, which included staff from the Defence Forces ISTAR component. A num- ber of Irish Personnel from the ISTAR company and the Force Headquarters participated in the overall exercise for the German Battlegroup in Germany in April 2016.

Ireland completed its national exercise for its contribution to the German Battlegroup in the and Kilworth in March 2016. No other countries participated in this exercise which was observed by the Force Commander of the German led Battlegroup.

In March 2016, the five Defence Force personnel who will be deployed to UK Battlegroup took part in a Force Headquarters Command and Staff exercise in the UK. In May 2016, they participated in a Battle Group Certification Exercise, comprising the entire Battlegroup which was conducted on Salisbury Plains, UK. In April 2016, a Political Exercise was conducted in London, UK, bringing together civilian and military decision makers from the participating countries, to exercise the political and policy decision making processes for the Battlegroup.

No further joint training exercises for either Battlegroup are currently envisaged.

Overseas Missions Data

21/06/2016WRA0260035. Deputy Éamon Ó Cuív asked the Taoiseach and Minister for Defence the number of overseas missions on which Irish soldiers are serving; the number of these that only involve troops from European Union States or North Atlantic Treaty Organisation States; the number that involve soldiers from states that are neither in the North Atlantic Treaty Organisation or the European Union; the names of these countries and the countries in which the missions are located; and if he will make a statement on the matter. [16891/16]

21/06/2016WRA02700Minister of State at the Department of Defence (Deputy Paul Kehoe): As of 1 June 2016, Ireland is contributing 474 personnel to 11 different missions throughout the world. The main overseas missions, in which Defence Forces personnel are currently deployed, are:

6 21 June 2016 1. the United Nations Interim Force in Lebanon (UNIFIL) with 199 personnel,

2. the United Nations Disengagement Observer Force (UNDOF) with 136 personnel and

3. the Naval Service Humanitarian mission in the Mediterranean with 57 personnel.

Ten of the eleven missions in which the Defence Forces include both European Union mem- ber States, North Atlantic Treaty Organisation members, and States who are members of neither organisation. Operation Pontus only involves Irish Personnel operating in co-ordination with the Italian authorities. Details of the contributors to all these missions are set out in the follow- ing table.

Contributors of personnel to the eleven missions in which the Defence Forces are participat- ing

Overseas Missions - UNIFIL - Lebanon Armenia, Austria, Bangladesh, Belarus, Belgium, Brazil, Brunei, Cambodia, China, Croatia, Cyprus, El Salvador, Estonia, Fiji, Finland, France, Germany, Ghana, Greece, Guatemala, Hungary, India, Indonesia, Ireland, Italy, Kenya, Malaysia, Mexico, Nepal, Nigeria, Qatar, Republic of Korea, Serbia, Sierra Leone, Slovenia, Spain, Sri Lanka, Turkey, The former Yugoslav Republic of Macedonia and United Republic of Tanzania. UNDOF - Syria Bhutan, Czech Republic, Fiji, India, Ireland, Nepal and Netherlands.

EUFOR – Bosnia & Herzegovina Albania, Austria, Bulgaria, Chile, Czech Republic, Fin- land, Former Yugoslav Republic of Macedonia, Greece, Hungary, Ireland, Italy, Netherlands, Poland, Romania, Slovakia, Slovenia, Spain, Sweden, Switzerland, Turkey, United Kingdom.

EUTM Mali Albania, Austria, Belgium, Bulgaria, Czech Republic, Estonia, Finland, France, Georgia, Germany, Greece, Hungary, Ireland, Italy, Latvia, Lithuania, Luxembourg, Montenegro, Netherlands, Portugal, Romania, Serbia, Slovenia, Slovakia, Spain, Sweden, United Kingdom.

KFOR – Kosovo Albania, Armenia, Austria, Bulgaria, Canada, Croatia, Czech Republic, Denmark, Estonia, Finland, France, Ger- many, Greece, Hungary, Ireland, Italy, Lithuania, Luxem- bourg, Moldova, Netherlands, Norway, Poland, Portugal, Romania, Slovenia, Sweden, Switzerland, Turkey, United Kingdom, United States and Ukraine.

Naval Service Humanitarian Mission in the Mediterranean Ireland.

UNTSO – Israel, Lebanon & Syria Argentina, Australia, Austria, Belgium, Bhutan, Canada, Chile, China, Denmark, Estonia, Fiji, Finland, France, India, Ireland, Nepal, Netherlands, New Zealand, Norway, Russian Federation, Serbia, Slovakia, Slovenia, Sweden, Switzerland and United States.

MINURSO – Western Sahara Argentina, Austria, Bangladesh, Bhutan, Brazil, China, Croatia, Djibouti, Egypt, El Salvador, France, Germany, Ghana, Guinea, Honduras, Hungary, India, Indonesia, Ire- land, Kazakhstan, Malawi, Malaysia, Mexico, Mongolia, Nepal, Nigeria, Pakistan, Poland, Republic of Korea, Rus- sian Federation, Sri Lanka, Switzerland, Togo, and Yemen.

7 Questions - Written Answers

Overseas Missions -

MONUSCO – Republic of Congo Bangladesh, Belgium, Benin, Bolivia, Bosnia and Herze- govina, Brazil, Burkina Faso, Cameroon, Canada, China, Côte d’Ivoire, Czech Republic, Egypt, France, Ghana, Guatemala, India, Indonesia, Ireland, Jordan, Kenya, Ma- lawi, Malaysia, Mali, Mongolia, Morocco, Nepal, Niger, Nigeria, Pakistan, Paraguay, Peru, Poland, Romania, Rus- sian Federation, Senegal, Serbia, South Africa, Sri Lanka, Sweden, Switzerland, Tunisia, Ukraine, United Kingdom, United Republic of Tanzania, United States, Uruguay, Ye- men and Zambia.

UNOCI – Ivory Coast Bangladesh, Benin, Bolivia, Brazil, Cameroon, Chad, China, Ecuador, Egypt, El Salvador, Ethiopia, France, Gambia, Ghana, Guatemala, Guinea, India, Ireland, Jordan, Kazakhstan, Malawi, Morocco, Namibia, Nepal, Niger, Nigeria, Pakistan, Paraguay, Peru, Philippines, Po- land, Republic of Korea, Republic of Moldova, Romania, Russian Federation, Senegal, Serbia, Spain, Togo, Tunisia, Uganda, Ukraine, Uruguay, United Republic of Tanzania, Yemen, Zambia and Zimbabwe.

OSCE – Bosnia & Herzegovina and Austria Albania, Andorra, Armenia, Austria, Azerbaijan, Belarus, Belgium, Bosnia & Herzegovina, Bulgaria, Canada, Croatia, Cyprus, Czech Republic, Denmark, Estonia, Finland, France, Georgia, Germany, Greece, Holy See, Hungary, Iceland, Ireland, Italy, Kazakhstan, Kyrgyzstan, Latvia, Liechtenstein, Lithuania, Luxembourg, Malta, Moldova, Monaco, Mongolia, Montenegro, Netherlands, Norway, Poland, Portugal, Romania, Russian Federation, San Marino, Serbia, Slovakia, Slovenia, Spain, Sweden, Switzerland, Tajikistan, the former Yugoslav Republic of Macedonia, Turkey, Turkmenistan, Ukraine, United King- dom, United States, Uzbekistan.

Defence Forces Personnel

21/06/2016WRB0020036. Deputy Lisa Chambers asked the Taoiseach and Minister for Defence the steps he will take to address the low number of female personnel in the Defence Forces; and if he will make a statement on the matter. [16900/16]

21/06/2016WRB00300Minister of State at the Department of Defence (Deputy Paul Kehoe): The number of women serving in the Permanent Defence Force at 31st May 2016 was 546, which represented 6% of the overall strength of the PDF at that date.

The number of women serving in the and Naval Service Reserve on the same date was 285 (effective), which represents 13% of the effective strength at that date.

The Government is committed to a policy of equal opportunity for men and women through- out the Defence Forces and to the full participation by women in all aspects of Defence Forces activities.

The Defence Forces have no restrictions as regards the assignment of men or women to the full range of operational and administrative duties and all promotions and career courses are open to both sexes on merit.

A key impediment in achieving greater numbers of women serving in the Defence Forces is the lack of women recruitment applicants relative to men. This may be as a result of societal

8 21 June 2016 perception and attitudes to female soldiers and officers. There have been a number of recent initiatives to encourage greater female participation. This has included social media campaigns, advertising targeted at women, and school visits and career exhibitions.

Over the course of the White Paper, further initiatives will be developed to encourage more women to apply for the Defence Forces and to increase female participation at all ranks. This will include a survey to identify any impediments to the advancement of women in the PDF, including the impact of the requirements of career courses and overseas service on female re- tention and advancement.

Defence Forces Reserve

21/06/2016WRB0040037. Deputy Michael Healy-Rae asked the Taoiseach and Minister for Defence the status of a recruitment drive to the Reserve Defence Forces (details supplied); and if he will make a statement on the matter. [16896/16]

21/06/2016WRB00500Minister of State at the Department of Defence (Deputy Paul Kehoe): As the Deputy may be aware, the White Paper on Defence published in 2015 reaffirms the Government’s com- mitment to the ongoing development of the Reserve Defence Forces.

Among other policy initiatives to be undertaken in the coming years in relation to the Re- serve, the White Paper provides that the overarching establishment of the Army Reserve (AR) and Naval Service Reserve (NSR) is to be set at 4,169 personnel, consisting of 3,869 Army Reservists and the expansion of the establishment of the four NSR Units from currently 200 personnel to 300.

As the Deputy will appreciate a key challenge is to recruit and train sufficient personnel to meet the reserve establishment provided for.

Generally speaking recruitment campaigns are conducted on an annual ongoing basis. The current recruitment competition started in September 2015. It has currently been paused, on an administrative basis, as the Defence Forces are administering the recruitment campaign for candidates to enter the Permanent Defence Forces. I am advised that it is intended to recom- mence the campaign for the Reservists in the coming months.

The Deputy can be assured that I share his belief in the value of providing young people with range of unique life experiences which can be got from participation in both the Permanent and Reserve Defence Forces.

Naval Service

21/06/2016WRB0060038. Deputy Thomas P. Broughan asked the Taoiseach and Minister for Defence the opera- tions of the Naval headquarters in County Cork and the impact of the planned waste incinerator currently in the planning permission process; and if he will make a statement on the matter. [16651/16]

21/06/2016WRB00700Minister of State at the Department of Defence (Deputy Paul Kehoe): The Department of Defence, following consultation with the Air Corps and the Naval Service, recently made an oral submission to an Bord Pleanála regarding the proposed development of the waste to energy facility in Ringaskiddy. The submission identified two areas of concern, the proximity of the incinerator stack to Haulbowline and the difficulties in the evacuation of the island if neces- sitated by an incident at the incinerator facility. 9 Questions - Written Answers The proximity of the stack of the waste-to-energy facility to the helicopter approach paths of Haulbowline Naval Base and Spike Island is a matter of concern due to the fact that this stack will be emitting significant amounts of exhaust gases and is seen by the Department as a potential hazard as it may in certain conditions render approaches by Air Corps helicopters into and out of Haulbowline Island as unsafe.

The second area of concern is due to the fact that Haulbowline Island is accessed by a bridge which is connected to the Ringaskiddy Road. The proposed incinerator is to be built adjacent to this road, before Haulbowline. Therefore, in the event of any accident at the incinerator, road access to and from Haulbowline is threatened. This could in some instances hinder the opera- tion of the Naval Base or in the event of a major incident necessitating local area evacuation, the evacuation of Haulbowline would be denied.

Defence Forces Properties

21/06/2016WRB0080039. Deputy Fiona O’Loughlin asked the Taoiseach and Minister for Defence if he is mak- ing progress in negotiations with residents in the in County Kildare who have been threatened with eviction; and if he will make a statement on the matter. [16865/16]

21/06/2016WRB00900Minister of State at the Department of Defence (Deputy Paul Kehoe): The policy is to withdraw the provision of married quarters to serving personnel as they are no longer required for the modern soldier.

Personnel who leave the Defence Forces or who vacate a married quarter property are re- quired to return vacant possession of that property. Most observe this requirement.

Failure to vacate the property means that those remaining are overholding as they have no right to hold or reside in military accommodation.

The ongoing illegal occupation of these properties cannot be supported. The Department does not have a role in the provision of housing accommodation for the general public. It can- not subsidise housing for people who have no entitlement and who may well have the means to supply housing for themselves.

Each case of overholding is dealt with on an individual basis. It is always preferable not to have to use legal means to obtain vacant possession. My officials have met with a number of overholders to discuss and examine what assistance can be provided to bring about a resolution.

Securing alternative housing is a matter for the individuals concerned. In certain cases in- dividuals may qualify for social housing or some level of housing assistance. When requested to do so the Department provides whatever documentation it can to support such applications.

Our engagement with overholders is ongoing. Legal advice is also being sought in relation to possible agreed solutions. It is hoped that the discussions will provide an opportunity for both sides to work together in order to bring about a resolution to the problem.

We are aware of a small number of overholders who are particularly vulnerable and the deputy can be assured that they will be treated in an appropriately sensitive manner.

Defence Forces Reorganisation

21/06/2016WRB0100040. Deputy Éamon Ó Cuív asked the Taoiseach and Minister for Defence his plans to

10 21 June 2016 restore the three brigade structure to the Army; and if he will make a statement on the matter. [16892/16]

21/06/2016WRB01100Minister of State at the Department of Defence (Deputy Paul Kehoe): Following a comprehensive review of expenditure carried out in 2011, the strength ceiling of the Permanent Defence Force (PDF) was established at 9,500 personnel.

It was clear from the review that the three brigade structure was no longer viable, par- ticularly when compared to international norms. Accordingly, a major re-organisation of the Defence Forces, encompassing the consolidation of the three under-strength brigades into two full strength brigades, took place. The decision to move to a two brigade structure involved full consultation with the Defence Forces, and final proposals approved by the then Minister for Defence were agreed between the Chief of Staff and the Secretary General. Key aspects of this re-organisation, which was introduced in 2012, included the consolidation of under-strength units into a smaller number of full strength units, a reduction in the number of headquarters and the associated re-deployment of personnel from administrative and support functions to operational units.

The revised two brigade structure has optimised the capacity of the Defence Forces to de- liver the required operational outputs, within available resources, and to continue to fulfil all roles assigned by Government. A return to a three brigade structure would cause a range of un- necessary inefficiencies, such as an increased administrative burden arising from the need to re- introduce a layer of non-operational middle management, and a return to under-strength units.

The White Paper on Defence 2015 resulted from a wide ranging consultation process and a comprehensive examination of Defence requirements over the next decade. It provides for retention of the Army’s two existing infantry Brigades and the Defence Forces Training Centre, and for measures to further enhance the capabilities of the . Furthermore, the recently agreed Programme for a Partnership Government specifically provides for imple- mentation of the White Paper. In all of these circumstances, I have no plans to make changes to the existing two brigade structure.

Defence Forces Training

21/06/2016WRB0120041. Deputy Fiona O’Loughlin asked the Taoiseach and Minister for Defence the status of the development of a new institute for leadership and peacekeeping studies at the Defence Forces Training Centre; and if he will make a statement on the matter. [16864/16]

21/06/2016WRB01300Minister of State at the Department of Defence (Deputy Paul Kehoe): The Programme for a Partnership Government includes a commitment to develop a new Institute for Peace Sup- port and Leadership Training at the Defence Forces Training Centre in the Curragh. This com- mitment was also reflected in the White Paper on Defence, published in August 2015, which sets out the policy framework for the Defence Organisation for the next ten years and beyond. It is foreseen that the new Institute will have international standing and contribute to the overall development of knowledge and experience in the areas of peace support, leadership and conflict resolution.

Good progress has been made since publication of the White Paper in development of the concept for the Institute. This has included intial consultation with national and international third level and research institutes and potential philanthropic contributors. While some fur- ther consultation with these and other organisations is required, an Institute Development Task Force to progress the concept and develop an implementation plan is now being established. It

11 Questions - Written Answers is also planned to hold a major international symposium to debate and consider the role which Ireland and the Institute can play in Peace and Leadership and to advise on the initial pro- gramme for the Institute.

In its Public Capital Programme, the Government has allocated an initial €10 million for a capital building programme towards financing the construction of this new Institute in the Curragh. I hope to showcase the initial shape of the Institute once specific timescales for the establishment and roll out of the proposal have been identified by the Institute Development Task Force.

Defence Forces Equipment

21/06/2016WRB0140042. Deputy Richard Boyd Barrett asked the Taoiseach and Minister for Defence the weap- ons and equipment his Department or the Defence Forces purchased in each of the years 2014 to 2016 to date; that they plan to purchase in the coming year, detailing the value and nature of the purchase and the country and companies from which these purchases were or will be made; and if he will make a statement on the matter. [16894/16]

21/06/2016WRB01500Minister of State at the Department of Defence (Deputy Paul Kehoe): The Defence Or- ganisation routinely publishes details of Purchase Orders for goods and services valued at more than €20,000. This information is published on the Department of Defence website – www. defence.ie.

The Department has published details of Purchase Orders including those placed for weap- ons and equipment by the Defence Organisation for the years 2014 and 2015. I am advised that a small number of Purchase Orders for that timeframe were excluded from the published list for security and operational reasons.

Details of Purchase Orders placed for the first quarter of 2016 are currently being compiled and will be published shortly.

Weapons and equipment procurement requirements for the Defence Forces are kept under constant review. The White Paper on Defence sets out the key capability requirements for the coming decade and the major equipment projects that will be implemented for the Defence Forces. There is a rigorous planning and decision making process in place which underpins equipment procurement priorities.

A significant amount of the information sought by the Deputy is included in the information as already published. However, further details in relation to this information are being com- piled and will be forwarded to the Deputy.

Departmental Properties

21/06/2016WRB0160043. Deputy Peadar Tóibín asked the Taoiseach the properties Department or agencies un- der its remit leased; the number it leased on upward-only rents; the number of the lessors that are involved with the National Asset Management Agency or any of the State banks; and the details of each lease. [16608/16]

21/06/2016WRB01700The Taoiseach: All properties occupied by my Department are provided by the Office of Public Works. My Department does not lease any properties. 12 21 June 2016 Employment Data

21/06/2016WRB0180044. Deputy Niall Collins asked the Taoiseach the number of persons on the national mini- mum wage and their percentage of the workforce; and if he will make a statement on the matter. [16537/16]

21/06/2016WRB01900Minister of State at the Department of the Taoiseach (Deputy Regina Doherty): The exact information requested by the Deputy is currently not available.

The National Employment Survey, which provided statistics on the structure of earnings, was discontinued in 2010 due to resource constraints.

However, the CSO is currently completing a project to provide corresponding statistics for the years 2011 to 2014 by using administrative data sources rather than direct surveying of businesses.

For this project, the CSO is analysing Revenue P35 data for employees, combined with other statistical and administrative data sources, to provide a structural breakdown of earnings by age, gender, occupation, sector and other variables.

All of the analysis is being conducted by the CSO under the Statistics Act, 1993.

The results for the years 2011 to 2014 will be published before the end of July 2016 and the requested tables will be provided as soon as the results are available.

Zero-hour Contracts

21/06/2016WRB0200045. Deputy Niall Collins asked the Taoiseach the number of persons on zero-hour contracts and their percentage of the workforce; and if he will make a statement on the matter. [16538/16]

21/06/2016WRB0210046. Deputy Niall Collins asked the Taoiseach the number of persons on if-and-when con- tracts and their percentage of the workforce; and if he will make a statement on the matter. [16539/16]

21/06/2016WRB02200Minister of State at the Department of the Taoiseach (Deputy Regina Doherty): I pro- pose to take Questions Nos. 45 and 46 together.

The exact information requested by the Deputy is not available.

The Quarterly National Household Survey (QNHS) is the official source of estimates of employment in the State.

The most recent figures available are for Q1 2016.

Table 1 shows the number of employees (ILO) aged 15 years and over classified by perma- nency of job from Q1 2008 to Q1 2016.

Table 2 shows the number of employees (ILO) aged 15 years and over not in permanent em- ployment classified by reasons for being in temporary employment from Q1 2008 to Q1 2016.

Table 3 shows the number of employees (ILO) aged 15 years and over with temporary job/ work contract of limited duration as a percentage of total employees (including not stated) from Q1 2008 to Q1 2016.

Table 1 Employees (ILO) aged 15 years and over classified by permanency of job 13 Questions - Written Answers

‘000 Q1 08 Q1 09 Q1 10 Q1 11 Q1 12 Q1 13 Q1 14 Q1 15 Q1 16 Person has 1,610.0 1,501.2 1,415.7 1,371.5 1,350.1 1,364.5 1,391.9 1,445.1 1,486.3 a perma- nent job or work contract of unlimited duration Person 139.8 135.5 139.6 150.7 157.1 145.1 142.5 134.7 126.4 has a temporary job/work contract of limited duration Not stated 22.0 11.3 13.9 13.7 15.8 17.7 21.1 18.7 23.7 Total 1,771.9 1,648.0 1,569.2 1,535.9 1,523.0 1,527.3 1,555.5 1,598.6 1,636.4 Data may be subject to future revision.

Data may be subject to sampling or other survey errors, which are greater in respect of smaller values or estimates of change.

Reference period: q1 = January - March.

Source: Quarterly National Household Survey, Central Statistics Office.

Table 2 Employees (ILO) aged 15 years and over not in permanent employment clas- sified by reasons for being in temporary employment

‘000

Q1 08 Q1 09 Q1 10 Q1 11 Q1 12 Q1 13 Q1 14 Q1 15 Q1 16 It is a contract 5.7 4.8 5.3 [2.9] 5.2 17.4 16.9 16.9 16.5 covering a pe- riod of training Person could 23.4 28.4 37.6 42.4 50.1 77.2 75.2 71.9 64 not find a per- manent job Person did not 29.5 19.4 18.5 17.9 15.2 18.1 18.6 20.2 17.6 want a perma- nent job It is a contract : : : : : [3.8] 5.3 [4.7] * for a proba- tionary period Not stated 81.2 82.9 78.1 87.6 86.7 28.6 26.6 21.1 24.7 Total 139.8 135.5 139.6 150.7 157.1 145.1 142.5 134.7 126.4 Note: Caution is warranted in comparing data captured from Q2 2012 to prior periods due to change in filter for question and adding of additional answer option impacting upon direct comparability of series.

: No data available

Data may be subject to future revision. 14 21 June 2016 * Estimates for numbers of persons or averages where there are less than 30 persons in a cell are not produced as estimates are too small to be considered reliable.

Parentheses [ ] indicate where there are 30-49 persons in a cell, estimates are considered to have a wider margin of error and should be treated with caution.

Data may be subject to sampling or other survey errors, which are greater in respect of smaller values or estimates of change .

Reference period: q1 = January - March.

Source: Quarterly National Household Survey, Central Statistics Office.

Table 3 Employees (ILO) aged 15 years and over with temporary job/work contract of limited duration as a percentage of total employees (including not stated)

%

Q1 08 Q1 09 Q1 10 Q1 11 Q1 12 Q1 13 Q1 14 Q1 15 Q1 16 Employees (ILO) 7.9 8.2 8.9 9.8 10.3 9.5 9.2 8.4 7.7 aged 15 years and over with temporary job/work contract of limited duration as a percentage of total employees (including not stated) Data may be subject to future revision.

Data may be subject to sampling or other survey errors, which are greater in respect of smaller values or estimates of change.

Reference period: q1 = January - March.

Source: Quarterly National Household Survey, Central Statistics Office.

21/06/2016WRC00075Cross-Border Projects

21/06/2016WRC0010047. Deputy Micheál Martin asked the Taoiseach his role in working with the Northern Ireland Executive in agreeing and implementing cross-Border projects that can benefit from European Union funding, as per the programme for Government; and if he will make a state- ment on the matter. [13104/16]

21/06/2016WRC0015048. Deputy Micheál Martin asked the Taoiseach the status of the next British-Irish Council meeting; and if he will make a statement on the matter. [13106/16]

21/06/2016WRC0017566. Deputy Peter Fitzpatrick asked the Taoiseach the status of the proposed Narrow Water Bridge project following the meeting of the North-South Ministerial Council. [15870/16]

21/06/2016WRC00200The Taoiseach: I propose to take Questions Nos. 47, 48 and 66 together.

The Irish Government outlined its commitment to supporting North-South development in the National Development Plan 2016 - 2021, in the “Fresh Start Agreement” and in the Pro- gramme for a Partnership Government. The Irish Government is committed to investing in in- frastructure to support North-South cooperation and unlock the full potential of the economies of both jurisdictions. 15 Questions - Written Answers As co-chair of the North South Ministerial Council, I ensure that the focus of Ministers is on the priorities for additional North South co-operation which they have identified in their respec- tive areas of responsibility, particularly as regards assisting economic recovery, job creation, the best use of public funds and the most effective delivery of services for citizens across the island.

As indicated in the recent Programme for a Partnership Government, I look forward to the further development of mutual North South benefit, particularly through the ongoing work of the NSMC.

Narrow Water Bridge:

Under “A Fresh Start - Stormont House Agreement” the Northern Ireland Executive and the Irish Government have agreed to undertake a review of the Narrow Water Bridge project with a view to identifying options for its future development, for consideration by the North South Ministerial Council next month. Discussions at official level have taken place between Northern Ireland Executive and Irish Government officials and a report has been prepared for consideration at the next NSMC.

British-Irish Council

I attended the 26th Summit of the British-Irish Council on 17 June hosted by the Scottish Government in Glasgow. The heads of delegation were welcomed by First Minister Sturgeon on behalf of the Scottish Government.

First Minister Sturgeon commenced the meeting with a minute’s silence to mark the shock- ing death of Jo Cox MP. The Council reflected on priorities for Member Administrations and discussed how the Council could continue to promote cooperation on issues of common con- cern. The Council also noted that there might be scope for new areas of cooperation based on emerging Government and Member Administration priorities across these islands, many of which have had elections in recent times. In the context of current issues, the forthcoming UK EU Referendum was discussed. I clearly set out the Irish Government position and our pro- gramme of outreach to Irish citizens in Britain who have a vote.

Minister of State, Helen McEntee met with counterparts with particular responsibility for unpaid carers in advance of the Summit to consider a detailed paper prepared by the Council’s Social Inclusion work sector. The discussion was then taken forward by Heads of Adminis- tration, where the Council agreed that carers are an integral part of society, caring for family, friends and neighbours affected by physical or mental illness, disability, frailty or substance misuse. It was agreed that further sharing of research and good practice in this important area will be pursued by Member Administrations.

The Scottish Government updated the Council on the work done in advance of the Summit to take forward work being undertaken to review the operation of the Council. It also noted the Secretariat’s end of year report and agreed a new Council Communications Plan for the period 2016-18. The Council also approved publication of the BIC Annual Report 2015.

The next British Irish Council Summit is scheduled for Wales in November 2016.

21/06/2016WRC00350Overseas Visits

21/06/2016WRC0050049. Deputy Micheál Martin asked the Taoiseach his plans for overseas visits for the re- mainder of 2016; and if he will make a statement on the matter. [14515/16]

16 21 June 2016

21/06/2016WRC00600The Taoiseach: Between now and the end of 2016, I will be attending three European Council meetings. I will travel to Brussels next week, on 28-29 June and again in October (20- 21) and December (15-16).

I do not have any firm plans, at present, for other overseas visits. My officials, in conjunc- tion with other relevant Government Departments and Agencies, are considering the matter with a view to developing recommendations. Careful consideration will be given to locations where significant opportunities exist to support and promote our efforts, at both EU and inter- national level, to enhancing trade and bilateral relations, attracting investment and tourism, and growing export markets.

21/06/2016WRC00650UK Referendum on EU Membership

21/06/2016WRC0070050. Deputy Paul Murphy asked the Taoiseach if he has discussed the exit of the United Kingdom from the European Union following the referendum scheduled for 23 June 2016 with the British Prime Minister, Mr. David Cameron. [14458/16]

21/06/2016WRC0075051. Deputy Micheál Martin asked the Taoiseach his role in encouraging Irish persons liv- ing in Britain to vote to remain in the European Union; and if he will make a statement on the matter. [14461/16]

21/06/2016WRC0077552. Deputy Micheál Martin asked the Taoiseach his views that Border controls will return in the event of Britain and Northern Ireland voting to leave the European Union; and if he will make a statement on the matter. [14463/16]

21/06/2016WRC0078753. Deputy Micheál Martin asked the Taoiseach if he has spoken to, written to or met with the British Prime Minister, Mr. David Cameron, regarding the UK’s referendum on 23 June 2016 on a British exit from the European Union; and if he will make a statement on the matter. [14465/16]

21/06/2016WRC0079354. Deputy Gerry Adams asked the Taoiseach to report on his visit to London; and if he will make a statement on the matter. [14466/16]

21/06/2016WRC0079655. Deputy Gerry Adams asked the Taoiseach to report on the meetings he had with Irish groups while in London; and if he will make a statement on the matter. [14467/16]

21/06/2016WRC0079756. Deputy Gerry Adams asked the Taoiseach to report on meetings he had with British politicians during his visit to London; and if he will make a statement on the matter. [14468/16]

21/06/2016WRC0079857. Deputy Eamon Ryan asked the Taoiseach his plans in securing a “Remain” vote in the United Kingdom’s European Union referendum on 23 June 2016. [14470/16]

21/06/2016WRC0079958. Deputy Gerry Adams asked the Taoiseach the security concerns expressed at the Eu- ropean Union Council at the implications of the possible British exit from the European Union; and if he will make a statement on the matter. [14495/16]

21/06/2016WRC00800The Taoiseach: I propose to take Questions Nos. 50 to 58, inclusive, together.

I recently spoke with Prime Minister Cameron by telephone on 6 May 2016, when we dis- cussed a number of matters, including the UK Referendum on EU Membership.

I meet with Prime Minister Cameron regularly in the normal course of events on a bilateral basis and at European Council meetings. Our recent bilateral discussions have centred on the importance of the UK remaining a member of the EU.

17 Questions - Written Answers When I met the Prime Minister on January 25th in Downing Street, the focus then was on the European Council on 18 February 2016 and the constructive role that Ireland could play in the negotiations. A package of reforms was subsequently agreed by Heads of State and Govern- ment which provides a strong basis for the Prime Minister’s campaign for the UK to stay in a reformed EU.

This is clearly of particular importance to Ireland, given that our relationship with the UK is closer than with any other EU Member State. The Prime Minister fully recognises our mutual interest in matters relating to Northern Ireland, the importance of a reformed, more effective European Union, the Common Travel Area and our strong economic ties.

The Irish Government has been very active in our engagement with the UK Government, and with our EU partners, in outlining our concerns and our interests in this matter. Our Gov- ernment’s position on the EU-UK question is clear: We want the UK, as our friend, closest neighbour and partner, to remain a member of a reformed EU.

We believe Ireland has a unique perspective and interest in the outcome of the referendum: as a neighbour sharing a land border; as a partner with the UK in transforming British-Irish relations in recent years; and as a facilitator and co-guarantor, with the UK, of successive agree- ments aimed at securing peace and prosperity in Northern Ireland.

It is for all these reasons that the Irish Government is working hard to ensure that Irish Com- munities in the UK are aware of our particular perspective on the outcome of the upcoming referendum.

Irish citizens living in the UK will have a vote in the upcoming referendum. It is important that they are aware that they are entitled to vote and that they are fully informed on the issues.

I and other members of the Government have completed a targeted programme of outreach in Britain and Northern Ireland to Irish and Irish-connected citizens in the UK who have a vote. Last week I had engagements in Belfast and a programme of events in Liverpool, Manchester and Glasgow where I engaged with the Irish community. I also attended a Summit meeting of the British Irish Council in Glasgow. However, my programme in Britain coincided with the shocking death of Jo Cox MP in West Yorkshire. Out of respect for her and her family, and in light of the suspension of the two official campaigns, I refrained from issuing any public state- ments about the referendum.

I travelled in a personal capacity to attend the Mayo v London game recently in Ruislip, London. Irish4Europe were launching their voter registration campaign and I met with some of the members informally in the margins of the match.

Naturally, the Government is alive to the risks associated with a possible overall leave result in the referendum.

One of the most beneficial effects of the peace process and our common membership of the EU has been the virtual elimination of the border. The re-establishment of customs checks on the border, or indeed of any customs arrangements, would be a regrettable and backward step for North-South trade and cooperation. The situation would depend on the future arrangements to be put in place between the UK and the remaining 27 EU member states, including Ireland, following a UK departure. We would certainly do our utmost to preserve the Common Travel Area. But it is difficult to imagine a situation where there would be no controls or checks on the movement of goods if the UK left the EU.

All EU partners share the view - for a variety of economic, political and social reasons - that continued UK membership of the Union is in all of our interests. Discussions on the EU-UK 18 21 June 2016 issue at the European Council to date were focussed on securing a constructive deal with the UK that would enable Prime Minister Cameron to recommend and campaign for a remain vote in the UK referendum, and that was also acceptable to all EU partners. Next week’s European Council meeting was rescheduled to 28/29 June to allow EU leaders have a first exchange on the referendum result, to reflect on its implications and to begin to chart a way forward: Ireland is ready for these discussions. Today, I gave my pre-European Council statement to the House, where I will outline Ireland’s position in more detail.

21/06/2016WRC01650Departmental Staff

21/06/2016WRC0250059. Deputy Micheál Martin asked the Taoiseach the changes to staffing and to the salaries of advisers in his Department since 1 March 2016; and if he will make a statement on the mat- ter. [14510/16]

21/06/2016WRC0255060. Deputy Micheál Martin asked the Taoiseach the changes he will implement in the structure and responsibilities of his Department subsequent to the formation of the new Govern- ment; and if he will make a statement on the matter. [14511/16]

21/06/2016WRC0257565. Deputy Brendan Howlin asked the Taoiseach his plans for the staffing and structure of his Department. [15845/16]

21/06/2016WRC02600The Taoiseach: I propose to take Questions Nos. 59, 60 and 65 together.

The Department of the Taoiseach’s core role will continue to be supporting me, the Govern- ment and the Ministers of State in the Department in relation to all our domestic, EU and inter- national responsibilities, and driving implementation of the new Programme for a Partnership Government.

The Programme for Government Office, based in my Department, will continue to monitor implementation of Programme for Government commitments across all Government Depart- ments and regularly report progress to me. The structures used to advance the Government’s key priorities will include Cabinet Committees; bi-laterals with relevant Ministers; and regular engagement at official level between the Programme for Government Office and Government Departments.

Taking account of the need to maintain a new relationship between the Oireachtas and the Government, the Office of the Chief Whip will have an expanded role in terms of more active management of all aspects of the legislative process, working with Government Departments and the Houses of the Oireachtas. As part of this process, the Chief Whip’s Office will work closely with a new Parliamentary Liaison Unit and the Programme for Government Office which will track progress across all policy and legislative commitments.

Minister of State Murphy’s portfolio has been expanded to include the EU Digital Single Market, in addition to responsibility for EU Affairs and Data Protection.

My Department no longer has responsibility at Ministerial level for International Financial Services. The member of staff who has been working full-time on International Financial Ser- vices will shortly transfer to the Department of Finance.

In relation to my role as Minister for Defence, Deputy Paul Kehoe has been appointed as Minister of State with special responsibility for the Department of Defence and will attend Government meetings. I signed the Defence (Delegation of Ministerial Functions) Order 2016 on 14th June 2016.

19 Questions - Written Answers The outcome of the UK referendum on EU Membership may also have structural implica- tions for my Department.

The resource implications of the above and other Programme for Government commitments for which my Department has direct responsibility for implementing will be addressed in the context of the Estimates process.

There have been no changes to the staffing and salaries of advisers in my Department since 1 March, 2016. There are currently five Special Advisers employed in my Department, four of whom are my Special Advisers and one of whom is Special Adviser to the Government Chief Whip.

21/06/2016WRC02750Citizens Assembly

21/06/2016WRC0290061. Deputy Ruth Coppinger asked the Taoiseach when he will issue the tender for a poll- ing company to source members of the citizens’ assembly; and his plans to publish full details of the call for tender, given the importance of the issues to be addressed. [15598/16]

21/06/2016WRC03000The Taoiseach: Members for the Citizens’ Assembly will be selected on the basis of being representative of the Irish electorate in terms of gender, age and regional spread by a polling company commissioned for that purpose.

Preparations for establishment of the Assembly in the Autumn are being worked on by my Department. The tender document will be published when finalised.

Constitutional Amendments

21/06/2016WRC0310062. Deputy Brendan Howlin asked the Taoiseach the constitutional referendums he will hold in the next 12 months. [15849/16]

21/06/2016WRC03200The Taoiseach: Under the Programme for a Partnership Government, the Government is committed to holding Constitutional referendums on the following matters:

- Article 41.2.1, regarding a “woman’s life within the home”

- Article 40.6.1, on the offence of blasphemy

- Ireland’s participation in the Unified Patent Court

- Giving the office of Ceann Comhairle constitutional standing.

A timescale has not yet been established for these referendums.

The Programme for a Partnership Government also commits the Government to establish a Citizens’ Assembly, within six months, with a mandate to look at a limited number of key issues over an extended period of time. While these issues will not be limited to those directly per- taining to the constitution, the Citizens’ Assembly will be asked to make recommendations to the Dáil on further constitutional changes, including on the Eighth Amendment, on fixed-term parliaments and on the manner in which referendums are held.

The Programme for a Partnership Government also says that on foot of the recommendation of the Banking Inquiry, the Government will seek a review of the powers of Oireachtas Com- mittees in conducting inquiries and, based on this review, will consider whether there should be 20 21 June 2016 a Constitutional Referendum to strengthen Committees’ powers.

21/06/2016WRC03250Child Care Services Staff

21/06/2016WRC0330063. Deputy Gino Kenny asked the Taoiseach the average wage for child care workers; how this compares to European Union and Organisation for Economic Co-operation and Develop- ment figures. [14721/16]

21/06/2016WRC03400Minister of State at the Department of the Taoiseach (Deputy Regina Doherty): The exact information requested by the Deputy is currently not available.

Statistics on average wages are compiled from the quarterly Earnings, Hours and Employ- ment Costs Survey (EHECS), and from the periodic Labour Costs Survey and Structure of Earnings Surveys conducted under EU Regulations. While these surveys include breakdowns by sector and by occupation, the figures do not include a separate breakdown in relation to childcare workers. Accurate results at this level of detail cannot be provided from sample sur- veys.

In the EU NACE Rev 2 economic activity classification system, Child day-care activities are classified as a sub-group of NACE code 88,Social work activities without accommodation. Figures are available for the latter sector from the quarterly EHECS survey, showing average earnings per week of €465.12 in quarter four of 2015. This is the average earnings for all oc- cupations working in the Social work activities without accommodation sector.

Some comparative EU statistics are available for the Social work activities sector from the 2012 EU-Labour Costs Survey. Average earnings in the Social work activities sector in Ireland were €32,896 in 2012, the fourth highest average earnings in the EU. The United Kingdom had a comparative figure of €28,583, while the European Union average was €25,468. See Table 1.

Average hourly earnings for the Social work activities sector are also available from the 2012 EU-Labour Cost Survey and show that Ireland earned the fourth highest average hourly earnings in the European Union, with mean hourly earnings of €20.21. This compares to mean hourly earnings of €15.87 in the UK and €15.76 as an EU average. Denmark recorded the high- est average hourly earnings in this sector with a figure of €26.70. See Table 1.

There are no comparative statistics available on childcare workers for Organisation for Eco- nomic Co-operation and Development (OECD) member countries.

The CSO is currently preparing updated results on the structure of earnings for the years 2011 to 2014, based primarily on administrative rather than survey data. Broad sectoral and oc- cupational statistics from this source will be published before the end of July and more detailed breakdowns will subsequently be available upon request, subject to CSO confidentiality rules. While this will enable more detailed structural tables than before, the CSO does not expect that it will be possible to provide tables on earnings on specific detailed occupation codes of child care workers. To identify the earnings of childcare workers, individual occupations would need to be coded at the 4 digit level of the International Standard Classification of Occupations.

21 Questions - Written Answers Table 1: Mean annual and hourly earnings in Social work activities sector (NACE code 88) - 2008 and 2012

Average Annual Average An- Average Average Earnings nual Earnings Hourly Earn- Hourly Earn- ings ings 2008 2012 2008 2012 € € € € European Union (28 23,901 25,468 14.93 15.76 countries) European Union (15 24,972 27,166 15.67 16.93 countries) Euro area (19 countries) 23,118 25,111 14.83 16.32 Belgium 21,785 26,250 16.20 18.94 Bulgaria 2,253 2,692 1.32 1.58 Czech Republic 7,800 8,643 4.51 4.99 Denmark 35,630 38,686 24.22 26.70 Germany 25,602 28,843 15.35 17.47 Estonia 7,203 7,084 4.14 4.02 Ireland 33,575 32,896 20.03 20.21 Greece 13,462 15,692 7.15 8.60 Spain 18,851 18,721 11.61 11.55 France 22,085 22,109 14.61 15.32 Croatia 13,750 10,423 7.62 6.51 Italy 17,283 19,015 10.51 12.13 Cyprus 22,511 18,700 14.99 11.42 Latvia 7,076 6,419 3.88 3.50 Lithuania 5,725 5,457 3.27 3.01 Luxembourg 34,809 40,342 21.12 23.78 Hungary 7,053 3,892 4.08 2.18 Malta 13,022 17,983 6.64 8.25 Netherlands 28,494 31,685 19.82 21.03 Austria 24,903 29,187 14.84 17.27 Poland 9,183 9,152 5.60 5.50 Portugal 10,391 11,571 6.40 7.11 Romania 2,982 2,761 1.58 1.48 Slovenia 20,514 19,961 12.48 12.41 Slovakia 6,422 7,438 3.76 4.73 Finland 24,085 27,370 15.27 17.79 Sweden 26,325 33,466 15.26 19.01 United Kingdom 28,972 28,583 16.48 15.87 Reference period: 2008 & 2012.

Source: Labour Cost Survey, Eurostat.

21/06/2016WRC03450Ministerial Responsibilities

22 21 June 2016

21/06/2016WRC0350064. Deputy Brendan Howlin asked the Taoiseach the elements of the programme for Gov- ernment for which he is responsible. [15843/16]

21/06/2016WRC03600The Taoiseach: The Programme for a Partnership Government was published on 11 May and sets out an ambitious programme of work to be implemented over the lifetime of the Gov- ernment.

My Department is working in consultation with all Departments to identify individual com- mitments and agree responsibility for these actions. These will be published in due course by each Department in their respective Statements of Strategy.

While this process has not yet concluded, my Department will have a number of commit- ments coming under its responsibility including the areas of Dáil reform, relations with Britain and Northern Ireland, and managing the new partnership approach between government and parliament. Officials in my Department will work with all stakeholders involved to progress these issues over the lifetime of the Government.

Question No. 65 answered with Question No. 59.

Question No. 66 answered with Question No. 47.

21/06/2016WRC04050Employment Data

21/06/2016WRC0410067. Deputy Brendan Griffin asked the Taoiseach further to Parliamentary Question No. 1 of 22 March 2016, the additional cost of producing detailed county estimates of the number of persons at work; the increase in sample size that would be required; and if he will make a state- ment on the matter. [17157/16]

21/06/2016WRC04200Minister of State at the Department of the Taoiseach (Deputy Regina Doherty): The Quarterly National Household Survey (QNHS) is the official source of employment estimates in the state and is conducted in line with the European regulation for the conduct of labour force surveys (577/98). The QNHS is based on a national sample of 26,000 households each quarter. This sample of households is designed to be representative of the population at both regional (NUTS 3) and national levels and the sample selection process takes account of the distribution of the population across the country.

The labour market estimates produced by the QNHS are designed to meet strict quality cri- teria set down by Eurostat, which specify the level of statistical accuracy that these estimates must achieve at national level. The CSO also produces regional labour market estimates from the QNHS (NUTS 3 level) although this level of detail is not required by Eurostat.

In order to produce labour market estimates at county level (NUTS 4) that would meet the same statistical standards as currently achieved for national and regional labour market statis- tics, it is estimated that the QNHS sample size would need to be increased by a factor of four in some counties. Potentially this could mean an increase in the QNHS sample size nationally from 26,000 households to 104,000 households.

In order to interview a sample of this size the CSO would need to increase the number of permanent field staff posts from 100 to 400 and increase the number of temporary or backup posts from 40 to 160. The current cost of the QNHS fieldwork is approximately €4m per an- num, which includes fieldstaff pay, travel costs and computer hardware costs. It is estimated that with a potential increase in the sample outlined above the cost of the QNHS could similarly increase the fieldwork costs to €16m per annum.

23 Questions - Written Answers The actual level of increase required in the household sample size in each county would require a more detailed statistical examination.

21/06/2016WRC04250UK Referendum on EU Membership

21/06/2016WRC0430068. Deputy Eugene Murphy asked the Tánaiste and Minister for Justice and Equality the measures British citizens living here will have to take in relation to Irish citizenship in the event of Britain leaving the European Union after their referendum on the matter; if they can apply for dual citizenship or a passport; and if she will make a statement on the matter. [16689/16]

21/06/2016WRC04400Tánaiste and Minister for Justice and Equality (Deputy Frances Fitzgerald): The enti- tlement to Irish citizenship, as well as the conditions pertaining to a grant of citizenship through naturalisation, are governed by the provisions of the Irish Nationality and Citizenship Act 1956, as amended. It is open to any individual to lodge an application for citizenship if and when they are in a position to meet the statutory requirements as prescribed in the Act. Under Irish law it is possible for an Irish national to hold dual citizenship; the acquisition of Irish citizenship is not contingent on the renunciation of the citizenship of another jurisdiction. Detailed informa- tion on Irish citizenship and naturalisation is available on the INIS website at www.inis.gov.ie.

As outlined, the grant of citizenship is a national competency and there are no current plans to amend the legislation. It would not be appropriate to pre-empt the outcome of the upcoming UK referendum except to say that it is our intention to maintain the existing arrangements with regard to the United Kingdom. Should an issue arise in this area during any possible future negotiations between the UK and the other EU member states, the position will be revisited at that stage.

Mental Health Policy

21/06/2016WRD0020069. Deputy Billy Kelleher asked the Tánaiste and Minister for Justice and Equality when she will appoint the director of decision support services in line with the Assisted Decision- Making (Capacity) Act 2015; the extent of the resourcing; when the director will commence working with professionals responsible for carrying out the codes of practice; and if she will make a statement on the matter. [16850/16]

21/06/2016WRD00300Tánaiste and Minister for Justice and Equality (Deputy Frances Fitzgerald): The As- sisted Decision-Making (Capacity) Act 2015 was signed into law by the President on 30 De- cember 2015 but has not yet been commenced. The Act provides for the setting up of the De- cision Support Service within the Mental Health Commission, which is an independent body under the aegis of the Department of Health. The appointment of the Director of the Decision Support Service is a matter for the Mental Health Commission, as provided by section 94 of the Act.

A Steering Group on the Implementation of the Decision Support Service was recently established to oversee the establishment and commissioning of the Decision Support Service, including overseeing the recruitment of the Director of the Service and matters relating to the resourcing of the Service. The Steering Group comprises senior officials from the Department of Justice and Equality, the Department of Health and the Mental Health Commission.

In relation to the codes of practice provided for in the Act, preparatory work has commenced on preparing guidance material for healthcare and non-health care related codes of practice. This guidance material will enable the Director, when appointed, to quickly engage with the 24 21 June 2016 relevant stakeholders in order to publish the necessary codes of practice.

Deputies will appreciate that careful planning and groundwork, and not just funding, has to be put in to ensure that the commencement of the legislation is correctly, appropriately and ef- fectively handled and my officials are working carefully on this at present in consultation with the Department of Health and the Mental Health Commission.

21/06/2016WRD00350Pension Provisions

21/06/2016WRD0040070. Deputy Dara Calleary asked the Tánaiste and Minister for Justice and Equality the pension to which a retired garda who served between 1966 and 1974 is entitled; the criteria for qualifying for a pension; the application procedure; and if she will make a statement on the matter. [16666/16]

21/06/2016WRD00500Tánaiste and Minister for Justice and Equality (Deputy Frances Fitzgerald): A mem- ber of An Garda Síochána, having served from 1966 and left the Force in 1974 through res- ignation or dismissal would not qualify for superannuation benefits under the Garda Síochána Pension Scheme. Any contributions already made to the scheme would have been refunded in accordance with the Garda Síochána Pension Orders.

21/06/2016WRD00550Spent Convictions

21/06/2016WRD0060071. Deputy Michael McGrath asked the Tánaiste and Minister for Justice and Equality if the convictions incurred by a person (details supplied) qualify as spent convictions under the Criminal Justice (Spent Convictions and Certain Disclosures) Act 2016. [16671/16]

21/06/2016WRD00700Tánaiste and Minister for Justice and Equality (Deputy Frances Fitzgerald): The Criminal Justice (Spent Convictions and Certain Disclosures) Act 2016 was commenced in full on 29th April 2016.

The effect of the Act is that a person is not obliged to disclose certain convictions which are over 7 years old, subject to specified limitations. A person will not be penalised in law or incur any liability for failing to disclose a spent conviction.

In accordance with the provisions of the Act the following convictions are now spent:

1) All convictions in the District Court for Motoring offences which are more than 7 years old subject to the proviso that spent convictions for dangerous driving are limited to a single conviction.

2) All convictions in the District Court for minor public order offences which are more than 7 years old.

3) In addition, where a person has one, and only one, conviction (other than a motoring or public order offence) which resulted in a term of imprisonment of less than 12 months (or a fine) that conviction is spent after 7 years. This provision applies to either a District Court or Circuit Court conviction.

Where a person appears before a court and is convicted of 2 or more offences which were committed at the same time or relate to the same event and more than one sentence is imposed by that court at that time, these are regarded as a single conviction for the purposes of the Act.

25 Questions - Written Answers Sexual offences or convictions in the Central Criminal Court are not eligible to become spent convictions.

21/06/2016WRD00750Equality Issues

21/06/2016WRD0080072. Deputy Barry Cowen asked the Tánaiste and Minister for Justice and Equality if it is illegal under the Equality Act 2015 or other legislation for private emergency accommodation, such as bed and breakfasts or hotels, to specifically deny accommodation to persons on the grounds that they are homeless; or on the basis that they are paying via emergency homeless assistance payments from local authorities. [16678/16]

21/06/2016WRD00900Tánaiste and Minister for Justice and Equality (Deputy Frances Fitzgerald): The Equality (Miscellaneous Provisions) Status Act 2015 introduced new provisions that prohibit discrimination in the provision of rented accommodation on the basis that a person is in re- ceipt of certain payments (such as rent supplement or housing assistance payment). Advice for persons who consider that they might have been discriminated against under these provisions or any provision of equality legislation is available from the Irish Human Rights and Equality Commission. A person who considers that their rights have been infringed under any provision of equality legislation can bring a claim for adjudication to the Workplace Relations Commis- sion. It would not be appropriate for me to offer legal advice or a legal interpretation of any statutory provision where compliance with that provision is or may be subject to adjudication by a Tribunal or consideration by a Court.

21/06/2016WRD00950Legal Services Regulation

21/06/2016WRD0100073. Deputy Mattie McGrath asked the Tánaiste and Minister for Justice and Equality the funding she is making available to the Legal Services Regulatory Authority as provided for in the Legal Services Regulation Act 2015; and if she will make a statement on the matter. [16686/16]

21/06/2016WRD01100Tánaiste and Minister for Justice and Equality (Deputy Frances Fitzgerald): As previ- ously conveyed to the House, preparations for the phased commencement of the Legal Services Regulation Act 2015 are already underway. A necessary first step in preparation for these com- mencements is the appointment of the Legal Services Regulatory Authority after which the Authority will appoint its own Chief Executive. Nominations to the new Legal Services Regu- latory Authority have been received from the relevant nominating bodies as set out in the Legal Services Regulation Act. The appointment by Government of the relevant nominees under the various criteria set out in the Act will then be subject to approval by resolution of both Houses of the Oireachtas. The Government will appoint one of the lay members of the Authority to be Chairperson. At the same time, preparations are also being made in support of the public re- cruitment, by the new Regulatory Authority, of its Chief Executive whose terms and functions are similarly set out in the 2015 Act. A start-up support team is being established and a suitable premises is being identified from which the new Authority can commence its operations.

These are the initial steps that will enable the members and Chief Executive of the new Regulatory Authority to spear-head the coming into operation of the new legal services regula- tory regime. This stage will also include the phased commencement of the Parts of the Act such as those dealing with legal costs, the new Office of the Legal Costs Adjudicator, the Roll of Practising Barristers and Pre-Action Protocols.

Following establishment of the Authority and appointment of a Chief Executive the key 26 21 June 2016 provisions centred around Part 6 of the Act dealing with the new public complaints and profes- sional conduct and disciplinary procedures, and the appointment of the new Legal Practitioners Disciplinary Tribunal, will be commenced. The managed commencement of these functions is planned for the Autumn in order to allow adequate time and preparation to ensure their effec- tiveness and success as crucial components of the new regulatory framework. It should be noted that, under the relevant transitional provisions contained in the 2015 Act, complaints already made to the Law Society under the Solicitors Acts will be brought to completion under that framework and existing law.

In support of the planned and managed commencement of the relevant Parts of the 2015 Act proposed, as I have outlined, between now and the end of this year, an allocation of €1 million has been made under the Justice Vote for 2016 as set-up support for the new Regulatory Author- ity. Any funding advanced from this allocation will be provided on a recoupable basis. As an independent corporate body under statute the new Authority will have the power to manage the conduct of its affairs and its finances. Once in operation, the new regulatory regime will be self- funding by means of a levy on the regulated legal professions. The levy will be applied under the terms set out for that specific purpose in Part 7 of the 2015Act.

21/06/2016WRD01150Garda Deployment

21/06/2016WRD0120074. Deputy Jonathan O’Brien asked the Tánaiste and Minister for Justice and Equality the number of members of An Garda Síochána in west County Dublin in each of the years from 2012 to May 2016. [16764/16]

21/06/2016WRD0130075. Deputy Jonathan O’Brien asked the Tánaiste and Minister for Justice and Equality the number of community gardaí in west County Dublin in each of the years from 2012 to May 2016. [16765/16]

21/06/2016WRD01400Tánaiste and Minister for Justice and Equality (Deputy Frances Fitzgerald): I propose to take Questions Nos. 74 and 75 together.

As the Deputy will appreciate, the Garda Commissioner is responsible for the distribution of personnel among the Garda Regions, Divisions, and Districts. Garda management keep this distribution under continuing review taking into account crime trends and policing priorities so as to ensure that the best possible use is made of these resources.

I have been informed by the Garda Commissioner the number of Gardaí and Community Gardaí in the Dublin Metropolitan Region (DMR) West on the 31 December 2012 to 2015 and on the 30 April 2016, the latest date for which figures are readily available, was as set out in the following table.

As the Deputy will be aware, when the financial crisis hit, the Government of the time in- troduced a moratorium on recruitment and the four year National Recovery Plan, published in 2010, envisaged a steady reduction in Garda numbers. Thankfully, in a recovering economy, we were able to reopen the Garda College in September 2014, and a total of 700 Garda trainees have been recruited with a further 450 planned to be recruited during the remainder of this year. So far 463 of the new Garda trainees have attested as members of An Garda Síochána and have been assigned to mainstream uniform duties nationwide. Another 76 will attest on the 7 July with 150 more to attest in November. I am assured by the Commissioner that the needs of all Garda Divisions are fully considered when determining the allocation of newly attested Gardaí and that 41 newly attested Gardaí have been assigned to the DMR West Garda Division to date.

In relation to the number of Community Gardaí, it is of course, the case that all Gardaí have 27 Questions - Written Answers a role to play in addressing community policing issues as and when the need arises. In that sense, community policing involves far more than a single unit within An Garda Síochána, a point highlighted by the Garda Inspectorate in their third report entitled “Policing in Ireland— Looking Forward” in which they stated that community policing is a fundamental policing philosophy and that there is a strong foundation for it in Ireland.

The Programme for a Partnership Government recognises that community policing is the embodiment of An Garda Síochána, providing a means of recognising that every community – both urban and rural – has its own concerns and expectations. It commits the Government to ensuring a strong and visible police presence throughout the country in order to maintain and strengthen community engagement, provide reassurance to citizens and deter crime. Key to achieving this goal is the commitment in the Programme to continue the ongoing accelerated Garda recruitment programme with a view to increasing Garda numbers to 15,000 as well as doubling the Garda Reserve and increasing the number of civilian staff so as to free-up Gardaí for front-line policing.

The Garda Commissioner’s Modernisation and Renewal Programme 2016-2021, published on 9 June 2016, places a strong emphasis on developing and supporting the community polic- ing ethos of the organisation and enhancing the current delivery model so that Gardaí spend more time in the community, gaining public confidence and trust and providing a greater sense of security. I welcome in particular, the proposal to establish local Community Policing Teams (CPTs) headed by an inspector and made up of Gardaí from across a range of areas to work with the community to prevent and detect crime. The CPTs will take proactive measures to prevent crime from happening in the area. This will entail a combination of crime prevention advice, targeting and disrupting criminals through operations and patrols, and where possible diverting people from committing crime. I also welcome the proposal to establish Community Safety Fora in every District comprising local Gardaí, local communities and key stakeholders. The implementation of these initiatives has started and will be rolled out nationwide.

DMR West Garda Division.

Date Garda Community 31/12/2012 744 80 31/12/2013 717 68 31/12/2014 699 62 31/12/2015 688 63 30/04/2016 684 63

21/06/2016WRD01450Garda Stations

21/06/2016WRD0150076. Deputy Niamh Smyth asked the Tánaiste and Minister for Justice and Equality the location of the new building acquired by the Office of Public Works for the new Garda station in Bailieborough, County Cavan; if funding is in place for this project; the date when works will go to tender and when it will be complete; and if she will make a statement on the matter. [16803/16]

21/06/2016WRD01600Tánaiste and Minister for Justice and Equality (Deputy Frances Fitzgerald): The Of- fice of Public Works (OPW) has the primary responsibility for the provision and maintenance of Garda accommodation and works closely with the Garda authorities in this regard.

The Deputy will be aware that the Garda Building and Refurbishment Programme 2016- 2021 includes the development of a new Garda station at the District Headquarters of Bailiebor-

28 21 June 2016 ough, Co. Cavan.

I am happy to inform the Deputy that, following consultation between Garda management and the OPW, a site has been identified for the new Garda Station and the OPW advises that it is progressing the legal matters relating to the acquisition of the site.

Until the site acquisition is completed, it is not possible to provide an exact timeline and details on procurement, construction and costing of the new station.

21/06/2016WRD01650Firearms Licences

21/06/2016WRD0170077. Deputy Michael Ring asked the Tánaiste and Minister for Justice and Equality when a reply will issue from her office to a query submitted on 23 May 2016 (details supplied). [16809/16]

21/06/2016WRD01800Tánaiste and Minister for Justice and Equality (Deputy Frances Fitzgerald): As indi- cated to the Deputy in the response issued on 16 June 2016, Section 3D of the Criminal Justice (Miscellaneous Provisions) Act 2009 provides that only those centre fire short firearms licensed before 19 November 2008 remain licensable. Therefore, it is not possible to substitute another centre fire handgun for a handgun licensed before 19 November 2008. However, it is open to the certificate holder to replace a part of the licensed handgun, for example, if the part was faulty.

21/06/2016WRD01850Criminal Injuries Compensation Tribunal

21/06/2016WRD0190078. Deputy Willie O’Dea asked the Tánaiste and Minister for Justice and Equality when she will appoint the criminal injuries compensation tribunal and when it will hear appeals; and if she will make a statement on the matter. [16964/16]

21/06/2016WRD02000Tánaiste and Minister for Justice and Equality (Deputy Frances Fitzgerald): I can in- form the Deputy that vacancies were advertised on the Criminal Injuries Compensation Tribu- nal in April and I expect the process of making appointments to be finalised in the near future. I am informed that the Tribunal, which is independent in the exercise of its functions, will then plan the scheduling of appeal hearings.

21/06/2016WRD02050Prison Service

21/06/2016WRD0210079. Deputy Jim O’Callaghan asked the Tánaiste and Minister for Justice and Equality if she has evaluated or plans to privatise part of the operations of the Prison Service. [17006/16]

21/06/2016WRD02200Tánaiste and Minister for Justice and Equality (Deputy Frances Fitzgerald): I have no plans currently to privatise any part of the operations of the Irish Prison Service.

The operation of the service is constantly kept under review. In this respect, it should be noted that section 5 of the Prisons Act 2007 provides that ‘the Minister may, with the consent of the Minister for Finance and the approval of Government, enter into an agreement with another person (in this Act referred to as a “contractor”) for the provision by that person of prisoner escort services.

21/06/2016WRD02250Prison Accommodation

29 Questions - Written Answers

21/06/2016WRD0230080. Deputy Clare Daly asked the Tánaiste and Minister for Justice and Equality if she will provide the policy document and latest information on the implementation of the cell-share risk assessment tool proposed as part of the Prison Service’s strategic plan; the locations where it is in operation; and if she will make a statement on the matter. [17014/16]

21/06/2016WRD02400Tánaiste and Minister for Justice and Equality (Deputy Frances Fitzgerald): I wish to advise the Deputy that the Director General of the Irish Prison Service accepted in full the recommendations contained in the Report of the Commission of Investigation into the Death of Gary Douche (commonly referred to as the McMorrow Report). As a result a “Cell Sharing Risk Assessment Policy” was drafted and piloted in the Midlands Prison.

As a result of this pilot, the Director General appointed a high-level Group to introduce a standard Risk Assessment Tool for use throughout the prison Estate. This Risk Assessment Tool will include provisions for the assessment of prisoners for suitability for cell-sharing, for moves within individual prisons and other operational decisions. The Assessment Tool is currently being developed with professional advice and assistance from the State Claims Agency, and is very much at an advanced stage.

Upon completion of this developmental stage of the introduction of the Assessment Tool, a decision will then be taken as to whether or not the policy will be published, subject to the usual operational and security considerations.

21/06/2016WRD02450Prison Service

21/06/2016WRD0250081. Deputy Clare Daly asked the Tánaiste and Minister for Justice and Equality the status of the development of a dignity at work charter in the Prison Service, including when she will publish information on it; if she has launched a dignity at work campaign; and if she will make a statement on the matter. [17015/16]

21/06/2016WRD02600Tánaiste and Minister for Justice and Equality (Deputy Frances Fitzgerald): I am ad- vised by the Irish Prison Service that, as part of the IPS Strategic Plan 2012 - 2015, a Dignity at Work steering group was established and five behaviours were identified by staff to ensure a dignified work environment – Respectful, Supportive, Fair and Inclusive, Openness, and Com- petence.

Following from this the Irish Prison Service developed a number of policies including Ac- ceptable Behaviour Standard, Anti Graffiti Policy, Competency Framework, Well-Being at Work Policy, to create the foundations to the Programme.

Support and development of staff to promote these behaviours is a key Strategic Action in the new Irish Prison Service Strategic Plan 2016 – 2018 ‘Creating a Better Environment’ which I will be publishing on Monday 27 June 2016. When developing this strategic action the Irish Prison Service took into account the report by the Office of the Inspector of Prisons Report on Culture and Organisation in the Irish Prison Service: A Roadmap for the Future which exam- ined the current culture within the Irish Prison Service and brought forward suggestions for the future organisational and cultural development of the Irish Prison Service.

As part of the new Strategy, the Irish Prison Service is committed to creating a better work environment for staff that promotes positive engagement, open communication, fairness, equal- ity and respect. This approach is underlined by the Irish Prison Service core organisational values of Teamwork, Integrity, Positivity, Safety and Support.

30 21 June 2016

21/06/2016WRD02650Prison Service Strategies

21/06/2016WRD0270082. Deputy Clare Daly asked the Tánaiste and Minister for Justice and Equality when she will publish the strategies for older prisoners, young prisoners, sex offenders and prisoners requiring protection; when these strategies and the joint strategy for women prisoners will be operational in each prison; and if she will make a statement on the matter. [17016/16]

21/06/2016WRD02800Tánaiste and Minister for Justice and Equality (Deputy Frances Fitzgerald): I can advise the Deputy the Irish Prison Service published its three year strategic plan 2012-2015 on 30 April 2012. Appended to this document were the Irish Prison Service’s strategy for the man- agement of older prisoners, the strategy for the management of young offenders and women offenders, the strategy for reducing re-offending by sex offenders, and the strategy for the man- agement of prisoners requiring protection.

The implementation of the actions contained in the individual strategies is ongoing at prison level.

I can further advice the Deputy that in regard to younger persons, since December 2013 all boys aged 17 years sentenced to detention in St. Patrick’s have been transferred to dedicated units in Wheatfield Place of Detention. The Irish Prison Service continues to engage extensive- ly with the Irish Youth Justice Service regarding the transfer of responsibility for 17 year olds to Oberstown Campus in line with the Government decision to close St. Patrick’s Institution. The Prisons Act 2015 was passed by the Oireachtas in December 2015 and is a significant step toward the closing of St. Patrick’s Institution.

In relation to older persons, having cognisance of the particular needs of older people in custody all prisoners over 55 years of age now have an active nursing care plan, in which their specific needs have been identified. This care planning process assists healthcare staff in iden- tifying patient specific issues and arranging effective through-care on release. A survey of older prisoners has been completed and the results are currently under consideration. A Policy on the management of Older Prisoners is also currently in development.

A joint Irish Prison Psychology Service/Probation Service National Programme of Excel- lence is in operation between the Midlands and Arbour Hill prisons, targeting treatment efforts at moderate to high risk offenders convicted of sexual violence. This is managed by a multi- disciplinary oversight group between both prisons, chaired by the Director of Care and Reha- bilitation.

Regular meetings of the Solitary Confinement Group, chaired by the Director General are held to bring about a reduction in the of prisoners on 23 hour lock up and to introduce measures to reduce the number of prisoners held on restricted regime. Since the introduction of the Irish Prison Service Solitary Confinement Group the number of prisoners on 22/23 hour lock up has decreased by 126 or from 211 to 85 in April 2015. An internal committee has been established to examine the area of protection in a wider context, aiming to reduce the number of prisoners seeking protection and to increase access to regimes.

21/06/2016WRD02850Prison Service Strategies

21/06/2016WRD0290083. Deputy Clare Daly asked the Tánaiste and Minister for Justice and Equality if the women in prison strategy published in March 2014 has been unsuccessful (details supplied); her plans to address the level of overcrowding and the continual increase in the number of women being committed to prison each year; and if she will make a statement on the matter.

31 Questions - Written Answers [17017/16]

21/06/2016WRD03000Tánaiste and Minister for Justice and Equality (Deputy Frances Fitzgerald): The Dep- uty will be aware that the Irish Prison Service must accept all persons committed by the courts and does not have the option of refusing committals.

I wish to advise the Deputy that the joint Probation Service/Irish Prison Service Women’s Strategy 2014 - 2016, “An Effective Response to Women Who Offend” is ongoing and will be reviewed at the end of its term. The Women’s Strategy sets out how the Irish Prison Service and the Probation Service work together, as well as with other statutory, community and voluntary sector partners, to provide women-focused interventions to help reduce offending, improve op- portunities for reintegration and to improve outcomes more generally.

As part of the Joint Strategy, the Irish Prison Service made a commitment to explore the development of an open centre/open conditions for women assessed as low risk of re-offending. This commitment was acknowledged in the Report on the Strategic Review of Penal Policy who also recommended that a greater focus on step down facilities, supported accommodation, use of more community based open conditions for female offenders and the provision of an open centre for women.

A joint Irish Prison Service/Probation Service working group considered an open centre for women and decided to recommend, rather than developing an open centre for women, that a more practical and cost effective way to address the deficit is to pursue step down facilities for women. In this regard, I have given approval to the Heads of the Irish Prison Service and Probation Service to proceed to scope the possibilities for the development of step down units for female offenders and female ex-offenders.

Alternatives to custody continue to be pursued and legislation has already been passed. This includes the Criminal Justice (Community Service) (Amendment) Act 2011 which requires the sentencing judge to consider the imposition of community service where a custodial sentence of 12 months or less is being considered.

Also the Fines (Payment and Recovery) Act 2014 provides that the Court imposing a fine shall take into account a person’s financial circumstances. It further provides, inter alia, that where a person fails to pay a fine by the due date, the Court may make an attachment order to earnings as a means of recovering the unpaid fine. As a result of this legislation, it is expected that we will see a reduction in the number of committals to prisons on short sentences.

To address the female accommodation within the prison estate, planning is well under way for the modernisation and expansion of facilities in Limerick Prison. Part of this redevelopment includes the provision of high quality prison accommodation for female prisoners with a capac- ity of approximately 50 individual cells and 8 transition units which will more than double its current capacity.

21/06/2016WRD03050Domestic Violence Policy

21/06/2016WRD0310084. Deputy John Lahart asked the Tánaiste and Minister for Justice and Equality her plans to provide better protection to victims of domestic violence by ratifying and implementing the Istanbul Convention; and if she will make a statement on the matter. [17036/16]

21/06/2016WRD03200Tánaiste and Minister for Justice and Equality (Deputy Frances Fitzgerald): The pro- tection of members of society from domestic violence, sexual violence and exploitation are primary concerns for me and my Department. The Istanbul Convention is a significant legal 32 21 June 2016 instrument in the fight against domestic and sexual violence. Ireland is fully supportive of the aims and terms of the Convention and the Deputy will be aware that Ireland signed the Conven- tion on 5 November 2015. The recently published Programme for a Partnership Government contains a commitment to implementing the Istanbul Convention. The 18 legislative and ad- ministrative actions which, when implemented, will enable Ireland to ratify the Convention are contained in the Second National Strategy on Domestic, Sexual and Gender-based Violence, which was published in January. A monitoring committee in relation to implementation of the Strategy, which involves the community and voluntary sector, has been established. It met earli- er this month and will shortly commence formal monitoring of implementation of the Strategy.

21/06/2016WRD03250Domestic Violence Policy

21/06/2016WRD0330085. Deputy John Lahart asked the Tánaiste and Minister for Justice and Equality her plans to amend the Domestic Violence Act to extend eligibility for safety orders to all parties who are or have been in an intimate relationship, regardless of cohabitation, and to provide for emer- gency barring orders; and if she will make a statement on the matter. [17037/16]

21/06/2016WRD03400Tánaiste and Minister for Justice and Equality (Deputy Frances Fitzgerald): Many in- cidences of abuse in dating relationships are offences under the Non-Fatal Offences Against the Person Act 1997, including assault, harassment and coercion. The provisions of a statutory defi- nition of dating relationships is problematic. Questions that would arise include issues around when a dating relationship can be said to begin and the behavioural threshold required for a meeting of two people to be classified in such a way. Given the complexities, these relation- ships have not been included in the General Scheme of the Domestic Violence Bill. However I am seeking legal advice on the issue of including dating relationships in domestic violence legislation.

A proposal in relation to emergency barring orders was included in the general scheme of the Domestic Violence Bill, approved by Government and following legal advice an appropri- ate provision will appear in the Bill when it is published.

21/06/2016WRD03450Legislative Programme

21/06/2016WRD0350086. Deputy John Lahart asked the Tánaiste and Minister for Justice and Equality her plans to introduce stalking legislation to cover traditional, online and digital stalking in their direct and indirect forms; and if she will make a statement on the matter. [17038/16]

21/06/2016WRD03600Tánaiste and Minister for Justice and Equality (Deputy Frances Fitzgerald): Section 10 of the Non-Fatal Offences Against the Person Act 1997 provides that any person who, with- out lawful authority or reasonable excuse, by any means including by use of the telephone, harasses another by persistently following, watching, pestering, besetting or communicating with him or her, is guilty of an offence. Harassment is deemed to occur where a person seri- ously interferes with the other’s peace and privacy or causes alarm, distress or harm to the other. Section 10(3) also provides for orders to be made by the court to prevent communication by any means with the victim or to approach within any distance specified by the court of the place of residence or employment of the victim. Such an order can be made even in circumstances where the accused person is not convicted of the offence if the court is satisfied, having regard to the evidence, that it is in the interests of justice.

I would also add that the Law Reform Commission (LRC) is currently carrying out a project on Cybercrime affecting personal safety, privacy and reputation, including cyberbullying. In 33 Questions - Written Answers November 2014 the LRC published an Issues Paper and sought the views of interested parties on the key issues it had identified. In April 2015 it held a seminar to discuss these issues. It is hoped that the LRC Report will be published in the coming months and its recommendations, including any proposals in relation to legislation, will then be considered.

21/06/2016WRD03650Family Reunification Applications

21/06/2016WRD0370087. Deputy Eamon Ryan asked the Tánaiste and Minister for Justice and Equality the sta- tus of an application by a person (details supplied) under the family reunification scheme; and if she will make a statement on the matter. [17076/16]

21/06/2016WRD03800Tánaiste and Minister for Justice and Equality (Deputy Frances Fitzgerald): I am in- formed by the Irish Naturalisation and Immigration Service (INIS) of my Department that the application for the person referred to by the Deputy is currently being processed and that the Family Reunification Unit recently requested more information. On receipt of the information requested the application for family reunification will be finalised.

The Deputy may wish to note that queries in relation to the status of individual immigra- tion cases may be made directly to INIS by e-mail using the Oireachtas Mail facility which has been established specifically for this purpose. This service enables up to date information on such cases to be obtained without the need to seek information by way of the Parliamentary Questions process. The Deputy may consider using the e-mail service except in cases where the response from INIS is, in the Deputy’s view, inadequate or too long awaited.

21/06/2016WRD03850Garda Station Closures

21/06/2016WRD0390088. Deputy Michael Healy-Rae asked the Tánaiste and Minister for Justice and Equality when she will review the Garda stations that were closed under the previous Government; and if she will make a statement on the matter. [17081/16]

21/06/2016WRD04000Tánaiste and Minister for Justice and Equality (Deputy Frances Fitzgerald): The Pro- gramme for a Partnership Government recognises that community policing is the embodiment of An Garda Síochána, providing a means of recognising that every community, both urban and rural, has its own concerns and expectations. It commits the Government to ensuring visible, effective and responsive policing in every community, including the most minimal response times possible.

In support of this objective, the Policing Authority will be asked to oversee a review of, among other things, both the boundaries of Garda districts and the dispersement of Garda sta- tions in rural areas, and in developing urban and suburban areas, with a view to ensuring both an efficient and optimum geographical distribution of stations and minimal response times, including taking account of station closures since 2012.

Under the Programme, the Government is committed to launching a pilot scheme to reopen 6 Garda stations, both urban and rural, to determine possible positive impacts that such open- ings will have on criminal activity, with special emphasis on burglaries, theft and public order.

I will shortly be in contact with the relevant authorities in the relation to these matters.

21/06/2016WRD04050Public Sector Pay

34 21 June 2016

21/06/2016WRD0410089. Deputy Bríd Smith asked the Tánaiste and Minister for Justice and Equality the amount saved by having a lower level of pay for new recruits in her Department; and if she will make a statement on the matter. [13012/16]

21/06/2016WRD04200Tánaiste and Minister for Justice and Equality (Deputy Frances Fitzgerald): The Dep- uty will be aware that in 2011 the Government introduced revised rates of pay for new entrant public servants in order to reduce the Public Service Pay and Pensions Bill. This was one of a range of measures introduced over the period of the fiscal crisis which directly affected the pay of public servants.

Since 2011, it is estimated that approximately €380,000 was saved by my Department as a result of the introduction of lower levels of pay for some recruitment grades. This figure applies to civil service grades in my Department and does not include others agencies under the aegis of my Department, such as An Garda Síochána.

The Deputy will be aware that the Government is currently in the process of establishing a Public Service Pay Commission to examine pay levels across the public service. The precise structure of such a commission and the technical aspects as to how it would operate have yet to be decided upon by Government and will require broad consultation, including engagement with staff representatives as committed to in the Lansdowne Road Agreement.

21/06/2016WRD04250Commissions of Investigation

21/06/2016WRD0430090. Deputy Micheál Martin asked the Tánaiste and Minister for Justice and Equality when she received the O’Higgin’s report from the commission of investigation into certain matters relative to the Cavan and Monaghan division of the Garda Síochána; and if she will make a statement on the matter. [11431/16]

21/06/2016WRD04400Tánaiste and Minister for Justice and Equality (Deputy Frances Fitzgerald): I received the report of the O’Higgins Commission of Investigation on 25 April 2016. I published the report on 11 May 2016.

As the Deputy is aware, the report was the subject of a debate in the House on 25 and 26 May 2016. In the course of my contribution I outlined the reasons why publication was not possible before the date on which I did publish it.

As I have already informed the House, the Policing Authority is overseeing the Garda re- sponse to the O’Higgins Report. You will be aware of the framework which the Authority has put in place in this regard, including two public meetings. Matters relating to the treatment of victims, protected disclosures and culture were addressed at the meeting of 13 June. The meeting scheduled for 30 June is intended to focus in particular on governance, performance management and supervision issues.

In addition, I have asked the Garda Commissioner to examine the report and to indicate to me what further measures might be taken to try to prevent the type of difficulties outlined in it in relation to An Garda Síochána arising again. I am also seeking her proposals concerning the recommendations which it contains in relation to the Garda service. I am committed to overseeing further reforms which are necessary to avoid a recurrence of the type of incidents highlighted in the report.

I very much appreciate that the events outlined in the report have been traumatic for many people who have been affected by them. It would be an injustice to those who brought events to light in the public interest and those who have lived under the shadow of these events for a 35 Questions - Written Answers long time, if we do not take on board the lessons from these events. I hope they can take some reassurance from the fact that the examination of those events in this report will help serve to consolidate a programme of reform which will ensure we continue to have a Garda Síochána in which its members and the community it serves can take great pride.

21/06/2016WRE00200Mortgage Debt

21/06/2016WRE0030091. Deputy Michael McGrath asked the Tánaiste and Minister for Justice and Equality the status of interest applying to a judgment mortgage; if a judgment mortgage lapses after a period of time; the steps the creditor may take to enforce the asset, the subject to the judgment mortgage; the rights of the borrower; and if she will make a statement on the matter. [17159/16]

21/06/2016WRE00400Tánaiste and Minister for Justice and Equality (Deputy Frances Fitzgerald): The posi- tion is that the Land and Conveyancing Law Reform Act 2009 updated the law in relation to judgment mortgages registered after 1 December 2009 by repealing the Judgment Mortgage (Ireland) Acts 1850 and 1858 and replacing them with updated provisions as recommended by the Law Reform Commission in 2004.

Section 116 of the 2009 Act provides that a creditor who has obtained a judgment against a person may apply to the Property Registration Authority (PRA) to register a judgment mortgage against that person’s estate or interest in land in the Land Registry or Registry of Deeds as ap- propriate. Registration of the judgment mortgage by the PRA operates to charge the debtor’s estate or interest in the land and entitles the judgment mortgagee (i.e. creditor) to apply to court for an order under section 117 of the Act, or section 31 in the case of co-owned property. A judgment mortgage remains valid until discharged and there is no need to re-register it in order to maintain its validity or enforceability.

In the case of an application under section 117, the court may make such order for enforce- ment of the judgment mortgage as the court thinks appropriate, including an order for sale of the land and distribution of the proceeds of sale. In the case of co-owned property, section 30 of the 2009 Act provides that the registration of a judgment mortgage against the property of a joint tenant no longer severs the joint tenancy and if the joint tenancy remains unsevered, the judg- ment mortgage is extinguished on the death of the judgment debtor, i.e. a surviving joint tenant has no liability for debts of the deceased joint tenant against whose interest in the property the judgment mortgage was registered.

A judgment mortgagee (i.e. creditor) may apply to court for an order under section 31 to enforce a judgment mortgage in the case of co-owned land. In dealing with such an application, the court may dismiss the application or, in light of the rights of all co-owners, make such order relating to the land as appears to the court to be just and equitable in the circumstances of the particular case. The wider discretion available to the court in the case of co-owned land is in- tended to ensure that the interests of all co-owners are taken into account in a fair and equitable manner. It should also be noted that subsection 5 of Section 31 makes it clear that this section does not affect the court’s jurisdiction under the Family Home Protection Act 1976.

As regards interest rates applicable in the case of judgment mortgages, I will communicate directly with the Deputy on receipt of this information.

21/06/2016WRE00500Parole Boards

21/06/2016WRE0060092. Deputy Jonathan O’Brien asked the Tánaiste and Minister for Justice and Equality the 36 21 June 2016 names of members of the Parole Board since 2011. [17175/16]

21/06/2016WRE00700Tánaiste and Minister for Justice and Equality (Deputy Frances Fitzgerald): Details of the membership of the Parole Board is set out each year in the Board’s Annual Report which is published on my Department’s website www.justice.ie.

Details of members of the Parole Board since 1 January, 2011 are set out in a table.

Current Members

Name Date of Appointment / Reappoint- Date of expiry of term ment Mr. John Costello (Chairman) 26th July 2014 25th July 2017 Ms. Ciarín de Buis 26th July 2014 25th July 2017 Mr. Vivian Geiran (Probation Aug-12 Ex-Officio: Termination of Cur- Service) rent Position Mr. Martin Smyth (IPS) Jan-15 Ex-Officio: Termination of Cur- rent Position Mr. Noel Dowling (Prisons Aug-15 Ex-Officio: Termination of Cur- Policy) rent Position Ms Nora McGarry 17-Jul-15 16-Jul-18 Mr. Shane McCarthy 17-Jul-15 16-Jul-18 Mr. Eddie Rock 17-Jul-15 16-Jul-18 Dr. Michael Mulcahy 11-Feb-14 10-Feb-17 Mr. William Connolly * 17-Jul-15 16-Jul-18 Mr Ray Kavanagh 01-Feb-13 31-Jan-16 *Willie Connolly retired as Director of Operations, Irish prison Service in February 2012 and was subsequently re-appointed in July 2012

Former Members who were in post on 1 January, 2011 or who were appointed since then

Name Date of latest Appointment / Reap- Date of expiry of term pointment Dr. Gordon Holmes R.I.P. 05/09/2005 Chairman (to January 2011) Mr. Michael Donnellan (Proba- Probation Service (to November tion) 2011) Mr. Pat Crummey R.I.P. 01/07/2009 to April 2012 Mr. Frank McCarthy 01/07/2009 to June 2012 Mr Tim O’Donoghue 01/07/2009 to June 2012 Mr. Brendan Waters 20th October 2010 19th October 2013 Mr. Eamonn Nolan 01-Dec-10 30th November 2013 Mr. Brian Murphy (IPS) - Ex- Aug-12 Sep-14 Officio Mr. Gerry McDonagh (Prison Oct-10 Jun-15 Policy)- Ex Officio Mr Mick Duff 01-Feb-13 31-Jan-16

21/06/2016WRE00800Equality Legislation

21/06/2016WRE0090093. Deputy Mattie McGrath asked the Tánaiste and Minister for Justice and Equality the prohibitions in place to prevent public officials or public authorities from imposing certain requirements contrary to the religious beliefs or moral convictions of employees in the public 37 Questions - Written Answers service; and if she will make a statement on the matter. [17223/16]

21/06/2016WRE01000Tánaiste and Minister for Justice and Equality (Deputy Frances Fitzgerald): Equality legislation prohibits discrimination in employment and in the provision of goods and services on the basis of nine grounds, including that of religion. Public servants are obliged to operate within the law regardless of their own religious beliefs or moral convictions and may not dis- criminate on grounds prohibited in the legislation. There is no exemption which enables a pub- lic servant to avoid the fulfilment of the obligations of their office on the grounds of religious or other beliefs. I should make the point, however, that I have no Ministerial responsibility in relation to either the codes of governance of public sector organisations or personnel manage- ment issues in the public sector.

21/06/2016WRE01100Assisted Suicide

21/06/2016WRE0120094. Deputy Mattie McGrath asked the Tánaiste and Minister for Justice and Equality the status of assisted suicide and euthanasia; her plans for changes; and if she will make a statement on the matter. [17225/16]

21/06/2016WRE01300Tánaiste and Minister for Justice and Equality (Deputy Frances Fitzgerald): It is an offence, under section 2(2) of the Criminal Law (Suicide) Act 1993, to assist another person in taking his or her life. The term euthanasia generally implies the intentional termination of a life by another at the request of a person wishing to die and could, therefore, be considered a form of assisted suicide. The 1993 Act provides for a maximum penalty of 14 years imprisonment on conviction on indictment for assisted suicide. The purpose of this legislation is to protect people who are nearing the end of their lives and might be vulnerable or at risk of abuse. Pros- ecutions under section 2(2) are at the discretion of the Director of Public Prosecutions.

The Supreme Court upheld the constitutionality of section 2(2) in its judgment in Fleming v Ireland and Others on 29 April 2013. The Government currently has no plans to bring forward legislative proposals in this area.

21/06/2016WRE01400Garda Resources

21/06/2016WRE0150095. Deputy Michael Harty asked the Tánaiste and Minister for Justice and Equality her plans to increase the resources of An Garda Síochána in County Clare and to end the morato- rium on recruitment of clerical staff; and if she will make a statement on the matter. [17229/16]

21/06/2016WRE01600Tánaiste and Minister for Justice and Equality (Deputy Frances Fitzgerald): As the Deputy will appreciate, the Garda Commissioner is responsible for the distribution of person- nel, including civilians, among the Garda Regions, Divisions, and Districts. I am assured by the Garda Commissioner that personnel assignments throughout the country, together with overall policing arrangements and operational strategy, are continually monitored and reviewed and are determined by a number of factors including population, crime trends and the policing needs of each individual Division and Unit. Such monitoring ensures that optimum use is made of Garda resources, and the best possible Garda service is provided to the general public.

I have been informed by the Garda Commissioner that as of the 30 April 2016, the latest date for which figures are readily available, there were 290 Gardaí, 8 Garda Reserves and 30 civilian staff assigned to the Clare Garda Division.

As the Deputy will be aware, when the financial crisis hit, the Government of the time in-

38 21 June 2016 troduced a moratorium on recruitment and the four year National Recovery Plan, published in 2010, envisaged a steady reduction in Garda numbers. Thankfully, in a recovering economy, we were able to reopen the Garda College in September 2014, and a total of 700 Garda trainees have been recruited with a further 450 planned to be recruited during the remainder of this year. So far 463 of the new Garda trainees have attested as members of An Garda Síochána and have been assigned to mainstream uniform duties nationwide. Another 76 will attest on the 7 July with 150 more to attest in November. I am assured by the Commissioner that the needs of all Garda Divisions are fully considered when determining the allocation of newly attested Gardaí and that 13 newly attested Gardaí have been assigned to the Clare Garda Division to date.

This Government is committed to ensuring a strong and visible police presence throughout the country in order to maintain and strengthen community engagement, provide reassurance to citizens and deter crime. Key to achieving this goal is the commitment in the Programme for Government, “A Programme for a Partnership Government” to continue the ongoing ac- celerated Garda recruitment programme with a view to increasing Garda numbers to 15,000, as well as doubling the Garda Reserve and increasing the number of civilian staff so as to free-up Gardaí for front-line policing.

Taking account of projected retirements, the current rate of recruitment will bring Garda numbers to around the 13,000 mark this year. We must, I believe, endeavour to make more rapid progress than this to reach our target of 15,000 and I am engaging with my colleague, the Minister for Public Expenditure and Reform, in relation to increasing the planned annual intake this year and in coming years. My officials are also engaging with Garda management as a matter of priority in relation to the preparation of a recruitment plan for the next five years that will deliver increased numbers of Garda without compromising the quality of the training and supervision of new recruits.

In relation to civilian staff there is no moratorium per se on recruitment. An Garda Síochána is subject to an employment control framework which sets out the maximum number of civilian staff it may have. It is, however, open to the Commissioner where a need is identified, to seek sanction for the recruitment of additional civilians and I understand that Garda management is currently engaged in a workforce planning exercise to identify critical gaps in its organisation with a view to seeking such sanction. In addition, as mentioned above, the Programme for Gov- ernment includes a commitment to further increase the number of civilians in order to facilitate the redeployment of Gardaí to front-line policing duties. My officials are also engaged with Garda management in relation to the development of a plan to deliver this commitment over the life time of the Government.

21/06/2016WRE01700Home Repossessions

21/06/2016WRE0180096. Deputy John Lahart asked the Tánaiste and Minister for Justice and Equality if she will urgently legislate for a moratorium on family home evictions until such time as a reasoned solution to the family housing crisis is formulated by the Houses of the Oireachtas; and if she will make a statement on the matter. [17238/16]

21/06/2016WRE01900Tánaiste and Minister for Justice and Equality (Deputy Frances Fitzgerald): The posi- tion is that the Oireachtas Committee on Housing and Homelessness has recommended in its recently-published report that, subject to the advices of the Attorney General, the Government should introduce legislation for a moratorium on home repossessions until such time as the Government’s proposals in relation to housing and homelessness are in place. I can confirm that this recommendation, together with the Committee’s other recommendations, will be con- sidered by the Government in the context of the formulation of an action plan to implement the 39 Questions - Written Answers Programme for Government commitments in relation to mortgage arrears.

I wish also to take this opportunity to draw the Deputy’s attention to the reform that I intro- duced in the Personal Insolvency (Amendment) Act 2015 in order to further assist insolvent ho- meowners in mortgage arrears. The Act has introduced a new ‘court review’ process applicable in cases where creditors such as a mortgage lender refuse a proposal for a Personal Insolvency Arrangement drawn up to resolve the borrower’s debts, including the mortgage on their home. Previously, the creditors’ rejection was final; under the new provision, the borrower can seek review by the court of the creditors’ refusal. The court will then consider the borrower’s pro- posal using the criteria set out in the amending legislation and may, if those criteria are satisfied, impose the Personal Insolvency Arrangement proposed by the borrower. The review provision came into effect on 20 November 2015 [SI No. 514/2015]. To coincide with its introduction, the Insolvency Service of Ireland (ISI) held a number of training sessions around the country with Personal Insolvency Practitioners (PIPs).

All applications for review under the amending legislation are notified to the ISI. It has ad- vised that 51 review applications have been notified to it as of 17 June, 4 of which have been approved and 2 of which have been rejected. A further 5 applications have been withdrawn and 5 struck out by consent, arising, it seems, from settlement negotiations between the parties. In a further 27 cases an initial hearing has been held but new affidavits or additional material has been requested and the cases have been adjourned to allow for filing of this material in advance of a full hearing. My officials will, of course, be carefully monitoring the operation of this important reform.

The Court review process is being complemented by a new scheme which provides access to independent legal and financial aid and advice for those in home mortgage arrears which I announced earlier this year. Intensive work on implementing this Aid and Advice Scheme has been under way across Government Departments and other relevant bodies. This work has been led by my Department and by the Department of Social Protection, with the participation of the ISI, the Legal Aid Board, the Citizens’ Information Board and the Money and Budgeting Service (MABS). The Scheme builds on a network of Dedicated Mortgage Arrears Advisers established in MABS offices across the country which was announced in December 2015.

MABS will act as a single gateway to the new Scheme for those who are deep in debt and at risk of losing their homes due to mortgage arrears. MABS advisers will advise these borrowers on options for returning to solvency and remaining in their homes as far as possible. This may involve obtaining financial advice from a PIP or accountant and legal aid and advice from a solicitor. The Scheme will also provide for support for insolvent borrowers in repossession pro- ceedings under a new ‘duty solicitor’ service operated by the Legal Aid Board. Work on imple- mentation is almost complete and I expect the Scheme to become operational in the near future.

21/06/2016WRE02000Garda Training

21/06/2016WRE0210097. Deputy John Lahart asked the Tánaiste and Minister for Justice and Equality what the training programme entails for a recruit to An Garda Síochána and if a group of trainee gardaí has ever been allowed to skip certain segments of the programme in order to quickly allocate gardaí to stations; and if she will make a statement on the matter. [17263/16]

21/06/2016WRE02200Tánaiste and Minister for Justice and Equality (Deputy Frances Fitzgerald): I am in- formed by the Garda Commissioner that all Gardaí recruited since the reopening of the Garda College in September 2014 undertake the new 2 year training programme which leads to a Bachelor of Arts in Applied Policing and is accredited by the University of Limerick. The pro- 40 21 June 2016 gramme is the outcome of a comprehensive review of foundation training for entrants to An Garda Síochána and exemplifies best practice.

Phase 1, which lasts 32 weeks, is based in the Garda College and places a strong emphasis on problem-based learning where students learn in small groups through engagement with re- alistic policing scenarios. This methodology emphasises the development of ‘learning to learn’ skills, the development of reflective practice and supports the transition into learning inan operational policing environment. At the end of Phase1 successful Garda trainees are attested (i.e. become members of the Garda Síochána with full police powers) and are dispersed to Garda stations throughout the country, where they are assigned to mainstream policing duties.

Phase 2, which lasts for 65 weeks, is primarily based in Garda stations with appropriate training and development structures in place that include access to a trained tutor Garda and a permanently appointed supervisory sergeant who is thoroughly familiar with their responsibili- ties under the training programme. During the course of their placement, trainees move through three development stages from assisting his or her Garda tutor to taking the lead role and being assisted by the tutor, to finally the autonomous stage where they are deployed in regular polic- ing activity and work independently within the operational unit. Over the course of the place- ment they also return to the Garda College for a number to weeks to further enhance their skills in specialist areas including sexual assault, intelligence –led policing, file preparation and court presentation and so on.

Phase 3 consists of seven weeks of preparation for final exams and assessments and I look forward to the first batch of new recruits graduating from the programme this year. It is, of course, too early to conduct a formal evaluation of the new programme but I understand from those leading the programme that the feedback from managers and colleagues on the ground is positive.

Participants are required to sit mandatory academic examinations and professional assess- ments and are required to pass all examinations and assessments in order to progress to the conclusion of the programme.

21/06/2016WRE02300Garda Transport Data

21/06/2016WRE0240098. Deputy Mary Butler asked the Tánaiste and Minister for Justice and Equality the number of new vehicles supplied to Garda Síochána stations in the Waterford, Dungarvan and Tramore divisional districts of County Waterford in the past 24 months; and if she will make a statement on the matter. [17286/16]

21/06/2016WRE02500Tánaiste and Minister for Justice and Equality (Deputy Frances Fitzgerald): As the Deputy will appreciate, decisions in relation to the provision and allocation of Garda vehicles are a matter for the Garda Commissioner in the light of her identified operational demands and the availability of resources and I, as Minister, have no direct role in the matter.

The tables supplied provide an overview in relation to the number of Garda vehicles as- signed to the Waterford Division as of 17 June, which is the latest date for which such infor- mation is available, and the number of new vehicles assigned to that Division since 2014. I am informed by the Garda authorities that the allocation of Garda vehicles is monitored and reviewed on a continual basis and vehicles are allocated between districts as required by opera- tional circumstances.

The Deputy will be aware that, in recent times, the Government has made considerable funding available to provide An Garda Síochána with additional high-powered vehicles, patrol 41 Questions - Written Answers cars (marked and unmarked) and motorcycles to ensure that Gardaí can be mobile, visible and responsive on the roads and in the community.

In summary, over €34 million has been invested in the fleet since 2012, with over 720 new vehicles coming on stream since the start of 2015, and a further €46 million is being provided under the Government’s Capital Plan 2016 - 2021.

Waterford Division Fleet Strength, as of 17 June 2016

CARS VANS Motor- 4X4 OTHER Total cycles DISTRICT MARKED UN- MARKED DUNGAR- 10 3 2 0 0 0 15 VAN TRA- 5 3 1 1 0 0 10 MORE WATER- 10 11 7 3 3 1 35 FORD

TOTAL 25 17 10 4 3 1 60

Vehicles allocated to the Waterford Division Fleet from 1 January 2014 to 17 June 2016

CARS VANS Motor- 4X4 OTHER Total cycles DISTRICT MARKED UN- MARKED DUNGAR- 6 1 0 0 0 0 7 VAN TRA- 4 1 0 1 0 0 6 MORE WATER- 4 5 3 1 2 1 16 FORD

TOTAL 14 7 3 2 2 1 29

21/06/2016WRE02600Community Policing

21/06/2016WRE0270099. Deputy Mary Butler asked the Tánaiste and Minister for Justice and Equality the number of members of An Garda Síochána who are community gardaí in each of the districts in Waterford city and county; and if she will make a statement on the matter. [17287/16]

21/06/2016WRE02800Tánaiste and Minister for Justice and Equality (Deputy Frances Fitzgerald): As the Deputy will appreciate, it is the Garda Commissioner who is responsible for the distribution of resources, including Community Gardaí, among the various Garda Divisions and I, as Minister, have no direct role in the matter. I am assured by the Garda Commissioner that the allocation of Gardaí is continually monitored and reviewed taking into account all relevant factors including crime trends, demographics, and security assessments relating to the area in question so as to ensure optimal use is made of Garda human resources.

I have been informed by the Garda Commissioner that, as of the 30 April 2016, the latest date for which figures are readily available, there were 282 members assigned to the Water- ford Garda Division. Of these, there were 44 dedicated Community Gardaí, 13 in Dungarvan 42 21 June 2016 District, 12 in Tramore District and 19 in Waterford Districts. Of course, it is the case that all Gardaí have a role to play in addressing community policing issues as and when the need arises. In that sense, community policing involves far more than a single unit within An Garda Síochána, a point highlighted by the Garda Inspectorate in their third report entitled “Policing in Ireland - Looking Forward” in which they stated that community policing is a fundamental policing philosophy and that there is a strong foundation for it in Ireland.

The Programme for a Partnership Government recognises that community policing is the embodiment of An Garda Síochána, providing a means of recognising that every community – both urban and rural – has its own concerns and expectations. It commits the Government to ensuring a strong and visible police presence throughout the country in order to maintain and strengthen community engagement, provide reassurance to citizens and deter crime. Key to achieving this goal is the commitment in the Programme to continue the ongoing accelerated Garda recruitment programme with a view to increasing Garda numbers to 15,000 as well as doubling the Garda Reserve and increasing the number of civilian staff so as to free-up Gardaí for front-line policing.

The Garda Commissioner’s Modernisation and Renewal Programme 2016-2021, published on 9 June 2016, places a strong emphasis on developing and supporting the community polic- ing ethos of the organisation and enhancing the current delivery model so that Gardaí spend more time in the community, gaining public confidence and trust and providing a greater sense of security. I welcome in particular the proposal to establish local Community Policing Teams (CPTs) headed by an Inspector and made up of Gardaí from across a range of areas to work with the community to prevent and detect crime. The CPTs will take proactive measures to prevent crime from happening in the area. This will entail a combination of crime prevention advice, targeting and disrupting criminals through operations and patrols, and where possible diverting people from committing crime. I also welcome the proposal to establish Community Safety Fora in every District comprising local Gardaí, local communities and key stakeholders. The implementation of these initiatives has started and will be rolled out nationwide.

21/06/2016WRE02900Anti-Social Behaviour

21/06/2016WRE03000100. Deputy Louise O’Reilly asked the Tánaiste and Minister for Justice and Equality if she is aware of the high level of anti-social behaviour in an area (details supplied) in Dublin 15 including but not limited to drug dealing, joyriding, intimidation of residents and so on; the actions she is taking to combat this; and if she will make a statement on the matter. [17290/16]

21/06/2016WRE03100Tánaiste and Minister for Justice and Equality (Deputy Frances Fitzgerald): I am, of course, conscious of the impact which anti-social behaviour and the types of crime referred to have on the quality of life for residents in local communities. An Garda Síochána continues to tackle public disorder and anti-social behaviour by working with communities to reduce this type of behaviour and enhance community safety. The Garda approach includes a strong focus on quality of life issues and collaboration with local authorities to help address the causes of anti-social behaviour. The Deputy will appreciate that I do not have a role in relation to Garda policing strategies in any particular local area but have brought Deputy’s specific concerns to the attention of the Garda authorities.

Anyone affected by the matters referred to should, of course, contact their local Garda sta- tion. Alternatively, information can be provided to An Garda Síochána by way of the Garda confidential line, 1800 666 111, which is a monitored freephone line that allows members of the public to contact An Garda Síochána with information of a confidential nature.

43 Questions - Written Answers I might add that there is a range of strong legislative provisions available to an An Gar- da Síochána to combat anti-social behaviour, including provisions under the Criminal Justice (Public Order) Acts, the Criminal Damage Act and the Intoxicating Liquor Acts. There are also the incremental provisions contained in the Criminal Justice Act 2006 which provide for warn- ings and civil proceedings in relation to anti-social behaviour by adults and Part 13 of the Act which provide for warnings, good behaviour contracts and civil proceedings in relation to anti- social behaviour by children.

The Programme for Government underlines the need for close engagement between An Garda Síochána and local communities as part of the strong community policing ethos which has long been central to policing in this jurisdiction. The Deputy will appreciate that the spe- cific allocation of personnel and other resources is a matter for the Garda Commissioner, but I am assured that the Garda National Model of Community Policing continues to play a key part in responding to crime by taking into account and responding to local conditions and needs. This commitment is also very evident in the new Garda Síochána Modernisation and Renewal Programme, which was recently launched by the Commissioner, and as part of which new Community Policing Teams are to be introduced in each Garda District.

Local policing measures to address the type of issues referred to by the Deputy will un- doubtedly benefit from the resources now coming on stream through the Garda recruitment programme, and in particular the Government’s commitment to increase Garda numbers to 15,000. We are working on measures to accelerate this programme so as to ensure that the Commissioner has the capacity to address the needs of communities throughout the country and into the future.

21/06/2016WRE03200Immigration Status

21/06/2016WRE03300101. Deputy Jim O’Callaghan asked the Tánaiste and Minister for Justice and Equality her proposals to introduce an administrative scheme for migrants who wish to regularise their undocumented status; and if she will make a statement on the matter. [17330/16]

21/06/2016WRE03400103. Deputy Jim O’Callaghan asked the Tánaiste and Minister for Justice and Equality the time the Irish Naturalisation and Immigration Service requires to administer applications from migrants seeking to regularise their status; and if she will make a statement on the matter. [17332/16]

21/06/2016WRE03500Tánaiste and Minister for Justice and Equality (Deputy Frances Fitzgerald): I propose to take Questions Nos. 101 and 103 together.

I have no plans to introduce a general regularisation scheme for those who are currently un- documented in the State as a proposal of this nature could give rise to very large, unpredictable and potentially very costly impacts across the full range of public and social services. Also any possible implication for the operation of the Common Travel Area would also have to be very carefully considered.

At EU Level, the Member States, in agreeing the European Pact on Immigration and Asy- lum at the European Council in October 2008 made specific commitments “to use only case-by- case regularisation, rather than generalised regularisation, under national law, for humanitarian or economic reasons”. While the Pact is not legally binding, the political commitment among Member States, then and now, is clearly against any form of process that would in any way legitimise the status of those unlawfully present without first examining the merits of their in- dividual cases. It is open to persons in an undocumented situation to present their case on its

44 21 June 2016 merits, to have it considered and to abide by the decision.

We are entitled to expect that people coming here will obey our laws, including those that relate to immigration. It is open to any foreign national who finds him or herself in an un- documented situation to apply to the authorities for permission to remain. Cases are carefully considered before a decision is made and it is reasonable for the State to expect that people will respect that decision. It should also be remembered that most people become undocumented through their own conscious actions or omissions. Recognising that in a minority of instances that may not be the case, the Department has operated a scheme - for those who already held an employment permit - to address the situation of those who became undocumented through no fault of their own. All cases are dealt with on an individual basis and if the Deputy has a particular case in mind he should contact the officials in my Department with the details.

Ireland has shown itself to be a country that is open to migration. In that regard, it should be noted that over the past 5 years nearly 100,000 persons have been naturalised.

21/06/2016WRE03600Refugee Data

21/06/2016WRE03700102. Deputy Jim O’Callaghan asked the Tánaiste and Minister for Justice and Equality the backlog in processing applications by migrants seeking humanitarian leave to remain; and if she will make a statement on the matter. [17331/16]

21/06/2016WRE03800Tánaiste and Minister for Justice and Equality (Deputy Frances Fitzgerald): I am ad- vised by the Irish Naturalisation and Immigration Service (INIS) of my Department that the statistics are being compiled and will be provided to the Deputy in due course.

Question No. 103 answered with Question No. 101.

21/06/2016WRE04000Migrant Integration

21/06/2016WRE04100104. Deputy Jim O’Callaghan asked the Tánaiste and Minister for Justice and Equality if the office for the promotion of migrant integration within her Department still operates; the work it has carried out in the past two years; and if she will make a statement on the matter. [17333/16]

21/06/2016WRE04200Minister of State at the Department of Justice and Equality (Deputy David Stanton): The Office for the Promotion of Migrant Integration (OPMI) in my Department continues to have responsibility for promoting the integration of migrants into Irish society. Details of its activities are to be found on its website www.integration.ie.

OPMI chairs a Cross-Departmental Group which is tasked with preparing a Draft Migrant Integration Strategy, having engaged in a public consultation process which involved key stake- holders including relevant NGOs. This work is very advanced and I expect the strategy to be published in the Autumn.

In addition, Ireland is one of 27 States that participate in a refugee resettlement programme led by the United Nations Commissioner for Refugees (UNCHR). OPMI is the Irish Govern- ment Office which manages this process. In recent years, the focus of the work of this Office has been on the resettlement of persons displaced by the Syrian conflict who are living in one of the hosting countries in the region, for example those living in Lebanon, Jordan or Turkey. In 2015, the Office resettled 176 refugees from Jordan and Lebanon. To date in 2016, 110 refu- gees have been resettled from Lebanon. A further 247 refugees were selected during a selection 45 Questions - Written Answers mission to Lebanon in January 2016. These cases have now been security screened and are in the process of being health screened at present. They are expected to arrive in Ireland between July and September 2016. Staff of OPMI have participated in selection missions to Lebanon in March and July 2015 and again in January 2016 to facilitate this process.

OPMI also oversees national and European funding for the promotion of integration. In 2014, 2015 and from January to May 2016, around €3.1 million in grants has been paid, includ- ing grants for resettlement and an European Social Fund measure to improve the employment prospects of immigrants. Preparations have been made for a call for applications under this measure which will take place in early September and for a call in respect of the EU Asylum, Migration and Integration Fund which will take place at the same time.

OPMI also participates in EU and international fora relevant to its responsibilities including the EU National Contact points on Integration (now called the European Migration Forum) and the Forum on Resettlement and Relocation.

21/06/2016WRE04300Migrant Integration

21/06/2016WRE04400105. Deputy Jim O’Callaghan asked the Tánaiste and Minister for Justice and Equality if she proposes to introduce an integration strategy for migrants; and if she will make a statement on the matter. [17334/16]

21/06/2016WRE04500Minister of State at the Department of Justice and Equality (Deputy David Stanton): The Office for the Promotion of Migrant Integration, an Office operating under the ambit of my Department, has a cross-Departmental mandate to develop, lead and coordinate migrant integration policy across other Government Departments, agencies and services. This Office is currently engaged in a review of the approach to the integration of migrants with a view to the preparation and publication of a new updated Migrant Integration Strategy.

The work undertaken by the Office thus far has included, inter alia, an extensive public con- sultation process including the involvement of key stakeholders. The work is very advanced and I expect that the Strategy will be published in the Autumn.

21/06/2016WRE04600Visa Applications

21/06/2016WRE04700106. Deputy Paul Murphy asked the Tánaiste and Minister for Justice and Equality if she will consider to the exceptional humanitarian circumstances and take measures to ensure the safe passage of persons (details supplied) to Ireland; and if she will make a statement on the matter. [17102/16]

21/06/2016WRE04800Tánaiste and Minister for Justice and Equality (Deputy Frances Fitzgerald): I am ad- vised by the Irish Naturalisation and Immigration Service (INIS) of my Department that the persons referred to are of a nationality required to hold a valid visa in order to travel to Ireland.

I am also advised that an official from INIS was in touch with their family member to advise on the procedure for making visa applications. However, it appears that the persons involved do not currently hold valid travel documents and it is therefore not possible to process visa ap- plications until such time as they do.

The Deputy may wish to note that queries in relation to the status of individual immigration cases may be made directly to INIS by e-mail using the Oireachtas Mail facility which has been specifically established for this purpose. This service enables up to date information on such 46 21 June 2016 cases to be obtained without the need to seek information by way of the Parliamentary Ques- tions process. The Deputy may consider using the e-mail service except in cases where the response from INIS is, in the Deputy’s view, inadequate or too long awaited.

In addition, applicants may themselves e-mail queries directly to the INIS Visa Office (visa- [email protected]).

21/06/2016WRE04900Motor Insurance Coverage

21/06/2016WRE05000107. Deputy James Browne asked the Minister for Finance if it is permissible for an insur- ance company not to insure a person (details supplied), given that each of the persons have a card approved by a general practitioner, GP, under the medical card scheme; and if he will make a statement on the matter. [16667/16]

21/06/2016WRE05100Minister for Finance (Deputy Michael Noonan): I am advised that the matter raised by the Deputy in this Parliamentary Question relates to motor insurance.

As Minister for Finance, I am responsible for the development of the legal framework gov- erning financial regulation. Neither I, nor the Central Bank of Ireland, may interfere in the pro- vision or pricing of insurance products. The EU framework for insurance expressly prohibits Member States from adopting rules which require insurance companies to obtain prior approval of the pricing, or terms and conditions of an insurance product.

The provision of insurance cover and the price at which it is offered is a commercial mat- ter for insurance companies and is based on an assessment of the risks they are willing to ac- cept and adequate provisioning to meet those risks. These are considered by insurance compa- nies on a case by case basis.

Insurance Ireland, which represents the insurance industry in Ireland, has informed me that motor insurers use a combination of rating factors in making their individual decisions on whether to offer cover and what terms to apply. Factors include those such as the age of the driver, the type of car, the claims record, driving experience, the number of drivers, how the car is used, etc.

Insurance Ireland has also informed me that if an individual has a valid driving licence and has medical confirmation that any ailment or sickness suffered does not render them unfit to drive, then their insurer is obliged to offer them an insurance quote. Also, the legislative frame- work does not impose individual age limits on motor insurance.

In the event that the person in question is unable to obtain a quotation for motor insurance or feels that the premium proposed or the terms are so excessive that it amounts to a refusal to give them motor insurance, they should contact Insurance Ireland, 5 Harbourmaster Place, IFSC, Dublin 1, Telephone +353 1 6761820 quoting the Declined Cases Agreement.

21/06/2016WRE05200Motor Insurance Coverage

21/06/2016WRE05300108. Deputy James Browne asked the Minister for Finance why insurance companies will not cover drivers due to their age; if it is permissible for insurance companies to have an upper age requirement; and if he will make a statement on the matter. [16669/16]

21/06/2016WRE05400Minister for Finance (Deputy Michael Noonan): As Minister for Finance, I am respon- sible for the development of the legal framework governing financial regulation. Neither I, nor 47 Questions - Written Answers the Central Bank of Ireland, may interfere in the provision or pricing of insurance products. The EU framework for insurance expressly prohibits Member States from adopting rules which require insurance companies to obtain prior approval of the pricing, or terms and conditions of an insurance product.

The provision of insurance cover and the price at which it is offered is a commercial mat- ter for insurance companies and is based on an assessment of the risks they are willing to ac- cept and adequate provisioning to meet those risks. These are considered by insurance compa- nies on a case by case basis.

There are no individual age limits on motor insurance. Insurance Ireland, which represents the insurance industry in Ireland, has informed me that motor insurers use a combination of rating factors in making their individual decisions on whether to offer cover and what terms to apply. Factors include those such as the age of the driver, the type of car, the claims record, driving experience, the number of drivers, how the car is used, etc.

In the event that a person is unable to obtain a quotation for motor insurance or feels that the premium proposed or the terms are so excessive that it amounts to a refusal to give them motor insurance, they should contact Insurance Ireland, 5 Harbourmaster Place, IFSC, Dublin 1, Telephone +353 1 6761820. quoting the Declined Cases Agreement.

21/06/2016WRE05500Insurance Costs

21/06/2016WRE05600109. Deputy Mary Lou McDonald asked the Minister for Finance his plans to tackle the extraordinary hike in car insurance premiums following the Private Members’ business motion on this issue last week. [17002/16]

21/06/2016WRE05700Minister for Finance (Deputy Michael Noonan): Differing reasons have been put forward by various interested parties to explain Ireland’s current increasing insurance costs. Motor in- surance appears to be particularly affected with the cost of premiums increasing significantly in the past 12 months. In fact, there are a number of factors that drive the cost of insurance.

Reasons often presented include the increased level of insurance claims and the increasing value of compensation awards. Others highlight that the highly competitive nature of the do- mestic market for non-life insurance in recent years has begun to impact on firms’ underwriting profitability with underwriting losses reported in 2014 for a number of high-impact firms.

To examine these issues in more detail and to assess what the options are for the Govern- ment, I have established a task force in my Department to undertake a Review of Policy in the Insurance Sector. The first phase of the work of the task force, which began in January 2016, is a review of the Framework for Motor Insurance Compensation. This is being carried out jointly with the Department of Transport, Tourism and Sport. This review also deals with broader is- sues around the Insurance Compensation Fund and its report will be submitted shortly to me and my colleague, the Minister for Transport, Tourism and Sport.

The broader work of the task force includes an examination of the issues debated during the Dáil Private Member’s Motion of the 8th and 9th of June. The work is being undertaken in consultation with the Central Bank of Ireland, other Government Departments, Agencies and interested bodies. The aim of the review is to identify the factors contributing to the increasing costs of insurance, and to recommended measures to improve the functioning and regulation of the insurance sector in Ireland, identifying the issues that can be addressed on a more immediate basis and those that need more long-term policy implementation.

48 21 June 2016 This work will be completed over the coming months.

21/06/2016WRE05800Disabled Drivers and Passengers Scheme

21/06/2016WRE05900110. Deputy Michael Healy-Rae asked the Minister for Finance the status of an applica- tion by a person (details supplied) who was declined a certificate under the primary care medi- cal scheme; and if he will make a statement on the matter. [17028/16]

21/06/2016WRE06000Minister for Finance (Deputy Michael Noonan): As the Deputy is aware, a Primary Med- ical Certificate is required to claim the tax reliefs provided under the Disabled Drivers and Disabled Passengers (Tax Concessions) Scheme. The Senior Medical Officer for the relevant local Health Service Executive administrative area makes a professional clinical determination as to whether an individual applicant satisfies the medical criteria to receive a Primary Medical Certificate. An unsuccessful applicant can appeal the decision of the Senior Medical Officer to the Disabled Drivers Medical Board of Appeal, which makes a new clinical determination in respect of the individual.

Hearings of the Disabled Drivers Medical Board of Appeal are held on average twice a month at the National Rehabilitation Hospital in Dun Laoghaire, which has the facilities to ca- ter for people with mobility impairing disabilities of the kind provided for under the Disabled Drivers and Disabled Passengers Scheme.

The Medical Board of Appeal holds regional clinics as demand arises. I’m informed that one clinic per year, for the past four years, has been held in Cork City. A regional clinic is sched- uled for September this year in the Mercy University Hospital, Cork City. The Medical Board of Appeal conducts appeals in the appropriate clinical environment, and cannot undertake ap- peals in appellants’ homes.

Regulation 6(1)(e) of the Disabled Drivers and Disabled Passengers (Tax Concession) Reg- ulations 1994 (S.I. 353 of 1994) mandates that the Medical Board of Appeal is independent in the exercise of its functions.

21/06/2016WRE06100Motor Insurance Coverage

21/06/2016WRE06200111. Deputy Robert Troy asked the Minister for Finance why the car insurance industry will not honour driving experience overseas for emigrants who are returning home (details sup- plied). [17312/16]

21/06/2016WRE06300Minister for Finance (Deputy Michael Noonan): As Minister for Finance, I am respon- sible for the development of the legal framework governing financial regulation but am prohib- ited from interfering in the provision or pricing of insurance products. The EU framework for insurance expressly prohibits Member States from adopting rules which require insurance com- panies to obtain prior approval of the pricing, or terms and conditions of an insurance product.

The provision of insurance cover and the price at which it is offered, including the granting of no claims discounts, is a commercial matter for insurance companies and is based on an as- sessment of the risks they are willing to accept and adequate provisioning to meet those risks. These are considered by insurance companies on a case by case basis. Variations in the costs and risks of providing motor insurance cover can vary between different countries.

With regard to no claims discounts for persons returning from abroad, Insurance Ireland has informed me that, in general terms, where there has been no motor insurance in an indi- 49 Questions - Written Answers vidual’s name and there is a gap of cover of two years or more since their last insurance, the no claims discount is deemed invalid. However, Insurance Ireland has further stated that if the individual can produce confirmation that they were continually insured and are claims free in their own name while they were away, this would be taken into consideration.

My Department has embarked on a review of policy in the insurance sector which is being undertaken in consultation with the Central Bank and other Departments and Agencies. The objective of the Review is to recommend measures to improve the functioning and regulation of the insurance sector.

The first phase of the Review is focussed on the motor insurance compensation framework and this work is nearing completion. The next phase of the Review involves examining the fac- tors contributing to the increasing cost of insurance and identifying what short-term measures can be introduced to help reduce the cost of insurance for consumers and businesses. Work on the Review of Policy in the Insurance sector will continue over the coming months and it will include an examination of the issues affecting persons returning from abroad when they attempt to obtain motor insurance.

I would also add that Insurance Ireland operates a free Insurance Information Service for those who have queries, complaints or difficulties in relation to obtaining insurance. In the event that a person is unable to obtain a quotation for motor insurance or feels that the premium proposed or the terms are so excessive that it amounts to a refusal to give them motor insurance, they should contact Insurance Ireland, 5 Harbourmaster Place, IFSC, Dublin 1, Telephone +353 1 6761820 quoting the Declined Cases Agreement.

21/06/2016WRE06400IBRC Staff

21/06/2016WRE06500112. Deputy Paul Murphy asked the Minister for Finance if he has agreed to pay four weeks’ redundancy for each year worked to former employees of the Irish Bank Resolution Corporation; if he will treat this as a valid unsecured creditor claim; the measures he will take to ensure the payment of redundancy to former staff; and if he will make a statement on the matter. [16675/16]

21/06/2016WRE06600Minister for Finance (Deputy Michael Noonan): I am informed by the Special Liquida- tors that employee claims for an additional four weeks pay per year of service over and above statutory redundancy entitlements have been reviewed and adjudicated by the Special Liquida- tors.

The Special Liquidators have confirmed that, based on legal advice, these claims for ad- ditional redundancy entitlements have not been admitted as unsecured creditors in the liquida- tion.

VAT Rate Application

21/06/2016WRF00200113. Deputy Danny Healy-Rae asked the Minister for Finance to ensure that the 9% value added tax rate is retained for the hospitality sector to safeguard tourism; and if he will make a statement on the matter. [16718/16]

21/06/2016WRF00300116. Deputy Pearse Doherty asked the Minister for Finance further to the statement in the document, Minister’s Brief 2016, about the reduced value added tax rate for the hospitality sector, the general recovery of the economy and increasing prices in the sector raises ques-

50 21 June 2016 tions about its future, his plans in this regard; and if he will make a statement on the matter. [16729/16]

21/06/2016WRF00400126. Deputy Imelda Munster asked the Minister for Finance his plans to raise the rate of value added tax for the tourism sector. [16830/16]

21/06/2016WRF00500Minister for Finance (Deputy Michael Noonan): I propose to take Questions Nos. 113, 116 and 126 together.

The 9% reduced VAT rate for tourism related services was introduced in July 2011 as part of the Government Jobs Initiative. The measure was designed to boost tourism and create ad- ditional jobs in that sector. In my Budget Day speech last year, I suggested that while the case for retaining the measure for the hotel sector in Dublin is diminishing each year with room rates rising particularly during major events, the case for retention of the measure for the rest of the country remained. The Programme for a Partnership Government published in May, includes a commitment to retain the hugely successful 9% VAT rate on tourism related services, providing that prices remain competitive.

Departmental Schemes

21/06/2016WRF00600114. Deputy Pearse Doherty asked the Minister for Finance the number of companies which participated in the research and development tax credit scheme, the amounts they claimed and the number of jobs they created under the scheme in each of the years 2013 to 2015, in tabular form; and if he will make a statement on the matter. [16727/16]

21/06/2016WRF00700Minister for Finance (Deputy Michael Noonan): I am informed by Revenue that statistics regarding the tax cost and number of claims for the Research and Development tax credit are available on the Revenue Statistics webpage at

http://www.revenue.ie/en/about/statistics/costs-expenditures.html. Information for the tax year 2015 is not yet available as tax returns for this year are not due to be returned in many cases until later in 2016.

It is not possible to provide data on the number of jobs created as this information is not required on the tax return.

Universal Social Charge

21/06/2016WRF00800115. Deputy Pearse Doherty asked the Minister for Finance further to the statement in the document, Minister’s Brief 2016, given its significance in revenue raising, that changes to the universal social charge would likely require the introduction of other discretionary revenue raising measures to fund it, what are these discretionary measures and when he envisages their implementation; and if he will make a statement on the matter. [16728/16]

21/06/2016WRF00900Minister for Finance (Deputy Michael Noonan): The Summer Economic Statement pub- lished by Government today reflects the tax and spending priorities set out in the Programme for a Partnership Government (PPG). As the Deputy is aware, there is a commitment in the PPG to ask the Oireachtas to continue to phase out the USC as part of a wider medium-term income tax reform plan that keeps the tax base broad, reduces excessive tax rates for middle income earners, and limits the benefit for high earners. Reductions will be introduced on a fair basis with an emphasis on low and middle income earners.

51 Questions - Written Answers The PPG states that reductions in personal tax rates - such as the phasing out of the USC needed to reward work and support enterprise and employment will be funded largely through:

- Extra revenues from not indexing personal tax credits and bands

- The removal of PAYE tax credit for high earners and other measures to ensure the tax system remains fair and progressive

- Higher excise duties on cigarettes and increased enforcement and sanctions on the illegal importation and sale of cigarettes

- Increased enforcement and sanctions on fuel laundering

- A new tax on sugar sweetened drinks

- Improving tax compliance

Work on development of the medium-term income tax reform plan is ongoing and as such the expected cost is not yet finalised. It is due to be published for consultation with the Oireach- tas Committee on Finance in July, and for approval by the Oireachtas in October.

The revenue to be raised by planned offsetting tax changes, such as a sugar-sweetened drinks tax for example, will depend on, among other things, the rate at which the tax is set and the scope of its application. These issues are budgetary matters which will be determined through the budgetary process.

The income tax reform plan will aim to support job creation and reward work which is a key driver of growth and prosperity in the economy. In developing the plan, available resources will be a key consideration along with safeguarding the economic stability of the public fi- nances and the wider economy.

Question No. 116 answered with Question No. 113.

Pension Provisions

21/06/2016WRF01100117. Deputy Pearse Doherty asked the Minister for Finance further to the statement in the document, Minister’s Brief 2016, about work his Department is doing on pensions, work on developing and introducing a workable system, including a tax or other State incentive, if he will elaborate on what he means by other State incentive and comment on his determination of the sustainability of current pension tax reliefs over the coming years; and if he will make a statement on the matter. [16730/16]

21/06/2016WRF01200Minister for Finance (Deputy Michael Noonan): The context in which the particular ref- erences to the pension issues of interest to the Deputy were raised in the briefing document was in relation to the development of a universal retirement savings system for that half of the Irish workforce who do not have supplementary pension coverage.

Officials of my Department, together with those of other Government Departments and Agencies (as well as representatives from other organisations), have been involved in the work of the Universal Retirement Savings Group (URSG) which was established last year under the aegis of the Department of Social Protection to develop a roadmap and timeline for the intro- duction of a new retirement savings system to progressively achieve universal pension with particular focus on the lower paid. The scope and direction of the work of the URSG is a mat- ter, in the first instance, for the new Minister for Social Protection, Mr. Leo Varadkar TD, who

52 21 June 2016 has indicated that the development of a universal retirement savings system will be a priority for his Department.

The development of a system, as referred to above, would be a long-term project which, in order to be successful, would have to involve appropriate decisions being taken by Government over time on detailed key design, operational and administrative structures. Among the many issues that would have to be decided, in this context, is the extent and nature of any State sup- port for pension savers in such a system. While I see no reason, at this point, to change from the marginal rate relief incentive on pension contributions which applies for current pension savers, arguments have been put forward in the past, for example, for a system of direct subvention or matching contributions to pension savings to be made by the State as opposed to the provision of tax relief. I would be open to examining and considering, without prejudice, any case that might be made in this regard and this is what the reference to “other State incentive” in the briefing document is getting at.

As regards the sustainability of current pension tax reliefs, I would point out that the esti- mated annual cost of tax reliefs on pension contributions, based on the statistical data published by the Revenue Commissioners and available on the Revenue website, has fallen considerably in the period since 2008. Among the reasons for this, is the range of measures taken over the years to restrict the reliefs on pension contributions, particularly for higher earners.

EU Meetings

21/06/2016WRF01300118. Deputy Joan Collins asked the Minister for Finance the details of the Bilderberg meeting held recently in Germany, including who attended, what was discussed, the minutes of the meeting and if he attended on behalf of the Government. [16733/16]

21/06/2016WRF01400Minister for Finance (Deputy Michael Noonan): I attended the Bilderberg meeting from 10-12 June 2016 in Dresden, Germany. I, like a number of European ministers, was invited to attend given my position as Minister for Finance. For further information, I would point the Deputy to the Bilderberg Meetings website (www.bilderbergmeetings.org) which includes in- formation on the organisation’s governance, steering committee, meetings, attendees, agendas and associated press releases. At this meeting and its workshops I took the opportunity to set out to my fellow attendees the opportunities that exist in Ireland for investors and multinational companies.

The Government is focussed on encouraging and supporting foreign direct investment into Ireland to provide jobs and continue to support economic growth. In January of this year, the IDA announced the highest level of employment in its client companies in its 67 year history. IDA client companies created 18,983 new jobs in 2015. These results mean that more than one- in-five private sector jobs in the economy are as a result of government supported FDI. I would point out to the Deputy that a number of the business attendees represented companies which have very significant investments in Ireland that support thousands of Irish jobs.

EU Meetings

21/06/2016WRF01500119. Deputy Pearse Doherty asked the Minister for Finance the meetings he held at the Bilderberg conference in 2016 including their agendas and who attended; and if he will make a statement on the matter. [16766/16]

21/06/2016WRF01600Minister for Finance (Deputy Michael Noonan): I attended the Bilderberg meeting from 53 Questions - Written Answers 10-12 June 2016 in Dresden, Germany. I, like a number of European ministers, was invited to attend given my position as Minister for Finance. For further information, I would point the Deputy to the Bilderberg Meetings website (www.bilderbergmeetings.org) which includes in- formation on the organisation’s governance, steering committee, meetings, attendees, agendas and associated press releases. At this meeting and its workshops I took the opportunity to set out to my fellow attendees the opportunities that exist in Ireland for investors and multinational companies.

The Government is focussed on encouraging and supporting foreign direct investment into Ireland to provide jobs and continue to support economic growth. In January of this year, the IDA announced the highest level of employment in its client companies in its 67 year history. IDA client companies created 18,983 new jobs in 2015. These results mean that more than one- in-five private sector jobs in the economy are as a result of government supported FDI. I would point out to the Deputy that a number of the business attendees represented companies which have very significant investments in Ireland that support thousands of Irish jobs.

Tax Settlements

21/06/2016WRF01700120. Deputy Michael McGrath asked the Minister for Finance the number of settlements the Revenue Commissioners reached in 2014 and 2015 with high-net-worth individuals, HNWI, that contained a clause guaranteeing that the details of the settlement would not be published; why the Revenue Commissioners enter into non-disclosure agreements; his plans to review the policy in relation to non-disclosure of large settlements; and if he will make a statement on the matter. [16796/16]

21/06/2016WRF01800Minister for Finance (Deputy Michael Noonan): I am informed by Revenue that non- publication of a tax settlement applies only where the details of the settlement are specifically excluded from publication by virtue of provisions of Section 1086 of the Taxes Consolidation Act, 1997 as set out in Revenue’s Code of Practice for Revenue Audit http://www.revenue.ie/ en/practitioner/codes-practice.html. I am advised by Revenue that there is no separate non- publication procedure operated by them for any particular class of taxpayer.

I am also advised by Revenue that their records relating to tax settlements where non-pub- lication arose by virtue of the provisions of Section 1086 of the Taxes Consolidation Act 1997 do not separately identify settlements with individuals who may be described as high net worth individuals.

Financial Services Regulation

21/06/2016WRF01900121. Deputy Pearse Doherty asked the Minister for Finance the number of prosecutions which have been brought against lenders for irresponsible lending in breach of Article 8(1) of the credit agreements for consumers’ directive; and if he will make a statement on the matter. [16798/16]

21/06/2016WRF02000Minister for Finance (Deputy Michael Noonan): I presume the Deputy is referring to Statutory Instrument 281 of 2010 European Communities (Consumer Credit Agreements) Reg- ulation 2010 (which transposed Directive 2008/48/EC of the European Parliament and of the Council of 23 April 2008 on credit agreements for consumers and repealing Council Directive 87/102/EEC). Regulation 11 of the Statutory Instrument transposed Article 8(1) the “Obliga- tion to assess creditworthiness of consumers”.

54 21 June 2016 I am advised by the Central Bank that it has not taken any sanctions against lenders for breaches of Regulation 11.

Tax Reliefs Eligibility

21/06/2016WRF02100122. Deputy Fergus O’Dowd asked the Minister for Finance his views on a matter (details supplied) regarding tax relief on social housing loans; and if he will make a statement on the matter. [16816/16]

21/06/2016WRF02200Minister for Finance (Deputy Michael Noonan): Some of the details supplied in connec- tion with this question are unclear. However, it would appear to refer primarily to a proposal to allow tax relief on the capital repayment element of a loan for new social housing.

The Deputy will be aware that I introduced a new tax relief in Finance Act 2015 which al- lows a full 100% mortgage interest deduction where a landlord undertakes, for a period of at least three years, to provide accommodation to tenants in receipt of social housing supports. This is in place of the 75% cap on mortgage interest relief deductibility which normally applies in respect of rented residential properties. The relief is designed to incentivise landlords to commit to letting their property to tenants in receipt of social housing supports over the longer term, thereby improving the stability of supply of property to such tenants.

This relief was one element of an overall package of measures designed by the Govern- ment aimed at stabilising rent and boosting supply in the housing market which, in my view, is the most appropriate and effective route to addressing rental price increases driven by supply constraints. To extend this relief further to allow for a deduction for the capital repayment ele- ment of a mortgage, as proposed in the Deputy’s question, would in effect see the State fully subsidise the purchase by a private individual of a residential rental investment property. I do not believe that this would be an appropriate use of State resources at this time.

In relation to social housing, the Deputy will be aware that there are a number of non-tax commitments in the Programme for a Partnership Government, including a commitment to significantly increase and expedite the delivery of social housing units.

Tax Reliefs Eligibility

21/06/2016WRF02300123. Deputy Peadar Tóibín asked the Minister for Finance if he will broaden the criteria for granting a tax relief (details supplied). [16821/16]

21/06/2016WRF02400Minister for Finance (Deputy Michael Noonan): The Disabled Drivers and Disabled Pas- sengers (Tax Concessions) Scheme provides relief from VAT and VRT (up to a certain limit) on the purchase of an adapted car for transport of a person with specific severe and permanent physical disabilities, payment of a fuel grant, and an exemption from Motor Tax.

To qualify for the Scheme an applicant must be in possession of a Primary Medical Cer- tificate. To qualify for a Primary Medical Certificate, an applicant must be permanently and severely disabled within the terms of the Disabled Drivers and Disabled Passengers (Tax Con- cessions) Regulations 1994 and satisfy one of the following conditions:

- be wholly or almost wholly without the use of both legs;

- be wholly without the use of one leg and almost wholly without the use of the other leg such that the applicant is severely restricted as to movement of the lower limbs; 55 Questions - Written Answers - be without both hands or without both arms;

- be without one or both legs;

- be wholly or almost wholly without the use of both hands or arms and wholly or almost wholly without the use of one leg;

- have the medical condition of dwarfism and have serious difficulties of movement of the lower limbs.

The Senior Medical Officer for the relevant local Health Service Executive administrative area makes a professional clinical determination as to whether an individual applicant satisfies the medical criteria. A successful applicant is provided with a Primary Medical Certificate, which is required under the Regulations to claim the reliefs provided for in the Scheme. An unsuccessful applicant can appeal the decision of the Senior Medical Officer to the Disabled Drivers Medical Board of Appeal, which makes a new clinical determination in respect of the individual. The Regulations mandate that the Medical Board of Appeal is independent in the exercise of its functions to ensure the integrity of its clinical determinations. After six months a citizen can reapply if there is a deterioration in their condition.

The Scheme represents a significant tax expenditure. Between the Vehicle Registration Tax and VAT foregone, and the repayment of excise on fuel used by members of the Scheme, the Scheme represented a cost of €50.3 million to the Exchequer in 2015, an increase from €48.6 million in 2014. These figures do not include the revenue foregone to the Local Government Fund in the respect of the relief from Motor Tax provided to members of the Scheme.

I am aware that the Ombudsman has made comments regarding the eligibility criteria of the Disabled Driver and Disabled Passengers Scheme. The Ombudsman stated that, in his opinion, the criteria were narrowly focused and prescriptive. The Scheme and qualifying criteria were designed specifically for those with severe physical disabilities and are, therefore, necessarily precise.

I recognise the important role that the Scheme plays in expanding the mobility of citizens with disabilities. I have managed to maintain the relief at current levels throughout the cri- sis despite the requirement for significant fiscal consolidation. From time to time I receive rep- resentations from individuals who feel they would benefit from the Scheme but do not qualify under the six criteria. While I have sympathy for these cases, given the scale and scope of the Scheme, I have no plans to expand the medical criteria beyond the six currently provided for in the Disabled Drivers and Disabled Passengers (Tax Concessions) Regulations 1994.

Tax Code

21/06/2016WRF02500124. Deputy Pearse Doherty asked the Minister for Finance if he still intends to appeal the decision in the event of the European Union ruling that a company (details supplied) should repay tax to Ireland; and if he will make a statement on the matter. [16828/16]

21/06/2016WRF02600125. Deputy Pearse Doherty asked the Minister for Finance if he has met directly with the European Commission regarding its ongoing investigation into the tax affairs of a company (details supplied) here; and if he will make a statement on the matter. [16829/16]

21/06/2016WRF02700Minister for Finance (Deputy Michael Noonan): I propose to take Questions Nos. 124 and 125 together.

56 21 June 2016 In June 2014, the Competition Directorate of the European Commission announced its in- tention to open formal state aid investigations into tax rulings provided to a number of compa- nies in various Member States of the European Union. Since October 2015, investigations in three other Member States have concluded. In each of these cases the Commission found that the Member States granted an illegal State Aid to the companies in question.

While the Commission has opened a formal investigation in relation to one particular case involving Ireland, it has not made a final determination in the matter. While there is no formal timeline for a when the final decision will be made in our case, I am aware of speculation about a possible decision in July.

This a priority matter and Ireland has co-operated fully with the process to date and will continue to do so. My Department has engaged closely with the Commission throughout this process. Detailed and comprehensive responses have been provided to the Commission dem- onstrating that the appropriate amount of Irish tax was charged in accordance with the relevant legislation, that no selective advantage was given and that there was no State Aid.

I remain of the view that there was no breach of State Aid rules in this case and that the leg- islative provisions were correctly applied. In the event that the Commission forms the view that there was state aid, Ireland is entitled to challenge this decision in the European Courts. As the Government has already indicated, we will take that course of action, if necessary, to continue to vigorously defend the Irish position.

Question No. 126 answered with Question No. 113.

21/06/2016WRF02850UK Referendum on EU Membership

21/06/2016WRF02900127. Deputy Pearse Doherty asked the Minister for Finance if he has analysed the impact on gross domestic product of a British withdrawal from the European Union; and if he will make a statement on the matter. [16839/16]

21/06/2016WRF03000Minister for Finance (Deputy Michael Noonan): There are numerous sources of uncer- tainty at present which pose risks to the central forecast set out in the Stability Programme Up- date 2016. Principal among them is the prospect of the UK voting to leave the European Union.

An assessment of the potential economic impact of such an outcome is set out below.

Recently published work by both the UK Treasury and the UK’s National Institute of Eco- nomic and Social Research (NIESR) using model based analysis suggest that a vote to leave the EU could reduce UK GDP by between 2.3 and 6.0 per cent, relative to baseline, under a range of scenarios.

In relation to the impact for Ireland, estimates made using the ESRI HERMES model sug- gest that a 1 per cent reduction in UK GDP would reduce Irish GDP by approximately 0.2 per cent, relative to baseline, over two years. This implies a possible fall in Irish GDP relative to baseline in the range of 0.5 to 1.2 per cent based on Treasury and NIESR estimates.

A shock to UK GDP would also be expected to impact on our other trading partners. Esti- mates from the ESRI HERMES model suggest that if euro area GDP were to also fall by 1 per cent, a level estimated in the Treasury’s “severe scenario”, Irish GDP would fall by a further 0.4 per cent relative to baseline.

Separately, both UK reports predict a depreciation of sterling in the event of a vote to leave.

57 Questions - Written Answers An assessment of the impact of a 5 per cent sterling depreciation, was presented in the 2016 Stability Programme Update published in April, and indicated a loss in Irish GDP of 1 per cent after 2 years. This result also incorporates the impact on Ireland of wider spillover effects on the global economic environment from a sterling depreciation.

In addition to the macroeconomic assessments set out above, it must also be recognised that a UK decision to leave the EU will increase short term uncertainty and volatility in the financial markets potentially leading to negative outcomes internationally and in Ireland beyond those set out above.

Inflation Rate

21/06/2016WRF03100128. Deputy Pearse Doherty asked the Minister for Finance the effects of a longer period of very low inflation on debt profile; and if he will make a statement on the matter. [16840/16]

21/06/2016WRF03200Minister for Finance (Deputy Michael Noonan): The debt profile, the ratio of general government debt to GDP, is driven by developments in the stock of general government debt and nominal, or current prices, GDP. Nominal GDP, in turn, is determined by both real eco- nomic activity and the GDP deflator - which measures price changes in the economy as whole. All else being equal, a lower than expected pace of inflation will lower the GDP deflator which, for any given level of debt, will mechanically increase the debt-to-GDP ratio relative to baseline i.e. compared to what it would have been otherwise.

My Department’s macroeconomic and fiscal forecasts, as published in the Stability Pro- gramme Update (SPU) in April of this year, show overall nominal GDP growth of 7.6 per cent this year and 4 ¾ per cent over the medium term, with the GDP deflator increasing by 2.6 per cent and 1¼ per cent, respectively, over the same period. This is consistent with continued im- provements in the debt-to-GDP ratio over the forecast period.

Flood Risk Insurance Cover Provision

21/06/2016WRF03300129. Deputy Pearse Doherty asked the Minister for Finance when he will report on the proposal to tackle the difficulty in some cases of acquiring flood insurance; and if he will make a statement on the matter. [16841/16]

21/06/2016WRF03400Minister for Finance (Deputy Michael Noonan): The provision of insurance cover is a commercial matter for insurance companies and is based on an assessment of the risks they are willing to accept and adequate provisioning to meet those risks.

In my role as Minister for Finance, I have responsibility for the development of the legal framework governing financial regulation. Neither I, nor the Central Bank of Ireland, can in- terfere in the provision or pricing of insurance products or have the power to direct insurance companies to provide flood cover to specific individuals or businesses.

I am aware of the difficulties that the absence or withdrawal of flood insurance cover can cause to homeowners and the flooding crisis last winter has raised issues in relation to insurance and flooding. To examine the issue of flooding an Inter-Departmental Flood Policy Co-ordi- nation Group has been established to ensure a whole of Government approach in the area of Flood Policy. The OPW are the lead agency and have responsibility for submitting the final report of the group to Government. Each Department involved will submit a report to the OPW with policy proposals in their own area which can directly improve preparation and response

58 21 June 2016 to flooding or strategic policies which impact on people’s risk and experience of dealing with flooding.

The Department of Finance is actively engaged with the Inter-Departmental Flood Policy Co-ordination Group and will contribute to the report in the area of a review of flood insurance with a particular focus on the strategies that other jurisdictions have implemented to increase the availability of flood insurance cover. This work is examining a number of policy options and within the coming weeks will feed into the final report of the Inter-Departmental Group, chaired by Seán Canney TD, Minister of State with special responsibility for the Office of Pub- lic Works and Flood Relief.

Financial Transactions Tax

21/06/2016WRF03500130. Deputy Pearse Doherty asked the Minister for Finance his views on the financial transaction tax, including why he is opposed to it; if this position is open to review; and if he will make a statement on the matter. [16844/16]

21/06/2016WRF03600Minister for Finance (Deputy Michael Noonan): Ireland already has a tax on financial transactions, a Stamp Duty on transfers of shares in Irish incorporated companies, which cur- rently stands at 1%. I am informed by the Revenue Commissioners that the yield from this charge in 2015 was €424.13 million.

The Financial Institutions Levy I announced as part of Budget 2014 is a revenue raising measure which provides for a contribution from the banking sector to Ireland’s economic re- covery. The levy is in place for the years 2014 to 2016 inclusive with an anticipated annual yield of €150 million. As the levy is a percentage of an institution’s DIRT liability in 2011, liability to the levy relates to the size of an institution’s Irish operation. The entire banking system has been underpinned by the strong Government support provided both here and abroad and I believe it is appropriate therefore that the banking sector should make a contribution to the State’s economic recovery. Accordingly, I announced in my Budget 2016 statement that I propose to extend the levy out to 2021, subject to a review taking place of the methodology used to calculate the levy. This will bring in an additional €750 million over the period, which is a very significant additional contribution to the Exchequer.

In relation to discussions at EU level, the Government’s position is that a Financial Trans- actions Tax would be best applied on a wide international basis to include the major financial centres to prevent the danger of activities gravitating to jurisdictions where taxes are not levied on financial transactions. Notwithstanding this, the Government is not prepared to stand in the way of EU Member States that wish to work together to implement a Financial Transactions Tax and in this regard adoption of a decision formally authorising enhanced cooperation took place during the Irish Presidency of the EU in January 2013.

The proposal for a Directive from the European Commission in the area of financial trans- action tax was published in February 2013. Ireland had many concerns about the proposal as drafted, not least of which were the potential impacts on, and the trading of, Irish Sovereign debt in the secondary market and in total, the potential negative impact on the liquidity of the financial sector as a whole. Members of the Economic and Financial Sub-Committee on EU Sovereign Debt Markets have stated that the introduction of the FTT would have a significantly negative effect on Sovereign Debt Markets and may impair the good-functioning of secondary markets for sovereign debt resulting in reduced liquidity, reduced investor demand and there- fore higher financing costs for States.

59 Questions - Written Answers Our concerns are widely shared amongst the Member States, including some of the partici- pating countries. These concerns have led to the issuing of a communique by the participating Member States, announcing that they have agreed to implement a financial transaction tax in a progressive manner, with the first step being a charge on shares and some derivatives.

More recently on 17th June 2016 the ECOFIN Council discussed the current state of play with regard to the proposal of a number of Member States to introduce a financial transaction tax. In the context of this discussion, ten of the original eleven Member States (Estonia has indicated that it no longer supports the proposal), issued a statement setting out their agreement on the core design principles of an FTT. The statement indicates that further reassurances were needed on two issues in particular, for which two task forces will be immediately set up. First, taxation of derivatives should not have a negative impact on public borrowing costs. Second, tax collection should be cost-effective. The outcome of these two task forces will be discussed in September.

There is still uncertainty therefore as to the form the FTT might take and more detail would be needed on the final shape of the tax before a definitive conclusion could be reached about its impact on Irish taxation revenue.

Banking Sector

21/06/2016WRF03700131. Deputy Pearse Doherty asked the Minister for Finance the options he is considering regarding his shares in Permanent TSB and the overall strategy for the bank; and if he will make a statement on the matter. [16845/16]

21/06/2016WRF03800Minister for Finance (Deputy Michael Noonan): Our strategy envisages Permanent TSB (“PTSB”) playing an important role in the future of Irish retail banking as a retail bank bring- ing competition to the marketplace which has consolidated significantly since 2008. PTSB is an important bank in a highly concentrated Irish market lending approximately €460 million in 2015.

PTSB has fundamentally de-risked its balance sheet under the intensive oversight of various authorities since 2011 through deleverage, improved funding profile and increased capital lev- els (fully loaded CET1 of 15.4% at Q1, 2016). It has made positive progress on arrears reduc- tions, with great than 90 day arrears in Homeloans c. 46% below peak levels in 2013. PTSB returned to underlying profitability in 2015 for the first time since 2007 and it announced that it was profitable and capital generative in Q1, 2016 (€39m Profit BeforeTax).

Notwithstanding the positive progress made PTSB in many areas its share price has fallen c. 57% to date in 2016. The share price has been under pressure for a myriad of reasons including, but not limited to, the lower interest rate environment, the delayed disposal of CHL due to the UK referendum, increasing regulatory costs, the Central Bank tracker mortgage review, exten- sion of the Bank Levy and higher provisions and lower growth in new lending than the market anticipated. I have also been informed that some investors are concerned about the policy that might be pursued by the new Government in relation to banking and the attitude of the Oireach- tas to issues around mortgage interest rates. Equity research analysts covering PTSB have writ- ten about this topic in detail recently.

I am of the view that the best way to protect the value of the State’s shareholding is to ensure PTSB continues the progress it has made with a view to reaching sustainable operat- ing profitability and an adequate Return on Equity (ROE) as soon as possible while striving to meet the terms of the European Commission restructuring plan.

60 21 June 2016 While I am strongly supportive of Permanent TSB in the delivery of their strategy I cannot discount the possibility that a strategic transaction could arise opportunistically at any time in- volving PTSB which could be in the best interests of the State. As part of their day-to-day role officials in the Shareholding Management Unit will consider all credible proposals and develop strategic options relating to our banking investments and will also consider from time to time whether the sale of shares by way of placing would be beneficial for the State. Having said that I have no current plans to sell shares in PTSB, notwithstanding the flexibility to do so within the Programme for Government.

As I have stated previously I would like to see more competition in the domestic banking system to provide the lending required for our growing economy and this could be achieved through new entrants or the continued growth of our domestic banks.

Legislative Reviews

21/06/2016WRF03900132. Deputy Pearse Doherty asked the Minister for Finance his plans to simplify and con- solidate financial consumer protection legislation. [16846/16]

21/06/2016WRF04000Minister for Finance (Deputy Michael Noonan): My Department continues to keep fi- nancial legislation relating to consumer protection under review to ensure consumers are pro- tected. Given that financial consumer protection law is contained within national and European legislation, its consolidation would be an extremely large and complex piece of work and is not something that is currently planned.

The Deputy should also be aware that there is already a large and heavy agenda in the area of consumer protection legislation.

By way of example, as you know the Consumer Protection (Regulation of Credit Servicing Firms) Act, 2015 was enacted on 8 July 2015. It was introduced to fill the consumer protection gap where loans were sold by the original lender to an unregulated entity.

In the area of residential mortgages, the European Union (Consumer Mortgage Credit Agreements) Regulations 2016 (SI 142/16) have transposed the Mortgage Credit Directive into Irish law. This Directive provides that certain minimum consumer information and protection measures such as the provision of standardised pre-contractual information, common require- ments for the calculation of the APRC, the requirement on a lender to conduct a credit worthi- ness assessment prior to offering mortgage credit are to apply across the EU. The Deputy may wish note that Directive also places an onus on the EU Commission to undertake a review of the effectiveness and appropriateness of the Directive’s provisions on consumers and the internal market by 2019.

Furthermore, there are a number of projects being progressed by my Department at present. By way of illustration, the Payment Accounts Directive must be transposed by 18 September 2016. The main elements of the Payment Accounts Directive focus on transparency and com- parability of payment account fees, payment account switching, and access to payment ac- counts with basic features for all EU consumers. The Directive also allows Member States to take additional measures to ensure access to basic payment accounts for unbanked, vulnerable consumers.

In addition, the Payment Services Directive 2 (PSD2) aims to further develop the aims of the first Payment Services Directive (PSD1) by regulating new market players, enhancing consumer protection, ensuring competition within the industry whilst maintaining a level play- ing field, and harmonising the regulations across Europe. PSD2 is due to be transposed in all 61 Questions - Written Answers Member States by 13 January 2018.

Consumer Protection

21/06/2016WRF04100133. Deputy Pearse Doherty asked the Minister for Finance his plans to amend the Con- sumer Protection (Regulation of Credit Servicing Firms) Act 2015; and if he will make a state- ment on the matter. [16847/16]

21/06/2016WRF04200Minister for Finance (Deputy Michael Noonan): As the Deputy will be aware, the Con- sumer Protection (Regulation of Credit Servicing Firms) Act, 2015 was enacted on 8 July 2015. It was introduced to fill the consumer protection gap where loans were sold by the original lender to an unregulated firm. The 2015 Act introduced a regulatory regime for a new type of entity called a ‘credit servicing firm’. Credit servicing firms are now subject to the provisions of Irish financial services law that apply to ‘regulated financial service providers’. This ensures that relevant borrowers, whose loans are sold to third parties, maintain the same regulatory protections they had prior to the sale, including under the various statutory codes (such as the Consumer Protection Code, Code of Conduct on Mortgage Arrears, Code of Conduct for Busi- ness Lending to Small and Medium Enterprises and the Minimum Competency Code) issued by the Central Bank of Ireland and the Central Bank (Supervision and Enforcement) Act 2013 (Section 48) (Lending to Small and Medium-Sized Enterprises) Regulations 2015 which comes into operation on 1 July 2016.

Our continued commitment on this issue is underlined by the Programme for a Partnership Government which provides that “we will ask the Central Bank and the Oireachtas Committee on Housing to examine the legislation introduced last year that ensured that borrowers, whose loans are sold to third parties, maintain the same regulatory protections they had prior to the sale. We will provide greater protection for mortgage holders and tenants and SMEs whose loans have been transferred to non-regulated entities (‘vulture funds’).” This is a Year 1 Action in the Programme. The nature of any proposed changes will be decided after further consider- ation of the issues.

Financial Services Sector

21/06/2016WRF04300134. Deputy Clare Daly asked the Minister for Finance his views on whether the incor- rect valuation of loan assets is enriching wealthy hedge funds at the expense of small business owners, given that according to the Tomlinson report banks are deliberately closing down busi- nesses and forcing small business owners to sell assets on distressed terms to favoured clients, including companies (details supplied); on the fact that a law firm has examined if an Irish bank is engaged in the same activities; on whether his assurances that banks are not systemically overvaluing assets are in direct contradiction of evidence to the banking inquiry in which bank- ers have admitted that by following International Accounting Standard 39 they are deliberately overvaluing assets. [16878/16]

21/06/2016WRF04400Minister for Finance (Deputy Michael Noonan): The Deputy will be aware that the valu- ation of assets for accounting purposes must be in accordance with International Accounting Standards. IAS 39 provided that assets must be valued at a point in time and it did not allow for the provision of potential future losses which was highlighted by the financial crisis, as noted by the Banking Inquiry. I understand that IAS 39 is being replaced by IFRS 9. The package of improvements introduced by IFRS 9 includes a logical model for classification and measure- ment, a single, forward-looking ‘expected loss’ impairment model and a substantially-reformed

62 21 June 2016 approach to hedge accounting. IFRS 9 is effective for annual periods beginning on or after 1 January 2018.

The Deputy will also be aware that the Central Bank has issued regulations aimed at protect- ing SMEs when dealing with regulated and unregulated firms as set out below.

The Consumer Protection (Regulation of Credit Servicing Firms) Act, 2015 was enacted on 8 July 2015. It was introduced to fill the consumer protection gap where loans were sold by the original lender to an unregulated firm. The 2015 Act introduced a regulatory regime for a new type of entity called a ‘credit servicing firm’. Credit Servicing Firms are now subject to the provisions of Irish financial services law that apply to ‘regulated financial service providers’. This ensures that relevant borrowers, whose loans are sold to third parties, maintain the same regulatory protections they had prior to the sale, including under the various statutory codes (such as the Consumer Protection Code, the Code of Conduct on Mortgage Arrears, the Code of Conduct for Business Lending to Small and Medium Enterprises and the Minimum Compe- tency Code) issued by the Central Bank of Ireland.

Also, following a review in 2015, the Code of Conduct for Business Lending to Small and Medium Enterprises, has been strengthened in certain areas resulting in the Central Bank (Supervision and Enforcement) Act 2013 (Section 48) (Lending to Small and Medium-Sized Enterprises) Regulations 2015 which comes into operation on 1 July 2016.

It is widely recognised that the financial crisis of the previous decade uncovered significant deficiencies in the financial regulatory and supervisory framework and its operation. These have been the subject of extensive and objective analysis. The reports from Professor Patrick Honohan, Regling and Watson, and the Nyberg commission point out the problems to be ad- dressed. A significant amount of legislation has been enacted at national and EU level to deal with these issues.

The Single Supervisory Mechanism (SSM) transfers key supervisory tasks for significant banks in the euro area to the European Central Bank, which means additional oversight beyond national Central Banks.

At a national level, there has been a substantial increase in the Central Bank’s staff resourc- es, particularly in the area of regulation and the revised Central Bank Acts have enhanced peer review and independent oversight arrangements for the Central Bank’s operations.

21/06/2016WRG00200EU Regulations

21/06/2016WRG00300135. Deputy Clare Daly asked the Minister for Finance his views on the European Union having indicated to this Deputy that it is possible to update a regulation by a later directive and that it has used this technique to justify the over-valuation of assets and the hiding of losses in Irish banks; and the circumstances in which a directive, passed after a regulation, can be used to update that regulation. [16879/16]

21/06/2016WRG00400Minister for Finance (Deputy Michael Noonan): I understand that this question relates to Regulation (EC) No 1606/2002 and the Directive is 2013/34/EU.

I have been informed by the Department of Jobs, Enterprise and Innovation that Regulation (EC) No 1606/2002 of the European Parliament and of the Council of 19 July 2002 on the appli- cation of international accounting standards was amended, in Article 3 and Article 6, by Regu- lation (EC) No 297/2008 of the European Parliament and of the Council of 11 March 2008.

63 Questions - Written Answers It was not amended by Directive 2013/34/EU of the European Parliament and of the Council of 26 June 2013 on the annual financial statements, consolidated financial statements and -re lated reports of certain types of undertakings, amending Directive 2006/43/EC of the European Parliament and of the Council and repealing Council Directives 78/660/EEC and 83/349/EEC.

On the valuation of assets I have also been informed by the Department of Jobs, Enterprise and Innovation that the EU-adopted Standard relevant to the measurement of financial assets in the financial statements of banks is International Accounting Standard No. 39 (IAS 39) on Financial Instruments: Recognition and Measurement.

That Standard was adopted by means of Commission Regulation (EC) No 2086/2004 of 19 November 2004 amending Regulation (EC) No 1725/2003 on the adoption of certain interna- tional accounting standards in accordance with Regulation (EC) No 1606/2002 of the European Parliament and of the Council as regards the insertion of IAS 39. This text was supported by a qualified majority of Member States at the Accounting Regulatory Committee (ARC) meeting on 1 October 2004 and by the European Parliament.

The International Accounting Standards Board has issued a Standard replacing IAS 39, International Financial Reporting Standard No. 9, Financial Instruments. The new Standard, among other things, replaces the “incurred loss model” for assessing loan impairment with an “expected loss model”.

The Commission is proposing that the EU adopt this new IFRS 9 standard. Accordingly, the draft Regulation to give effect to that adoption will soon be considered by the European Parlia- ment and Council.

21/06/2016WRG00500Credit Register Establishment

21/06/2016WRG00600136. Deputy Pearse Doherty asked the Minister for Finance when the central credit regis- ter will come into operation and will operate it; and if he will make a statement on the matter. [16938/16]

21/06/2016WRG00700Minister for Finance (Deputy Michael Noonan): The Central Credit Register (CCR) is a national mandatory database of credit and related data established, maintained and operated by the Central Bank, in accordance with the provisions of the Credit Reporting Act 2013 (‘the Act’).

The Central Bank continues to make progress on the implementation of the CCR since the enactment of the legislation in December 2013. In the last twelve months the Central Bank has:

- Completed a design stage to specify the detailed data requirements and technical CCR solution;

- Published feedback on 11 February 2016 to a Public Consultation Paper which sought views on keys issues relating to the CCR;

- Carried out a privacy impact assessment (PIA) to ensure the appropriate controls are in place to safeguard personal data across the end to end processes. This PIA has been shared with the Data Protection Commissioner.

- Prepared draft CCR regulations which were approved by the Commission of the Central Bank of Ireland on 25 February 2016.

As required by the Act, the Central Bank is currently consulting with the Data Protection 64 21 June 2016 Commissioner on the draft regulations which will specify the precise data to be collected and published by the CCR. Pending the outcome of this consultation, the exact timing of the mak- ing of the final regulations remains to be finalised.

I am informed by the Central Bank that following the making of the regulations the take on of data will be implemented on a phased basis, with Phase 1 focusing on lending to con- sumers and Phase 2 focusing on lending to businesses. It is expected that data submissions by lenders will commence six months after the finalisation of regulations but the final deadline for data submission will be influenced by the scale of the technical and operational changes to be implemented by over 500 lenders. Enquiries by lenders against CCR data are expected to commence around twelve months after the finalisation of regulations and once data quality has been assured.

The Central Bank will own the CCR data and it will act as data controller under the Data Protection Acts. Following a public procurement process in 2014, the Central Bank appointed CRIF Ireland Limited as its agent to build and operate the CCR on its behalf. The Bank and CRIF continue to work closely together on the project implementation and ongoing operations of the CCR.

21/06/2016WRG00800Code of Conduct on Mortgage Arrears

21/06/2016WRG00900137. Deputy Pearse Doherty asked the Minister for Finance his plans to place the code of conduct on mortgage arrears on a statutory footing; and if he will make a statement on the matter. [16939/16]

21/06/2016WRG01000Minister for Finance (Deputy Michael Noonan): The Code of Conduct on Mortgage Ar- rears (CCMA) was issued by the Central Bank pursuant to the provisions of Section 117 of the Central Bank Act 1989 and regulated entities are required to comply with the provisions of the Code as a matter of law. The CCMA is therefore issued under statute and, as stated by the Su- preme Court, it forms part of the law. The Central Bank has the power to administer sanctions for a contravention of this Code under Part IIIC of the Central Bank Act 1942.

It is clear that where a borrower actively engages with their lender under the CCMA with a view to agreeing a sustainable arrangement to address their mortgage arrears, it is more likely that an equitable arrangement will be found and that borrower will be able to remain in their family home. Over 120,400 PDH mortgage accounts have been restructured up to the end of Q1-2016.

As the Deputy is aware, the CCMA has already been reviewed and updated over time and the Government will work with the Central Bank to ensure that the code continues to be rel- evant, fair and balanced in respect of the legitimate interests of debtors and creditors all the while promoting the availability of sustainable solutions to address genuine mortgage difficulty.

21/06/2016WRG01100Central Bank of Ireland

21/06/2016WRG01200138. Deputy Pearse Doherty asked the Minister for Finance his plans to amend section 33AK of the Central Bank Act 1942 in line with the banking inquiry’s recommendation; and if he will make a statement on the matter. [16940/16]

21/06/2016WRG01300Minister for Finance (Deputy Michael Noonan): The Report of the Joint Committee of Inquiry into the Banking Crisis (Banking Inquiry) recommended that: “A full review should

65 Questions - Written Answers take place of section 33AK of the Central Bank Act 1942, to ensure that only documents deemed ‘secret’ which are independently reviewed by a High Court judge are withheld from any future Oireachtas inquiry.”

Section 33AK of the Central Bank Act 1942, as amended, deals with the circumstances un- der which the Central Bank can disclose confidential information to third parties.

Section 33AK was most recently amended under the Central Bank (Amendment) Act 2015 (No. 1), to ensure that information could be disclosed to the Banking Inquiry.

My Department and the Central Bank are currently examining all of the recommendations emanating from the Banking Inquiry Report, including the proposal for a further amendment to Section 33AK.

On completion of that examination, I will consider the need to bring forward amending leg- islation to provide for any recommendations arising from the Banking Inquiry Report.

21/06/2016WRG01400Departmental Contracts

21/06/2016WRG01500139. Deputy Pearse Doherty asked the Minister for Finance the amount paid to a firm (details supplied) to date and the expected final costs in respect of work on the Central Bank consolidation Bill; and if he will make a statement on the matter. [16941/16]

21/06/2016WRG01600Minister for Finance (Deputy Michael Noonan): A tender was awarded to the firm in question in November 2014 to assist the Department in the drafting of a Central Bank Consoli- dation Bill.

This is a very complex project that requires comprehensive examination of all of the col- lectively cited Central Bank Acts and that work is still ongoing.

The tender award price is a fixed fee of €100,000 irrespective of the time required to com- plete the project. No monies have been paid to date.

21/06/2016WRG01700Promissory Notes

21/06/2016WRG01800140. Deputy Pearse Doherty asked the Minister for Finance the status of the planned dis- posal of the floating bonds held by the Central Bank related to the promissory notes; his plans to alter this strategy; the discussions he had with the Central Bank and with the European Central Bank on the issue; and if he will make a statement on the matter. [16942/16]

21/06/2016WRG01900Minister for Finance (Deputy Michael Noonan): The Central Bank of Ireland is indepen- dent in the exercise of its functions and the management of its investment holdings are a matter for the bank themselves, neither I nor the Department of Finance have any role in the matter.

The Central Bank of Ireland (“CBI”) indicated a minimum disposal schedule of €0.5 billion up to the end of 2014, €0.5 billion per annum 2015 2018, €1 billion per annum 2019 2023 and €2 billion per annum after that until all the bonds are sold. However, the CBI also stated that it would dispose of the government bonds as soon as possible, provided conditions of financial stability permit. This position remains unchanged. Due to improved financial stability condi- tions, the disposals of fixed and floating rate government bonds from the Special Portfolio have been faster than the minimum. However, any decision to accelerate sales cannot be permanent or predetermined by the CBI as the sales programme must be able to adjust to market conditions

66 21 June 2016 taking into account the views of the NTMA regarding the management of the State’s interest rate risk, the market absorption capacity, the State’s general issuance programme and target maturity profile as well as the financial impact.

21/06/2016WRG02000Central Bank of Ireland Reports

21/06/2016WRG02100141. Deputy Pearse Doherty asked the Minister for Finance if the figures given in the Cen- tral Bank’s published insurance statistics are provided by the industry or if they are audited by the Central Bank before publication; and if he will make a statement on the matter. [16944/16]

21/06/2016WRG02200Minister for Finance (Deputy Michael Noonan): The Central Bank of Ireland has in- formed me that its Insurance Statistics publication contains a summary of the Life Assurance and Non-Life Insurance annual returns as provided by insurers to the Central Bank. In accor- dance with the European Communities (Life Assurance) Framework Regulations 1994 and the European Communities (Non-Life Insurance Accounts) Regulations 1995, the Life Assurance and Non-Life Insurance annual returns are audited by a person appointed by the firm and duly qualified. The insurance returns are reviewed by the Central Bank prior to inclusion in the In- surance Statistics publication.

Separately, the annual Private Motor Insurance Statistics Report is prepared by the Central Bank, which is charged with the responsibility of continuing the statistical work initiated by the Motor Insurance Advisory Board. The primary purpose of the Report is to examine the level of accident frequency and costs and the related impact upon allocated premium differentials by driver profile. The Report is primarily based on the analysis of policy, premium and claim data. The Central Bank also advises that its Private Motor Insurance Statistics Report is based on policy level data submitted by Insurance Ireland to the Statistics Division of the Central Bank on an annual basis.

The submissions are provided annually in a raw disaggregated format, listing individual policy and claims records for all categories of cover, i.e. comprehensive, third party fire and theft and third party only. A total of twenty variables are associated with each individual record. The Central Bank performs a manual quality check of the data received (sense checking, outlier detection etc.) and follows up with Insurance Ireland on a bilateral basis regarding any que- ries. Additionally, the Central Bank also employs the services of an external insurance expert to independently assess the annual findings of the Report.

As I have stated previously in the House, the availability of relevant and timely data is necessary to facilitate an in-depth analysis of the insurance sector and, in particular, the factors driving the increasing cost of insurance.

The issues being examined in my Department’s review of policy in the insurance sector includes the issue of data and will take into account what information is currently available and identify any shortfalls.

21/06/2016WRG02300IBRC Liquidation

21/06/2016WRG02400142. Deputy Mick Wallace asked the Minister for Finance if he will publish the legal opin- ion of a company (details supplied) mentioned in the KPMG progress report on the Irish Bank Resolution Corporation issued to him on 27 May 2016, which suggests that delayed recognition of losses is legal and complies with Irish company law; and if he will make a statement on the matter. [16996/16] 67 Questions - Written Answers

21/06/2016WRG02500Minister for Finance (Deputy Michael Noonan): I am informed that the legal report ref- erenced in the question does not relate to the issues the Deputy raises in the question and so cannot be answered without further clarification. My officials have sought but have not re- ceived such clarification from the Deputy in advance of this response. I am happy to provide the Deputy with an appropriate response following clarification of the question.

21/06/2016WRG02600Tax Code

21/06/2016WRG02700143. Deputy James Browne asked the Minister for Finance the availability of agricultural relief from capital acquisitions tax, where the farming land that is the subject of the relief has an already commissioned solar farm or is wholly or partially the subject of an unexercised option in favour of a solar farm developer or the subject of a burden in the Land Registry prohibiting the alienation or transfer of the landholding without the consent of a solar farm developer; and if he will make a statement on the matter. [17073/16]

21/06/2016WRG02800Minister for Finance (Deputy Michael Noonan): I am advised by the Revenue Com- missioners that gifts and inheritances of agricultural property, including land, qualify for re- lief (known as ‘agricultural relief’) from the payment of Capital Acquisitions Tax (CAT) once certain conditions are satisfied. Section 89 of the Capital Acquisitions Tax Consolidation Act (CATCA) 2003 provides for ‘agricultural relief’. The relief takes the form of a 90% reduction in the taxable market value of the gifted or inherited agricultural property.

The person taking the gift or inheritance (the ‘beneficiary’) of the agricultural property must qualify as a ‘farmer’ for the purpose of section 89 CATCA 2003. This means that a benefi- ciary’s agricultural property must comprise at least 80% by gross market value of the benefi- ciary’s total property at a particular date. The Revenue Commissioners take the view that land on which solar panels are installed is not agricultural property for the purpose of establishing whether or not a beneficiary satisfies this ‘80%’ test. Thus, depending on the amount of an individual’s land that is actually occupied by solar panels (and not merely subject to an option over the land), the use of agricultural land for a solar farm may result in a beneficiary’s failure to satisfy the ‘80%’ test and to qualify for agricultural relief.

A condition for agricultural relief that applies in relation to gifts and inheritances taken on or after 1 January 2015 is that a beneficiary, or a lessee where the beneficiary leases the agri- cultural land, must actually farm the land for a period of at least 6 years after taking the gift or inheritance. It is unlikely that the ability to farm the land would be affected by the granting of an option over the land to a solar farm developer or the registration of a burden over the land with the Land Registry. However, the ability to farm the land would be affected where the op- tion is exercised by the solar farm developer and solar panels are actually installed. As it would not generally be possible to farm any part of the land occupied by a solar farm, the change in the use of land from farming to the generation of solar energy within the required 6-year period would result in a withdrawal of some, or all, of any agricultural relief that had been granted, depending on how much of the land is diverted to this alternative use.

21/06/2016WRG02900Motor Insurance

21/06/2016WRG03000144. Deputy Michael Healy-Rae asked the Minister for Finance his views on the motor insurance crisis (details supplied); and if he will make a statement on the matter. [17080/16]

21/06/2016WRG03100Minister for Finance (Deputy Michael Noonan): As Minister for Finance, I am respon- sible for the development of the legal framework governing financial regulation. Neither I, 68 21 June 2016 nor the Central Bank of Ireland, can interfere in the provision or pricing of insurance products. The EU framework for insurance expressly prohibits Member States from adopting rules which require insurance companies to obtain prior approval of the pricing or terms and conditions of insurance products. The provision of insurance cover and the price at which it is offered is a commercial matter for insurance companies and is based on an assessment of the risks they are willing to accept and adequate provisioning to meet those risks.

The question of the cost of insurance is a complex one involving a number of Government Departments, State Bodies and private sector organisations and while the provision and the pricing of insurance policies is a commercial matter for insurance companies, this does not pre- clude the Government from introducing measures that may, in the longer term, lead to a better claims environment that could facilitate a reduction in claims costs.

My Department has embarked on a review of policy in the insurance sector which is being undertaken in consultation with the Central Bank and other Departments and Agencies. The objective of the Review is to recommend measures to improve the functioning and regulation of the insurance sector.

The first phase of the Review is focussed on the motor insurance compensation framework and this work is nearing completion. The next phase of the Review involves examining the factors contributing to the increasing cost of insurance and aims to identify what short-term measures can be introduced to help reduce the cost of insurance for consumers and businesses. Work on the Review of Policy in the Insurance sector will continue over the coming months.

I would also add that Insurance Ireland operates a free Insurance Information Service for those who have queries, complaints or difficulties in relation to obtaining insurance. In the event that a person is unable to obtain a quotation for motor insurance or feels that the premium proposed or the terms are so excessive that it amounts to a refusal to give them motor insurance, they should contact Insurance Ireland, 5 Harbourmaster Place, IFSC, Dublin 1, Telephone +353 1 6761820 quoting the Declined Cases Agreement.

21/06/2016WRG03200Tax Code

21/06/2016WRG03300145. Deputy Martin Heydon asked the Minister for Finance if he will consider increases to thresholds of inheritance tax to take account of different types of family units, including those who wish to pass on family property to nieces, nephews and other family members, outside of threshold A; and if he will make a statement on the matter. [17158/16]

21/06/2016WRG03400Minister for Finance (Deputy Michael Noonan): As with all tax areas capital acquisitions tax (CAT), including issues such rates of tax, tax-free thresholds, reliefs and exemptions, is kept under review. As part of Budget 2016 I raised the Group A tax-free threshold applying to gifts and inheritance from parents to their children from €225,000 to €280,000. This represented an increase of about 25%. I did this in recognition of the improving state of the public finances and of the concerns expressed to me by people making and receiving gifts and inheritances, particularly in a context of rising property prices.

I indicated at the time that I saw the change to the Group A tax- free threshold in the last Budget as the start of a process. The extent to which increases in the various tax-free thresholds for CAT purposes can be accommodated in the future will depend, among other things, on the continuing economic recovery and the competing choices for limited resources. The Deputy will be aware of the commitment in the Programme for a Partnership Government to work with the Oireachtas to raise the Group A CAT tax-free threshold (including all gifts and inheritances

69 Questions - Written Answers from parents to their children) to €500,000.

21/06/2016WRG03500Tax Credits

21/06/2016WRG03600146. Deputy Martin Heydon asked the Minister for Finance the reason a person (details supplied) on low income is paying a higher proportion of tax; if he will review this case to as- certain additional credits are or will be available in the future; and if he will make a statement on the matter. [17228/16]

21/06/2016WRG03700Minister for Finance (Deputy Michael Noonan): I am advised by Revenue that based on the information provided to them by the person concerned, all appropriate credits have been granted to her.

In the context of an examination by them of the tax records of the person concerned, I am advised by Revenue that a surcharge for late filing of the individual s tax return for 2014 was in- correctly levied. An amended Notice of Assessment and a refund of €297.99 will issue shortly to the person concerned.

21/06/2016WRG03800Tax Code

21/06/2016WRG03900147. Deputy Brendan Griffin asked the Minister for Finance if he has considered reducing capital gains tax on the transfer of assets, including shares to children; and if he will make a statement on the matter. [17242/16]

21/06/2016WRG04000Minister for Finance (Deputy Michael Noonan): Capital gains tax applies on the disposal of assets, including where assets are transferred by way of gift, at the rate of 33% on any charge- able gains arising beyond the annual exempt amount of €1,270.

A transfer of assets can give rise to a gift tax charge in the form of capital acquisitions tax on the beneficiary of the transfer as well as a capital gains tax charge. However, capital gains tax paid in respect of such a transfer can be offset against any gift tax that is due.

Individuals may qualify for a capital gains tax relief known as retirement relief on a transfer of assets, including shares, to children in certain circumstances. In general, the relief applies to a disposal of business or farming assets that have been owned and used for such purposes for a period of 10 years prior to the disposal. In order to qualify for the relief, an individual must be aged 55 or over. The relief also applies to a disposal by an individual of all or part of the shares of a company which is a trading or farming company and the individual’s family company or a member of a trading group of which the holding company is the individual’s family company.

Where the disposal is to a child and the individual is aged between 55 and 65, there is no restriction on the amount of relief that is available. However, where the individual is aged 66 or over, a cap of €3 million applies in respect of the value of the assets on which relief is given.

I have no plans for further reliefs from capital gains tax on transfers of assets beyond those detailed.

21/06/2016WRG04100Financial Services Ombudsman Data

21/06/2016WRG04200148. Deputy Michael McGrath asked the Minister for Finance the number of complaints

70 21 June 2016 the Financial Services Ombudsman received, and the number upheld, regarding the sale of whole-of-life insurance policies in each of the years from 2012 to 2015; if he is aware of wide- spread concern regarding very large premium increases being demanded by insurance firms when whole-of-life policies are reviewed; if he is satisfied that consumers are adequately in- formed of the risks associated with taking out such policies; and if he will make a statement on the matter. [17248/16]

21/06/2016WRG04300Minister for Finance (Deputy Michael Noonan): I am informed by the Financial Services Ombudsman that the number of complaints received by the him in relation to whole-of-life policies in the years requested were as follows (the figures below include both mis-selling and policy review complaints):

Year Number 2011 152 2012 218 2013 193 2014 105 2015 64 Total 732 I am further informed by the Financial Services Ombudsman that no complaints have been upheld regarding the sale of whole-of-life insurance policies in any of the years 2012 to 2015, as all complaints received have been in relation to a sale which occurred more than 6 years prior to the date of complaint, the time limit within which complaints are required to be made.

Whole-of-life policies are plans that are designed to provide consumers with life cover for their whole life. As long as the policy holder makes regular payments and the payments are sufficient to maintain the chosen benefits, this type of cover will pay a lump sum on the death of the policy holder.

The regular payment into the plan covers the cost of providing the benefits chosen on the plan. In the early years the payments are higher than the cost of the policy holder’s benefits. The extra money paid goes into the plan fund. Protection benefits get more expensive as policy holders get older; usually as the plan progresses the payments begin to equal the cost of the chosen benefits. In the later years of reviewable protection plans, the cost of the benefits -in creases significantly. In order to keep the level of benefits at the current level of payments, the difference is made up from the plan fund.

The insurance company carries out regular reviews to see if the policyholder’s regular pay- ment plus any fund that has been built up is enough to cover their chosen benefits for their reviewable protection plan. During a policy review, the insurance company may find that the consumer’s current level of payments is not enough to maintain the level of cover that the con- sumer wants. The insurance company may also find that the current level of payments is not enough to maintain the level of cover desired by the consumer.

The Central Bank of Ireland’s Consumer Protection Code was introduced in 2006 and re- vised in 2012. It requires firms to act honestly fairly and professionally in the best interest of consumers, acts with due care and diligence, and prohibits firms from misleading customers.

The Central Bank has informed me that it expects that when consumers are sold any prod- uct, including unit linked whole of life insurance, that the risks in that product are fully ex- plained. When assessing suitability, a regulated entity must ensure that the product or service is consistent with the consumer’s attitude to risk.

71 Questions - Written Answers

21/06/2016WRG04400Property Tax Administration

21/06/2016WRG04500149. Deputy Alan Farrell asked the Minister for Finance under section 13 of the Finance (Local Property Tax) Act 2012 whereby home owners are responsible for the local property tax for their property for the whole of the subsequent year following 1 November of a given year, even if they sell the property soon after; if he will amend the legislation to modify this to make local property tax payable by persons only for the timeframe for which they have ownership of a given property; and if he will make a statement on the matter. [17253/16]

21/06/2016WRG04600Minister for Finance (Deputy Michael Noonan): I am advised by the Revenue Com- missioners that in accordance with Section 13 of the Finance (Local Property Tax) Act 2012 (as amended), liability for Local Property Tax (LPT) arises where a person owns a residential property on a “liability date”, which for 2014 and subsequent years is 1 November.

A definite liability date is essential to ensure certainty as to the identity of person liable for LPT in respect of a given property for a year.

Section 119 of the Act sets out the dates for payment of the Local Property Tax and clarifies that while the LPT is due by reference to a liability date it is not payable until the later date of 1 January of the following year.

Having a liability date two months prior to the payment date gives liable persons time to make the necessary provisions and to choose from the wide range of options available for pay- ing the tax. In particular, the liable person can put the required arrangements in place to ensure that phased payments, by way of direct debit or deduction at source from employment income, occupational pension or from certain Government payments.

Where an individual sells their residential property after 1 November in any given year, provided that they owned the property on 1 November, they are liable to pay LPT on that prop- erty in respect of that liability date. It is worth noting that if such a person then buys a differ- ent property after the 1 November liability date, s/he will not incur a liability in respect of that property until the next liability date of 1 November in the following year.

I have no plans to change the legislation in this regard at this time.

21/06/2016WRG04700Financial Services Regulation

21/06/2016WRG04800150. Deputy Michael McGrath asked the Minister for Finance if his Department or the Central Bank has a record of the number of commercial loans that have been sold on by the original underwriter; the details of this record; if the Central Bank must be notified when a com- mercial loan has been sold on; if he has concerns that a commercial loan could be sold on to a competitor of the borrowers; and if he will make a statement on the matter. [17265/16]

21/06/2016WRG04900Minister for Finance (Deputy Michael Noonan): I have been informed by the Central Bank that it does not maintain a record of the number of commercial loans sold on by the origi- nal underwriter. Portfolio sales are considered as part of normal supervisory engagement where they are sufficiently material.

However, the Deputy will be aware that legislation and regulations have been implemented by the Oireachtas and Central Bank to protect SMEs when dealing with regulated and unregu- lated firms.

The Consumer Protection (Regulation of Credit Servicing Firms) Act, 2015 was enacted on 72 21 June 2016 8 July 2015. It was introduced to fill the consumer protection gap where loans were sold by the original lender to an unregulated firm. The 2015 Act introduced a regulatory regime for a new type of entity called a ‘credit servicing firm’. Credit Servicing Firms are now subject to the provisions of Irish financial services law that apply to ‘regulated financial service providers’. This ensures that relevant borrowers, whose loans are sold to third parties, maintain the same regulatory protections they had prior to the sale, including under the various statutory codes (such as the Consumer Protection Code, the Code of Conduct on Mortgage Arrears, the Code of Conduct for Business Lending to Small and Medium Enterprises and the Minimum Compe- tency Code) issued by the Central Bank of Ireland.

Under the 2015 Act, therefore, purchasers of loan books must either be regulated by the Central Bank themselves or else the loans must be serviced by a credit servicing firm who is regulated by the Central Bank. Furthermore, it is important to highlight that the transfer of a loan from one entity to another does not change the terms of the contract or the borrower’s rights and obligations under the original contract.

Also, following a review in 2015, the Code of Conduct for Business Lending to Small and Medium Enterprises, has been strengthened in certain areas resulting in the Central Bank (Supervision and Enforcement) Act 2013 (Section 48) (Lending to Small and Medium-Sized Enterprises) Regulations 2015 which comes into operation on 1 July 2016.

21/06/2016WRG05000NAMA Transactions

21/06/2016WRG05100151. Deputy Clare Daly asked the Minister for Finance the steps the National Asset Man- agement Agency is obliged to take should it come to its attention that a person has lobbied it regarding a transaction on behalf of a person with whom the person has a business relationship. [17284/16]

21/06/2016WRG05200152. Deputy Clare Daly asked the Minister for Finance if it is permissible for public rep- resentatives to lobby the National Asset Management Agency on behalf of persons with whom they have a business relationship without disclosing that relationship to the agency. [17285/16]

21/06/2016WRG05300Minister for Finance (Deputy Michael Noonan): I propose to take Questions Nos. 151 and 152 together.

Section 221 of the NAMA Act makes it an offence to lobby NAMA and an offence to not report such lobbying to the Garda.

The section of the Act sets out what is meant by lobbying and contains explicit provisions regarding communication with NAMA in an individual’s professional capacity or in the public interest.

Section 221(5) of the NAMA Act sets out that any person who believes that he or she has been communicated with in contravention of Section 221 must report “that the communication was made, the details of the communication made, and the name of the person who communi- cated with him or her, to a member of the Garda Síochána”. A person who fails to comply with Section 221(5) commits an offence.

In accordance with Section 221, NAMA seeks to facilitate public representatives in rais- ing constituency-related queries directly with NAMA. In this respect NAMA operates a dedi- cated email address, [email protected], through which Deputies and Senators can directly commu- nicate with it in the course of their public duty.

73 Questions - Written Answers NAMA has no way of knowing the nature of any relationship between public representa- tives and their correspondents and, in that context, is obliged to treat queries from public rep- resentatives as bona fide representations. In circumstances where a public representative fails to disclose a relationship to NAMA, or is seeking to influence NAMA commercial decision- making process in order to give preferential treatment to an individual, or individuals, and to the detriment of taxpayers in general, NAMA is obliged to consider that the representative is in breach of Section 221 of the Act.

I am advised by NAMA that the Agency does not believe that any of its engagements with public representatives to date could be considered as a breach of Section 221 of the NAMA Act.

More generally, NAMA is also defined as a “public service body” under Section 7 of the Regulation of Lobbying Act 2015. This Act outlines the requirements for individuals involved in lobbying. Depending on the circumstances, individuals involved in lobbying may need to register on the Register of Lobbying website, and/or provide information to the Standards Com- mission about lobbying activities three times a year. The offences related to relevant contraven- tions are outlined in Section 20 of the Regulation of Lobbying Act.

Further information, including the definition of lobbying, is available from www.lobbying. ie.

21/06/2016WRG05400Tax Code

21/06/2016WRG05500153. Deputy Sean Fleming asked the Minister for Finance if the legislation on close com- pany professional service surcharge (details supplied), section 441 of the Taxes Consolidation Act 1997, distorts competition; if the lack of clarity about what is and is not a professional services company may become damaging to Ireland’s reputation for clear taxation policies and rules; and if he will make a statement on the matter. [17288/16]

21/06/2016WRG05600Minister for Finance (Deputy Michael Noonan): The 12.5% rate of corporation tax ap- plies to the relevant trading profits of all companies regardless of their size and regardless of whether they are close companies or not.

Broadly, a close company is a company that is under the control of five or fewer participa- tors or any number of participators who are directors. A close company could be subject to a close company surcharge of 20% in respect of its undistributed estate and investment income. This would only apply to the extent that it does not distribute that income to its shareholders within 18 months of the end of the accounting period to which it relates.

A close company that is a service company, being a company engaged in professional ser- vices, could also be liable to a close company surcharge of 15% in respect of half of its un- distributed trading income. Again, this would only apply to the extent that this income is not distributed to its shareholders within 18 months of the end of the accounting period to which it relates, and therefore, a surcharge can be avoided entirely by a service company if it distributes income to its shareholders. To allow a close service company to reinvest in its business, the legislation allows such a company to retain up to half of its trading income without triggering a close company surcharge on that income.

The intention behind the surcharge in section 441 of the Taxes Consolidation Act 1997 is not to distort competition but rather to discourage professional persons from channelling their ac- tivities through a controlled company so that income can accumulate without being distributed and, thereby, allowing those professional persons to avoid income tax at higher rates on such income. In the absence of the surcharge, professional persons operating through a corporate 74 21 June 2016 structure may enjoy a tax advantage over self-employed professionals.

For those companies for which control is spread over a larger number of participators, and which do not fall within the definition of a close company, the close company surcharges do not apply. This is because close company legislation operates by reference to objective criteria that are targeted towards circumstances where the income tax base is most likely to be at risk. It is not perceived that the same level of risk exists, that activities have been channelled through a corporate structure to avoid income tax, where a company is not closely held. This is the case for companies of all sizes, whether SMEs or larger firms, which are not closely held.

The service company surcharge applies to closely held companies that are engaged in pro- fessional services. Tax legislation does not define “profession” and the term must therefore be given its ordinary meaning, taking into account general principles of statutory construction and relevant case law. Revenue issued a Tax Briefing in June 2002 (Issue 48) which provides guid- ance on the activities which would fall within the definition of profession and lists a number of professions that are regarded as falling within the ambit of section 441 TCA 1997. The Tax Briefing also lists a number of activities which are not considered to be a profession.

21/06/2016WRG05700School Transport Provision

21/06/2016WRG05800154. Deputy James Lawless asked the Minister for Education and Skills if he will work with Bus Éireann to provide a school bus from Cluain Dara, Derrinturn, County Kildare, to Oakland secondary school in Edenderry, County Offaly; and if he will make a statement on the matter. [16849/16]

21/06/2016WRG05900Minister of State at the Department of Education and Skills (Deputy John Halligan): Under the terms of my Department’s Post Primary School Transport Scheme children are eli- gible for transport where they reside not less than 4.8 kilometres from and are attending their nearest education centre as determined by my Department/Bus Éireann, having regard to ethos and language.

Bus Éireann, which operates the school transport scheme, has advised that there is a school transport service operating into the school in question and a number of children from the Cluain Dara area travelled on this service in the 2015/16 school year.

Preschool Services

21/06/2016WRG06100155. Deputy Clare Daly asked the Minister for Education and Skills further to Parliamen- tary Question No. 301 of 17 May 2016, to expedite an application by a school (details sup- plied) for a naíonra as a matter of urgency, given that it has submitted all the documentation. [16655/16]

21/06/2016WRG06200Minister for Education and Skills (Deputy Richard Bruton): I wish to advise the Deputy that my Department is making arrangements to meet the school in question to discuss the matter to which she refers.

21/06/2016WRG06300Special Educational Needs Service Provision

21/06/2016WRG06400156. Deputy Pat Breen asked the Minister for Education and Skills if he will provide ad- ditional hours to a person (details supplied) under the special needs assistance scheme; and if 75 Questions - Written Answers he will make a statement on the matter. [16665/16]

21/06/2016WRG06500Minister for Education and Skills (Deputy Richard Bruton): I wish to advise the Deputy that the National Council for Special Education (NCSE) is responsible, through its network of Special Needs Organisers (SENOs) for allocating Special Needs Assistants (SNAs) support to mainstream Primary, Post Primary and Special Schools, to assist children with special educa- tional needs who also have additional and significant care needs. The NCSE makes such alloca- tions in accordance with my Departments criteria for the scheme.

Circular 0030/2014, which is available on my Department’s website www.education.ie, sets out my Department’s policy in relation to the Special Needs Assistant (SNA) scheme. The Cir- cular explains that SNA support is not provided to pre-school services which operate outside of the primary or special school provision, other than in early intervention classes in recognised primary schools, or special schools, where support is provided for as part of the schools total quantum of SNA support.

The vast majority of supports for childcare, including pre-school education, are provided by the Department of Children and Youth Affairs. The principal vehicle for the delivery of pre- school education is the free Pre-School Year in Early Childhood Care and Education (ECCE) programme which was introduced in January 2010 and provides for early learning in a formal setting to children in the year before they commence primary school.

Children with disabilities will now have better access to pre-school services under a new Access and Inclusion Model (AIM) programme of supports, which was recently announced, on 15th June 2016, by the Minister for Children and Youth Affairs.

AIM is a child-centred model, involving seven levels of progressive support, moving from the universal to the targeted, depending on the needs of the child and the pre-school.

The supports include: A new Inclusion Charter for the Early Years sector, alongside updated and strengthened Diversity, Equality and Inclusion Guidelines for Early Childhood Care and Education. A new higher education programme for early years practitioners (LINC) which will commence from September 2016. A new national specialist service which is based in the Better Start National Early Years Quality Development Service will provide expert advice, mentoring and support to pre-school providers from a team of 50 specialists in early years care and education for children with disabilities. A new national scheme will provide specialised equipment, appliances and minor alterations which are necessary to support a child’s participa- tion in the ECCE programme. A new national scheme will also provide additional capitation to pre-school providers where this is critical to fund extra support in the classroom and enable a child’s participation in pre-school. It is estimated that only 1 to 1.5% of children in pre-school will require, and therefore be eligible for, this scheme of additional capitation.

Details of the supports which will be available under AIM can be found at www.preschoo- laccess.ie which contains comprehensive information on the access and inclusion model and on how to apply for the new schemes and supports.

Questions relating to provision of such services in preschool settings should be addressed my colleague, the Minister for Children and Youth Affairs.

21/06/2016WRH00200Bus Éireann

21/06/2016WRH00300157. Deputy Michael Fitzmaurice asked the Minister for Education and Skills further to Parliamentary Question No. 102 of 26 May 2016, if he monitors payments to Bus Éireann 76 21 June 2016 under state aid rules; if Bus Éireann is market oriented; and how he defines market oriented. [16668/16]

21/06/2016WRH00400Minister of State at the Department of Education and Skills (Deputy John Halligan): The Deputy will be aware that payments made to Bus Éireann, which operates the School Transport Scheme on behalf of my Department are in line with the Summary of Accounting Arrangements relating to the Transport Scheme for Primary and Post-Primary School Children dated 1 January 1975.

My Department receives an Annual Statement of Account in respect of the School Transport Scheme, which includes an audit opinion provided by independent external auditors.

For 2015 and prior years the opinion states that the financial information of Bus Éireann’s School Transport Scheme is prepared, in all material respects, in accordance with the Summary of Accounting Arrangements relating to the Transport Scheme for Primary and Post-Primary School Children dated 1 January 1975. The auditors have stated that they believe that the au- dit evidence they have obtained is sufficient and appropriate to provide a basis for their audit opinion.

School transport services operated by Bus Éireann are provided using a mix of publicly owned buses and private vehicles supplied by private operators under contract to Bus Éireann. About 90% of the vehicles used to provide school transport services each day under the School Transport Scheme are sourced from private contractors.

21/06/2016WRH00500Student Grant Scheme

21/06/2016WRH00600158. Deputy Billy Kelleher asked the Minister for Education and Skills the outcome of meetings between his Department officials and representatives of the Wake Up SUSI campaign who are seeking grants for low-income students attending Quality and Qualifications Ireland degree courses in recognised private colleges from autumn 2016, as well as access to the Higher Education Authority hardship fund; and if he has taken any decision on their request that will ensure they will be able to continue their studies. [16703/16]

21/06/2016WRH00700Minister for Education and Skills (Deputy Richard Bruton): Officials in my Depart- ment met with representatives from the ‘Wake up SUSI’ campaign on 25th. May. A proposal in relation to extending student supports to students in certain private colleges was subsequently submitted to my Department on 8th. June. This proposal is currently being considered by of- ficials in my Department, in the context of overall budgetary constraints.

An Expert Group chaired by Peter Cassells was established to examine funding arrange- ments for higher education and to present options for developing a sustainable long term fund- ing strategy for the sector. The Expert Group has completed its work and its report will allow for a real and meaningful discussion on the funding requirements of the sector and how those requirements should be met. As signalled in the Programme for Government, it is my intention to publish the report shortly and to refer it to the cross-party Oireachtas Committee for consul- tation as part of the process for formulating a plan for the future of this sector. This will ensure that all views can be heard and considered on the most appropriate way forward for the Irish higher education system.

21/06/2016WRH00800Teacher Redeployment

77 Questions - Written Answers

21/06/2016WRH00900159. Deputy Danny Healy-Rae asked the Minister for Education and Skills the plans he is putting in place to create equal supplementary panel rights for regulation 2 and regulation 3 teachers (details supplied); and if he will make a statement on the matter. [16725/16]

21/06/2016WRH01000Minister for Education and Skills (Deputy Richard Bruton): The core function of the redeployment arrangements is to facilitate the redeployment of all surplus permanent teachers to other schools that have vacancies.

Thereafter, schools are required under the panel arrangements to fill permanent vacancies from supplementary panels comprised of eligible fixed-term (temporary/substitute) and part- time teachers.

The supplementary panel application process for eligible fixed-term (temporary/substitute) and part-time teachers for the 2016/17 school year is set out in Circular 0058/2015.

The Supplementary Special National Panel is in the process of being phased out since the end of the 2012/13 school year and is used only for filling permanent teaching posts in special schools that are not otherwise required for the redeployment of surplus permanent/CID holding teachers. No new applications have been accepted to the Special Supplementary National Panel since 2012.

Arrangements for the Supplementary Panel will be reviewed with the relevant education stakeholders later this year to determine what adjustment, if any, is required for the 2017/18 school year.

21/06/2016WRH01100School Curriculum

21/06/2016WRH01200160. Deputy Jan O’Sullivan asked the Minister for Education and Skills his views on the removal of rule 68 from the Rules for National Schools; the progress he hopes to make in amending the rules to reflect the changes here since they were written; and if he will make a statement on the matter. [16726/16]

21/06/2016WRH01300Minister for Education and Skills (Deputy Richard Bruton): As the Deputy will be aware Rule 68 of the Rules for National Schools was rescinded by Circular 0009/2016 in Janu- ary of this year in accordance with a recommendation in the Report of the Advisory Group to the Forum on Patronage and Pluralism in the Primary Sector.

The Report of the Advisory Group to the Forum on Patronage and Pluralism in the Primary Sector was published in April 2012 and recommended a review and updating of the Rules for National Schools and in particular, Rule 68. The Report acknowledged that many of the Rules are outdated and have been overtaken in whole or in part by legislation and directions in Department circulars. In addition to rescinding Rule 68, Circular 0009/2016 outlined that the Department would proceed to identify other Rules for rescinding particularly where primary legislation deals with the subject matter of the rule in question.

In relation to the education sector generally, the Programme for Government recognises that Government, working in conjunction with the Oireachtas must introduce measures that, inter alia, will modernise our education system and address the demands and pressures that face school leaders. In light of this and in line with Circular 0009/2016, my Department will be examining the issue of identifying further Rules for rescinding.

21/06/2016WRH01400Preschool Services 78 21 June 2016

21/06/2016WRH01500161. Deputy John Brassil asked the Minister for Education and Skills the current policy towards the opening and facilitation of preschools in existing primary schools where there is adequate space to provide a facility and a demonstrated need; and if he will make a statement on the matter. [16737/16]

21/06/2016WRH01600Minister for Education and Skills (Deputy Richard Bruton): The Governance Manual for Primary Schools 2015 - 2019 specifies that the use of school premises during the school day, for purposes other than regular school business, must have the prior approval of the Minister.

Proposals to the Minister to allow use of accommodation during the school day have been facilitated in the past, provided that the school’s current and future accommodation needs are not compromised and the consent of the patron has been received. In general, future accommo- dation needs for the primary school intake will take precedence over the needs of the preschool. A proposal from the school concerned will be considered in that regard.

In facilitating the establishment of preschools in schools or co-located with schools, my Department looks favourably on the establishment of preschool facilities on school sites. How- ever, there are a number of complex issues that need to be considered in respect of such arrange- ments. These issues, including property, governance, insurance and liability, are being consid- ered as part of a review within my Department of the requirements to be established around the use of school premises/property for preschool services and the preparation of guidelines in relation to same.

My Department is seeking legal advice in relation to certain aspects and when this advice is received, it will be considered by officials in my Department with a view to putting arrange- ments in place to facilitate this matter.

My Department is also working closely with the Department of Children and Youth Affairs in relation to the Programme for Government commitment to facilitate the use of school prem- ises outside of the school day for afterschool care and other community purposes.

21/06/2016WRH01700Ministerial Appointments

21/06/2016WRH01800162. Deputy Alan Kelly asked the Minister for Education and Skills if he will appoint a new chair of the Higher Education Authority as a matter of urgency given that the authority has been without a chair since January 2016; and if and when he will fill the vacancies that currently exist on the board of the Higher Education Authority. [16761/16]

21/06/2016WRH01900Minister for Education and Skills (Deputy Richard Bruton): The term of office of the previous Chair of the Higher Education Authority expired at the end of January 2016. The pro- cess for the recruitment of his replacement was undertaken by the Public Appointments Service (PAS). This was done in line with the procedure put in place in September 2014 by the previ- ous Government, which gave the PAS responsibility for putting in place an open, accessible, rigorous and transparent system to support Ministers in making appointments to State Boards.

The PAS process has recently concluded and a short-list of suitable candidates have been put forward for consideration following the process undertaken by the PAS.

A decision in relation appointing a new Chair of the Higher Education Authority will now be taken as soon as practicable.

21/06/2016WRH02000University Governance 79 Questions - Written Answers

21/06/2016WRH02100163. Deputy Alan Kelly asked the Minister for Education and Skills the number of univer- sities that currently provide a residence for their president; if he is aware of any universities in the process of purchasing such properties; if such expenditure is an appropriate use of public funds; and if he will make a statement on the matter. [16762/16]

21/06/2016WRH02200Minister for Education and Skills (Deputy Richard Bruton): At present four Universi- ties provide residences for their Presidents. I am not aware of any University in the process of or planning to purchase such properties. The use of a residence for official purposes was dealt with by the Review Body on Higher Remuneration in the Public Sector, and the provision of residences by Universities is operated in line with the recommendations of that Body.

21/06/2016WRH02300School Transport Provision

21/06/2016WRH02400164. Deputy Dara Calleary asked the Minister for Education and Skills if a school trans- port service will be provided to a primary school (details supplied); if he will ensure that a scheme is put in place given the circumstances of the school and its enrolment; and if he will make a statement on the matter. [16813/16]

21/06/2016WRH02500Minister of State at the Department of Education and Skills (Deputy John Halligan): The Deputy will be aware that a minimum number of 10 eligible children residing in a distinct locality, as determined by Bus Éireann, is required before consideration may be given to the establishment of a school transport service, provided this can be done within reasonable cost limits.

In this regard, Bus Éireann has advised that a sufficient number of children, who are eligible for school transport, have applied for transport for the 2016/17 school year and it is envisaged that a new service will be established provided these families submit ticket payment details to Bus Éireann by the closing date of 29th July 2016.

The families, in question, should liaise with their local Bus Éireann office regarding the timetabling and routing of this proposed new service.

21/06/2016WRH02600Special Educational Needs

21/06/2016WRH02700165. Deputy Billy Kelleher asked the Minister for Education and Skills why he did not put in place proper individual education and care plans and make provision for physiotherapy, oc- cupational therapy, speech and language therapy and so on at a school (details supplied); and if he will make a statement on the matter. [16814/16]

21/06/2016WRH02800166. Deputy Billy Kelleher asked the Minister for Education and Skills why a school (de- tails supplied) that is specifically built for autism has fewer resources available than a standard school; if he is aware of the huge pressure placed on the school to provide the basic resources; and if he will make a statement on the matter. [16815/16]

21/06/2016WRH02900Minister for Education and Skills (Deputy Richard Bruton): I propose to take Questions Nos. 165 and 166 together.

I wish to advise the Deputy that the provision of health therapy supports for children with disabilities, including speech, language, physical and sensory therapy supports are provided by the Health Service Executive (HSE).

The HSE, which has statutory responsibility for the management and delivery of health 80 21 June 2016 and personal social services, is currently engaged in a reconfiguration of existing health re- lated therapy resources for children with complex needs, into multi-disciplinary geographically based teams as part of its National Programme on Progressing Disability Services for Children and Young People (0-18 years).

The aim of this programme is to achieve a national, unified approach to delivering disability health services so that there is a clear pathway to services for all children, regardless of where they live, what school they go to or the nature of their disability.

The education sector, which is working very closely with Health, is fully involved in the development of the Programme, to ensure that, from the children’s and parents’ perspective, the services provided by each sector are integrated, in so far as is possible.

The Deputy will be aware that the Programme for Partnership Government includes a com- mitment to introduce a new in-school speech and language service creating stronger links be- tween parents, teachers and speech and language therapists. This Government has also commit- ted to increase the number of HSE speech and language therapists by some 25%, bringing the number up to 1,102.

Officials of my Department will meet counterparts in the Departments of Health, Children and Youth Affairs and the HSE in the coming weeks to consider the proposals and to commence the development of an implementation plan to give effect to the commitment.

Schools are currently encouraged to use Individual Education Plans (IEP) through policy guidance, support, training and inspection. The DES Inspectorate’s advice is that the majority of schools are now using some form of individual education planning for children with special needs.

DES Circular 30/2014 in relation to the SNA scheme also requires schools to provide per- sonalised pupil plans for new pupils for whom SNA support is being applied for.

As the Deputy is aware each special class for autism at primary level has a 6:1 pupil teacher ratio and also have Special Needs Assistant support normally amounting to 2 SNAs for a class of 6 children.

Scoil Cara has 10 classes with 60 students enrolled for 2016/2017 school year supported by an Administrative Principal, class teachers and SNAs. The NCSE has also allocated additional SNAs to this school to meet the complex needs of the students enrolled. This level of educa- tional support significantly exceeds the resource allocation provision currently available to any mainstream school in the State and is fully consistent with the resource levels provided for other special schools in the State.

I am confident that the proposed development of in-school provision, supported by the pro- posed additional resources and the re-configuration of HSE multi-disciplinary geographically based teams in all regions across the country, will ensure that services for children with dis- abilities will be significantly improved.

21/06/2016WRH03000Torthaí Scrúdaithe

21/06/2016WRH03100167. D’fhiafraigh Deputy Peadar Tóibín den Aire Oideachais agus Scileanna cad iad na torthaí atá ar fáil ó na cineálacha éagsúla scoileanna iar-bhunoideachais sa chóras oideachais idir mheánscoileanna saorálacha, phobalscoileanna, Ghaelcholáistí agus scoileanna cuimsitheacha; agus an ndéanfaidh sé an t-idirdhealú eatarthu a lua. [16822/16]

81 Questions - Written Answers

21/06/2016WRH03200Minister for Education and Skills (Deputy Richard Bruton): Tá sonraí ar na ngnóthach- táil choibhneasta ag mic léinn 15 bliana d’aois in iar-bhunscoileanna deonacha, i scoileanna po- bail agus cuimsitheacha, agus i scoileanna faoi phátrúnacht na mBord Oideachais agus Oiliúna i Litearthacht na Léitheoireachta, sa Mhatamaitic agus san Eolaíocht ar fáil ó na tuarascála- cha náisiúnta a d’eascair mar thoradh ar an anailís ar shonraí a bailíodh mar chuid den Chlár Idirnáisiúnta um Measúnú Daltaí (PISA) agus atá foilsithe ag an bhForas Taighde ar Oideachas (ERC). In 2014, foilsíodh na torthaí ar an staidéar PISA a tharla in 2013 agus tá fáil orthu ar láithreán gréasáin an ERC. Bailíodh na sonraí do PISA 2015 in earrach na bliana 2015 agus foilseofar na sonraí ón staidéar sin i mí na Nollag 2016.

In earrach na bliana 2015 bailíodh sonraí den chineál céanna, a bhaineann leis an ngnóthach- táil choibhneasta sa Mhatamaitic agus san Eolaíocht ag mic léinn dara bliana na scoileanna seo, sa Staidéar ar Threochtaí Idirnáisiúnta ar Mhatamaitic agus ar Eolaíocht (TIMSS 2015) agus táthar ag súil go ndéanfar iad seo a fhoilsiú ag deireadh na bliana 2016. Cruthóidh na sonraí deis comparáide idir ghnóthachtálacha na mac léinn i ngach ceann de na trí chineál iar- bhunscoileanna. Níl fáil i PISA ná i TIMSS ar shonraí ar leith don ghnóthachtáil ag mic léinn i nGaelcholáistí.

21/06/2016WRH03300Schools Building Projects Status

21/06/2016WRH03400168. Deputy John Lahart asked the Minister for Education and Skills the status of the provision of permanent buildings for schools (details supplied); and if he will make a statement on the matter. [16827/16]

21/06/2016WRH03500Minister for Education and Skills (Deputy Richard Bruton): Officials from my Depart- ment have worked closely with officials from South Dublin County Council under the Memo- randum of Understanding for the acquisition of school sites towards procuring a suitable site for this building project. A suitable site has been identified and the conveyancing process in respect of this is under way.

In tandem with the site acquisition process, the design work for the schools has been ad- vanced so the application for planning permission can be submitted as the site acquisition is closed.

Once the site has been acquired and planning permission secured, the project will then be progressed to tender and construction stages.

21/06/2016WRH03600Teacher Training Places

21/06/2016WRH03700169. Deputy Martin Heydon asked the Minister for Education and Skills if he is aware of difficulties being experienced by students of post-primary teaching qualification courses in finding suitable work experience places for 2017 given that it is now a two-year course; the role his Department can play in this issue; and if he will make a statement on the matter. [16875/16]

21/06/2016WRH03800Minister for Education and Skills (Deputy Richard Bruton): The Teaching Council is the statutory body that reviews and accredits programmes of initial teacher education. The school placement experience is integral to all initial teacher education programmes and is gov- erned by the Council’s accreditation criteria, published in June 2011 (Initial Teacher Education: Criteria and Guidelines for Programme Providers) which must be adhered to by all providers of initial teacher education providers.

82 21 June 2016 An extended and enhanced school placement component is a particular feature of all pro- grammes of initial teacher education. The new placement process is based on the development of a partnership approach between Higher Education Institutions and schools. Much progress has been made towards the development of that partnership approach. Based on engagement with the Higher Education Authority, the Teaching Council has identified the need for a forum that includes all HEIs providing programmes of ITE in Ireland, and a clear, time bound commit- ment to agreeing practical measures, including a national IT-based system, that will enhance the school placement experience for all parties to the process, and facilitate access by students to opportunities for same. I understand that the HEA and the Council will meet shortly to progress this matter.

21/06/2016WRH03900School Accommodation

21/06/2016WRH04000170. Deputy Carol Nolan asked the Minister for Education and Skills to consider co-loca- tion as an option to resolve the situation whereby parents in north County Dublin cannot access a school through the medium of the Irish language following the decision to grant patronage of the new school in Drumcondra to Educate Together; and if he will make a statement on the matter. [16915/16]

21/06/2016WRH04100Minister for Education and Skills (Deputy Richard Bruton): As the Deputy will be aware, since 2011 new schools are generally only established in areas of demographic growth. My Department uses a Geographical Information System to identify the areas under increased demographic pressure nationwide. The system uses data from the Central Statistics Office, Ord- nance Survey Ireland, the Department of Social Protection and information from my own De- partment’s databases. With this information, nationwide demographic exercises are carried out to determine where additional school accommodation is needed at primary and post-primary levels.

When it is decided that a new school is required to meet demographic needs in an area, the Department runs a separate patronage process to decide who will operate the school. It is open to all patrons and prospective patrons to apply for patronage of the school under this process and the level of parental preference for each patron is key to decisions in relation to the outcome of the process.

With regard to the decision on the patronage of the new school which will open this Septem- ber to serve the Drumcondra/Marino/Dublin 1 area, my Department assessed the applications received from the prospective patrons, including examining the parental preferences received in the applications to ensure they represented children living within the school planning area which will be served by this school. My Department prepared an assessment report for the consideration of the New Schools Establishment Group (NSEG) which then submitted a report with recommendations to me for consideration and final decision. The patronage assessment report which is published on my Department’s website, while indicating substantial support for an Irish medium school, reflects nearly twice as many valid preferences for an Educate Together school in this area as for a Gaelscoil under the patronage of An Foras Pátrúnachta. I accepted the recommendation of the NSEG and the patronage of the new school remains as announced.

As the Deputy may be aware, there are currently three Gaelscoileanna operating within the school planning area.

The NSEG, in their report to me, noted the strong level of demand from parents for Irish medium education in the area and recommended that this should be kept under review in the context of future demographic exercises. My Department continues to keep the demographic 83 Questions - Written Answers data for the area under ongoing review to take into account updated child benefit data and enrol- ment data and the impact of ongoing and planned capacity increases in this and adjacent school planning areas.

21/06/2016WRH04200Irish Language

21/06/2016WRH04300171. Deputy Carol Nolan asked the Minister for Education and Skills if he will carry out a projection of the future demand among parents for the provision of education through the me- dium of the Irish language; and if he will make a statement on the matter. [16916/16]

21/06/2016WRH04400Minister for Education and Skills (Deputy Richard Bruton): My Department uses a Geographical Information System (GIS) to identify where the pressure for school places will arise nationwide. The GIS uses data from the Central Statistics Office, Ordnance Survey Ire- land, the Department of Social Protection and information from my Department’s own data- bases. With this information my Department carries out demographic analyses to determine where additional school places are required nationwide.

When it is decided that a new school is required to meet demographic needs in an area, my Department runs a separate patronage process to decide who will operate the school. It is open to all patrons and prospective patrons to apply for patronage of the school under this process and the level of parental preference for each patron, including for those providing Irish-medium education, is key to decisions in relation to the outcome of the process.

The criteria used in assessing the patronage applications received from prospective patrons place a particular emphasis on parental demand for plurality and diversity of provision, with parental preferences at the centre of the process. In this regard, prospective patrons must collect and demonstrate evidence of the level of parental demand for their preferred model of provision and medium of instruction, English or Irish, as part of their application under this process.

In this context, I recently announced the patronage of the three new primary schools which will open in September of this year. The outcome of the patronage process is that, Educate Together, a multi-denominational provider, is patron of the two new schools to serve the Ca- bra/Phibsborough/Dublin 7 area and the Drumcondra/Marino/Dublin 1 area and An Foras Pátrúnachta is the patron of the new school to serve the Goatstown/Stillorgan school planning area, providing education through the medium of Irish.

My Department is currently carrying out an analysis of the applications received for the patronage of the nine new post-primary schools which will open in coming years - six in Sep- tember 2017 and three in September 2018. In line with the arrangements referred to above, pro- spective patrons were required to submit with their application a parental preference template in which parents have been requested to state their preference for their child(ren) to be educated through the medium of English or Irish. On completion of this assessment, my Department will draft a report for the consideration of the New Schools Establishment Group who will submit a report with recommendations to me for consideration and final decision.

21/06/2016WRH04500Departmental Funding

21/06/2016WRH04600172. Deputy Carol Nolan asked the Minister for Education and Skills if he provides any type of grant towards the costs of courses in the Gaeltacht for students who attend them; and if he will make a statement on the matter. [16917/16]

84 21 June 2016

21/06/2016WRH04700Minister for Education and Skills (Deputy Richard Bruton): My Department does not provide any type of grant towards the costs of courses in the Gaeltacht for primary or post- primary students. My Department does provide funding towards the cost of Campaí Samhraidh which are summer courses in Irish for primary school children in designated disadvantaged schools.

21/06/2016WRH04800Special Educational Needs Service Provision

21/06/2016WRH04900173. Deputy Carol Nolan asked the Minister for Education and Skills the number of schools that currently have no special needs unit; and if he will make a statement on the matter. [16918/16]

21/06/2016WRH05000Minister for Education and Skills (Deputy Richard Bruton): I wish to advise the Deputy that my Department’s policies focus on ensuring that all children can have access to an educa- tion appropriate to their needs, preferably in mainstream school settings through the primary and post-primary school network.

Section 2 of the Education of Persons with Special Educational Needs Act provides that a child with special educational needs shall be educated in an inclusive environment with chil- dren who do not have such needs, unless the nature or degree of the needs of the child is such that it would be inconsistent with the best interests of the child, or children with whom the child is to be educated.

Accordingly, most children with special educational needs will attend mainstream schools with additional resources provided to assist the child in that setting.

For pupils who have needs which require more specialist interventions, my Department provides for special classes and special school placements.

The National Council for Special Education (NCSE), through its network of local Special Educational Needs Organisers (SENOs), is responsible for processing applications from pri- mary and post-primary schools for special educational needs supports, including the establish- ment of special classes in various geographical areas as required. The NCSE operates within my Department’s criteria in allocating such support.

The NCSE recently published a list of special classes for the 2016/17 school year, details of which are available at www.ncse.ie.

In total there will be 1153 special classes available next year, which is an increase of almost 100% in the number of special classes which were available in 2011, which was 548.

There will be 842 special classes in 421 primary schools from September 2016, which means that approximately 13% of primary schools now have one, or more, special classes.

There will be 311 special classes in 193 post-primary schools from September 2016, which means that approximately 26% of post-primary schools now have one, or more, special classes.

The NCSE continues to monitor and review the requirement for special class places in par- ticular areas and has capacity to establish such new special classes where necessary subject to the willingness of schools to open classes.

Full details of all special classes are available at www.ncse.ie.

85 Questions - Written Answers

21/06/2016WRH05100Pupil-Teacher Ratio

21/06/2016WRH05200174. Deputy Carol Nolan asked the Minister for Education and Skills the number of chil- dren currently being taught in classes that have fewer than 25 pupils, have between 25 and 28 pupils, and have over 28 pupils, by county, in tabular form; and if he will make a statement on the matter. [16919/16]

21/06/2016WRH05300Minister for Education and Skills (Deputy Richard Bruton): Provisional results from the 2015/2016 National School Annual Census showing the information requested by the Dep- uty are shown in the following table. Final figures will be published at the end of June. Further details on individual school class size are available on my Department’s website at www.educa- tion.ie/en/Publications/Statistics/Data-on-Individual-Schools/. Due to the moveable nature of classes at post-primary level depending on subject choices, class size data are not available at post-primary level.

Provisional Data - Number of Pupils In Mainstream Chasses classified by class size and County, 2015/2016 National School Annual Census

- 0-25 25-28 OVER 28 OVER- ALL TOTALS COUNTY PUPILS CLASS- PUPILS CLASS- PUPILS CLASS- PUPILS CLASS- ES ES ES ES Dublin 40,877 2038 46,809 1750 46,602 1544 134,288 5332 Cork 21,438 1071 18,508 698 21,139 681 61,085 2450 Galway 12,234 639 8,335 315 8,994 289 29,563 1243 Kildare 6,830 317 12,079 454 10,125 334 29,034 1105 Meath 5,402 255 9,778 367 11,140 364 26,320 986 Limerick 7,272 370 6,939 261 8,031 259 22,242 890 Donegal 7,262 385 5,584 211 6,010 195 18,856 791 Tipperary 7,392 371 5,818 220 5,198 169 18,408 760 Wexford 5,872 283 6,276 237 6,218 198 18,366 718 Wicklow 4,411 213 6,879 258 6,180 201 17,470 672 Louth 5,504 268 5,308 200 5,938 192 16,750 660 Kerry 6,496 335 4,671 176 4,698 152 15,865 663 Mayo 6,466 357 3,960 150 3,892 125 14,318 632 Waterford 4,095 199 4,926 184 5,116 168 14,137 551 Clare 5,721 300 3,778 144 4,101 131 13,600 575 Westmeath 3,755 188 3,371 126 3,920 130 11,046 444 Kilkenny 3,323 164 3,753 141 3,800 122 10,876 427 Laois 3,631 178 3,802 143 3,262 106 10,695 427 Offaly 4,322 212 3,053 115 2,460 79 9,835 406 Cavan 4,002 198 3,333 125 2,234 73 9,569 396 Monaghan 2,832 140 2,265 86 2,464 79 7,561 305 Carlow 1,758 84 2,882 109 2,881 94 7,521 287 Roscommon 3,158 176 2,186 83 2,056 67 7,400 326 Sligo 3,109 163 1,835 69 2,420 78 7,364 310 Longford 2,150 108 1,417 54 1,573 50 5,140 212 Leitrim 1,563 81 1,250 48 833 27 3,646 156 Grand Total 180875 9093 178795 6724 181285 5907 540955 21724

86 21 June 2016

21/06/2016WRH05400EU Funding

21/06/2016WRH05500175. Deputy Carol Nolan asked the Minister for Education and Skills the closing date for the submission of final claims and documentation for the 2007-2013 Human Capital Invest- ment Operational Programme; if he will draw down all of the moneys available under this programme; and if he will make a statement on the matter. [16920/16]

21/06/2016WRH05600Minister for Education and Skills (Deputy Richard Bruton): The closing date for the submission to the European Commission of the final claim for European Social Fund (ESF) funding under the 2007-2013 Human Capital Investment Operational Programme is 31 March 2017. It is expected that Ireland’s allocation of just over €375 million of ESF funding under this programme will be drawn down in full.

21/06/2016WRH05700Literacy Programmes

21/06/2016WRH05800176. Deputy Carol Nolan asked the Minister for Education and Skills the amount of money he has allocated for each of the years 2006 to 2016 to date for the provision of adult literacy services; and if he will make a statement on the matter. [16921/16]

21/06/2016WRH05900Minister of State at the Department of Education and Skills (Deputy John Halligan): Some €23 million in funding was provided by my Department in 2006 for the provision of adult literacy services. Funding in the region of €30 million was provided by my Department for each of the years 2007 to 2013. At the beginning of 2014 SOLAS took over the allocation of monies for all further education and training programmes, including the literacy service, with funding provided by my Department. SOLAS provided funding in the region of €30 million for the provision of adult literacy services for each of the years 2014 and 2015.

It is expected that this level of funding will be maintained for the current year.

21/06/2016WRH06000Student Assistance Fund

21/06/2016WRH06100177. Deputy Carol Nolan asked the Minister for Education and Skills the amount of money he has allocated for each of the years 2006 to 2016 to date under the student assistance fund; and if he will make a statement on the matter. [16922/16]

21/06/2016WRH06200Minister for Education and Skills (Deputy Richard Bruton): The information requested by the Deputy is contained in the table.

Academic Year Student Assistance Fund 2006-07 6.0m 2007-08 6.2m 2008-09 5.0m 2009-10 5.0m 2010-11 5.0m 2011-12 9.0m 2012-13 11.0m 2013-14 8.35m 2014-15 6.6m 2015-16 6.34m 2016-17 *** 7.99m ***Provisional - funding not allocated to individual Institutions until the 2016 academic 87 Questions - Written Answers year.

21/06/2016WRH06300Fund for Students with Disabilities

21/06/2016WRH06400178. Deputy Carol Nolan asked the Minister for Education and Skills the amount of money he has allocated for each of the years 2006 to 2016 to date under the fund for students with dis- abilities; and if he will make a statement on the matter. [16923/16]

21/06/2016WRH06500Minister for Education and Skills (Deputy Richard Bruton): The information requested by the Deputy is provided in the following table.

Academic Year Fund for Students with Disabilities 2006-07 9.9m 2007-08 13.5m 2008-09 11.7m 2009-10 12.0m 2010-11 13.7m 2011-12 10.8m 2012-13 10.6m 2013-14 10.3m 2014-15 10.4m 2015-16 10.4m 2016-17 *** 10.3m *** Provisional - funding not allocated to individual institutions until into the 2016 aca- demic year.

21/06/2016WRJ00200School Patronage

21/06/2016WRJ00300179. Deputy Carol Nolan asked the Minister for Education and Skills if he will carry out an independent review of the operation of the community national school model; and if he will make a statement on the matter. [16924/16]

21/06/2016WRJ00400Minister for Education and Skills (Deputy Richard Bruton): In 2008, a new model of primary school provision was established on a pilot basis. This pilot was in response to increased demand for parental choice in the patronage of primary schools. The Community National School (CNS) model was initially piloted in two schools and has since expanded to a total of eleven schools. CNSs are multi-denominational schools and cater for the diversity present within the community in a single school setting, rather than in more than one school. The operation of Community National Schools, like all other recognised schools, is subject to ongoing inspection and evaluation. My Department’s school inspectorate inspects the schools on an ongoing basis and this has included a Whole School Evaluation in certain cases. The findings of such Whole School Evaluations are published on my Department’s website. In ad- dition, the schools all engage in school self-evaluation, the findings of which are published on school websites.

21/06/2016WRJ00500Gaeltacht Policy

21/06/2016WRJ00600180. Deputy Carol Nolan asked the Minister for Education and Skills when he will finalise

88 21 June 2016 and publish the Gaeltacht education policy; and if he will make a statement on the matter. [16925/16]

21/06/2016WRJ00700Minister for Education and Skills (Deputy Richard Bruton): The drafting of a Policy on Education in the Gaeltacht is at an advanced stage. This is a key response by my Department to the Government’s commitment to implement the Twenty Year Strategy for the Irish Language 2010 – 2030. I will consider the Policy and its recommendations and will present it to my col- leagues in Government for approval. I expect that the report will be published before the end of the year.

21/06/2016WRJ00800Youth Employment Initiative

21/06/2016WRJ00900181. Deputy Carol Nolan asked the Minister for Education and Skills why his Department had to refund €19.8 million under the programme for employability, inclusion and learning; the impact of this on the delivery of the programme; and if he will make a statement on the matter. [16926/16]

21/06/2016WRJ01000Minister for Education and Skills (Deputy Richard Bruton): The EU co-financed Youth Employment Initiative (YEI), which aims to tackle youth unemployment and implement the Youth Guarantee, is integrated into European Social Fund programming and is being delivered in Ireland as a dedicated priority axis within the ESF Programme for Education, Inclusion and Learning (PEIL) 2014-2020. The specific YEI funding allocation for Ireland of €68m is matched by equal amounts from our ESF allocation and from the Exchequer, giving an overall allocation of €204m. The rules governing the ESF provide for initial pre-financing of EU sup- port at 1% (or 1.5% for Member States under financial assistance) of the full programme sup- port, payable for each year from 2014 to 2016. However, in order to mobilise faster YEI actions on the ground, the initial pre-financing of the YEI EU support was exceptionally increased to 30% in 2015. Accordingly Ireland received additional YEI pre-financing of €19.8m. The Reg- ulations providing for this exceptional increase also specified that where a Member State did not submit an interim payment application by 23 May 2016 for at least 50% of the additional pre-financing amount, that the Member State must reimburse the Commission the total YEI pre- financing contribution. As work on the designation of the relevant ESF authorities, including the provision of a national computerised accounting and information system for EU funds to meet the 2014-2020 functionality requirements is continuing, Ireland did not comply with the requirement to submit an interim claim by the due date. In accordance with the relevant Regu- lations, the €19.8m additional pre-financing received and currently held in a Suspense Account, is to be reimbursed to the Commission. My Department is liaising with the Commission in this regard. However, it should be noted that the reimbursement of this pre-financing will not result in any loss of EU monies to the Exchequer, as the full YEI allocation of €68m to Ireland will still be available for drawdown before year-end 2018, and it is expected that this funding will be fully drawn down. Furthermore, as the activities concerned are fully funded up-front by the Exchequer there is no resulting reduction in the funding available to those activities.

Youth Employment Initiative Funding (€m) Back to Work Enterprise Allowance (BTWEA) 4.648 Scheme JobsPlus Incentive Scheme 7.992 Tús 35.347 JobBridge, the National Internship Scheme 57.240

89 Questions - Written Answers Youth Employment Initiative Funding (€m) Youthreach 79.876 Momentum 13.333 Social Inclusion and Activation Programme (SICAP) 6.000 TOTAL 204.436

21/06/2016WRJ01100Education Policy

21/06/2016WRJ01200182. Deputy Carol Nolan asked the Minister for Education and Skills when the science, technology, engineering and maths report will be published and referred to the Oireachtas com- mittee; and if he will make a statement on the matter. [16927/16]

21/06/2016WRJ01300Minister for Education and Skills (Deputy Richard Bruton): I am awaiting the final re- port for the STEM Education Review Group. When I receive the final report, I will request the relevant Oireachtas Committee to review the recommendations contained in the report in line with the commitment given in the Programme for Government.

21/06/2016WRJ01400Access to Higher Education

21/06/2016WRJ01500183. Deputy Carol Nolan asked the Minister for Education and Skills if the independent examination of the supports and barriers to accessing higher education for lone parents, as re- ferred to in the programme for Government, has commenced; if he will engage in a wide con- sultation process with relevant stakeholders during this review; and if he will make a statement on the matter. [16928/16]

21/06/2016WRJ01600Minister for Education and Skills (Deputy Richard Bruton): Officials in my Depart- ment have consulted with relevant officials in the Departments of Social Protection and Chil- dren and Youth Affairs about this commitment in the Programme for Government. A process has been initiated to engage an independent expert to undertake the review. I expect that con- sultation with relevant stakeholders will inform the review process. The review is expected to be completed in the autumn.

21/06/2016WRJ01700Delivering Equality of Opportunity in Schools Scheme

21/06/2016WRJ01800184. Deputy Carol Nolan asked the Minister for Education and Skills the amount that has been allocated over each of the years 2006 to 2016 to date to the delivering equality of oppor- tunity in schools (DEIS) programme and the number of schools that have qualified for DEIS status; and if he will make a statement on the matter. [16929/16]

21/06/2016WRJ01900Minister for Education and Skills (Deputy Richard Bruton): The information requested by the Deputy is not readily available. I have asked my officials to compile the information and forward it to the Deputy as soon as possible.

21/06/2016WRJ02000Third Level Staff

21/06/2016WRJ02100185. Deputy Carol Nolan asked the Minister for Education and Skills if he will imple- ment the recommendations of the Cush report; and if he will make a statement on the matter. 90 21 June 2016 [16930/16]

21/06/2016WRJ02200Minister for Education and Skills (Deputy Richard Bruton): The Report of the Expert Group on Fixed Term and Part Time Employment in Lecturing contains a number of recom- mendations that will assist in addressing the concerns raised about precarious employment by Union representatives. My officials are currently taking steps to begin implementation of the recommendations to academic staff who have signed up to the Lansdowne Road Agreement.

21/06/2016WRJ02300Technological Universities

21/06/2016WRJ02400186. Deputy Carol Nolan asked the Minister for Education and Skills if he remains of the view that the merger of institutes of technology is a pre-requisite for the application for, as op- posed to the granting of, technological university status; and if he will make a statement on the matter. [16931/16]

21/06/2016WRJ02500Minister for Education and Skills (Deputy Richard Bruton): As the Deputy will be aware, the new Programme for Government outlines that this new Government will continue to support the creation of Technological Universities. This is in line with the National Strategy for Higher Education to 2030 which provides a framework for the development of the higher education sector to 2030. With regard to the institute of technology sector, the Strategy recom- mended significant reforms to position the sector to meet national strategic objectives. In par- ticular, the Strategy recommended consolidation within the sector and a pathway of evolution for those consolidated institutes of technology, to allow them to demonstrate significant progress against robust performance criteria and to apply to become technological universities. Mission of Technological Universities: The intention has always been that a Technological University would be distinctly different from traditional universities and institutes of technology by virtue of their mission relating to graduate formation, applied research and scholarship, dissemination of knowledge to meet the needs of society and enterprise with a very strong regional focus. However, the mission of a university in Ireland as set out in the 1997 Universities Act is broad and comprehensive and as such it is not surprising that there may be some evidence of some ele- ments of overlap between a University and an Institute of Technology or Technological Univer- sity. This differentiated mission for a Technological University includes a systematic focus on the preparation of graduates for complex professional roles in a changing technological world, the advancement of knowledge through applied research and scholarship and the dissemination of this knowledge to meet the needs of society and enterprise, and the particular contribution the university will make to the needs of the region in which it is located. There is no point in mak- ing the TU project a simple renaming exercise and falling into traps seen in similar processes in other countries. The institutions concerned are required to achieve high standards across a range of areas before being designated as technological universities. These include standards relating to the qualifications of staff, the quality of research output, the proportion of students engaged in lifelong learning, and other relevant issues. The following is a list of some of the key features expected of a Technological University: the range of teaching provision (levels 6-10), with the associated scope for progression via appropriate pathways; the strength of provision for work based, and lifelong, learning; the weight of postgraduate research activity relative to that in In- stitutes of Technology; the key role of experienced practitioners in teaching and research; very close employer links with involvement in curriculum design, teaching and supervision; a strong vocational/professional orientation; a strong regional as well as a wider focus; highly developed responsiveness and flexibility in delivery, with demand (via feedback from employers) as a key driver; and an approach to research that entails building strength in focused areas in close partnership with users. The mission and vision for each of the individual proposed technologi- cal universities is developed by the consortia, in consultation with stakeholders including staff 91 Questions - Written Answers and students, and is contained in the implementation plans subsequently developed. Regional Focus: There will also be a specific focus on the particular contribution the university will make to the needs of the region in which it is located. The development of technological universities has the potential to deliver greater opportunity to students in these regions, to staff working in the institutions, and to the broader local economy and society. Position of Technological Uni- versities Bill: The first Government Legislative Programme of this new Partnership Govern- ment was published on 8 June 2016, and the Technological Universities Bill has been restored to the Dáil Order Paper at Committee Stage. I recognise that there were a significant number of matters raised in respect of the Bill at both Committee and Report Stage. It is my intention to now consult with all of the relevant stakeholders in relation to both the matters raised dur- ing the legislative process and the commitments contained in the Programme for Government. Following the finalisation of this consultation process I will then advance the legislation having determined a position in relation to any matters raised as part of this consultation process.

21/06/2016WRJ02600National Educational Psychological Service Data

21/06/2016WRJ02700187. Deputy Carol Nolan asked the Minister for Education and Skills the number of pupils currently awaiting assessment by a National Educational Psychological Service psychologist, by county; the number that have been waiting less than 13 weeks, over 13 weeks, for over six months or longer; and if he will make a statement on the matter. [16932/16]

21/06/2016WRJ02800Minister for Education and Skills (Deputy Richard Bruton): The Deputy will be aware from my previous responses to her in this regard that my Department’s National Educational Psychological Service (NEPS) provides educational psychology service to all primary and post primary schools through an assigned NEPS psychologist and in some cases through the Scheme for Commissioning Psychological Assessments (SCPA).

NEPS does not maintain waiting lists but, in common with many other psychological ser- vices and best international practice, NEPS has adopted a consultative model of service. The focus is on empowering teachers to intervene effectively with pupils whose needs range from mild to severe and transient to enduring. Psychologists use a problem solving and solution ori- ented consultative approach to maximise positive outcomes for these pupils. NEPS encourages schools to use a continuum based assessment and intervention process whereby each school takes responsibility for initial assessment, educational planning and remedial intervention for pupils with learning, emotional or behavioural difficulties. Teachers may consult their NEPS psychologist should they need to at this stage in the process. Only in the event of a failure to make reasonable progress, in spite of the school’s best efforts in consultation with NEPS, will the psychologist become involved with an individual child for intensive intervention or assess- ment.

This system allows psychologists to give early attention to urgent cases and also to help many more children indirectly than could be seen individually. It also ensures that children are not referred unnecessarily for psychological intervention.

I would advise if there are concerns in relation to the educational development of any stu- dent that these should be raised, in the first instance, with the Principal of the school he/she is attending, with a view to the Principal discussing the situation with the assigned NEPS psy- chologist. In this regard where it is agreed that an assessment is appropriate it is conducted within an average of six weeks.

21/06/2016WRJ02900Schools Building Projects Applications 92 21 June 2016

21/06/2016WRJ03000188. Deputy Carol Nolan asked the Minister for Education and Skills the reason for the delay in granting permission for a new walkway between a school (details supplied) and the local shopping centre; and if he will make a statement on the matter. [16933/16]

21/06/2016WRJ03100Minister for Education and Skills (Deputy Richard Bruton): I can confirm that my Department received a revised proposal for funding towards the proposed walkway from the school to which the Deputy refers.

This proposal was considered and my Department wrote to the school requesting further information in March 2016. This information has not yet been received.

21/06/2016WRJ03200Summer Works Scheme Applications

21/06/2016WRJ03300189. Deputy Carol Nolan asked the Minister for Education and Skills the status of an ap- plication for the summer works scheme by a school (details supplied); and if he will make a statement on the matter. [16934/16]

21/06/2016WRJ03400Minister for Education and Skills (Deputy Richard Bruton): I wish to confirm to the Deputy that in the case of the school, referred to, an application was received under Category 6 of the Summer Works Scheme (SWS) 2016-17. Valid SWS (2016-2017) applications from schools in respect of Categories 3 to 10 that were not reached under round one approvals an- nounced on 28th April last, will, subject to the overall availability of funding, qualify to be as- sessed under future rounds of the Scheme. If this arises, the terms and conditions of the scheme as outlined in Circular Letter (0055/2015) which may be accessed on my Department’s website, will continue to apply when allocating funding to such projects. The SWS (2016-2017) applica- tion from the school, in question, is available to be considered in this context.

21/06/2016WRJ03500SOLAS Training and Education Programmes

21/06/2016WRJ03600190. Deputy Michael Healy-Rae asked the Minister for Education and Skills his views on a matter (details supplied) regarding the construction skills certification scheme, quarry skills certification scheme and safe pass courses. [16963/16]

21/06/2016WRJ03700Minister for Education and Skills (Deputy Richard Bruton): The functions of SOLAS are set out in section 7 of the Further Education and Training Act 2013 and include the provi- sion of training as well as the development of new and existing further education and training programmes. The role of SOLAS in relation to the CSCS and Safe Pass derives from the Safety, Health and Welfare at Work (Construction) Regulations 2013. The regulations provide that FÁS is responsible for the issue of valid construction skills registration cards under the provisions of Schedule 5 of the Regulations. Schedule 4 of the regulations require the success- ful completion of either the FÁS Safe Pass training programme, or an equivalent safety aware- ness scheme approved by FÁS for the issue of a valid safety awareness registration card. The role of SOLAS in relation to the QSCS derives from the Safety, Health and Welfare at Work (Quarries) Regulations, 2008. It provides that FÁS is responsible for issue of the quarries skills registration card under the provisions of Schedule 1 of the Regulations. SOLAS was estab- lished in October 2013 under the Further Education and Training Act, 2013. Section 38 of the Act provides that all functions that were vested in FÁS are transferred to SOLAS including any references to FÁS in any enactment or instrument.

21/06/2016WRJ03800SOLAS Training and Education Programmes 93 Questions - Written Answers

21/06/2016WRJ03900191. Deputy Michael Healy-Rae asked the Minister for Education and Skills his views on a matter (details supplied) regarding construction skills certification scheme, quarries skills certi- fication scheme and safe pass courses; and if he will make a statement on the matter. [16966/16]

21/06/2016WRJ04000Minister for Education and Skills (Deputy Richard Bruton): FÁS previously had re- sponsibility for administering the Construction Skills Certification Scheme (CSCS). In 2012 FÁS commissioned a report to consider proposed changes to a number of programmes for which it had responsibility including the CSCS. This included an examination of the potential transfer of the CSCS to another state agency. Amongst a number of conclusions, this report recommended that no imminent transfer of the CSCS should be undertaken. As a result, FÁS continued in its role and the Further Education and Training Awarding Council (FETAC) re- tained its role as the awarding body for the CSCS. Quality and Qualifications Ireland (QQI) was established in November 2012 and many of the functions of FETAC were assumed by QQI, including its role in relation to the CSCS. The Common Awards System (CAS) was introduced by FETAC for further education and training awards at levels 1 to 6 of the National Framework of Qualifications and is now maintained by QQI. Under the 2012 Qualifications and Quality Assurance (Education and Training) Act QQI is empowered to issue a wide range of awards both inside and outside of the CAS. SOLAS was established in October 2013 under the Further Education and Training Act 2013. Section 38 of that Act provides that all functions that were vested in FÁS are now transferred to SOLAS. The role of SOLAS in relation to the CSCS and Safe Pass derives from the Safety, Health and Welfare at Work (Construction) Regulations 2013. The regulations provide that FÁS (now SOLAS) is responsible for issuing valid construc- tion skills registration cards under the provisions of Schedule 5 of the Regulations. Schedule 4 of the regulations require the successful completion of either the FÁS Safe Pass training programme, or an equivalent safety awareness scheme approved by FÁS (now SOLAS), for the issuance of a valid safety awareness registration card. The role of SOLAS in relation to the Quarries Skills Certification Scheme (QSCS) derives from the Safety, Health and Welfare at Work (Quarries) Regulations, 2008. It provides that FÁS (now SOLAS) is responsible for issu- ing the Quarries Skills Registration card under the provisions of Schedule 1 of the Regulations.

21/06/2016WRJ04100Special Educational Needs Data

21/06/2016WRJ04200192. Deputy Thomas Byrne asked the Minister for Education and Skills the number of stu- dents with special needs in mainstream classes at primary and post-primary level in the school years 2014/2015 and 2015/2016, in tabular form. [16994/16]

21/06/2016WRJ04300Minister for Education and Skills (Deputy Richard Bruton): The information requested is presented in the following table.

Number of Students accessing additional learning supports in Primary and Post Pri- mary Mainstream Classes

- Primary Post Primary Total GAM* LITH HITH* LITH 2014/15 110,813 25,647 10,623 12,767 159,850 2015/16 113,888 28,714 10,835 14,217 167,654 *Estimated.

21/06/2016WRJ04400Third Level Fees

94 21 June 2016

21/06/2016WRJ04500193. Deputy Jim O’Callaghan asked the Minister for Education and Skills if third level institutions in receipt of considerable State funding, should be allowed to discriminate against Irish citizens, in the matter of postgraduate fees (details supplied). [17004/16]

21/06/2016WRJ04600Minister for Education and Skills (Deputy Richard Bruton): The position is that higher education universities are autonomous institutions as provided for in the Universities Act 1997 and therefore the criteria governing the level of tuition fees to be charged in the case of post- graduate study is solely a matter for the relevant institution to determine in line with its own criteria. Accordingly, my Department has no function in relation to such matters. Tax relief is available on postgraduate tuition fees paid. Details in relation to this relief are available from the Revenue Commissioners at www.revenue.ie.

21/06/2016WRJ04700Schools Building Projects Applications

21/06/2016WRJ04800194. Deputy Seán Haughey asked the Minister for Education and Skills if he will sanction a building project at a school (details supplied) to enable it to open another class for children with autism; and if he will make a statement on the matter. [17013/16]

21/06/2016WRJ04900Minister for Education and Skills (Deputy Richard Bruton): I can confirm that the school referred to by the Deputy has submitted an application for additional accommodation to establish a second class for pupils with autism. The application is currently being consid- ered and a decision will be conveyed to the school authority as soon as this process has been finalised.

21/06/2016WRJ05000Special Educational Needs Data

21/06/2016WRJ05100195. Deputy Thomas Byrne asked the Minister for Education and Skills the number of ap- plications from schools he received for special educational needs posts in 2016 and the number of applications by county. [17019/16]

21/06/2016WRJ05200Minister for Education and Skills (Deputy Richard Bruton): I wish to advise the Deputy that the National Council for Special Education (NCSE) is responsible, through its network of local Special Educational Needs Organisers (SENOs), for allocating special educational needs supports, including Special Needs Assistants (SNAs) and Resource Teaching support to schools to support children with special educational needs. The NCSE operates within my Department’s criteria in allocating such support. All schools were asked to submit applications for SNA and Resource Teaching support for the 2016/17 school year to the NCSE by 29th February this year. This year, 12,900 SNAs are available for allocation to schools, which is an increase of almost 22% in the number of posts available since 2011, which was 10,575 posts. The NCSE advised all schools of their allocations for SNA support for the coming 2016/17 school year, on 14th June, 2016, based on the number of valid applications received to date. The NCSE has now allocated 12,635 Whole Time Equivalent SNA posts to schools, which is 711 posts additional to 11,924 SNA posts which were allocated at the end of the 2015/16 school year. A total of 7452 resource teaching posts is also available to the NCSE to allocate to schools for the coming 2016/17 school year, which is an increase of over 600 posts on the current years allocation, and an increase of 41% over the numbers allocated since 2011, which was 5265 posts. The NCSE advised all schools, on 28th April, 2016, of their allocations for resource teaching support for September 2016. 7015 posts were allocated to schools at that time. Details of all of the alloca- tions which have been made to schools for SNA and resource teaching support, on a per county and per school basis, have been published on www.ncse.ie.

95 Questions - Written Answers The NCSE continues to accept applications in recognition that enrolments may not have been completed or where assessments were not completed. The NCSE will consider these ap- plications and make further allocations to schools in respect of valid applications which have been received to September. As the Deputy’s question relates to the number of invalid applica- tions received, additional to the allocations made, for resource hours and SNA support, I have referred this question to the NCSE for their consideration and direct reply to the Deputy.

21/06/2016WRJ05300Special Educational Needs Service Provision

21/06/2016WRJ05400196. Deputy Michael Healy-Rae asked the Minister for Education and Skills his views on whether the July provision should be provided for children with special needs (details sup- plied); and if he will make a statement on the matter. [17020/16]

21/06/2016WRJ05500Minister for Education and Skills (Deputy Richard Bruton): My Department currently provides July Provision for children with a diagnosis of ASD and/or severe or profound general learning disability. It is not currently proposed to extend the 2016 July Provision Scheme to further disability categories. The 2016 July Provision scheme commenced in April 2016, the deadline for receipt of applications was May 13th 2016. As the Deputy is aware the Programme for a Partnership Government, published in May 2016, includes a commitment to examine the adequacy of current special education access and funding provision, in particular for children with Down syndrome. In addition the National Council for Special Education (NCSE) has submitted its Policy Advice on Educational Provision for Children with Autism Spectrum Dis- orders. In developing this policy advice, the NCSE reviewed a range of provision for children with Autism including the Department’s July Provision scheme. The NCSE consulted widely with parents, professionals and other stakeholders and interested parties while also conducting research. An implementation group has now been established to review the recommendations of this policy advice. Any recommendations which relate to the July Provision Scheme will be fully considered by this group which will prepare an implementation plan for my consideration.

21/06/2016WRJ05600Emergency Works Scheme Applications

21/06/2016WRJ05700197. Deputy Michael Ring asked the Minister for Education and Skills if and when he will provide funding to a school (details supplied) under works given the urgent and substantial works required; and if he will approve funding Scheme. [17023/16]

21/06/2016WRJ05800Minister for Education and Skills (Deputy Richard Bruton): I wish to advise the Deputy that my Department has no record of receiving an application for funding for emergency works from the school in question. However, I can confirm that the school in question submitted an application under Category 6 of the Summer Works Scheme (SWS) (2016-17), for roof works. I wish to confirm to the Deputy that valid SWS (2016-2017) applications from schools in re- spect of Categories 3 to 10 that were not reached under round one approvals announced on 28th April last, will, subject to the overall availability of funding, qualify to be assessed under future rounds of the Scheme. If this arises, the terms and conditions of the scheme as outlined in Circular Letter (0055/2015) which may be accessed on my Department’s website, will continue to apply when allocating funding to such projects. The SWS (2016-2017) application from the school, in question, is available to be considered in this context.

21/06/2016WRJ05900School Expulsions

96 21 June 2016

21/06/2016WRJ06000198. Deputy Stephen S. Donnelly asked the Minister for Education and Skills if he can challenge an overturning by third parties directly affected with regard to the expulsion of a student from a secondary school and the subsequent overturning of the expulsion by the school board; and if he will make a statement on the matter. [17026/16]

21/06/2016WRJ06100Minister for Education and Skills (Deputy Richard Bruton): The position regarding expulsions is that Section 29 of the Education Act, 1998 provides for an appeal by a parent or guardian to the Secretary General of my Department or in the case of an Education and Training Board (ETB) school to the ETB in the first instance, where a Board of Management of a school, or a person acting on behalf of the Board, expels a student. In the case of a school which is established or maintained by an ETB, an appeal against a decision of the board of such a school shall lie, in the first instance, to the ETB and thereafter to the Secretary General of my Department. In general, appeals must be made within 42 days of receipt of the Board of Management’s decision. My Department has no authority to compel a school to admit a pupil, except in the case of an appeal under section 29 of the Education Act, 1998 being upheld. Ap- plication forms for Section 29 Appeals are available on my Departments website at the follow- ing link: http://www.education.ie/en/Parents/Services/Appeal-against-Permanent-Exclusion- Suspension-or-Refusal-to-Enrol/Section-29-Appeals-Application-Form.doc or from Section 29 Appeals Unit, Friars Mill Road, Mullingar, Co. Westmeath, Phone: 0761 108588.

21/06/2016WRJ06200Student Support Schemes

21/06/2016WRJ06300199. Deputy Carol Nolan asked the Minister for Education and Skills the amount of the €407 million he allocated for the purposes of student support and related expenses, that is paid to education institutions to cover the student contribution; the amount that is paid for the purposes of student maintenance and the amount paid for staff or administration purposes; the additional moneys not accounted for, in this allocation, including the purpose of same; and if he will make a statement on the matter. [17040/16]

21/06/2016WRJ06400Minister for Education and Skills (Deputy Richard Bruton): The allocation of the €407.656m is broken down as follows:

- €377.2m - Student grants and related activities.

- €19.2m - Third Level Access Fund.

- €9.5m – SUSI pay and non-pay costs.

- €1.7m – Costs associated with scholarships, bursaries, the European University Institute and the Student Grants Appeals Board. The Deputy will be aware that the student support subhead is a demand led subhead. As such, the breakdown of the 2016 expenditure between its constituent parts will not be available until 2017. However, the breakdown of expenditure in 2015 was as follows:

- €219m (56%) was spent on student contributions & fees, and

- €169m (44%) was spent on maintenance.

21/06/2016WRJ06500School Services Staff

21/06/2016WRJ06600200. Deputy Michael Healy-Rae asked the Minister for Education and Skills the status of an application by a school (details supplied) which wishes to receive a second caretaker in the 97 Questions - Written Answers school; and if he will make a statement on the matter. [17048/16]

21/06/2016WRJ06700Minister for Education and Skills (Deputy Richard Bruton): Under arrangements in- troduced in 2009 as part of budgetary measures, schools like the one referred to by the Deputy are permitted to fill a caretaker vacancy provided they have no other caretaker. Given that this school already has a caretaker it is not permitted to fill a second caretaker post. Any improve- ment in these arrangements is dependent on the budgetary position and will, in the first instance, be targeted at the largest schools. In the meantime, if any additional caretaking is required it should be met from within the School Services Support Fund grant that is paid to the school. This grant will amount to €66,052 for the school year 2015/16, in respect of the school in ques- tion. Schools are permitted discretion in the application of the School Services Support Fund in accordance with their own needs and priorities. However, the provision of secretarial and maintenance services is a particular focus of this fund. The SSSF is a significant resource for schools in regard to employing additional administrative and maintenance staff.

21/06/2016WRK00150Emergency Works Scheme Applications

21/06/2016WRK00200201. Deputy Charlie McConalogue asked the Minister for Education and Skills the status of an emergency works application by a school (details supplied); if the application will be suc- cessful, given that funding is vital to ensure the health and safety of the children attending this school; and if he will make a statement on the matter. [17060/16]

21/06/2016WRK00300Minister for Education and Skills (Deputy Richard Bruton): I wish to advise the Deputy that my Department has no record of receiving an application for funding for emergency works from the school in question. However, I can confirm that the school in question submitted an application under Category 5 of the Summer Works Scheme (SWS) (2016-17), for upgrade of toilet facilities.

I wish to confirm to the Deputy that valid SWS (2016-2017) applications from schools in respect of Categories 3 to 10 that were not reached under round one approvals announced on 28th April last, will, subject to the overall availability of funding, qualify to be assessed under future rounds of the Scheme. If this arises, the terms and conditions of the scheme as outlined in Circular Letter (0055/2015) which may be accessed on my Department’s website, will con- tinue to apply when allocating funding to such projects. The SWS (2016-2017) application from the school, in question, is available to be considered in this context.

21/06/2016WRK00350Schools Building Projects Status

21/06/2016WRK00400202. Deputy Michael Healy-Rae asked the Minister for Education and Skills the status of an application by a school (details supplied) under the capital works scheme; and if he will make a statement on the matter. [17065/16]

21/06/2016WRK00500Minister for Education and Skills (Deputy Richard Bruton): I can confirm to the Deputy that the school referred to made an application to my Department for significant capital funding for building works.

I wish to advise the Deputy that a project for the school, referred to, is not included in my Department’s 6 year Construction Programme (2016 - 2021), which is focused on providing ad- ditional capacity to meet demographic demand. In the circumstances, it is not possible to pro- vide an indicative timeframe for the progression of a major project for the school, in question, at this time. I also wish to advise the Deputy however that my Department approved capital 98 21 June 2016 funding in 2014 for the provision of additional permanent classroom accommodation, ancil- lary accommodation and conversion works to address the school’s immediate accommodation needs. When completed, these works will provide greatly improved facilities at the school in question.

21/06/2016WRK00550Student Grant Scheme Eligibility

21/06/2016WRK00600203. Deputy Brendan Griffin asked the Minister for Education and Skills if there is addi- tional student grant financial support available to students who are living in isolated rural areas; and if he will make a statement on the matter. [17070/16]

21/06/2016WRK00700Minister for Education and Skills (Deputy Richard Bruton): The Deputy will appreciate that, in the absence of all of the relevant details that would be contained in an individual’s appli- cation form and supporting documentation, it is not possible to say whether or not a particular student(s) would qualify for a grant or other supports.

Under the terms of the student grant scheme, grant assistance is awarded to students who meet the prescribed conditions of funding, including those relating to nationality, residency, previous academic attainment and means. The eligibility of the individual to which the Deputy refers is a matter for SUSI (Student Universal Support Ireland) to determine; upon receipt of the relevant application form and supporting documentation.

There are no specific supports targeted at students living in “isolated rural areas”. However, if the student in question qualifies for a SUSI grant and is living more than 45kms from college, then s/he may qualify for a higher maintenance grant.

Other possible supports include:

1) Tax Relief on Tuition Fees - Further information on this tax relief is available from the Revenue Commissioners on www.revenue.ie and

2) The Student Assistance Fund - Students in third-level institutions experiencing excep- tional financial need can apply for support under the Student Assistance Fund. This Fund assists students, in a sensitive and compassionate manner, who might otherwise be unable to continue their third level studies due to their financial circumstances. Information on the fund is available through the Access Officer in the third level institution attended. This fund is ad- ministered on a confidential, discretionary basis.

21/06/2016WRK00750Schools Establishment

21/06/2016WRK00800204. Deputy Eamon Ryan asked the Minister for Education and Skills how he will meet the growing demand for school places in the Harold’s Cross and Terenure areas, including cur- rent demand for primary school places, current demand for secular non-religious schools and the demand for secondary schools. [17075/16]

21/06/2016WRK00900Minister for Education and Skills (Deputy Richard Bruton): The Programme for a Part- nership Government reflects the Government’s objective of strengthening parental choice and further expanding diversity in our school system, specifically by increasing the number of non- denominational and multi-denominational schools with a view to reaching 400 by 2030.

In addition to the phased transfer of Catholic schools to new patrons already under way, this will require considering new ideas and new approaches to substantially accelerate the changes 99 Questions - Written Answers in school provision in order to achieve greater diversity. The Government is committed to working with all stakeholders to facilitate this process and I am progressing these matters as a priority.

Regarding the areas to which the Deputy refers, my Department is engaged with an exist- ing school patron under the patronage divesting process in relation to a former school building which can be returned to use as a school under new patronage to serve the areas concerned and work is well advanced in this regard.

In relation to new schools required to meet demographic needs, my Department uses a Geo- graphical Information System (GIS) to identify where the pressure for school places will arise nationwide. The GIS uses data from the Central Statistics Office, Ordnance Survey Ireland, the Department of Social Protection and information from my Department’s own databases.

The outcome of the latest demographic exercises, based on the GIS, was the announcement, in November 2015, of the establishment of four new primary schools and nine new post-prima- ry schools to open in 2017 and 2018 to cater for increased demographic needs. These schools include a new primary school which will open in 2018 to serve the Dublin South City Centre areas of Dublin 2, 4 and 6 and will provide additional capacity. A new post-primary school will also open in 2018 to serve the areas of Dublin 2, 4, 6 and 8.

My Department is keeping the demographic data under ongoing review to take into account updated enrolment data, updated child benefit data and the impact of ongoing and planned ex- pansion of capacity.

21/06/2016WRK00950School Transport Provision

21/06/2016WRK01000205. Deputy Michael Ring asked the Minister for Education and Skills if he will provide school transport to a school (details supplied), given that it has more than the required number of pupils who meet the criteria and qualify for transport. [17173/16]

21/06/2016WRK01100Minister of State at the Department of Education and Skills (Deputy John Halligan): The Deputy will be aware that a minimum number of 10 eligible children residing in a distinct locality, as determined by Bus Éireann, is required before consideration may be given to the establishment of a school transport service, provided this can be done within reasonable cost limits.

In this regard, Bus Éireann has advised that a sufficient number of children, who are eligible for school transport, have applied for transport for the 2016/17 school year and it is envisaged that a new service will be established provided these families submit ticket payment details to Bus Éireann by the closing date of 29th July 2016.

The families, in question, should liaise with their local Bus Éireann office regarding the timetabling and routing of this proposed new service.

21/06/2016WRK01150Schools Building Projects Applications

21/06/2016WRK01200206. Deputy Charlie McConalogue asked the Minister for Education and Skills if he will make funding available to a school (details supplied); and if he will make a statement on the matter. [17187/16]

21/06/2016WRK01300Minister for Education and Skills (Deputy Richard Bruton): I can confirm that my 100 21 June 2016 Department received an application from the school referred to by the Deputy to extend the school’s General Purpose Room and Staff Room, and also to address some remedial works.

My Department has indicated to the school that due to the competing demands on its capital budget imposed by the need to prioritise available funding for the provision of essential class- room accommodation, it is not possible to consider the school’s application for the proposed extensions at this time. The school’s application will however be kept on file for consideration in the context of future funding becoming available.

My Department also advised the school that it may use its minor works grant, which all primary schools received last November, to carry out the remedial works in full or on a phased basis as that grant permits.

21/06/2016WRK01350Special Educational Needs Service Provision

21/06/2016WRK01400207. Deputy Charlie McConalogue asked the Minister for Education and Skills if he will reverse a decision regarding the supply of the July provision for a person (details supplied); and if he will make a statement on the matter. [17222/16]

21/06/2016WRK01500Minister for Education and Skills (Deputy Richard Bruton): My Department currently provides July Provision for children with a diagnosis of ASD and/or severe or profound general learning disability. I understand that the application in respect of the child to whom the Deputy has referred does not indicate either of these disabilities. It is not currently proposed to ex- tend the 2016 July Provision Scheme to further disability categories. The 2016 July Provision scheme commenced in April 2016, the deadline for receipt of applications was May 13th 2016.

As the Deputy is aware the Programme for a Partnership Government, published in May 2016, includes a commitment to examine the adequacy of current special education access and funding provision.

In addition the National Council for Special Education (NCSE) has submitted its Policy Advice on Educational Provision for Children with Autism Spectrum Disorders.

In developing this policy advice, the NCSE reviewed a range of provision for children with Autism including the Department’s July Provision scheme. The NCSE consulted widely with parents, professionals and other stakeholders and interested parties while also conducting re- search.

An implementation group has now been established to review the recommendations of this policy advice. Any recommendations which relate to the July Provision Scheme will be fully considered by this group which will prepare an implementation plan for my consideration.

21/06/2016WRK01550Student Universal Support Ireland

21/06/2016WRK01600208. Deputy Róisín Shortall asked the Minister for Education and Skills if he is aware of an anomalous situation whereby a SUSI maintenance grant is not payable to a person (details supplied) who is on a work placement and remains registered a student; and if he will make a statement on the matter. [17224/16]

21/06/2016WRK01700Minister for Education and Skills (Deputy Richard Bruton): Article 15 (3) of the Stu- dent Grant Scheme 2015 states that a maintenance grant may continue to be paid, where a stu- dent is required to participate in an off-campus placement as a compulsory part of their course, 101 Questions - Written Answers as long as the off-campus placement does not extend the normal duration of the course.

The normal duration of the course at the Institute referred to by the Deputy is three years. The student referred to has received grant support for the full three years of her course in the academic years, 2013/14, 2014/15 and 2015/16 and cannot be considered for any further fund- ing for this course.

21/06/2016WRK01750Schools Building Projects

21/06/2016WRK01800209. Deputy Michael Ring asked the Minister for Education and Skills the name of the ar- chitect or architectural firm which designed a school (details supplied); the name of the builder who, or building company which, built the school; the contractual price for this project; the date the contract was signed; the date the project was completed; the amount withheld under the re- tention clause; the final amount actually paid under the contract; his views on two reports; and if he will make a statement on the matter. [17230/16]

21/06/2016WRK01900210. Deputy Michael Ring asked the Minister for Education and Skills if he is aware that there is a roof leak in a school (details supplied); the information on this problem, and his pro- posals and timeline to remedy. [17231/16]

21/06/2016WRK02000211. Deputy Michael Ring asked the Minister for Education and Skills if his Department or the Office of Public Works had a site manager for the building of a school (details supplied). [17232/16]

21/06/2016WRK02100Minister for Education and Skills (Deputy Richard Bruton): I propose to take Questions Nos. 209 to 211, inclusive, together.

The Architectural Services for the school building project referred to by the Deputy was provided by an in-house architectural team in the Department of Education & Skills.

The builder was J.J. Rhatigan & Company the contract sum was €1,697,524.53 including VAT. The project commenced on site in June 2010 and reached substantial completion in Au- gust 2011.

The Final Account was approved and paid in December 2012 the final cost of the project was €1,601,511.76 incl VAT.

My Department is currently seeking to arrange a joint inspection of the roof with the Con- tractor in order to decide on the best course of action to resolve the issues identified in the reports.

21/06/2016WRK02150Schools Refurbishment

21/06/2016WRK02200212. Deputy Michael Ring asked the Minister for Education and Skills the cost of carrying out repairs to faulty roofs in newly built schools in each of the years 2011 to 2016 to date; and if he will make a statement on the matter. [17233/16]

21/06/2016WRK02300Minister for Education and Skills (Deputy Richard Bruton): I wish to advise the Deputy that the defects period on a public works contract is 6 years where a contract is not sealed and 12 years where a contract is sealed. If a roof defect arises within that time, the Department’s position would be primarily that it is a matter for the contractor to resolve without recourse to additional funding from the Department. In that regard, therefore, my Department does not

102 21 June 2016 maintain statistical information specifically in relation to roof defect costs.

21/06/2016WRK02350Schools Grievance Procedures

21/06/2016WRK02400213. Deputy Robert Troy asked the Minister for Education and Skills why he has not intro- duced an adequate school complaints procedure despite calls for reform from the Ombudsman for Children and the National Parents Council; when he will resolve this issue; and if he will make a statement on the matter. [17234/16]

21/06/2016WRK02500Minister for Education and Skills (Deputy Richard Bruton): As the Deputy will be aware the Programme for a Partnership Government provides that the Government will intro- duce a stronger complaints procedure and charter for parents, and commence the Fitness to Teach provisions of the Teaching Council Act.

The Programme also provides that the role and power of an Ombudsman for Education, to whom a parent could complain and appeal on foot of a decision by a board of management, will be examined by the relevant Oireachtas Committee to ensure its consistency with the need to ensure better local decision making and accountability to parents.

Both of these commitments are part of a continuum because having a dedicated ombudsman with a power to deal with parental complaints would be a residual, but potentially important function where local resolution has failed.

I think that the relevant Oireachtas Committee will need to consider both programme com- mitments together. Work already under way in my Department envisages legislative change to Section 28 of the Education Act 1998, and creating an ombudsman with powers to externally review school actions would require new legislation that could be progressed in tandem.

21/06/2016WRK02550Schools Site Acquisitions

21/06/2016WRK02600214. Deputy Louise O’Reilly asked the Minister for Education and Skills the status of the identification and acquisition of sites for the two new secondary schools to serve the communi- ties of Malahide, Portmarnock and Swords in County Dublin and for the permanent building for the Malahide Portmarnock Educate Together national school; the timeframe for the identifica- tion and acquisition; and if he will make a statement on the matter. [17244/16]

21/06/2016WRK02700Minister for Education and Skills (Deputy Richard Bruton): As the Deputy may be aware, the building projects for the new post primary schools to serve Malahide/Portmarnock and Swords are included in the 6 year construction programme that was announced last No- vember, and these schools are listed to go to construction in 2019-21. Malahide Portmarnock Educate Together NS is currently in temporary accommodation on a Teagasc-owned property in Kinsealy and the project to deliver permanent accommodation for this school is also on the construction programme.

Officials in my Department are liaising with officials in Fingal County Council in accor- dance with the Memorandum of Understanding in relation to the identification and acquisition of suitable locations for all three schools.

Agreement in principle has been reached in respect of a site for the new post primary school in Swords and my Department is currently progressing with its acquisition.

In relation to the new post primary school to serve Malahide/Portmarnock the technical 103 Questions - Written Answers feasibility of an identified site option is currently being comprehensively assessed by officials from my Department in liaison with officials from the Local Authority.

Similarly, for Malahide Portmarnock Educate Together NS, an assessment of technical suit- ability in respect of an identified site option is currently being undertaken.

Due to commercial sensitivities relating to site acquisitions generally I am unfortunately not in a position to provide further details at this time. I can, however, assure you that the site acquisitions for all three schools are being advanced by my Department.

21/06/2016WRK02750Schools Site Acquisitions

21/06/2016WRK02800215. Deputy Louise O’Reilly asked the Minister for Education and Skills the status of where two new secondary schools to serve the communities of Malahide, Portmarnock and Swords in County Dublin will open in September 2017 in temporary accommodation until a new permanent building is delivered; how long the temporary accommodation will be func- tional; the temporary accommodation that is available; and if he will make a statement on the matter. [17245/16]

21/06/2016WRK02900Minister for Education and Skills (Deputy Richard Bruton): My Department uses a Geographical Information System (GIS) to identify where the pressure for school places will arise. The GIS uses data from the Central Statistics Office, Ordnance Survey Ireland, the De- partment of Social Protection and information from my Department’s own databases.

The outcome of the latest demographic exercises, based on the GIS, was the announcement in November 2015 of the establishment of 13 new schools to open nationwide in 2017 and 2018 to cater for increased demographic needs. One of the new post-primary schools will serve the Swords school planning area and one will serve the Malahide and Portmarnock school plan- ning areas. Both new schools will open in 2017 and both are included in my Department’s six year construction programme (2016-2021). Details on the progress of all projects on this programme will be published on my Department’s website and this will be updated regularly.

My Department are liaising with Fingal County Council in accordance with the Memoran- dum of Understanding in relation to the identification and acquisition of suitable locations for these schools.

Agreement in principle has been reached in respect of a site for the new post-primary school in Swords and my Department is currently progressing with its acquisition.

In relation to the new post-primary school to serve the Malahide and Portmarnock areas, the technical feasibility of an identified site option is currently being comprehensively assessed by officials from my Department in liaison with officials from the LocalAuthority.

Due to commercial sensitivities relating to site acquisitions generally, I am, unfortunately, not in a position to provide further details at this time. I can, however, assure you that the site acquisitions for both of these schools are being advanced by my Department and that pending the delivery of the permanent school buildings, temporary accommodation will be provided.

21/06/2016WRK02950Physical Education Facilities

21/06/2016WRK03000216. Deputy Alan Farrell asked the Minister for Education and Skills the action he will take in relation to a school (details supply) which urgently requires funding for a new floor in 104 21 June 2016 the physical education hall; if the provision of an adequate and usable physical activity facili- ties is of the utmost importance in terms of ensuring the health of children; and if he will make a statement on the matter. [17251/16]

21/06/2016WRK03100Minister for Education and Skills (Deputy Richard Bruton): I wish to advise the Deputy that my Department has no record of receiving an application from the school in question for refurbishment of their PE hall facilities.

I also wish to advise the Deputy that the works in question are appropriate to the Summer Works Scheme, the purpose of which is to devolve funding to individual school authorities to facilitate carrying out small and medium scale capital works that will improve and upgrade ex- isting school buildings including PE halls. Under the scheme, responsibility for identifying and prioritising the most urgently required project rests with the school authority. It will be open to the school, if it so decides, to make an application under a future summer works scheme.

In the meantime, the school may use its minor works grant, which all primary schools re- ceived last November, to carry out the remedial works in full or on a phased basis as that grant permits if the school considers them to be a priority.

21/06/2016WRK03150Schools Building Projects Status

21/06/2016WRK03200217. Deputy Alan Farrell asked the Minister for Education and Skills the action he will take about the requirement for the replacement of the K block in a school (details supplied); and if he will make a statement on the matter. [17252/16]

21/06/2016WRK03300Minister for Education and Skills (Deputy Richard Bruton): The Deputy will be aware that a project for the school to which he refers was included in my Department’s 6 Year Capital Programme which was announced last November. The project for the school in question will be progressed with a view to proceeding to tender and construction stages in 2019-2021, as outlined in the programme.

I understand that my Department is currently finalising the school project brief and once this process is complete, the project will be progressed into the Architectural Planning process.

In the meantime, my Department has advised the school authority that it is open to it to submit an Emergency Works Grant Application for the most minimal and urgent works as an interim measure.

21/06/2016WRK03350Ministerial Meetings

21/06/2016WRK03400218. Deputy David Cullinane asked the Minister for Education and Skills if he has met with the Irish National Teachers Organisation since his appointment; and if he will make a state- ment on the matter. [17274/16]

21/06/2016WRK03500219. Deputy David Cullinane asked the Minister for Education and Skills if he has met with the Association of Secondary Teachers Ireland since his appointment; and if he will make a statement on the matter. [17275/16]

21/06/2016WRK03600220. Deputy David Cullinane asked the Minister for Education and Skills if he has met with the Teachers Union of Ireland since his appointment; and if he will make a statement on the matter. [17276/16]

105 Questions - Written Answers

21/06/2016WRK03700221. Deputy David Cullinane asked the Minister for Education and Skills if has met with the Irish Federation of University Teachers since his appointment; and if he will make a state- ment on the matter. [17277/16]

21/06/2016WRK03800Minister for Education and Skills (Deputy Richard Bruton): I propose to take Questions Nos. 218 to 221, inclusive, together.

I met with ASTI on 15 June on the issue of Junior Cycle reform. In the context of the Lans- downe Road Agreement, I have on several occasions invited ASTI to discuss issues of mutual concern, but ASTI have to date declined such a meeting. The invitation to them remains open.

I have not yet met with INTO, TUI and IFUT but I intend to do so as soon as is practicable.

21/06/2016WRK03850Technological Universities

21/06/2016WRK03900222. Deputy David Cullinane asked the Minister for Education and Skills the dialogue he has had with the Teachers Union of Ireland regarding technological universities; and if he will make a statement on the matter. [17278/16]

21/06/2016WRK04000Minister for Education and Skills (Deputy Richard Bruton): As the Deputy will be aware, the new Programme for Government outlines that this new Government will continue to support the creation of Technological Universities.

This is in line with the National Strategy for Higher Education to 2030 which provides a framework for the development of the higher education sector to 2030. With regard to the insti- tute of technology sector, the Strategy recommended significant reforms to position the sector to meet national strategic objectives. In particular, the Strategy recommended consolidation within the sector and a pathway of evolution for those consolidated institutes of technology, to allow them to demonstrate significant progress against robust performance criteria and to apply to become technological universities.

Position of Technological Universities bill: The first Government Legislative Programme of this new Partnership Government was published on 8 June 2016, and the Technological Univer- sities Bill has been restored to the Dáil Order Paper at Committee Stage.

I recognise that there were a significant number of matters raised in respect of the Bill at both Committee and Report Stage. It is my intention to now consult with all of the relevant stakeholders in relation to both the matters raised during the legislative process and the com- mitments contained in the Programme for Government.

Following the finalisation of this consultation process I will then advance the legislation having determined a position in relation to any matters raised as part of this consultation pro- cess.

As part of this consultation process, the General Secretary of the Teachers’ Union of Ireland has recently written to me seeking a meeting to discuss relevant issues in regard to the Tech- nological Universities Bill. It is my intention to facilitate this meeting as soon as practicable.

21/06/2016WRL00150Technological Universities

21/06/2016WRL00200223. Deputy David Cullinane asked the Minister for Education and Skills his plans to pub- lish the report jointly submitted to the Higher Education Authority by the chairs and the presi-

106 21 June 2016 dents of Waterford Institute of Technology and Institute of Technology Carlow, in respect of the technological university process; and if he will make a statement on the matter. [17279/16]

21/06/2016WRL00300Minister for Education and Skills (Deputy Richard Bruton): Technological Universi- ties: The new Programme for Government outlines that this new Government will continue to support the creation of Technological Universities. It is envisaged that the development of a Technological University for the South East will act as a catalyst and stimulus of economic, social and cultural development for the region.

The development of Technological Universities is in line with the National Strategy for Higher Education to 2030 which provides a framework for the development of the higher educa- tion sector to 2030. With regard to the institute of technology sector, the Strategy recommended significant reforms to position the sector to meet national strategic objectives. In particular, the Strategy recommended consolidation within the sector and a pathway of evolution for those consolidated institutes of technology, to allow them to demonstrate significant progress against robust performance criteria and to apply to become technological universities.

The development of technological universities has the potential to deliver greater oppor- tunity to students in these regions, to staff working in the institutions, and to the broader local economy and society.

Technological University for the South-East (TUSE):With regard to the Technological Uni- versity for the South-East, consisting of Institute of Technology, Carlow (ITC) and Waterford Institute of Technology (WIT), following the publication of the Kelly report, a preliminary fa- cilitation process which was underway since September 2015, has recently been completed and a Report of the process was received by my Department on 13 May 2016.

The Report of the facilitation process is currently being considered by my officials and a decision on publication of the Report will be taken in due course.

I should point out that there was strong engagement in the process by both parties. This facilitation process has been an important building block in terms of building trust between the parties and in developing a strong working relationship between the Presidents and Chairs of both institutions. As part of this process, the Presidents of the two institutions, have jointly developed an initial work-plan to support the development of a joint TU proposal.

In addition to this both Institutions made a recent joint presentation and submission to the Higher Education Authority, seeking funding to underpin the next phase of engagement. This was part of the latest call issued by the Higher Education Authority regarding the restructuring of the higher education landscape. The call sought submissions for funding support in respect of the costs arising from mergers as part of the implementation of the National Strategy for Higher Education.Position of Technological Universities Bill: The first Government Legisla- tive Programme of this new Partnership Government was published on 8 June 2016, and the Technological Universities Bill has been restored to the Dáil Order Paper at Committee Stage.

I recognise that there were a significant number of matters raised in respect of the Bill at both Committee and Report Stage. It is my intention to now consult with all of the relevant stakeholders in relation to both the matters raised during the legislative process and the com- mitments contained in the Programme for Government.

Following the finalisation of this consultation process I will then advance the legislation having determined a position in relation to any matters raised as part of this consultation pro- cess.

107 Questions - Written Answers

21/06/2016WRL00350Cearta Painéil Mhúinteoirí Scoile

21/06/2016WRL00400224. D’fhiafraigh Deputy Pearse Doherty den Aire Oideachais agus Scileanna cathain a bheidh duine i nDún na nGall (sonraí curtha ar fáil) curtha ar phainéal forlíontach na Roinne; agus an ndéanfaidh sé ráiteas ina thaobh. [17280/16]

21/06/2016WRL00500Minister for Education and Skills (Deputy Richard Bruton): Is í an phríomhfheidhm atá ag na socruithe ath-imlonnaithe ná éascú le hath-imlonnú na múinteoirí buana barrachais go léir go dtí scoileanna eile a bhfuil folúntais iontu. Dá réir sin, ceanglaítear ar scoileanna faoi na socruithe painéil folúntais bhuana a líonadh ó phainéil fhorlíontacha atá déanta suas de mhúin- teoirí incháilithe ar théarma seasta (sealadach/ionadaíoch) agus de mhúinteoirí páirtaimseartha.

Tá na socruithe maidir le rochtain ar an bpainéal i gcás múinteoirí ar théarma seasta (seala- dach/ionadaíoch) agus múinteoirí páirtaimseartha, don scoilbhliain 2016/17, leagtha amach i gCiorclán 0058/2015 na Roinne a foilsíodh ar 16 Samhain 2015 ar shuíomh idirlín na Roinne.

Ba é an spriocdháta d’iarratais a chur isteach chuig mo Roinnse ar bheith curtha ar an bPai- néal Forlíontach 2016/17 ná 11 Nollaig 2015. Ní bhfuair mo Roinn iarratas ón mhúinteoir dá dtagraíonn an Teachta. Tá an Painéal Forlíontach ag oibriú faoi láthair agus, de réir mar atá leagtha amach i gCiorclán 0058/2015, ní féidir glacadh le hiarratais dhéanacha.

Foilseofar na socruithe don Phainéal Forlíontach don scoilbhliain 2017/18 ar shuíomh idirlín na Roinne sa bhFómhar. The core function of the redeployment arrangements is to fa- cilitate the redeployment of all surplus permanent teachers to other schools that have vacancies. Thereafter, schools are required under the panel arrangements to fill permanent vacancies from supplementary panels comprised of eligible fixed-term (temporary/substitute) and part-time teachers.

The arrangements for panel access for fixed term (temporary/substitute) and part-time teachers for the 2016/17 school year are set out in Department Circular 0058/2015 which was published on 16 November 2015 on the Department website. The deadline for submission of applications to the Supplementary Panel 2016/17 to my Department was 11th December 2015. My Department has not received an application from the teacher referred to by the Deputy. The Supplementary Panel is currently in operation and, as set out in Circular 0058/2015, late ap- plications cannot be accepted.

Arrangements for the Supplementary Panel for the 2017/18 school year will be published on the Department website in the Autumn.

21/06/2016WRL00550Home School Community Liaison Scheme

21/06/2016WRL00600225. Deputy Marc MacSharry asked the Minister for Education and Skills if he will ac- cede to the request from the principals of schools (details supplied) who share a home school liaison community co-ordinator resource to extend the assignment of the teacher in this posi- tion; and if he will make a statement on the matter. [17304/16]

21/06/2016WRL00700Minister for Education and Skills (Deputy Richard Bruton): The Deputy will be aware from my written reply to his representation that the arrangements governing the assignment of Home School Community Liaison (HSCL) is governed by my Department’s Circular 0058/2013. The circular in question was agreed with Teacher Unions and Management Bodies. A key feature of the HSCL scheme is the rotation of the HSCL post as it gives eligible staff members the opportunity to apply to serve as HSCL Coordinators in order for them to gain in-

108 21 June 2016 sight and experience of educational disadvantage first hand.

My officials have received correspondence from the school that an appointment has been made to the post which is effective from the next school year.

21/06/2016WRL00725Schools Refurbishment

21/06/2016WRL00800226. Deputy Marc MacSharry asked the Minister for Education and Skills to expedite a decision on an application for funding under the schools capital appraisal scheme for urgent refurbishment work in a school (details supplied) in preparation for the opening of a special class for children with autism in September 2016; and if he will make a statement on the matter. [17329/16]

21/06/2016WRL00900Minister for Education and Skills (Deputy Richard Bruton): I can confirm to the Deputy that my Department recently approved a grant to the school in question to refurbish existing available accommodation to provide suitable accommodation for an ASD class for September 2016. This decision has been conveyed to the school authority.

21/06/2016WRL00950Teachers’ Remuneration

21/06/2016WRL01000227. Deputy Timmy Dooley asked the Minister for Education and Skills if he will address a matter (details supplied) raised in correspondence regarding teachers. [17340/16]

21/06/2016WRL01100Minister for Education and Skills (Deputy Richard Bruton): The criteria for the award of incremental credit to recognised primary and post-primary teachers under Circulars 10/2001, 29/2007 and 29/2010 was agreed under the auspices of the Teachers Conciliation Council. This Council is comprised of representatives of teachers, school management, the Department of Public Expenditure and Reform, the Department of Education & Skills and is chaired by an of- ficial of the Workplace Relations Commission.

In accordance with the criteria as set out in the Scheme and on appointment to a teach- ing post, incremental credit may be awarded in respect of period(s) of non-teaching service if among other things, the experience gained is considered to be directly relevant to the require- ments of the post to which the teacher has been appointed. Where a decision is made to limit or refuse to award credit in respect of a claim a recognised teacher may appeal to a committee comprised of representatives of teacher unions, school management, the Department of Educa- tion and Skills and is chaired by an Independent Chairperson.

Regarding the further queries and requests for data contained in the details accompanying this question I will arrange for the information to be forwarded directly to the Deputy.

21/06/2016WRL01150Property Tax Yield

21/06/2016WRL01200228. Deputy Danny Healy-Rae asked the Minister for the Environment, Community and Local Government the amount of funding collected from the property tax charges in County Kerry in 2015; the amount of this funding provided to the local authority in County Kerry; and if he will make a statement on the matter. [16723/16]

21/06/2016WRL01300Minister for the Environment, Community and Local Government (Deputy Simon Coveney): I assume the Question refers to the Local Property Tax (LPT). LPT is collected and

109 Questions - Written Answers administered by the Revenue Commissioners and is remitted to the Central Fund. The Revenue Commissioners publish LPT information including details of LPT collected in all local author- ity areas on their website at the following link: http://www.revenue.ie/en/about/statistics/local- property-tax.html.

Section 157 of the Finance (Local Property Tax Act) 2012 provides that an amount equiva- lent to the overall amount of LPT collected in a given year shall, beginning in 2014, be paid into the Local Government Fund in the same year by the Minister for Finance.

In accordance with the Government’s decisions in relation to local retention of Local Prop- erty Tax, my Department paid €11,659,186 to Kerry County Council in respect of the author- ity’s LPT allocation for 2015.

The 2015 LPT allocation to Kerry County Council along with details of allocations in each local authority area are published on my Department’s website at the following link: http:// www.environ.ie/housing/chargestaxes/local-property-tax/local-property-tax-final-allocations- local-authorities-2015.

21/06/2016WRL01307Waste Disposal Charges

21/06/2016WRL01400229. Deputy Billy Kelleher asked the Minister for the Environment, Community and Local Government the allowance exemption he will provide given to households with medical needs and in respect of the upcoming pay-by-weight refuse charges; and if he will make a statement on the matter. [16662/16]

21/06/2016WRL01500256. Deputy Michael McGrath asked the Minister for the Environment, Community and Local Government further to Parliamentary Question No. 398 of 31 May 2016, the status of the steps he is taking, in respect of families that include a person with a disability, and whose bins are particularly heavy, as a result of that person’s condition, in relation to the introduction of pay-by-weight charges for the collection of domestic refuse; and if he will make a statement on the matter. [17205/16]

21/06/2016WRL01600268. Deputy Róisín Shortall asked the Minister for the Environment, Community and Local Government the action he will take to ensure that families who rely on adult nappies in the care of a dependent with a disability or older children who are not toilet ready due to a dis- ability, are not seriously disadvantaged by the change to a pay-by-weight waste charge system as currently is the case. [17310/16]

21/06/2016WRL01700Minister for the Environment, Community and Local Government (Deputy Simon Coveney): I propose to take Questions Nos. 229, 256 and 268 together.

My Department has met with groups who represent persons with special needs to discuss and explore the impact of pay-by-weight charging on households with special needs. My De- partment has also been in consultation with the HSE and the waste management industry to discuss the issue and to try to find a workable and fair solution to this important matter.

As part of a way forward in relation to the phased implementation of pay-by-weight charg- ing for household waste, the waste management industry has committed to provide a full weight allowance to the estimated 60,000 patients supplied with incontinence wear to reduce their annual household waste charges; the additional weight attributed to non-infancy incontinence wear will be collected free of charge. For its part, the Government is prepared to allow a 50% exemption from the landfill levy in respect of such waste.

110 21 June 2016

21/06/2016WRL01750Wind Energy Guidelines

21/06/2016WRL01800230. Deputy Thomas Pringle asked the Minister for the Environment, Community and Local Government if he will increase the 500 m setback distance proposed in the 2013 targeted revision to the Wind Energy Development Guidelines 2006 to be in line with current 2016 international best practices for wind turbine setback exemplified by the ten times tip height rule in Bavaria and the 2 km policy in Poland; and if he will make a statement on the matter. [16664/16]

21/06/2016WRL01900267. Deputy Willie Penrose asked the Minister for the Environment, Community and Lo- cal Government in the context of the consultation process, concerning Wind Energy Develop- ment Guidelines 2006, and the revision that was embarked upon, which involved public consul- tation, if he is aware that best international practice has moved on significantly since November 2013, with the most recent example being the recommendation of a 2 km setback by the Polish National Institute of Public Health on 8 March 2016, and the judgment by the Constitutional Court of Bavaria on 9 March 2016, upholding the Bavarian State setback rule of ten times tip height that has been in force since 2014; if he will take steps to issue fresh draft guidelines that reflect best international practice, as enforced by colleague European countries, which would necessitate undertaking the public consultation process, and then enacting enabling legislation, in order that the public can have confidence that the issue has been dealt with, in a fair and transparent manner; and if he will make a statement on the matter. [17307/16]

21/06/2016WRL02000Minister for the Environment, Community and Local Government (Deputy Simon Coveney): I propose to take Questions Nos. 230 and 267 together.

In December 2013, my Department published proposed “draft” revisions to the noise, set- back distance and shadow flicker aspects of the 2006 Wind Energy Development Guidelines.

These draft revisions proposed:

- the setting of a more stringent day and night noise limit of 40 decibels for future wind energy developments,

- a mandatory minimum setback distance of 500 metres between a wind turbine and the nearest dwelling for amenity considerations, and

- the complete elimination of shadow flicker between wind turbines and neighbouring dwellings.

A public consultation process was also initiated on these proposed draft revisions to the Guidelines, which ran until 21 February 2014. My Department received submissions from 7,500 organisations and members of the public during this public consultation process.

As outlined in the Programme for a Partnership Government published last month, the Government is committed to finalising the revisions to the 2006 Wind Energy Development Guidelines within 3 to 6 months. The revisions to the Guidelines will be informed by the public consultation process and best international practice. My Department will continue to advance work on the Guidelines, in conjunction with the Department of Communications, Energy and Natural Resources.

The revisions to the Wind Energy Development Guidelines 2006, when finalised, will be issued under Section 28 of the Planning and Development Act 2000, as amended. Planning authorities, and, where applicable, An Bord Pleanála are required to have regard to such guide- lines issued under Section 28 in the performance of their functions under the Planning Acts.

111 Questions - Written Answers

21/06/2016WRL02050Water Quality

21/06/2016WRL02100231. Deputy Clare Daly asked the Minister for the Environment, Community and Local Government the steps he has taken in relation to the do not swim notices which have been repeatedly issued in June 2016 in Portmarnock, and Skerries, County Dublin as a result of overflows in the Portmarnock Strand Pumping Station and Hampton Cove Pumping Station; and how he will address this matter. [16685/16]

21/06/2016WRL02200Minister for the Environment, Community and Local Government (Deputy Simon Coveney): I am aware of concerns regarding discharges in the functional area of Fingal County Council. However, neither I nor my Department has any direct role in monitoring or supervis- ing the delivery of water services or any pollution incidents arising therefrom. Irish Water has statutory responsibility for all aspects of water services planning, delivery and operation at national, regional and local levels for public water services, including the delivery of water services capital infrastructure, encompassing the management of urban waste water collection and treatment infrastructure. All discharges to the aquatic environment from sewerage systems owned, managed and operated by Irish Water require a waste water discharge licence or certifi- cate of authorisation from the Environmental Protection Agency (EPA), and the authorisation process provides for the EPA to place stringent conditions on the operation of such discharges to ensure that potential effects on the receiving water bodies are strictly limited and controlled.

The EPA is a key statutory body for investigating complaints of pollution and for the en- forcement , both directly and through oversight of local authorities, of environmental legisla- tion in Ireland, including compliance in relation to licensed urban waste water discharges. De- tails of all prosecutions taken by the EPA for pollution incidents and details of its enforcement activities are published on the EPA’s website (http://www.epa.ie/enforcement/).

The EPA’s Bathing Water Quality Report for 2015 identifies several bathing waters adjacent to urban areas as being prone to episodic pollution events and being of less than ‘good’ water quality status. These pollution events are generally associated with overflows from pumping stations or storm outfalls as a result of sewer network blockages or following heavy rainfall. The EPA report highlights that significant infrastructural investment will be required to reduce the likelihood of recurrence of pollution events in these urban areas.

The imposition of bathing prohibitions by local authorities in the event of discharges or following intense localised rainfall is done with regard to public health on a precautionary principle. All such incidents are reported to the EPA’s wastewater enforcement system and are publicised on the SPLASH website at http://splash.epa.ie/#, which is the national bathing water information website for identified bathing waters around Ireland.

21/06/2016WRL02250Local Authority Housing Mortgages

21/06/2016WRL02300232. Deputy Fergus O’Dowd asked the Minister for the Environment, Community and Local Government the number of local authority mortgagees, by local authority; and if he will make a statement on the matter. [16734/16]

21/06/2016WRL02400Minister for the Environment, Community and Local Government (Deputy Simon Coveney):

Information in relation to the number and value of local authority housing loans is available on my Department’s website at:

112 21 June 2016 http://www.environ.ie/housing/statistics/house-prices-loans-and-profile-borrowers/local- authority-loan-activity.

The most up-to-date information relates to Quarter 1, 2016. Of the 17,696 local authority housing loans as at end Q1, 10,134 have just one named borrower and t he remaining 7,562 loans have more than one named borrower.

21/06/2016WRL02450Water and Sewerage Schemes Grants

21/06/2016WRL02500233. Deputy John Brassil asked the Minister for the Environment, Community and Local Government his plans to increase the grants payable under the group water scheme to ensure schemes are sufficiently maintained, given that recent cuts have made them unmanageable for proper maintenance to take place; and if he will make a statement on the matter. [16751/16]

21/06/2016WRL02600Minister for the Environment, Community and Local Government (Deputy Simon Coveney): The Government is committed to introducing and supporting legislation in the Oireachtas, within six weeks of its appointment, to suspend domestic water charges for a period of nine months from the end of the current bill cycle. The proposed details of the suspension are set out in the recently published Water Services (Amendment) Bill 2016, which is available at www.oireachtas.ie. In line with the suspension of domestic water charges, and to ensure eq- uity of treatment with households on public water supplies, the subsidisation for those on group water schemes will be restored to the level that pertained prior to the introduction of domestic water charges for households connected to the public water supply for the same time period.

21/06/2016WRL02650Local Authority Housing Funding

21/06/2016WRL02700234. Deputy Danny Healy-Rae asked the Minister for the Environment, Community and Local Government the amount of funding Kerry County Council requested for housing in 2015; and if he will make a statement on the matter. [16758/16]

21/06/2016WRL02800235. Deputy Danny Healy-Rae asked the Minister for the Environment, Community and Local Government the amount of funding Kerry County Council received in 2015 for the provi- sion of social housing; and if he will make a statement on the matter. [16759/16]

21/06/2016WRL02900Minister for the Environment, Community and Local Government (Deputy Simon Coveney):

I propose to take Questions Nos 234 and 235 together.

In April 2015, funding allocations were announced of over €1.5 billion in respect of social housing to be provided by all local authorities via a combination of building, buying and leas- ing schemes over the period 2015-17. Those allocations and associated targets for each local authority, including Kerry which has an allocation for this purpose of some €62 million, are available on my Department’s website at the following link:

http://www.environ.ie/housing/social-housing/minsters-kelly-coffey-announce-eu15-bil- lion-social-housing-targets-local.

Following this notification of targets, announcements for substantial new social housing projects were made in May 2015, in July 2015 and in January 2016, on the basis of project proposals made the local authorities, including Kerry County Council. Between these three an- nouncements, almost €680 million has been allocated for over 3,900 social housing new builds, 113 Questions - Written Answers turnkey developments and acquisitions. Details of these project approvals, including those for Kerry, are available on my Department’s website at the following links:

http://www.environ.ie/en/DevelopmentHousing/Housing/News/MainBody,41340,en.htm

http://www.environ.ie/en/DevelopmentHousing/Housing/News/MainBody,42225,en.htm

http://www.environ.ie/housing/social-housing/ministers-kelly-coffey-announce-further- 1000-social-housing-units.

I am keen that all local authorities, including K erry County Council, advance these projects as soon as possible and I have assured them that funding is available to fully support their ef- forts in this regard. Across the range of schemes under the social housing programme, my De- partment provided over €11.9 million to Kerry County Council in 2015. I expect funding draw- down to increase as the construction projects are advanced by the local authorities and approved housing bodies. While social housing construction projects are being advanced, acquisitions of new and second-hand houses and apartments remain an effective means of meeting immediate social housing need . In 2015, Kerry County Council secured the purchase of 29 housing units. In addition, under the Social Housing Current Expenditure Programme, approved housing bod- ies can purchase, lease or construct housing units and make them available for social housing; 51 new units became operational for Kerry under this programme in 2015.

The funding provided also includes allocations to local authorities under the Rental Accom- modation Scheme (RAS). The Scheme provides social housing support for persons who are in receipt of long-term rent supplement and information on the amounts paid to local authori- ties, including Kerry Council, can be found at the following link on my Department’s website: http://www.environ.ie/housing/social-housing/social-and-affordble/overall-social-housing- provision.

21/06/2016WRL02913Planning Issues

21/06/2016WRL03000236. Deputy Robert Troy asked the Minister for the Environment, Community and Local Government the number of one-off houses built in each local authority area in each year from 2011 to 2016 to date, in tabular form. [16789/16]

21/06/2016WRL03100Minister for the Environment, Community and Local Government (Deputy Simon Coveney): The numbers of one-off houses built in each local authority area from 2011 to date are as follows:

Local Authority 2011 2012 2013 2014 2015 2016 to date (April 2016) Carlow 110 72 77 78 101 24 Cavan 184 149 123 184 196 50 Clare 248 210 162 190 242 58 Cork 867 639 504 604 702 193 Donegal 451 385 296 337 362 130 D/L Rathdown 64 69 77 64 61 27 Fingal 83 75 68 115 136 54 Galway 554 397 354 390 461 149 Kerry 308 272 247 234 295 72 Kildare 205 170 143 167 224 48 Kilkenny 222 126 143 140 162 47

114 21 June 2016

Local Authority 2011 2012 2013 2014 2015 2016 to date (April 2016) Laois 120 120 108 113 98 47 Leitrim 88 62 73 62 98 26 Limerick 266 200 191 182 254 53 Longford 63 61 54 66 86 32 Louth 170 138 128 155 173 37 Mayo 349 253 244 291 302 84 Meath 266 226 164 247 280 77 Monaghan 128 110 110 95 135 34 Offaly 141 110 97 102 132 41 Roscommon 160 117 122 128 125 41 Sligo 162 134 124 106 115 39 South Dublin 43 44 44 37 64 19 Tipperary 288 224 201 219 284 69 Waterford 176 150 126 147 194 34 Westmeath 149 130 130 150 143 37 Wexford 332 229 247 259 281 101 Wicklow 180 119 142 131 161 36 City Councils Cork 42 58 92 60 70 32 Dublin 78 98 106 92 109 33 Galway 29 15 33 26 25 5 TOTALS 6,526 5,162 4,730 5,171 6,071 1,729 This data are available as part of the housing completions data published by the Central Statistics Office and are based on the number of new dwellings connected by the ESB to the electricity supply, but excludes conversions and may not accord precisely with local authority boundaries.

21/06/2016WRL03150Planning Issues

21/06/2016WRL03200237. Deputy Robert Troy asked the Minister for the Environment, Community and Local Government the national policy with regard to one-off local housing and development in the high amenity area; if there are any inconsistencies between local authorities; his plans to amend guidelines with a view to ensuring support for rural development in the interest of long-term sustainability of communities; and if he will make a statement on the matter. [16790/16]

21/06/2016WRL03300Minister for the Environment, Community and Local Government (Deputy Simon Coveney): My Department published Guidelines for Planning Authorities on Sustainable Rural Housing in April 2005 as a broad national level policy framework for local planning authorities in drawing up their statutory development plans and addressing the issue of housing in rural areas, including high amenity areas, taking into account the particular circumstances of the relevant local authority. The Guidelines require planning authorities to frame their local plan- ning policies in a balanced and measured way that ensures that the housing requirements arising from rural communities are met on the one hand , while avoiding excessive urban generated housing and ribbon development on the other hand. The Guidelines also aim at ensuring that sites developed for rural housing purposes are suitable with regard to access, wastewater dis- posal, landscape and design perspectives.

The Guidelines further elaborate on the physical development policies for rural areas as set 115 Questions - Written Answers out in the National Spatial Strategy (2002) which, inter alia, promotes sustainable rural settle- ment as a key component of delivering more balanced regional development.

It is therefore a matter for individual planning authorities to prepare their development plans and the policies contained within such plans , taking account of local and varying geographical and landscape circumstances as well as the approaches set out under the Guidelines. Further- more, it is a matter for individual planning authorities as well as An Bord Pleanála to implement the Guidelines and local planning policies within the assessment of individual planning applica- tions and appeals under the development management process.

Given the need for local authorities to respond to local circumstance s in the implementation of the Guidelines, I consider that the Guidelines strike a reasonable balance between setting out overall national policies and the application of those policies locally. Therefore, I have no plans to amend these Guidelines at present but will continue to keep them under review.

21/06/2016WRL03350Local Authority Staff Data

21/06/2016WRL03400238. Deputy Richard Boyd Barrett asked the Minister for the Environment, Community and Local Government the number of extra staff that have been employed in local authorities and to which authorities, to support the housing assistance payment and rent accommodation scheme, and to which scheme they have been allocated; and if he will make a statement on the matter. [16804/16]

21/06/2016WRL03500Minister for the Environment, Community and Local Government (Deputy Simon Coveney): Under section 159 of the Local Government Act 2001, each Chief Executive is re- sponsible for staffing and organisational arrangements necessary for carrying out the functions of the local authorities for which he or she is responsible. In this regard, it is a matter, in the first instance, for Chief Executives, to ensure that the appropriate staffing and staffing structures are in place. My Department, however, works closely with local authorities with regard to the implementation of the Government’s Social Housing Strategy and in 2015 all local authorities were invited to submit staffing sanction requests to facilitate its delivery.

Furthermore, the staffing approval process was also streamlined to ensure the fast tracking of staffing sanction requests. As a result, to date some four hundred and forty local authority posts have been sanctioned for recruitment, to support the delivery of the Social Housing Strat- egy, which include posts across the full breadth of housing support services, including support- ing existing areas of delivery and resources already deployed in schemes such as the Rental Accommodation Scheme (RAS).

Since the beginning of 2015, my Department has approved 82 staffing applications from local authorities in relation to both the Housing Assistance Payment (HAP) and RAS schemes. Four of the posts sanctioned related to the RAS scheme. A further 34 sanctioned posts related to staffing needs of the central transactional shared service being provided by Limerick City and County Council to all local authorities operating the HAP scheme. The shared service centre, which is the primary administrative support to HAP local authorities, is responsible for a range of functions including monthly payments to landlords and the collection of weekly differential rent from tenants and provides a range of efficiencies to the operation of the scheme. The re- maining 44 HAP related posts sanctioned, once recruited, would be assigned to a range of hous- ing related tasks impacted by the introduction of HAP. The table below sets out a breakdown of posts by local authority.

116 21 June 2016 Local Authority Number of Post(s) Sanctioned Carlow Co 3 Clare Co 4 Cork City 2 Galway City 4 Galway Co 4 Kildare Co 7 Limerick City & Co 34 Louth Co 7 Mayo Co 3 Meath Co 4 Offaly Co 2 Roscommon Co 1 Tipperary Co 4 Westmeath Co 3 Total 82

21/06/2016WRL03550Local Authority Staff

21/06/2016WRL03600239. Deputy Richard Boyd Barrett asked the Minister for the Environment, Community and Local Government if he received a request from Dún Laoghaire-Rathdown County Council for extra staff to facilitate the roll-out of the housing assistance payment and rent accommoda- tion scheme; and if he will make a statement on the matter. [16805/16]

21/06/2016WRL03700Minister for the Environment, Community and Local Government (Deputy Simon Coveney): My Department has not received an application from Dún Laoghaire-Rathdown County Council for extra staff to facilitate the roll-out of Housing Assistance Payment and Rental Accommodation Scheme.

21/06/2016WRL03750Local Authority Housing

21/06/2016WRL03800240. Deputy Richard Boyd Barrett asked the Minister for the Environment, Community and Local Government to provide copies of memos, instructions and directives to local authori- ties or individual communications to particular councils in relation to all aspects of housing, including but not limited to, staffing levels, emergency accommodation, building programmes, acquisitions, Part 5 and Part 8 and allocation priorities since 2011; and if he will make a state- ment on the matter. [16806/16]

21/06/2016WRL03900Minister for the Environment, Community and Local Government (Deputy Simon Coveney): The information requested is being compiled and will be forwarded to the Deputy as soon as possible.

21/06/2016WRL03950Housing Policy

21/06/2016WRL04000241. Deputy Richard Boyd Barrett asked the Minister for the Environment, Community and Local Government his plans to allow all persons in emergency homeless situations to self- accommodate; his further plans to cease to limit this option to families and to cease the require-

117 Questions - Written Answers ment that persons be put in emergency accommodation; and if he will make a statement on the matter. [16807/16]

21/06/2016WRL04100Minister for the Environment, Community and Local Government (Deputy Simon Coveney): My Department’s role in relation to homelessness involves the provision of a na- tional framework of policy, legislation and funding to underpin the role of housing authorities in addressing homelessness at local level. Statutory responsibility in relation to the provision of accommodation and related services for homeless persons, including the actual provision of housing solutions for those currently in emergency accommodation, rests with housing authori- ties.

However, I understand that the Dublin Region Homeless Executive’s placement teams ac- tively work with homeless families and individuals in emergency accommodation in order to try to provide more permanent housing solutions, including through the Homeless Housing As- sistance Payment Pilot in the Dublin Region whereby the families and individuals themselves can source available accommodation which meets their needs.

21/06/2016WRL04125Tenant Purchase Scheme Eligibility

21/06/2016WRL04200242. Deputy David Cullinane asked the Minister for the Environment, Community and Local Government if different local authorities can derive different interpretations from the implementation of the incremental purchase scheme; and if he will make a statement on the matter. [16837/16]

21/06/2016WRL04300243. Deputy David Cullinane asked the Minister for the Environment, Community and Local Government if he is aware of an anomaly that exists under the incremental purchase scheme in that an income of €15,000 per year is the minimum required to make an application but a minimum of €25,000 per year is required for tenants wishing to use the local authority house purchase loan scheme; if it is therefore possible to qualify for the 60% discount for earn- ing between €15,000 and €20,000 and to apply under the latter scheme with a €25,000 mini- mum earnings per year requirement; and if he will make a statement on the matter. [16838/16]

21/06/2016WRL04400Minister for the Environment, Community and Local Government (Deputy Simon Coveney): I propose to take Questions Nos. 242 and 243 together.

The new Tenant (Incremental) Purchase Scheme came into operation on 1 January 2016. The Scheme is open to eligible tenants, including joint tenants, of local authority houses that are available for sale under the Scheme. To be eligible, tenants must meet certain criteria, including having a minimum reckonable income of €15,000 per annum.

The new scheme is a national scheme and the terms and conditions are set down in Part 3 of the Housing (Miscellaneous Provisions) Act 2014 and prescribed in the Housing (Sale of Lo- cal Authority Houses) Regulations 2015 and the associated Ministerial Income Directions. The consideration of individual applications and the determination of eligibility under the scheme is a matter for the local authority concerned in accordance with the relevant legislative provisions and the guidance underpinning the scheme.

The financing of any house sold under the Tenant (Incremental) Purchase Scheme is a sepa- rate matter from the eligibility criteria for the scheme. In order to participate in the scheme, the tenant must, in the first instance, meet the eligibility criteria as set out in the relevant legislation, including having a minimum reckonable income of €15,000 per annum. If the tenant is deemed eligible under the scheme, he or she may fund the purchase of a house from one, or a combi- nation of, own resources or a mortgage provided by a financial institution or a local authority 118 21 June 2016 house purchase loan.

The terms and conditions governing the operation of standard annuity mortgages provided by local authorities to qualifying first time buyers are set out most recently in the Housing (Lo- cal Authority Loans) Regulations 2012. A copy of the Regulations is available at the following link: http://www.irishstatutebook.ie/eli/2012/si/408/made/en/print.

Article 5(b) and (c) of the regulations provide that loans shall not be available in the case of a single borrower, where the annual gross income is more than €50,000 nor in the case of a joint application, where the combined gross income of the borrowers is more than €75,000. The Regulations do not prescribe a minimum gross income for loan applicants.

However, all loans must be issued in accordance with the statutory Credit Policy issued in 2009 in accordance with the Housing (Local Authority Loans) Regulations 2009, in order to en- sure lending prudence and to assist local authorities across the sector to engage consistently in responsible mortgage lending, in the best interest of borrowers and local authorities alike. This Credit Policy determines that all decisions in relation to loans are made based on a borrowers’ capacity to repay the loan, credit history and savings record. The final decision on loan approval is a matter for each local authority and its credit committee on a case-by-case basis.

The new Tenant (Incremental) Purchase Scheme is in the early stages of implementation and my Department is monitoring the operation of the scheme in consultation with housing authorities. In line with the commitment in the new Programme for a Partnership Government to make the scheme more attractive for social housing tenants and to raise new funds for hous- ing development, I intend to undertake a review of the scheme following the first 12 months of operation. I will bring forward any changes to the terms and conditions of the scheme which are considered necessary based on the evidence gathered at that stage.

21/06/2016WRL04450Air Quality

21/06/2016WRL04500244. Deputy James Browne asked the Minister for the Environment, Community and Lo- cal Government if and to what extent his Department or any agency under its remit monitors the level of nitrogen dioxide in the atmosphere; the towns and cities that regularly exceed a nitrogen dioxide level of 40 micrograms per cubic metre in tabular form; and if he will make a statement on the matter. [16866/16]

21/06/2016WRL04600Minister for the Environment, Community and Local Government (Deputy Simon Coveney): Air quality monitoring in Ireland is carried out by the Environmental Protection Agency to meet the requirements of EU Directive 2008/50/EC on ambient air quality and clean- er air for Europe, known as the CAFE Directive. The Directive requires that certain minimum levels of monitoring are carried out for the purposes of assessment and management of air qual- ity. The EPA monitors a range of atmospheric pollutants, based on data obtained from the 31 monitoring stations that form the national ambient air quality network.

The EPA publishes an annual report on air quality, primarily based on the monitoring re- quirements of the CAFE Directive. The most recent report, Air Quality in Ireland 2014 – Key Indicators of Ambient Air Quality, was published in September 2015 and provides an assess- ment of air quality in Ireland for 2014, including Nitrogen Dioxide (NO2) levels, monitored at 17 locations across Ireland in 2014. NO2 values for all monitoring sites in Ireland in 2014 were below the annual limit value. Details regarding locations, annual averages and other statistics are available from the following link, with the table relating to NO2 levels found at page 52: http://www.epa.ie/pubs/reports/air/quality/Air%20Quality%20Report%202014.pdf.

119 Questions - Written Answers There has only ever been one exceedance of the limit value for NO2 in Ireland: that exceed- ance occurred at a City Centre site in Dublin in 2009.

The 2015 report on air quality in Ireland is currently in production and is due for publication in September.

21/06/2016WRM00200Local Authority Housing Data

21/06/2016WRM00300245. Deputy Peter Fitzpatrick asked the Minister for the Environment, Community and Local Government the number of capital assistance scheme housing authority properties in Dundalk, County Louth; and if he will make a statement on the matter. [16965/16]

21/06/2016WRM00400Minister for the Environment, Community and Local Government (Deputy Simon Coveney): Details on the number of housing units completed under the Capital Assistance Scheme are published by my Department as part of a wide range of housing statistics. This information is available by local authority area and is at the following links:

http://www.environ.ie/sites/default/files/attachments/1d3-cas-area-00-13.xlsx

http://www.environ.ie/sites/default/files/attachments/1d2-cas-area-14-todate_0.xlsx.

Detailed breakdown of these figures for individual towns or areas may be obtained from individual local authorities.

21/06/2016WRM00500Waste Disposal Charges

21/06/2016WRM00600246. Deputy Dessie Ellis asked the Minister for the Environment, Community and Local Government to respond to complaints from persons that waste collection companies are charg- ing fees that far exceed the existing standing charge; his plans to implement measures to regu- late private companies to prevent them charging exorbitant waste disposal charges, that in one instance amounted to a 130% increase made under the guise of the pay-by-weight legislation; and if he will make a statement on the matter. [16991/16]

21/06/2016WRM00700247. Deputy Seán Haughey asked the Minister for the Environment, Community and Lo- cal Government if he is aware that waste management companies are introducing substantial increases in bin charges with effect from 1 July 2016; the legislative basis at national or EU level for these increases; the measures he will take to ensure that the householder can obtain the best competitive price for this service, given that householders are reporting difficulties in comparing prices; and if he will make a statement on the matter. [17007/16]

21/06/2016WRM00800254. Deputy John Lahart asked the Minister for the Environment, Community and Local Government his views on the plans to introduce a new billing system by private collectors for bin collections from 1 July 2016; if he is aware of the serious public annoyance at the dramatic increases in the flat rate for collection; the increase among some companies in their weight collection charges; to explain the legislative basis to the changes and/or European Union direc- tive changes; the measures he will implement regarding a waiver system for those on social protection payments and low incomes, as promised in the programme for Government in 2011; the regulatory framework there is in place to ensure that customers are not at a disadvantage, as a result of the changes in the pricing regime; and if he will make a statement on the matter. [17190/16]

21/06/2016WRM00900Minister for the Environment, Community and Local Government (Deputy Simon 120 21 June 2016 Coveney): I propose to take Questions Nos. 246, 247 and 254 together.

Government waste policy is predicated on the waste hierarchy, as set out in the EU’s Waste Framework Directive. A number of specific measures have been, and will continue to be, intro- duced to reduce the amount of waste generated in the State and to increase the segregation of waste which cannot be prevented in line with Government policy. The introduction of pay-by- weight charges for the collection of household waste is one such measure.

In response to the pricing plans proposed earlier this month by the waste industry in rela- tion to the nationwide roll-out of pay-by-weight charging, both I and my Ministerial colleagues have highlighted significant concerns regarding the reported escalation of waste bills for some customers from July, particularly in relation to some proposed increases in service charges.

Following intense engagement and meetings with representatives of the waste industry over the last few days, the Government has agreed a way forward that results in customers paying no more than they are currently paying for waste over the next 12 months, by means of a price freeze. This transition period should give households time to adapt to the new system and learn how to more effectively manage their waste.

During this transition period, customers will be able to see details in their bills of the amount of waste they are disposing, their costs under their current price plan and details of the compara- tive pay-by-weight charge. Where households become more efficient in segregating waste and reduce their waste bills, they will have the opportunity, from 1 January 2017 at latest, to choose to transition to pay-by-weight charging or, if they prefer, they can opt to remain on their current price plan.

Over the coming months, the Government, in partnership with the waste industry, will drive an intensive public awareness, information and promotion campaign to support customers in understanding the new system, how they can change their waste management behaviour and manage better their waste costs under the pay-by-weight system.

At the end of this 12-month transition period, the Government will review the operation of pay-by-weight, including the effectiveness of the transition process, and make decisions regard- ing its further roll-out, regulation and oversight.

In relation to the issue of waiver for certain households, during the period in which local authorities were directly involved in the collection of household waste, a minority of individual Councils offered different levels of discount to selected households, based on different qualifi- cation criteria. As local authorities exited the waste collection market, some required the private operators which took on the Councils’ customers to provide a level of discount for existing waiver customers only, and even then for a limited time.

The vast majority of such contractual commitments for private operators to provide a waiver have now expired. In that context, the number of households in receipt of waiver discounts is likely to decline further over time, especially as some householders were able to take advan- tage of special reduced offers elsewhere which actually undercut the waiver price. However, selected private operators still offer some level of discount to former waiver customers on a voluntary basis.

In addition, a very limited number of local authorities make financial contributions toward the cost of the collection of waste from certain households. Again, the qualification criteria and level of support differ from area to area.

With the exception of one or two municipal districts, local authorities no longer collect waste and the market is now serviced by a diverse range of private operators, where the fees 121 Questions - Written Answers charged are primarily a matter between service provider and customer and the services offered vary across the country.

21/06/2016WRM01000Homeless Accommodation Provision

21/06/2016WRM01100248. Deputy Richard Boyd Barrett asked the Minister for the Environment, Commu- nity and Local Government if he will provide funds for emergency accommodation in Dún Laoghaire-Rathdown to prevent the relocation of homeless people to emergency shelters in the city where they are away from family and support networks. [17027/16]

21/06/2016WRM01200Minister for the Environment, Community and Local Government (Deputy Simon Coveney): My Department’s role in relation to homelessness involves the provision of a na- tional framework of policy, legislation and funding to underpin the role of housing authorities in addressing homelessness at local level. Statutory responsibility in relation to the provision of accommodation and related services for homeless persons, including emergency accommo- dation, rests with housing authorities. My Department does not fund any homeless accommo- dation or service directly but provides funding to lead housing authorities on a regional basis. Housing authorities also contribute to the funding costs for their areas from their own resources.

The various Management Groups (comprising representatives of the housing authorities within each region) of the regional Joint Homelessness Consultative Fora, are responsible for the preparation of regional statutory Homelessness Action Plans which specify the measures to be undertaken to address homelessness in their region. Section 37 of the Housing (Miscel- laneous Provisions) Act 2009 provides that such plans be informed by available data, the costs of measures and likely financial resources, as well as Government policies and objectives. It is a requirement of the 2009 Act that these Action Plans be submitted to each of the relevant hous- ing authorities for adoption by the elected representatives of the Council. Issues around service provision, including access to emergency accommodation within the Region, are integral to the Action Plans. Therefore, the provision of emergency accommodation in Dún Laoghaire–Rath- down, and funding for same, are a matter for the Council and the Dublin Region Homelessness Management Group.

21/06/2016WRM01300Land Ownership

21/06/2016WRM01400249. Deputy Michael Healy-Rae asked the Minister for the Environment, Community and Local Government the status of a query regarding the ownership of land by a person (details supplied); and if he will make a statement on the matter. [17058/16]

21/06/2016WRM01500Minister for the Environment, Community and Local Government (Deputy Simon Coveney): Solicitors acting for the person named have been in touch with my Department and have been advised by my Department that the area in question appears to be reclaimed fore- shore. They were further advised that all of Ireland’s foreshore is considered to be owned by the State , unless valid alternative title is provided.

21/06/2016WRM01600Social and Affordable Housing Provision

21/06/2016WRM01700250. Deputy Louise O’Reilly asked the Minister for the Environment, Community and Local Government if he is aware of the plan to build 24 homes adjacent to Pinewood Estate in Balbriggan; his plans to provide funding to ensure that the access road which must be built, can

122 21 June 2016 be facilitated, given unacceptable levels of traffic congestion in the Pinewood Estate; and if he will make a statement on the matter. [17061/16]

21/06/2016WRM01800Minister for the Environment, Community and Local Government (Deputy Simon Coveney): My Department has not as yet received a formal proposal for the construction of 24 social houses adjacent to Pinewood Estate, Balbriggan. The development of such a proposal, including issues around the housing provision, access, services, etc will, in the first instance, be a matter for Fingal County Council to advance. My Department will work with the local authority on receipt of a proposal in this case.

21/06/2016WRM01900Scheme to Support National Organisations

21/06/2016WRM02000251. Deputy Dara Calleary asked the Minister for the Environment, Community and Lo- cal Government why he refused SSNO funding to an organisation (details supplied) in view of the fact that it adhered to a Pobal request in 2014, and rolled out a national service; the appeal process in place; and if he will make a statement on the matter. [17069/16]

21/06/2016WRM02100Minister for the Environment, Community and Local Government (Deputy Simon Coveney): The organisation in question submitted an application to Pobal under the Scheme to Support National Organisations in the Community and Voluntary Sector. In total 148 applica- tions were received and assessed by Pobal on behalf of my Department with 70 organisations to receive funding. The standard of application received was generally of a very high quality ensuring the appraisal process was particularly competitive.

I understand that the organisation in question was deemed ineligible as it did not meet the basic eligibility requirements of the Scheme. The application was deemed not to provide suf- ficient evidence of having a national membership or extensive reach or engagement across the country in terms of the supports and services it provides.

The organisation in question should contact Pobal if it requires further information on its application.

21/06/2016WRM02200Planning Issues

21/06/2016WRM02300252. Deputy Pat Deering asked the Minister for the Environment, Community and Local Government if he will introduce the submission of planning applications and comments elec- tronically by e-mail with reference to changes to the planning process (details supplied); and if he will make a statement on the matter. [17169/16]

21/06/2016WRM02400Minister for the Environment, Community and Local Government (Deputy Simon Coveney): The Planning and Development (Amendment) Bill 2016, which was published in January 2016 and is currently on the Dáil Order Paper awaiting Second Stage, proposes to amend section 33 of the Planning and Development Act 2000 to provide that the Minister may, subsequent to the enactment of the Bill, make regulations to facilitate the online submission of planning applications and appeals, including relevant fees, by electronic means (referred to as e-planning) to planning authorities and An Bord Pleanála respectively. The Bill further pro- poses that decisions in relation to planning applications and appeals may also be dealt with by planning authorities and An Bord Pleanála respectively by electronic means.

These proposals are amongst a broad package of measures currently being developed by my Department to facilitate the streamlining of the planning process, to bring it up to date techno-

123 Questions - Written Answers logically, and to make it more administratively efficient. I would be anxious to make the neces- sary regulations in this regard at an early date, following enactment of the Bill.

21/06/2016WRM02500Homeless Accommodation Funding

21/06/2016WRM02600253. Deputy Eoin Ó Broin asked the Minister for the Environment, Community and Local Government the funding he provided to local authorities for homeless services and its percent- age of the total local Government spending on homeless services in each of the years 2008 to 2016 to date in tabular form; and if he will make a statement on the matter. [17179/16]

21/06/2016WRM02700Minister for the Environment, Community and Local Government (Deputy Simon Coveney): My Department does not fund any homeless service directly but provides funding to housing authorities towards the operational costs of homeless accommodation and related services under Section 10 of the Housing Act 1988. Under Section 10 funding arrangements, housing authorities must provide at least 10% of the cost of such services from their own re- sources. Furthermore, housing authorities may also incur additional expenditure on homeless related services outside of the Section 10 funding arrangements. Therefore, the exact amounts spent by housing authorities on homeless services are a matter for those authorities.

“Section 10” funding is provided through my Department to housing authorities on a re- gional basis; an annual breakdown for the years 2008 to 2015 is set out in the following table:

Funding Provided under Section 10 of the Housing Act 1988

Region 2008 2009 2010 2011 2012 2013 2014 2015 Dublin €38,377,003 €38,967,542 €40,521,375 €35,732,525 €32,398,290 €31,322,223 €35,322,223 €48,270,201 Mid-East €1,188,303 €885,801 €906,842 €697,665 €839,146 €811,275 €928,489 €1,716,245 Midlands €683,037 €872,375 €666,344 €624,897 €672,027 €649,707 €649,707 €805,995 Mid-West €3,254,800 €4,110,166 €3,435,607 €2,905,457 €3,516,119 €3,399,336 €3,405,060 €3,499,898 North-East €1,128,576 €1,197,451 €1,037,617 €912,250 €910,194 €879,963 €879,963 €996,849 North-West €593,286 €444,206 €423,355 €406,335 €360,865 €348,879 €348,879 €387,643 South-East €1,789,704 €1,945,792 €1,742,886 €2,110,476 €1,848,877 €1,787,469 €1,818,969 €2,394,619 South-West €4,415,916 €5,458,211 €4,270,143 €3,505,749 €4,418,967 €4,272,197 €4,307,772 €5,000,686 West €1,804,375 €2,175,312 €1,699,288 €1,227,696 €1,581,478 €1,528,951 €1,544,712 €1,698,834 Total €53,234,999 €56,056,856 €54,703,457 €48,123,050 €46,545,963 €45,000,000 €49,205,774 €64,770,970 The national budget for 2016 is confirmed at €70 million. While the 2016 regional alloca- tions are currently being finalised, the funding needs of the various regions are kept under review on an on-going basis, taking account of known and emerging costs of services, and my Department has provided reassurances to all housing authorities in this regard.

Question No. 254 answered with Question No. 246.

21/06/2016WRM02900Waste Disposal

21/06/2016WRM03000255. Deputy Peadar Tóibín asked the Minister for the Environment, Community and Lo- cal Government if he is satisfied that there are sufficient top soil disposal locations to service the construction sector in the greater Dublin area; and if he is satisfied with the costs of disposal involved. [17201/16]

21/06/2016WRM03100Minister for the Environment, Community and Local Government (Deputy Simon Coveney): The Eastern-Midlands Regional Waste Management Plan published in May 2015 indicated at the time , based on 2012/2013 available data , that there was sufficient available ca- 124 21 June 2016 pacity for Construction and Demolition Waste. However, there has been a significant increase in activity in the sector over the last two years and additional capacity will be required to meet current and future demands.

The Regional Plan supports balanced development to meet both Local and Regional needs with a preference for larger sites with longer lifespans. A detailed analysis by the 3 Regional Waste Management Planning Offices of the Construction & Demolition Waste capacity is cur- rently being undertaken.

With regard to the determination of prices at such facilities, this is a contractual matter be- tween individual facilities and their customers.

Question No. 256 answered with Question No. 229.

21/06/2016WRM03300Road Projects Expenditure

21/06/2016WRM03400257. Deputy Peadar Tóibín asked the Minister for the Environment, Community and Lo- cal Government if he will ensure that the €200 million earmarked for infrastructural investment is used for the building of two major roads around the town of Navan, County Meath (details supplied). [17207/16]

21/06/2016WRM03500Minister for the Environment, Community and Local Government (Deputy Simon Coveney): I announced recently the Government’s approval to the establishment of a Local Infrastructure Fund of €200 million to relieve critical infrastructural blockages to enable the accelerated delivery of housing on key development sites and improve the economic viability of new housing projects in Dublin and in urban areas of high demand for housing.

The Fund will be composed of an Exchequer allocation of €150 million, matched by a €50 million contribution from participating local authorities. Funding is to be allocated to local authorities, following a call for proposals on a competitive bid process basis which would be disposed to early delivery of the requisite additional housing at scale, including at affordable prices.

It will be a matter for local authorities to determine their bid proposals to identify the prior- ity development sites which are proposed to be activated for housing in the immediate and short term through the removal of the infrastructural blockages.

The detailed arrangements on the implementation of the funding are being finalised by my Department, in conjunction with the Department of Public Expenditure and Reform.

21/06/2016WRM03600Proposed Legislation

21/06/2016WRM03700258. Deputy Clare Daly asked the Minister for the Environment, Community and Local Government if he will consider introducing legislation to halt evictions from family homes until solutions to the housing crisis are in place. [17227/16]

21/06/2016WRM03800259. Deputy John Halligan asked the Minister for the Environment, Community and Local Government if he has considered urgently legislating for a moratorium on family home evic- tions until such time as a reasoned solution to the family housing crisis is formulated by the Houses of the Oireachtas; and if he will make a statement on the matter. [17236/16]

21/06/2016WRM03900Minister for the Environment, Community and Local Government (Deputy Simon

125 Questions - Written Answers Coveney): I propose to take Questions Nos. 258 and 259 together.

The Residential Tenancies Act 2004 regulates the landlord-tenant relationship in the private rented residential sector and sets out the rights and obligations of landlords and tenants. Secu- rity of tenure under the Act is based on rolling four-year tenancy cycles. Where a tenant has been in occupation of a dwelling for a continuous period of 6 months and no notice of termina- tion has been served in respect of that tenancy before the expiry of the period of 6 months, the tenancy is established for the remainder of the four year period. This is referred to in the Act as a ‘Part 4’ tenancy.

A landlord may not terminate a Part 4 tenancy except on clearly defined grounds which are set out in the Table to section 34 of the Act. These grounds include that the landlord intends to sell the dwelling, that the landlord wishes to change the use of the dwelling, and that the land- lord requires the dwelling for a family member.

Amendments introduced in the Residential Tenancies (Amendment) Act 2015 strengthen the protections around tenancy terminations by providing for measures that will guard against, for example, landlords falsely declaring that the property is needed for a family member, or that it is going to be sold. In addition, the Act provides for graduated increases in the notice periods that must be given to tenants of the termination of a tenancy so that a landlord must now give a tenant up to a maximum of 224 days’ notice for tenancies of 8 years or more.

Housing is an absolute priority for this Government and, accordingly, a key priority of the Programme for a Partnership Government is the preparation and publication of an ‘Action Plan for Housing’ within the Government’s first 100 days. The Action Plan will be drafted with input from a number of key Departments, and will draw on the Report of the Special Oireachtas Committee on Housing and Homelessness, which published its final report on the 17 June 2016. This plan will map out a vision for the evolution of the entire housing sector over the coming years, including the rental sector where over one-fifth of our population now resides.

For the rental sector specifically, the Programme for a Partnership Government contains a specific commitment to review the regulatory regime for the rented sector to ensure that an ap- propriate balance is struck between the rights, interests and responsibilities of both tenants and landlords. The Action Plan for Housing will also contain a commitment to publish a strategy for the rental sector this Autumn. This strategy will chart a path for the future of the rental sec- tor. It will also list the wide ranging actions this Government will take to achieve a mature and stable rented sector which will appropriately balance the rights and responsibilities of landlords and tenants, protect the rights of vulnerable tenants, reflect our changing patterns of housing demand and support continued recovery for our modern economy.

21/06/2016WRM04000Building Regulations

21/06/2016WRM04100260. Deputy Dessie Ellis asked the Minister for the Environment, Community and Local Government if he is aware that over the past number of years, a clerk of works has not been mandatory on site, and if he will reintroduced this measure in the interest of better building regulations; and if he will make a statement on the matter. [17255/16]

21/06/2016WRM04200Minister of State at the Department of the Environment, Community and Local Gov- ernment (Deputy Damien English): In broad terms, the role of a clerk of works is to monitor the quality of work on site, check and supervise construction details, record progress and report to the project design team. The Building Control (Amendment) Regulations 2014 which came into effect on 1 March 2014, introduced requirements for the appointment of design certifiers

126 21 June 2016 and assigned certifiers. The role of the assigned certifier is to draw up and execute an appropri- ate inspection plan, in conjunction with the builder and the project team, and to jointly certify with the builder that the building is in compliance with the Building Regulations when com- plete.

Whereas heretofore local authorities, who frequently employed clerks of works on local authority construction projects, were exempt from building control requirements, this general exemption was removed under the Building Control (Amendment) (No. 2) Regulations 2015 with effect from 1 September 2015 following a review of the 2014 Regulations after 12 months in operation.

In response to this change , my Department has written to all local authorities outlining the revised funding arrangements that will apply typically in respect of social housing projects funded by the Department. The revised funding arrangements make provision for the addi- tional costs associated with the new design certifier and assigned certifier roles now required under Building Control Regulations. These inspection and certification roles are a statutory obligation unlike the non-statutory system of inspections previously undertaken by the clerks of works.

The design and assigned certifier roles must be undertaken by registered construction pro- fessionals (i.e. an architect, a building surveyor or a chartered engineer), who may be directly employed by the local authority or whose services may be procured by way of public tender. It is unlikely that persons who previously undertook the role of clerk of work will be in a position to undertake the design or assigned certifier role although some clerks of works may be quali- fied to do so.

Staffing and resourcing issues are in the first instance a matter for the Chief Executive of each local authority. Standard funding arrangements communicated by my Department to lo- cal authorities are necessarily based around the general arrangements that typically apply to construction projects. If, in relation to specific projects, circumstances exist which require alternative approaches, these can be considered on a case by case basis by the Department in consultation with the local authority concerned. Reasonable and appropriate proposals that represent good value for money having regard to the public funds invested will always receive due consideration.

21/06/2016WRM04300Departmental Schemes

21/06/2016WRM04400261. Deputy Aindrias Moynihan asked the Minister for the Environment, Community and Local Government the timeframe for decisions on the national taking-in-charge initiative measure 2 schemes recently submitted by local authorities, and if adequate funding is in place in measure 2 for the schemes put forward. [17258/16]

21/06/2016WRM04500Minister for the Environment, Community and Local Government (Deputy Simon Coveney): A fund of €10m has been made available from the proposed carry-over of capital savings in 2015 to be used in 2016 to fund a new initiative to accelerate the taking-in-charge of estates, irrespective of the wider development context that they are located within.

Local authorities were invited to submit proposals based on an indicative allocation of €3.5m proposed to be made available to seven local authorities (Cork County, Clare, Donegal, Galway County, Kerry, Tipperary and Wexford County Councils) for demonstration projects under Measure 2 relating to housing estates served by Developer Provided Infrastructure (DPI) based on an earlier pilot study, although consideration may also be given to other counties

127 Questions - Written Answers within the indicative funding available.

The experience gained through the initiative in 2016 and associated demonstration projects is intended to establish a co-ordinated taking-in-charge programme to progressively address the issue on a multi-annual basis, in the light of available local authority, Irish Water, bond and other funding sources and the general government financial position.

Consideration of the submissions put forward by the local authorities on the basis of the provisional funding levels available will be weighted towards those local authorities that dem- onstrate a high level of collaborative problem solving and leveraging of complimentary funding from developer, bond provider, receiver or other sources.

My Department is currently considering all applications and I expect to be in a position to award funding allocations to local authorities shortly.

21/06/2016WRM04600Regeneration Projects Expenditure

21/06/2016WRM04700262. Deputy Dessie Ellis asked the Minister for the Environment, Community and Local Government the cost of the urban regeneration scheme in Ballymun, Dublin 11. [17259/16]

21/06/2016WRM04800Minister for the Environment, Community and Local Government (Deputy Simon Coveney): Over the period 1999 to the end of 2015, my Department provided over € 764 mil- lion to Ballymun Regeneration Limited and Dublin City Council towards the physical, social and economic regeneration of Ballymun.

While investment in the physical regeneration of Ballymun is largely complete, my Depart- ment is continuing to support certain social and economic interventions in Ballymun over the medium term with funding allocated on the basis of agreed annual work programmes.

21/06/2016WRM04900Regeneration Projects

21/06/2016WRM05000263. Deputy Dessie Ellis asked the Minister for the Environment, Community and Local Government the number and status of urban regeneration schemes. [17260/16]

21/06/2016WRM05100264. Deputy Dessie Ellis asked the Minister for the Environment, Community and Local Government the cost of planned further regeneration projects. [17261/16]

21/06/2016WRM05200Minister for the Environment, Community and Local Government (Deputy Simon Coveney): I propose to take Questions Nos. 263 and 264 together.

Regeneration projects being funded by my Department target the country’s most disadvan- taged communities, including those defined by the most extreme social exclusion, unemploy- ment and anti-social behaviour. My Department currently supports a programme of large-scale regeneration projects in Dublin, Cork and Limerick and smaller projects in Tralee, Sligo and Dundalk. The 2016 allocation for existing commitments on regeneration projects is €50 mil- lion.

The regeneration project at Limerick has been in operation for over seven years, with a cu- mulative investment to date of some €245 million. It is expected that 2016 will see significant building works, which will include the major part of a contract for 83 new social housing units. The regeneration project at Cork City is continuing with 23 new social houses delivered during 2015 and work on a further 23 units commenced in 2016. Regeneration works consisting of

128 21 June 2016 major refurbishment and renewal in Dundalk and Sligo are ongoing, while Tralee regeneration is largely complete. In the Dublin City area, two new large regeneration projects, Dolphin House and St. Teresa’s Gardens, are expected to move to construction in 2016.

I am confident that in the context of the priority on urban regeneration set out in the Pro- gramme for a Partnership Government, I will be in a position to provide increased funding in 2017 and subsequent years for a range of measures that address deep-rooted disadvantage, while developing an approach to urban regeneration that empowers people to work together to improve their communities, to reduce poverty, disadvantage and inequality.

21/06/2016WRM05300Environmental Policy

21/06/2016WRM05400265. Deputy Dessie Ellis asked the Minister for the Environment, Community and Local Government his policy on urban regeneration. [17262/16]

21/06/2016WRM05500Minister for the Environment, Community and Local Government (Deputy Simon Coveney): The new Programme for Partnership Government sets out the ambitious priority attached to urban regeneration by the new Government, with a series of specific actions set out therein as to how it is proposed to facilitate the regeneration of our urban centres.

In this context, I have established a working group of officials (comprising senior represen- tatives from my Department, local authorities and other relevant actors ) to bring forward pro- posals for new urban regeneration measures as early as possible. In this regard, it is intended that the new measures will complement the existing regeneration programme under the Social Housing Capital Programme, as well as other social regeneration initiatives already under way. The Group meetings will be chaired by my colleague, Minister of State Damien English, TD, who has responsibility for urban renewal policy.

The series of specific actions set out in the Programme for Partnership Government are examples of the Government’s commitment and approach to regenerating our urban centres, many of which were adversely impacted by our recent economic difficulties. In this context, the Government will seek to introduce a new Town and Village Renewal Scheme to support the revitalisation of our towns and villages and improve the living and working environment of communities. Using the €30 million available to local authorities for town and village re- generation from this year, it is envisaged that the scheme will, among other things, particularly seek to increase the attractiveness and sustainability of our towns and villages as places to live and work.

In addition to this new Scheme, it is also intended under the new Programme for Partnership Government to examine a series of further initiatives, including:

- the introduction of a similar scheme to the ‘Living City Initiative’ to regenerate urban cen- tres and villages throughout Ireland (Year 1 action),

- the establishment of a national register of derelict sites, in addition to the new vacant site levy, to bring vacant and underutilised sites into beneficial use for housing and urban regenera- tion purposes (Year 2 action),

- the mandating of local authorities with better land management powers (Year 2 action),

- reclassifying and incentivising the use of underutilised or vacant areas over ground floor premises in urban areas, for both residential and commercial use (Year 1 action), and

129 Questions - Written Answers - examining the scope to reform the Derelict Sites Act to tackle the under-use and hoarding of derelict land by the State, semi-State and private sectors (Year 1 action).

In terms of existing measures already in place, my Department’s current Regeneration Pro- gramme targets the country’s most disadvantaged communities, i.e. those defined by the most extreme social exclusion, unemployment and anti-social behaviour. My Department currently supports an ambitious programme of large-scale regeneration projects in Dublin, Cork and Limerick, encompassing social housing improvements as well as social/community and eco- nomic renewal, as well as a number of smaller-scale projects in Tralee, Sligo and Dundalk. As outlined, the aim of the programme is to seek to address the causes of disadvantage in these communities through a holistic programme of physical, social and economic regeneration.

Under the current Programme, the 2016 allocation for existing commitments on regenera- tion projects is €50 million. In the Dublin City area, two new large regeneration projects (Dolphin House and St. Teresa’s Gardens) are expected to move to construction under the Pro- gramme in 2016, with a number of further regeneration projects in the pipeline.

In the context of the priority being attached to urban regeneration by the current Govern- ment, I will be aiming to provide increased funding in 2017 and beyond for a range of measures that will address deep-rooted disadvantage, while simultaneously developing an approach to urban regeneration that empowers people to work together to improve their communities, to reduce poverty, disadvantage and inequality. It is intended that these new measures will also include strengthening the Social Inclusion and Community Activation Programme (SICAP) as well as reactivating funding to areas covered by the RAPID (Revitalising Areas through Plan- ning, Investment and Development) Programme through the local authorities.

21/06/2016WRM05600Social and Affordable Housing Provision

21/06/2016WRM05700266. Deputy Niall Collins asked the Minister for the Environment, Community and Local Government if he will expedite a social housing development which has being ongoing over seven years (details supplied); why this process is taking such a long time; and if he will make a statement on the matter. [17266/16]

21/06/2016WRM05800Minister for the Environment, Community and Local Government (Deputy Simon Coveney): The site in question, at Glin, County Limerick, was accepted into the Land Aggre- gation Scheme in 2011. Under that Scheme, the Housing Agency is responsible for engaging with local authorities, and approved housing bodies, regarding the appropriate development of the lands, prior to seeking the necessary approval from the Minister for the Environment, Com- munity and Local Government.

The Housing Agency wrote to my Department requesting the sale of the land to the ap- proved housing body for the proposed development. In order for this proposal to be considered, my Department sought clarification from the Housing Agency on the number of houses to be delivered, the timeframe for delivery, and that the proposal was in line with Limerick County Council’s social housing delivery programme.

The clarifications have been received and I expect that a decision on the proposal will be made in the coming days.

Question No. 267 answered with Question No. 230.

Question No. 268 answered with Question No. 229.

130 21 June 2016

21/06/2016WRN00400Carer’s Allowance Applications

21/06/2016WRN00500269. Deputy Pearse Doherty asked the Minister for Social Protection the status of an ap- plication by a person (details supplied) under the carer’s allowance scheme; and if he will make a statement on the matter. [16792/16]

21/06/2016WRN00600Minister for Social Protection (Deputy Leo Varadkar): I confirm that my department received an application for carer’s allowance from the person concerned on 24 February 2016. Additional information in relation to the person’s application, which was omitted by the ap- plicant, has been requested by a deciding officer. Once the information is received the applica- tion will be processed without delay and the person concerned will be notified directly of the outcome.

I hope this clarifies the matter for the Deputy.

21/06/2016WRN00700Carer’s Allowance Applications

21/06/2016WRN00800270. Deputy Pearse Doherty asked the Minister for Social Protection the status of an ap- plication by a person (details supplied) under the carer’s allowance scheme; and if he will make a statement on the matter. [16793/16]

21/06/2016WRN00900Minister for Social Protection (Deputy Leo Varadkar): The application for carer’s al- lowance in respect of the person concerned was awarded on 13 June 2016 and the first payment is due to issue to the person’s nominated post office on 23 June 2016.

Arrears of allowance due from 24 March 2016 have issued to the post office for payment on 23 June 2016. The person concerned was notified of these details on 13 June 2016.

I hope this clarifies the matter for the Deputy.

21/06/2016WRN01000Training Support Grant

21/06/2016WRN01100271. Deputy Tom Neville asked the Minister for Social Protection the State supports avail- able by way of training under FÁS or any other avenues that can be explored to provide nec- essary training for a person (details supplied); and if he will make a statement on the matter. [16820/16]

21/06/2016WRN01200Minister for Social Protection (Deputy Leo Varadkar): The Training Support Grant (TSG) provided by the Department of Social Protection is designed to fund quick access to short-term training where this cannot be provided by a state provider within a reasonable time or where a intervention is identified that can support individual jobseekers to access work op- portunities.

The scheme is activation focused and the jobseeker is required to provide reasonable evi- dence or grounds of need (for example in the form of a potential job offer) or a requirement to maintain or achieve accreditation, for example, safe pass, driving licences certain certification such as required for the catering sector (HACCP) or the security industry permit.

The maximum fund available to jobseekers under the scheme is €500 p.a. Access to the scheme is via the Department’s case officers.

The training sought by the jobseeker, such as driving lessons must form part of the agreed 131 Questions - Written Answers personal progression or action plan for the individual to enter/re-enter the labour market.

In this case the person concerned should contact their case officer to discuss their options.

I hope this clarifies the matter for the Deputy.

21/06/2016WRN01300Carer’s Allowance Applications

21/06/2016WRN01400272. Deputy Peter Burke asked the Minister for Social Protection to expedite an applica- tion by a person (details supplied) under the carer’s allowance scheme; and if he will make a statement on the matter. [16656/16]

21/06/2016WRN01500Minister for Social Protection (Deputy Leo Varadkar): The application for carer’s al- lowance in respect of the person concerned was awarded on 16 June 2016 and the first payment is due to issue to the person’s nominated post office on 23 June 2016.

Arrears of allowance due from 7 January 2016 have issued to the post office for payment on 23 June 2016.

The person concerned was notified of these details on 16 June 2016.

I hope this clarifies the matter for the Deputy.

21/06/2016WRN01600Community Employment Schemes Places

21/06/2016WRN01700273. Deputy Kevin O’Keeffe asked the Minister for Social Protection if he will permit all persons on community employment schemes who are over 60 years of age, where desired, to continue on the scheme until they reach retirement age given the difficulties they will meet in endeavouring to secure alternative employment. [16657/16]

21/06/2016WRN01800Minister for Social Protection (Deputy Leo Varadkar): Community Employment (CE) is a labour market activation programme which helps long-term unemployed people (those 12-months or more in receipt of a jobseeker’s payment) and other disadvantaged groups to re-enter the active workforce by breaking their experience of unemployment through a return to work routine. It is designed to be a temporary fixed-term activation intervention. The pro- gramme assists participants to enhance and develop both their technical and personal skills which can then be used in the workplace.

The CE programme is open to participants aged up to 65 years of age who meet the current eligibility criteria. A pilot initiative was introduced in December 2015 to enable those aged 62 and over to extend their participation beyond the standard maximum participation limits (cur- rently set at 6 years cumulative lifetime participation) providing the eligibility conditions are met. Participants aged 62 and over, are allowed to participate on a continuous basis up to the state pension age, subject to satisfactory performance on the scheme and to annual approval by the Department. The places allocated for these participants within each individual CE scheme are limited to 7% of each schemes overall budgeted places.

I hope this clarifies the matter for the Deputy.

21/06/2016WRN01900State Pension (Contributory)

132 21 June 2016

21/06/2016WRN02000274. Deputy Clare Daly asked the Minister for Social Protection further to Parliamentary Question No. 9 of 30 September 2015, if he will take steps to address the anomaly in how PRSI contributions are calculated for pension purposes in order to correct the anomaly that in some cases women who were subject to the marriage bar and who worked thereafter, are not entitled to a full contributory pension and are worse off than if they had not worked at all prior to the lifting of the marriage bar. [16677/16]

21/06/2016WRN02100Minister for Social Protection (Deputy Leo Varadkar): The ‘marriage bar’ describes a rule that existed in most of the public service and some private sector employments, where women were required to leave their employment upon marriage. This practice was abolished in 1973 when Ireland joined the EEC. As employees in the public service generally paid a reduced rate of PRSI, which provided no cover for the State pension (contributory), the marriage bar would not generally have impacted on State pension entitlement, as they would not have quali- fied for that payment had they remained in public sector employment.

Instead, by impacting upon their continuing public service employment, the marriage bar’s pension implications, where they exist, more generally relate to a person’s eventual entitlement to a Public Service pension. Any questions regarding this issue are a matter for the Minister for Public Expenditure and Reform.

To qualify for a state pension (contributory) a person must have at least 520 paid contribu- tions and satisfy a yearly average condition (a yearly average of 48 contributions paid or cred- ited is required for a full rate State pension (contributory), and reduced rates of payment may be payable for pensioners with lower averages). Once over 16 years of age, the date a person enters into insurable employment is the date used for averaging purposes.

For those who have insufficient contributions to qualify for a full State pension (contribu- tory), there are supports available in the overall State pension system which assists qualification for a contributory payment, based on factors such as the contributions made by their spouse, and/or other factors likely to impact upon their needs. These include –

- The Homemaker’s scheme, which was introduced to make qualification for State pension (contributory) easier for those who take time out of the workforce for caring duties.

- Increases for Qualified Adults.

- Credits for periods of unemployment and illness .

- For those who do not qualify for a full rate contributory pension, they may qualify for a means-tested non-contributory pension of up to 95% of the full contributory pension.

The most recently published Actuarial Review of the Social Insurance Fund confirms that the Fund provides better value to female rather than male contributors. It further confirms that those with lower earnings and those with shorter contribution histories, mostly women, have and will continue to obtain the best value for money from the Fund due to the distributive nature of the Fund.

It is expected that a total contributions approach to pension qualification will replace the yearly average contributions test for State pension (contributory) for new pensioners from 2020. The aim of this approach is to make the rate of contributory pension more closely match contributions made by a person, while dealing with anomalies inherent in the yearly average approach. This is a very significant reform with considerable legal, administrative, and tech- nical elements in its implementation. When proposals are agreed, legislation will be brought forward to underpin the necessary changes. It is important that the changes be announced well in advance of introduction, to enable those affected to include the new factors into their retire- 133 Questions - Written Answers ment planning.

21/06/2016WRN02200Community Employment Schemes Administration

21/06/2016WRN02300275. Deputy Clare Daly asked the Minister for Social Protection the amount of money be- ing spent by community employment schemes on mandatory health and safety training, which in many instances can absorb the entire training allowance; and his plans to enable this training to be funded separately, as it leaves very little else for other training which would assist in the employability of community employment participants; and if he will make a statement on the matter. [16682/16]

21/06/2016WRN02400Minister for Social Protection (Deputy Leo Varadkar): The annual CE training budget for 2016 is €6m. The Sponsor as the employer must comply with the relevant health and safety legislation. Over 8,000 components of learning in mandatory CE training (including health and safety training) were completed in 2015. A breakdown of expenditure for health and safety training is not feasible due to the variation in fees and type of award and the fact that a propor- tion of these awards incur no costs.

A training grant is allocated to community employment (CE) schemes in respect of all approved places on a “per-place per annum” basis. Training is in accordance with a tailored individual learning plan (ILP) compiled by the CE supervisor with the participant and designed to promote progression opportunities given the participant’s training requirements and employ- ment goals. Flexibility can be applied in approval of training rates, particularly when the train- ing is leading to a major award e.g. in childcare. All training is approved at Divisional level. Any proposed training that exceeds the budget allocated is assessed based on the value of the training provided to furthering the employment prospects of the individual.

I hope this clarifies the matter for the Deputy.

21/06/2016WRN02500Jobseeker’s Allowance Payments

21/06/2016WRN02600276. Deputy John Brady asked the Minister for Social Protection if the report and review on the impact of the reduced jobseeker payment rates for jobseekers 18 to 25 years of age, as committed to in the Pathways to Work 2016-2020 strategy has commenced; and if not, when it will be undertaken. [16698/16]

21/06/2016WRN02700Minister for Social Protection (Deputy Leo Varadkar): Reduced rates for younger job- seeker’s allowance recipients were first introduced in 2009. Budget 2014 further extended the reduced rates to recipients under 26 years of age. This is a targeted measure aimed at protecting young people from welfare dependency. To guard against the development of welfare depen- dency I believe that it is necessary to provide young jobseekers with a strong financial incentive to engage in education or training or to take up employment. If a young jobseeker in receipt of the reduced jobseeker’s allowance rate participates on an education or training programme they will receive a higher weekly payment of €160.

The Youth Guarantee sets a medium-term objective of ensuring that all young people re- ceive an offer of employment within four months of becoming unemployed. There is now monthly engagement with all young jobseekers. Youth unemployment rose rapidly in the reces- sion to over 30% in 2012. According to the Quarterly National Household Survey for Quarter 1 2016, youth unemployment continues to fall in line with the overall recovery and now stands at 16.9%. 134 21 June 2016 The review of jobseeker’s allowance rates for young persons under 26 years of age will examine the effectiveness of the reduced rates in encouraging young jobseekers to avail of education, training, employment programmes and opportunities. Some preliminary analysis has commenced and I hope that this review will be completed later this year, subject to the neces- sary data being available and the required level of analysis involved being completed.

I am committed to ensuring my Department identifies effective measures to incentivise and support young people in finding and securing sustainable jobs. The best way to do this is through engagement processes and by incentivising them to avail of educational and training opportunities, thereby enhancing their employment prospects.

21/06/2016WRN02800Domiciliary Care Allowance Applications

21/06/2016WRN02900277. Deputy Michael Healy-Rae asked the Minister for Social Protection the status of an application by a person (details supplied) under the domiciliary care scheme; and if he will make a statement on the matter. [16702/16]

21/06/2016WRN03000Minister for Social Protection (Deputy Leo Varadkar): An application for domiciliary care allowance (DCA) was received in respect of the person concerned on the 24th March 2016. This application has been forwarded to one of the Department’s Medical Assessors for their medical opinion. Following receipt of this opinion, a decision will be made by a Deciding Officer and notified to the applicant. Applications are processed in date of receipt order. It can currently take 12 weeks to process an application for DCA.

I hope this clarifies the matter for the Deputy.

21/06/2016WRN03100Invalidity Pension Applications

21/06/2016WRN03200278. Deputy Tom Neville asked the Minister for Social Protection his views on the case of a person (details supplied) regarding payments from his Department; and if he will make a statement on the matter. [16704/16]

21/06/2016WRN03300Minister for Social Protection (Deputy Leo Varadkar): Upon review, my Department is satisfied to treat the application for disability allowance (DA) made by the person concerned in 2014 as equivalent to a claim for invalidity pension (IP) having been made at that time.

Consequently, IP has now been awarded to the person concerned from 10 July 2014, i.e. from the first Thursday following receipt of his original DA application. Payment for the period 10 July 2014 to the 22 April 2015 will issue to his nominated bank account on the 23 June 2016. The person in question was notified of this decision on the 20 June 2016.

I hope this clarifies the matter for the Deputy.

21/06/2016WRN03400Carer’s Allowance Applications

21/06/2016WRN03500279. Deputy Pat Breen asked the Minister for Social Protection when he will issue a deci- sion to a person (details supplied) under the carer’s allowance scheme; and if he will make a statement on the matter. [16768/16]

21/06/2016WRN03600Minister for Social Protection (Deputy Leo Varadkar): The application for carer’s al-

135 Questions - Written Answers lowance in respect of the person concerned was awarded on 15 June 2016 and the first payment is due to issue to the person’s nominated bank account on 23 June 2016.

Arrears of allowance due from 3 December 2015 have also issued to the bank.

The person concerned was notified of these details on 15 June 2016.

I hope this clarifies the matter for the Deputy.

Invalidity Pension Appeals

21/06/2016WRN03800280. Deputy Pat Deering asked the Minister for Social Protection when he will issue a decision on an application by a person (details supplied) under the invalidity pension scheme. [16775/16]

21/06/2016WRN03900Minister for Social Protection (Deputy Leo Varadkar): The Social Welfare Appeals Office has advised me that the appeal from the person concerned was referred to an Appeals Officer who has decided to hold an oral hearing in this case on 29th June 2016. The person concerned has been notified of the arrangements for the hearing.

The Social Welfare Appeals Office functions independently of the Minister for Social Pro- tection and of the Department and is responsible for determining appeals against decisions in relation to social welfare entitlements.

I hope this clarifies the matter for the Deputy.

21/06/2016WRN04000Disability Allowance Appeals

21/06/2016WRN04100281. Deputy Richard Boyd Barrett asked the Minister for Social Protection the reason a person (details supplied) was refused payment under the disability allowance scheme; and if he will make a statement on the matter. [16786/16]

21/06/2016WRN04200Minister for Social Protection (Deputy Leo Varadkar): The person in question appealed to the independent Social Welfare Appeals Office (SWAO) a decision by a deciding officer to disallow his application for disability allowance (DA). The deciding officer disallowed his claim on the grounds that he failed to show his means did not exceed the statutory limit.

Following due consideration, the appeal of the person in question was disallowed by an appeals officer (AO) on 24 November 2015. He was notified of this decision in writing by the SWAO on the same date.

An AO’s decision is final and conclusive in absence of any fresh facts or evidence.

An initial examination of the PRSI contribution record of the person concerned indicates that he would not qualify for invalidity pension based on the PRSI contributions paid within this State. However, if he has worked and paid social insurance contributions within the EU he may qualify based upon his EU record. It is open to him to apply for IP so that his eligibility may be definitively ascertained.

Also, he may reapply for DA if he feels that he would satisfy the means test and is prepared to cooperate in that regard.

Under the legislation governing the means test for DA, when a deciding officer is assessing 136 21 June 2016 means for a property that is not the primary residence of the person, the net capital value of the property is assessed whether or not the property is rented out. Full information in relation to the means test and the DA scheme is available on my Department’s website www.welfare.ie.

I trust this clarifies matters for the deputy.

21/06/2016WRN04300Disability Allowance Applications

21/06/2016WRN04400282. Deputy Michael Healy-Rae asked the Minister for Social Protection the status of an application by a person (details supplied) under the disability allowance scheme; and if he will make a statement on the matter. [16788/16]

21/06/2016WRN04500Minister for Social Protection (Deputy Leo Varadkar): My Department informs me that there is no record of an application for disability allowance from the person in question. If the person in question wishes to make an application, he should fully complete and submit an ap- plication form (DA1) so that his eligibility may be determined.

If the person in question has submitted an application for DA in the last few days, there is no need to reapply and the Department will be in touch with him in due course.

I trust that this clarifies the matter for the Deputy.

21/06/2016WRN04600Carer’s Allowance Applications

21/06/2016WRN04700283. Deputy Michael Healy-Rae asked the Minister for Social Protection the status of an application by a person (details supplied) under the carer’s allowance scheme; and if he will make a statement on the matter. [16808/16]

21/06/2016WRN04800Minister for Social Protection (Deputy Leo Varadkar): I confirm that my department re- ceived an application for carer’s allowance (CA) from the person concerned on 2 March 2016. Unfortunately, there are currently delays in the processing of new applications. Additional resources have been provided to the CA section in order to improve the waiting times for new applications and they are working hard to make this happen.

Frequently, delays are outside the control of the Department and are caused by the customer failing to fully complete the claim form or failing to attach the supporting documentation that is requested on the application form.

This application will be processed as quickly as possible and the person concerned will be notified directly of the outcome. In the meantime, the person concerned is in receipt of a weekly social welfare support.

I hope this clarifies the matter for the Deputy.

21/06/2016WRN04900Carer’s Allowance Applications

21/06/2016WRN05000284. Deputy Bernard J. Durkan asked the Minister for Social Protection the status of an application by a person (details supplied) under the carer’s allowance scheme; and if he will make a statement on the matter. [16817/16]

21/06/2016WRN05100Minister for Social Protection (Deputy Leo Varadkar): I confirm that my department

137 Questions - Written Answers received an application for carer’s allowance (CA) from the person concerned on 23 May 2016. Unfortunately, there are currently delays in the processing of new applications. Additional resources have been provided to the CA section in order to improve the waiting times for new applications and they are working hard to make this happen.

Frequently, delays are outside the control of the Department and are caused by the customer failing to fully complete the claim form or failing to attach the supporting documentation that is requested on the application form.

This application will be processed as quickly as possible and the person concerned will be notified directly of the outcome.

In the meantime, if the means of the person concerned are insufficient to meet her needs she should apply for a means-tested supplementary welfare allowance from her local community welfare service.

I hope this clarifies the matter for the Deputy.

21/06/2016WRN05200Social Welfare Offices

21/06/2016WRN05300285. Deputy John Brady asked the Minister for Social Protection his views on the future of the social welfare office in Swinford, County Mayo; if he can assure people in east County Mayo that there are no plans to close the office; and if he will make a statement on the matter. [16818/16]

21/06/2016WRN05400Minister for Social Protection (Deputy Leo Varadkar): The current manager of the So- cial Welfare Branch Office in Swinford, County Mayo has tendered his resignation with effect from 30th July 2016. The Department is examining the provision of services in the area and no decision has been made on the future of this office at this time. The Deputy will be kept informed of any future decision.

21/06/2016WRN05500Training Support Grant

21/06/2016WRN05600286. Deputy Brendan Griffin asked the Minister for Social Protection the additional fund- ing he will make available under the training support grant when a person’s retraining costs exceed €500; and if he will make a statement on the matter. [16819/16]

21/06/2016WRN05700Minister for Social Protection (Deputy Leo Varadkar): The Training Support Grant (TSG) is designed to fund quick access to short-term training where this cannot be provided by a state provider within a reasonable time or where a intervention is identified that can support individual jobseekers to access work opportunities.

The scheme is activation focused and the jobseeker is required to provide reasonable evi- dence or grounds of need (for example in the form of a potential job offer) or a requirement to maintain or achieve accreditation, for example, safe pass, driving licences certain certification such as required for the catering sector (HACCP) or the security industry permit. Access to the scheme is via the Department’s case officers. A maximum amount of €500 may be made available to a jobseeker within a 12 month period, where appropriate. There are no proposals at present to increase this amount.

I hope this clarifies the matter for the Deputy

138 21 June 2016

21/06/2016WRN05800Carer’s Allowance Applications

21/06/2016WRN05900287. Deputy Pat Breen asked the Minister for Social Protection when he will issue a deci- sion to a person (details supplied) under the carer’s allowance scheme; and if he will make a statement on the matter. [16831/16]

21/06/2016WRN06000Minister for Social Protection (Deputy Leo Varadkar): I confirm that my department received an application for carer’s allowance from the person in question on the 3 December 2015. The application was referred to a local social welfare inspector (SWI) to assess the level of care being provided, assess means and confirm that all the conditions for receipt of carer’s allowance are satisfied. Once the SWI has reported, a deciding officer will make a full decision and the person concerned will be notified directly of the outcome.

I hope this clarifies the matter for the Deputy.

Question No. 288 withdrawn.

21/06/2016WRO00200Child Benefit Eligibility

21/06/2016WRO00300289. Deputy Sean Fleming asked the Minister for Social Protection if he will consider extending child benefit to students who are in full-time secondary education even though they have reached 18 years of age, given that it is a very expensive time for parents and families when the students are coming up to leaving certificate to withdraw this payment; and if he will make a statement on the matter. [16867/16]

21/06/2016WRO00400Minister for Social Protection (Deputy Leo Varadkar): Child Benefit is a monthly pay- ment made to families with children in respect of all qualified children up to the age of 16 years. The payment continues to be paid in respect of children up to their 18th birthday who are in full-time education, or who have a disability.

Child Benefit is currently paid to around 624,000 families in respect of some 1.2 million children, with an estimated expenditure in the order of over €2 billion in 2016. Child Benefit is an important source of income for all families and in Budget 2016 the Government increased Child Benefit by €5 per month, at a cost of €72 million.

Budget 2009 reduced the age for eligibility for Child Benefit from 19 years to less than 18 years. A value for money review of child income supports, published by the Department of Social Protection in 2010, found that the participation pattern of children in education supports the current age limit for Child Benefit.

Families on low incomes can avail of a number of provisions to social welfare schemes that support children in full-time education until the age of 22, including:

- qualified child increases (IQCs) with primary social welfare payments;

- family income supplement (FIS) for low-paid employees with children;

- the back to school clothing and footwear allowance for low income families (paid at the full-time second level education rate).

These schemes provide targeted assistance that is directly linked with household income and thereby supports low-income families with older children participating in full-time education.

Extending Child Benefit to students who are in full-time secondary education who are over 139 Questions - Written Answers 18 years of age can be examined between now and the budget with the caveat that there are potentially other areas of need that also have to be accounted for.

Any changes to the eligibility criteria for Child Benefit, therefore, would have to be consid- ered in an overall budgetary context.

21/06/2016WRO00500Pensions Reform

21/06/2016WRO00600290. Deputy Michael Healy-Rae asked the Minister for Social Protection the status of the retirement age for persons (details supplied); and if he will make a statement on the matter. [16905/16]

21/06/2016WRO00700Minister for Social Protection (Deputy Leo Varadkar): The Social Welfare and Pensions Act 2011 provided that State pension age will be increased gradually to 68 years. This began in January 2014 with the abolition of the State pension (transition) available from 65 for those who satisfied the qualifying conditions, thereby standardising State pension age for all at 66 years, which is the current State pension age. This will increase to 67 in 2021 and 68 in 2028. The changes introduced in 2011 were on foot of a Government commitment included in the Na- tional Recovery Plan published in 2010, and in the subsequent Memorandum of Understanding with the EU/ECB/IMF

Each year more people are living to pension age and living longer in retirement. As a result of this demographic change, the number of State pension recipients is increasing by approxi- mately 17,000 annually. This has significant implications for the future costs of State pension provision which are currently increasing by close to €1 billion every 5 years. The purpose of changes to the State pension age is to make the pension system more sustainable in the context of increasing life expectancy.

The Deputy should note that there is no general retirement age in the State, and the age at which employees retire is a matter for the contract of employment between them and their employers.

In terms of financial supports, social welfare benefits will continue to be available to the age of 66 for those who are required to leave employment. Jobseekers whose benefit expires in their 65th year will continue to be paid benefit up until the age of 66. Where a jobseeker’s benefit claim spans two benefit years, a new Governing Contribution Year requirement is not applied to the second benefit year of a claimant aged 65 (effectively this means that they may receive payment in both years based upon eligibility in the first year).

21/06/2016WRO00800Invalidity Pension Applications

21/06/2016WRO00900291. Deputy Michael Ring asked the Minister for Social Protection when he will issue a decision on an application by a person (details supplied) under the invalidity pension scheme; and if he will make a statement on the matter. [16906/16]

21/06/2016WRO01000Minister for Social Protection (Deputy Leo Varadkar): The person concerned has been awarded IP with effect from the 21 April 2016. Payment will issue to his nominated bank ac- count on the 07 July 2016. Any arrears due from the 21 April 2016 to the 06 July 2016 (less any overlapping social welfare payment and/or outstanding overpayment) will issue in due course. The person in question was notified of this decision on the 17 June 2016.

I hope this clarifies the matter for the Deputy. 140 21 June 2016

21/06/2016WRO01100Jobseeker’s Allowance Payments

21/06/2016WRO01200292. Deputy Thomas Byrne asked the Minister for Social Protection why a person (details supplied) was awarded the jobseeker’s allowance from December 2015, rather than the date of application, which was September 2015. [16909/16]

21/06/2016WRO01300Minister for Social Protection (Deputy Leo Varadkar): The person concerned applied for a jobseeker’s payment on 30 September 2015, 5 November 2015 and 22 December 2015, and at the time of making each application they were requested to submit proof of residency. As this documentation was not submitted, each application closed without going into payment. Proof of residency was submitted to the Intreo Office concerned on 19 February and jobseeker’s allowance was awarded from 22 December 2015 to 20 February 2016 when the applicant com- menced employment.

The person concerned has been advised by their Intreo Office to provide clarification as to his ore her residency and circumstances prior to 22 December in order to establish whether an entitlement to a jobseeker’s payment existed for that period. When this requested clarification is provided we will be in a position to make a decision in respect of the earlier periods.

I trust this clarifies the matter for the Deputy.

Social Welfare Benefits Waiting Times

21/06/2016WRO01500293. Deputy Michael Healy-Rae asked the Minister for Social Protection the current wait- ing times of processing all social protection applications at present; and if he will make a state- ment on the matter. [16951/16]

21/06/2016WRO01600Minister for Social Protection (Deputy Leo Varadkar): The information requested (where available) by the Deputy is detailed in the following tabular statement.

Social Welfare claims by average waiting times - 31 May 2016

Scheme Average Waiting Time (weeks) Jobseeker’s Benefit 1 Jobseeker’s Allowance 2 One-Parent Family Payment 5 State Pension Contributory (Dom) 6 Widow’s, Widower’s or Surviving Civil 1 Partner’s Contributory Pension Widowed Parent Grant 1 Living Alone/Island Allowances N/a State Pension Non-Contributory 14 Household Benefits 2 Free Travel 2 Domiciliary Care Allowance 14 Supplementary Welfare Allowance 1 Child Benefit (Domestic & FRA) 2 Child Benefit (EU Regulation) 32 Treatment Benefit N/a

141 Questions - Written Answers Scheme Average Waiting Time (weeks) Maternity Benefit N/a Family Income Supplement (New) 3 Family Income Supplement (Renew) N/a Carer’s Allowance 22 Disability Allowance 10 Invalidity Pension 12 Illness Benefit 1 Occupational Injury Benefit N/a

Carer’s Allowance Appeals

21/06/2016WRO01800294. Deputy Michael Healy-Rae asked the Minister for Social Protection the status of an application by a person (details supplied) under the carer’s allowance scheme; and if he will make a statement on the matter. [16956/16]

21/06/2016WRO01900Minister for Social Protection (Deputy Leo Varadkar): I am advised by the Social Wel- fare Appeals Office that an Appeals Officer, having fully considered all of the available evi- dence, has decided to allow the appeal of the person concerned by way of a summary decision. The person concerned has been notified of the Appeals Officer’s decision.

The Social Welfare Appeals Office functions independently of the Minister for Social Pro- tection and of the Department and is responsible for determining appeals against decisions in relation to social welfare entitlements.

I hope this clarifies the matter for the Deputy.

21/06/2016WRO02000Community Employment Schemes Funding

21/06/2016WRO02100295. Deputy Seán Haughey asked the Minister for Social Protection to reverse the cuts to the community employment scheme at a community programme (details supplied) introduced in 2012 in respect of the training and materials budget; and if he will make a statement on the matter. [16960/16]

21/06/2016WRO02200Minister for Social Protection (Deputy Leo Varadkar): Each Community Employment (CE) scheme has an individually-set weekly materials rate per participant based on a detailed appraisal of their actual annual expenditure levels in previous years.

Similarly a training grant is allocated to schemes in respect of all approved places on a “per- place per annum” basis. Training is in accordance with a tailored individual learning plan com- piled by the CE supervisor with the participant, given the participant’s training requirements and employment goals. Any proposed training that exceeds the budget allocated is assessed based on the value of the training provided to furthering the employment prospects of the indi- vidual. There is flexibility that can be applied in approval of training rates, particularly when the training is leading to a major award e.g. in childcare.

All expenditure must be accounted and vouched for by way of receipts. It is the responsibil- ity of the scheme to ensure that they have obtained the best price and that they have achieved value for money. While the Department is not in a position to allocate additional monies to the 2016 budget, if there is a CE scheme experiencing particular difficulties the sponsor should

142 21 June 2016 contact their local DSP office and the matter will be reviewed on a case by case basis.

I hope this clarifies the matter for the Deputy.

21/06/2016WRO02300Carer’s Allowance Applications

21/06/2016WRO02400296. Deputy Michael Healy-Rae asked the Minister for Social Protection the status of an application by a person (details supplied) under the carer’s allowance; and if he will make a statement on the matter. [16987/16]

21/06/2016WRO02500Minister for Social Protection (Deputy Leo Varadkar): The application for carer’s al- lowance in respect of the person concerned was awarded on 15 June 2016 and the first payment is due to issue to the person’s nominated bank account on 23 June 2016.

Arrears of allowance due from 31 March 2016 have issued to the bank for payment on 23 June 2016.

The person concerned was notified of these details on 15 June 2016.

I hope this clarifies the matter for the Deputy.

Carer’s Allowance Payments

21/06/2016WRO02700297. Deputy Michael Ring asked the Minister for Social Protection why he has refused a person’s (details supplied) request to have a carer’s allowance payment backdated to the date that the caring commenced; and if he will make a statement on the matter. [16997/16]

21/06/2016WRO02800Minister for Social Protection (Deputy Leo Varadkar): I confirm that my department received an application for carer’s allowance (CA) from the person concerned on 15 May 2015. The application was awarded from 21 May 2015 which was the first Thursday after the date of receipt of the application.

The person concerned requested a review of the date of award of this decision but no grounds for the request were submitted. A deciding officer (DO) refused this request for back-dating and the person in question was notified of this decision on 4 April 2016.

In order for back-dating to be considered, it is important for an applicant to supply the DO with the reasons for the late submission of their CA application and the grounds upon which they wish the application to be back-dated. If the person in question wishes the decision on back-dating of his CA application to be reviewed by the Department, he should submit as soon as possible the reasons and grounds for the late submission of his application for CA so a DO may reconsider the back-dating request.

I hope this clarifies the matter for the Deputy.

Social Welfare Code

21/06/2016WRO03000298. Deputy John Brady asked the Minister for Social Protection the cost of allowing lone parents in employment whose children are between seven and 14 years of age to receive the jobseeker’s transition payment and the family income supplement if they meet the qualifying criteria. [16999/16]

143 Questions - Written Answers

21/06/2016WRO03100Minister for Social Protection (Deputy Leo Varadkar): The jobseeker’s transitional pay- ment (JST) is available to lone parents (both former one-parent family payment recipients and new lone parents), who have a youngest child aged 7 to 13 years inclusive. These customers are exempt from the jobseeker’s allowance conditions that require them to be available for, and genuinely seeking, full-time work.

While it is a condition of the JST scheme that recipients must continue to parent alone, this is not a qualifying condition of the family income supplement (FIS) and so this information is not maintained for FIS recipients. From the data currently available on household composition within the FIS scheme, it is not possible to determine which FIS recipients, who are also lone parents, would be eligible for JST, given the specific conditionality, which applies to the JST scheme.

Social Welfare Code

21/06/2016WRO03300299. Deputy John Brady asked the Minister for Social Protection the cost of making the jobseeker’s transitional payment and the Student Universal Support Ireland grant payable to lone parents who are undertaking an educational or training course for the duration of the course regardless of the age of the youngest child in the family or whether the family is in re- ceipt under the rent supplement scheme. [17000/16]

21/06/2016WRO03400Minister for Social Protection (Deputy Leo Varadkar): In order to provide the costings requested by the Deputy it would require my Department to anticipate the number of lone parents who intend to undertake a course of education where the course will continue beyond their youngest child reaching 14 years of age. My Department would not have access to these projections and therefore cannot provide these costings.

However, the Deputy should note that the current supports provided, outlined below, ensure that lone parents can complete a course of education with support from my Department regard- less of the age of their youngest child.

Former one-parent family payment (OFP) recipients who transitioned from the scheme on 2 July 2015 and who were in the middle of a course of education were allowed to complete their course on OFP and then transition to an appropriate payment.

From 2 July 2015 onwards lone parents who transition to the jobseeker’s transitional pay- ment (JST) have a youngest child of 7 years of age. These customers then have two options if they wish to complete a course of education while on JST. Where the course can be completed within the 7 years that they have access to JST (i.e. until their youngest child is 14 years of age), these customers can avail of the SUSI grant and a JST payment subject to the conditions of both schemes (eligibility for a SUSI grant is a matter for my colleague, the Minister for Education and Skills).

Where the course cannot be completed within the potentially 7 years available on JST, for example if the course is over 3 years and the youngest child is now 12, the customer can instead avail of the Back to Education Allowance (BTEA) when s/he commences the course. This al- lowance will continue to be paid, subject to the conditions of the BTEA scheme, until the course is completed even when the customer’s youngest child reaches 14 years of age.

All lone parents on a jobseeker’s allowance or JST payment have access to activation sup- ports from my Department.

JST recipients receive for the first time a one to one meeting with a case officer from my 144 21 June 2016 Department who will assist them to produce a personal development plan and guide them to- wards appropriate education, training and employment opportunities. While the customer is on the JST payment this support is available and is not limited to the 12 month engagement that applies for other jobseekers from their one to one meeting.

Through the JST payment, lone parents with children aged between 7 and 13 years are pro- vided with a very long transition period of seven years within which to engage with the Depart- ment’s Intreo service. The aim of this broader support is to improve the individual’s education and employment prospects.

With regard to rent supplement, in general, persons engaged in full-time education are ex- cluded from receiving rent supplement. However, those participating in approved courses un- der the back-to-education allowance (BTEA) or the vocational training opportunities scheme (VTOS) may retain entitlement to rent supplement, subject to meeting all other conditions of the scheme.

21/06/2016WRO03500Disability Act Employment Targets

21/06/2016WRO03600300. Deputy Catherine Connolly asked the Minister for Social Protection the extent to which he has implemented the comprehensive employment strategy for people with disabilities; the resources he has allocated to it; and if he will make a statement on the matter. [17008/16]

21/06/2016WRO03700Minister for Social Protection (Deputy Leo Varadkar): The Comprehensive Employ- ment Strategy for people with disabilities (CES) sets out the Government’s commitment to increasing the employment opportunities for people with a disability.

The strategy reflects a cross-government approach that brings together various measures and actions to be taken by different Departments and state agencies in a concerted effort to address the barriers and challenges that impact on the employment of people with disabilities. The overall CES is being coordinated by the Department of Justice Equality and Law Reform (DJELR).

The aim of the strategy is to increase the proportion of people with disabilities in a job. The measures contained in the strategy seek to address the barriers to employment and improve pathways to work, with the impact on employment building up over time.

The specific DSP commitments in the CES include: enhanced engagement with people with disabilities (drawing on profiling analysis etc.); provision of employment support services (Wage Subsidy Scheme, EmployAbility service etc.); engagement with employers to highlight benefits of employing people with disabilities; early intervention strategies to address drift into ill-health; the establishment of an interdepartmental group, to be led by DSP, under an inde- pendent chair, to review and make recommendations on ‘making work pay’ for people with disabilities.

I am pleased to inform the Deputy that my Department is making progress in delivering on its commitments under the CES, for example, my Department is progressively rolling out its full Intreo service to people with disabilities who wish to avail of the service on a voluntary basis. People with disabilities who present at INTREO Centres offices will be offered an in- terview by a case officer who will agree suitable personal progression plan with the individual and record the action plan for the person on the case management system. In addition, relevant officers in INTREO Centres have received training, to enable them to interact effectively with people with disabilities.

145 Questions - Written Answers This service is being provided across 58 Intreo Centres. Training of over 60 Employment Support officers and other INTREO support staff was completed in March 2016. Training in- cluded inputs from the National Disability Authority and Epilepsy Ireland.

Other actions completed include, the review, by Indecon, of the EmployAbility and DACT services. A survey, conducted by my Department, of a random sample 12,000 people in receipt of disability allowance. The survey is to inform the design of supports and interventions which will assist people with disabilities in receipt of Disability Allowance to participate in the labour force according to their capacity.

In addition my Department has established an interdepartmental expert group, with an in- dependent chair to examine the complex interaction between the benefits system, including the medical card, and the net income gains in employment. The interdepartmental group will also identify any significant disincentives for people with disabilities in taking up or returning to work. The interdepartmental group will report to Government by the end 2016.

Finally, my Department promotes workplace supports for people with disabilities and en- gages with employers on an ongoing basis. The Employer Information Pack contains informa- tion on supports available to people with disabilities and employers and the range of supports and services available to employers are highlighted at DSP employer road shows and Job fairs.

I hope this gives the Deputy an indication of the progress being made by my Department in meeting its CES commitments.

21/06/2016WRO03800Fit for Work Programme

21/06/2016WRO03900301. Deputy Catherine Connolly asked the Minister for Social Protection the nature of work competency assessment in respect of the fit for work programme, including by whom such assessments will be carried out; the criteria for such assessments; and if he will make a statement on the matter. [17009/16]

21/06/2016WRO04000302. Deputy Catherine Connolly asked the Minister for Social Protection the timeframe for the introduction of the fit for work programme; and if he will make a statement on the mat- ter. [17010/16]

21/06/2016WRO04100303. Deputy Catherine Connolly asked the Minister for Social Protection the penalties or sanctions he will apply to persons who do not engage with the fit for work programme; and if he will make a statement on the matter. [17011/16]

21/06/2016WRO04200304. Deputy Catherine Connolly asked the Minister for Social Protection the Department or agency or private sector operator, that will have responsibility for the implementation of the fit for work programme; and if he will make a statement on the matter. [17012/16]

21/06/2016WRO04300Minister for Social Protection (Deputy Leo Varadkar): I propose to take Questions Nos. 301 to 304, inclusive, together.

The recently agreed programme for a partnership government provides for a range of ac- tions that are designed to improve the quality of life for people with disabilities. One of these actions is a proposal for the Departments of Health and Social Protection to work together to pursue a “Fit for Work Programme” to support more people with an illness or disability to get back to work through early intervention.

The “Fit for Work” proposal is based on the findings of a pan European study, which exam-

146 21 June 2016 ined the impact of musculoskeletal disorders (MSDs) on an individual’s ability to work. The Irish module of this study was progressed by a coalition of key stakeholders and was led by Arthritis Ireland. The Irish College of General Practitioners, ICTU, IBEC and the Health & Safety Authority also contributed to the study.

Early interventions and developing return to work practices in the case of people with mus- culoskeletal incapacities is consistent with my Department’s approach to illness/disability in- come support, which aims to reduce the number of people progressing to chronic disability and long-term social welfare dependency. To this end, my Department has in 2015, issued a set of certification guidelines for GPs, which sets out defined periods of recovery for common medi- cal conditions, including MSDs.

The certification guidelines build on the “Renaissance project”, which is an initiative of my Department since 2003. This has shown that early intervention reduced the incidence of pro- gression from the acute simple low back pain to chronic disability in 64% of claimants.

While specific proposals around the design and operation of a “Fit for Work Programme” require further development and scoping out so at this time it is not possible to answer all your question, I would like to assure the Deputy that any such proposals will be in line with the wealth of evidence which shows that generally employment is good for one’s mental and physi- cal health and wellbeing and, conversely, that unemployment is damaging.

I hope this clarifies the matter for the Deputy.

21/06/2016WRO04400Carer’s Allowance Payments

21/06/2016WRO04500305. Deputy Michael Ring asked the Minister for Social Protection when he will issue correspondence confirming the payment rate to a person (details supplied) under the carer’s al- lowance scheme; and if he will make a statement on the matter. [17029/16]

21/06/2016WRO04600Minister for Social Protection (Deputy Leo Varadkar): I can confirm that my Depart- ment has issued a statement on 8 June 2016 which gives details of carer’s allowance payments made to the person concerned for the year ended 2015.

I hope this clarifies the matter for the Deputy.

21/06/2016WRO04700Carer’s Allowance Applications

21/06/2016WRO04800306. Deputy Bernard J. Durkan asked the Minister for Social Protection the status of an application for carer’s allowance for a person (details supplied); and if he will make a statement on the matter. [17051/16]

21/06/2016WRO04900Minister for Social Protection (Deputy Leo Varadkar): I confirm that my department received an application for carer’s allowance (CA) from the person concerned on 6 April 2016. Unfortunately, there are currently delays in the processing of new applications. Additional resources have been provided to the CA section in order to improve the waiting times for new applications and they are working hard to make this happen.

Frequently, delays are outside the control of the Department and are caused by the customer failing to fully complete the claim form or failing to attach the supporting documentation that is requested on the application form.

147 Questions - Written Answers This application will be processed as quickly as possible and the person concerned will be notified directly of the outcome.

In the meantime, the spouse of the person concerned is in receipt of a weekly social welfare support for both of them.

I hope this clarifies the matter for the Deputy.

21/06/2016WRO05000Carer’s Allowance Applications

21/06/2016WRO05100307. Deputy Lisa Chambers asked the Minister for Social Protection why there has been a delay of over three months in assessing the carer’s allowance application for a person (details supplied) and if he can ensure a response is issued as soon as possible. [17165/16]

21/06/2016WRO05200Minister for Social Protection (Deputy Leo Varadkar): Carer’s Allowance (CA) is a social assistance payment made to persons who are providing full-time care and attention to a relevant person/persons and whose income falls below certain limits.

My department informs me that, to date, no application by the person concerned for CA has been received/registered and that they have been in touch with your office directly to clarify to whom this question relates so that you may receive an update.

I hope this clarifies the matter for the Deputy.

21/06/2016WRO05300Carer’s Allowance Applications

21/06/2016WRO05400308. Deputy Michael Ring asked the Minister for Social Protection when he will reach a decision on a carer’s allowance application for a person (details supplied). [17172/16]

21/06/2016WRO05500Minister for Social Protection (Deputy Leo Varadkar): I confirm that my department received an application for carer’s allowance (CA) from the person concerned on 11 February 2016.

The application was referred to a local social welfare inspector (SWI) to assess the level of care being provided, assess means and confirm that all the conditions for receipt of carer’s al- lowance are satisfied. Once the SWI has reported, a deciding officer will make a full decision and the person concerned will be notified directly of the outcome.

I hope this clarifies the matter for the Deputy.

21/06/2016WRO05600Community Employment Schemes Places

21/06/2016WRO05700309. Deputy Charlie McConalogue asked the Minister for Social Protection why a person (details supplied) who has been offered a job on a community employment scheme is unable to accept it; and if he will make a statement on the matter. [17174/16]

21/06/2016WRO05800Minister for Social Protection (Deputy Leo Varadkar): The person concerned partici- pated in a TÚS Scheme following which he was selected for referral to the JobPath service and he completed the initial engagement process with JobPath. He also applied to participate in a Community Employment (CE) scheme.

148 21 June 2016 While TÚS and CE schemes are intended to help long-term unemployed people to re-enter the active workforce, by breaking their experience of unemployment through a return to work routine for a limited duration, they are not full-time sustainable jobs.

Participation on a CE scheme immediately on completion of a TÚS scheme is subject to re- view and approval of a Department case officer. In such cases, the case officer must be satisfied that participation on the CE scheme will improve the client’s employment prospects. In making this assessment, the case officer takes account of other options, including JobPath, available to support the client progress into employment.

I have asked my officials to review the case in question so as to ensure that the person concerned is provided with the most appropriate opportunity of securing full-time sustainable employment.

I hope this clarifies the matter for the Deputy.

21/06/2016WRO05900Community Services Programme

21/06/2016WRO06000310. Deputy Charlie McConalogue asked the Minister for Social Protection if he has in- vited any organisations to make an application to the community services programme; if not, when he expects to do so; and if he will make a statement on the matter. [17188/16]

21/06/2016WRO06100Minister for Social Protection (Deputy Leo Varadkar): The community services pro- gramme (CSP) is designed to address gaps in the delivery of key local services, to tackle dis- advantage and to ensure that community facilities are utilised. It provides valuable resourcing for service delivery undertaken by not-for-profit companies and cooperatives in communities around the country. There are some 3,000 people in full and part time posts which are sup- ported in employment by the programme across approximately 400 not-for-profit companies. Funding of just over €45m has been provided for the programme in 2016 which will maintain the current level of activity.

Service providers must operate community or social enterprises that deliver tangible ser- vices and are capable of generating non-public revenues from their operations by way of charg- ing fees, sales and/or fundraising. The programme is not intended to represent full funding for any operation.

Given the level of resources available and the number of companies currently supported the Department has been unable to make an open call for new proposals to the programme for a number of years. However, from time to time as resources allow, the Department does process new applications received from organisations that have expressed an interest in and are con- sidered eligible under the programme. Any not-for-profit company wishing to be considered for the programme can send an expression of interest to the Department by email to cspinfo@ welfare.ie.

I trust that this clarifies the matter for the Deputy.

Farm Assist Scheme

21/06/2016WRP00200311. Deputy Brendan Smith asked the Minister for Social Protection his proposals to re- verse the changes to the farm assist scheme that were introduced in budgets 2012 and 2013; if he is aware that these changes have had a very negative effect on many low income farmers; if he will give consideration to the immediate reversal of those measures, in view of the serious 149 Questions - Written Answers income difficulties across all farm sectors at present; and if he will make a statement on the matter. [17313/16]

21/06/2016WRP00300Minister for Social Protection (Deputy Leo Varadkar): The farm assist scheme pro- vides support for farmers on low incomes and is similar to jobseeker’s allowance. Farm assist recipients retain the advantages of the jobseeker’s allowance scheme such as the retention of secondary benefits and access to activation programmes. The 2016 Revised Estimates for my Department provide for expenditure of approximately €85 million on the farm assist scheme.

Changes introduced in Budgets 2012 and 2013 brought farm assist into closer alignment with the jobseeker’s allowance scheme’s treatment of self-employed people. Farm families with the lowest income were least impacted by these changes as the headline rates of farm as- sist were maintained.

The Programme for Government contains the commitment to undertake a “Review of the Farm Assist Scheme, recognising the challenges facing farmers on low incomes”. On foot of this commitment I have asked my officials to review the farm assist scheme. However, any changes to the scheme will have to be considered in a budgetary context.

Pension Provisions

21/06/2016WRP00400312. Deputy David Cullinane asked the Minister for Social Protection the status of an ap- peal by a person (details supplied) under the Waterford Crystal pension scheme; and if he will make a statement on the matter. [17315/16]

21/06/2016WRP00500Minister for Social Protection (Deputy Leo Varadkar): The mediated agreement for for- mer Waterford Crystal workers covers 1,774 members of the Waterford Crystal Pension Factory and Staff Schemes. During the mediation process it was agreed by the parties that a dispute resolution procedure would be set up whereby disputes in relation to individual cases would be referred to a dispute resolution process under the auspices of the Workplace Relations Commis- sion. The dispute resolution procedure is solely and exclusively for individual disputes con- cerning personal employment information particular to the individual used for the calculation of lump sums and monies in respect of pensionable service as set out in the letter of offer and/ or deed of release including scheme service dates and calculations.

It is my understanding that the Dispute Resolution Officer has examined all cases which have been submitted to him. The Department has not received any recommendation from the Dispute Resolution Officer in relation to the named person. Any change in the terms of the offer made to any individual, which were based on the records of the Administrator of the schemes, can only be made on the provision of documentary evidence to the increased entitle- ment claimed. I understand that the Administrator examined this case thoroughly and found no evidence of a normal retirement age of 63 applying.

The finalisation of all aspects of the mediated agreement is a matter of priority, both for me and for the Department.

I hope this clarifies the matter for the Deputy.

Human Rights

21/06/2016WRP00600313. Deputy John Paul Phelan asked the Minister for Foreign Affairs and Trade the action he will take to highlight the situation of the Rohingya people, displaced from their homeland in 150 21 June 2016 Myanmar; and if he will make a statement on the matter. [16693/16]

21/06/2016WRP00700Minister for Foreign Affairs and Trade (Deputy Charles Flanagan): I welcome the democratic transition which is currently underway in Myanmar following the historic elections which took place there in November 2015. The transition is still in its early phases, and there are many challenges facing the new Government in its efforts to bring peace and reconciliation, democratic reforms, and development and economic growth to all its people. I welcome the positive steps that the new Government has taken to date which have signalled their intention to address these issues. One of the most critical and sensitive of these challenges involves the need to comprehensively address the situation of the Rohingya people.

Ireland continues to follow the situation of the Rohingya minority closely, including those that have been displaced from their homes, many of whom remain within the country. The un- derlying root causes of this displacement, which include the ethnic tensions in Rakhine State, must be addressed as part of a long term solution led by the Government of Myanmar. The need to guarantee respect for human rights and the rule of law for all people in Myanmar remains central to achieving this. Ireland has also called for more investment in early recovery and sup- port for livelihood opportunities for both the Rohingya and Buddhist communities in Rakhine State.

Ireland regularly highlights the situation of the Rohingya people at a bilateral level and in the context of our membership of the EU. Yesterday, I attended the Foreign Affairs Council in Brussels, at which conclusions were adopted on the EU’s strategy on Myanmar. In addition to addressing the wider human rights concerns, the Conclusions made particular reference to the situation of the Rohingya, taking note of the efforts of the Government of Myanmar to begin work towards addressing the challenges of Rakhine State. The conclusions furthermore high- lighted the need for inclusive development in all areas and a political process, including solving citizenship for stateless persons in a non-discriminatory manner through a transparent, volun- tary and accessible procedure and ending displacement. The Council committed to engage in dialogue with the Union and State governments as well as all local actors to help build trust with the ultimate aim of promoting human rights and prosperity for all.

Ireland also supported the EU-led resolution on the situation of human rights in Myanmar at the 31st session of the UN Human Rights Council (HRC) in March 2016, which emphasised the HRC’s serious concerns over human rights violations, including the situation in Rakhine State and particularly with respect to persons belonging to religious and ethnic minorities, including the Rohingya minority. Ireland also raised this issue during the review of Myanmar’s human rights record at the HRC’s Universal Periodic Review in November 2015. We have read with interest the report of the United Nations High Commissioner for Human Rights, Zeid Ra’ad Al Hussein, on the human rights situation of Rohingya Muslims and other minorities in Myanmar, which was published yesterday and will be presented at the current session of the HRC in Ge- neva on 29 June.

In addition to these efforts, Ireland has provided development assistance and humanitarian aid to address the needs of people in Rakhine State, including the Rohingya community. Since October 2012 outbreaks of inter-communal violence in Rakhine State have left 140,000 people internally displaced (IDPs) and 70,000 extremely vulnerable people in need of humanitarian assistance. In this context of continued conflict and hardship, it is essential that access to basic education and essential psycho-social support to nearly 15,000 children in IDP camps contin- ues. To this end, since 2014, through our Embassy in Hanoi, Ireland has provided €234,500 to Plan International for their interventions in Rakhine State. Ireland’s contribution specifically supports the much needed provision of water, sanitation and hygiene education in the schools and safe spaces for children which complements and strengthens Plan’s broader education pro- 151 Questions - Written Answers gramme. In addition, 2015, Plan International Ireland received €50,000 under the Emergency Response Fund Scheme (ERFS) for its support to communities in Rakhine State that were af- fected by the floods of July 2015. As part of this project 20 villages of Minbya Township were selected in collaboration with the Township authorities. Across the 20 targeted villages, a total of 4,407 households have increased access to safe drinking water.

Ireland will continue, through relevant bilateral and multilateral channels, to highlight the situation of the Rohingya people and to urge the authorities in Myanmar to address the underly- ing causes as part of their continuing transition.

21/06/2016WRP00750Trade Fairs

21/06/2016WRP00800314. Deputy Robert Troy asked the Minister for Foreign Affairs and Trade if he will pro- vide details of the next upcoming trade fair in China, including the date and venue, the areas of expertise specifically dealt with in the eligibility criteria for businesses wishing to take part and the steps that must be taken. [16907/16]

21/06/2016WRP00900Minister for Foreign Affairs and Trade (Deputy Charles Flanagan): As the world’s sec- ond largest economy, with a population of approximately 1.35 billion people, there are numer- ous trade fairs taking place in China each day. Some of the most well-known trade fairs that take place in China this year include the 120th Canton Fair, the 22nd China Yiwu Commodities Fair and Bauma China 2016.

Officials in my Department are not aware of any forthcoming trade fair that is being or- ganised or part-organised by Irish companies in China. Nonetheless, the Irish Embassy in Beijing, Irish Consulate Generals in Shanghai and Hong Kong, as well as Irish businesses and institutions often take part in trade fairs in China. For example, the Consulate General in Hong Kong will participate in the Hong Kong International Education Expo (HKIEE) Education Fair to be held there from 9-10 July 2016 along with Enterprise Ireland and some Irish third level institutes. It is also expected that Irish organisations will participate in the China Education Expo which takes place in Beijing on 22 October 2016 where Ireland will be awarded the title of Country of Honour.

Officials in my Department remain available to provide you with the details of any specific trade fair in which Irish businesses or organisations might seek to take part in China upon re- quest.

Military Aircraft Landings

21/06/2016WRP01000315. Deputy Clare Daly asked the Minister for Foreign Affairs and Trade the steps he has taken to ascertain whether the sharp increase in military aircraft of the US landing and overfly- ing Shannon Airport in County Clare between 4 June and 12 June 2016, (details supplied) was linked to the North Atlantic Treaty Organization Annaconda military exercises in Poland; and if he will make a statement on the matter. [16949/16]

21/06/2016WRP01100Minister for Foreign Affairs and Trade (Deputy Charles Flanagan): I refer to the list of aircraft movements provided by the Deputy and note that it comprises overflights and landings by military and civil aircraft. The Air Navigation (Foreign Military Aircraft) Order 1952 gives the Minister for Foreign Affairs primary responsibility for the regulation of activity by foreign military aircraft in Ireland while the Minister for Transport, Tourism and Sport has primary responsibility for the regulation of foreign civil aircraft in Ireland. I shall therefore confine my 152 21 June 2016 reply to the activity of foreign military aircraft referred to by the Deputy.

Permission must be sought in advance for landings by all foreign military aircraft, includ- ing US aircraft and, if granted, is subject to strict conditions. These include stipulations that the aircraft must be unarmed, carry no arms, ammunition or explosives and must not engage in intelligence gathering, and that the flights in question must not form part of military exercises or operations.

I can confirm that all applications submitted by the US Embassy concerning landings by US military aircraft for the period covered by the Deputy’s list stated in each case that the flight met each of the conditions which I have outlined above, including that the flight did not form part of a military exercise.

My Department received 32 requests to land at Shannon Airport on dates between 1 and 12 June 2016. This compares to 19 requests received for landings on the equivalent dates in 2015. Department officials have asked the US Embassy to re-confirm the details provided in respect of two of the aircraft to which the Deputy refers.

I would note that the number of requests to land at Shannon received from the US Embassy for the wider period 1 January to 12 June is broadly comparable in 2015 and 2016. My Depart- ment received a total of 283 such requests in 2015 compared to 288 in 2016.

Overflights by US military aircraft are permitted without prior notification, on the basis that the aircraft are unarmed, carry only cargo and passengers and comply with navigational requirements.

Military Aircraft Landings

21/06/2016WRP01200316. Deputy Clare Daly asked the Minister for Foreign Affairs and Trade if he is concerned at the activity of two aircraft (details supplied) which made multiple take-offs and landings from Shannon Airport during the period 4 June to 12 June 2016; if these aircraft were involved in the North Atlantic Treaty Organization exercises in Poland; and if he will make a statement on the matter. [16950/16]

21/06/2016WRP01300Minister for Foreign Affairs and Trade (Deputy Charles Flanagan): I can confirm that the requests received by my Department for landings by KC10 aircraft at Shannon Airport dur- ing the period in question stated that the aircraft were unarmed, carried no arms, ammunition or explosives and that they would not engage in intelligence gathering. The requests also stated that the flights did not form part of any military operation or exercises.

Permission for the landings was granted subject to these conditions. Department officials have asked the US Embassy to re-confirm the details provided in respect of these aircraft.

21/06/2016WRP01350Passport Services

21/06/2016WRP01400317. Deputy Jim O’Callaghan asked the Minister for Foreign Affairs and Trade if he will assign additional staff to the Passport Office during the peak holiday season or between May and September in order that telephone calls made to it can be answered by an official, passports can be issued without delay, and queries regarding passports can be followed up accordingly. [17003/16]

21/06/2016WRP01500Minister for Foreign Affairs and Trade (Deputy Charles Flanagan): The Passport Ser- 153 Questions - Written Answers vice is in peak season with 60,638 applications currently in the system. The level of demand is very high this year with an 11% rise in the number of applications year-to-date compared to the same period last year.

To respond to the seasonal spike in demand and the more general increase in applications, a total of 233 Temporary Clerical Officers have been recruited so far this year. This is an increase of 62 officers on last year. The officers are assisting permanent staff with passport entitlement checking and customer care, including the response to queries received via phone calls, emails and social media. Furthermore, processing work has been redistributed across passport offices and staff re-deployed form other areas of the Department as needed.

The Passport Service will continue to closely monitor the situation to ensure the effective deployment of staff resources and to minimise the impact of the high volume of applications on turnaround times and on customer service.

Undocumented Irish in the USA

21/06/2016WRP01600318. Deputy Charlie McConalogue asked the Minister for Foreign Affairs and Trade the status of addressing the undocumented Irish in the US; and if he will make a statement on the matter. [17078/16]

21/06/2016WRP01700Minister for Foreign Affairs and Trade (Deputy Charles Flanagan): Achieving relief for undocumented Irish migrants in the US and agreement on a facility for future legal mi- gration between Ireland and the US remains a priority for the Government. Our Embassy in Washington and Consulates elsewhere in the US are active in advocating immigration reform and the issue is also regularly addressed in high level political contacts between Ireland and the US Government.

While in Washington for St. Patrick’s Day, the Taoiseach and I met President Obama, Vice President Biden, Speaker Ryan, Senator Schumer and other key Congressional contacts from both sides of the aisle. During these meetings we emphasised the plight of the undocumented Irish in the US and encouraged both Houses of Congress to work on a bipartisan basis towards a comprehensive package of immigration reform measures.

In his speech at the Speaker’s lunch at Capitol Hill, the Taoiseach addressed the issues of immigration reform and urged Republicans and Democrats to work together to address the concerns of the undocumented Irish in the US. The Taoiseach also spoke of the need for im- proved legal migration channels for those Irish people who wish to live, work and develop their skills in the US for a time and emphasised the exceptional contribution made by Irish people to American society over the course of centuries of shared history.

Our Embassy in Washington continues to closely monitor the progress of the judicial case on President Obama’s Executive Actions on immigration reform. This case, which was brought by a coalition of US States seeking an injunction against the Executive Action, has now reached the US Supreme Court, and it is expected that the Court will soon declare its decision. If the ap- peal succeeds and the measure is implemented, the Executive Action could benefit a significant number of our citizens by lifting the threat of deportation and allowing those undocumented im- migrants who have been in the US for more than five years and who have children who are US citizens or legal permanent residents to work and travel more freely within the United States.

The new Government, the Department of Foreign Affairs and Trade and our Embassy in Washington, will continue to actively pursue all opportunities to advance immigration reform that would be of benefit to our citizens, with the US Administration, both sides of the aisle in 154 21 June 2016 Congress and the US Embassy in Ireland.

Consular Services Provision

21/06/2016WRP01800319. Deputy Mattie McGrath asked the Minister for Foreign Affairs and Trade the con- sular assistance he is providing to a person (details supplied); and if he will make a statement on the matter. [17226/16]

21/06/2016WRP01900Minister for Foreign Affairs and Trade (Deputy Charles Flanagan): My Department is providing ongoing consular assistance to this Irish citizen and his family. The focus in such cases is to ensure that the imprisoned Irish citizen has access to legal representation and is not discriminated against in terms of the legal processes or treatment in the prisons system, as well as addressing any specific welfare concerns that arise.

Diplomatic and consular officials from the Embassy of Ireland in Abu Dhabi have regularly visited this individual at Al Wathba prison. The most recent visit by a consular official took place last Wednesday, 15 June, and a further consular visit is planned in the coming weeks.

During consular visits, Embassy officials bring messages and facilitate practical support for the detainee on behalf of the family and the Irish Council for Prisoners Overseas (ICPO).

As and when appropriate, the Embassy has also been facilitating correspondence on the case with the authorities in Abu Dhabi, including in relation to a clemency request which is being pursued on behalf of the detained citizen.

In 2015 the Abu Dhabi Judicial Department advised that this citizen could lodge a formal complaint with regard to his detention in the United Arab Emirates. The Embassy has ex- plained this option and provided the relevant complaint forms to the detained Irish citizen and his family in Ireland.

The Embassy in Abu Dhabi and the Consular Assistance Unit in Dublin will continue to provide all appropriate consular assistance to the detainee and the family as required.

National Monuments

21/06/2016WRP02000320. Deputy Imelda Munster asked the Minister for Public Expenditure and Reform his plans to close St. Laurence’s Gate in Drogheda, County Louth, to vehicular traffic to protect and preserve the structure of this ancient and historical gate, which is listed as a national monument. [16835/16]

21/06/2016WRP02100Minister of State at the Department of Public Expenditure and Reform (Deputy Seán Canney): As I have indicated in recent Parliamentary responses, (Questions numbers 12405/16; 12490/16 & 13497/16) the Office of Public Works, which is responsible for the care and main- tenance of the St. Laurence’s Gate National Monument in Drogheda, is in favour of closing the roadway beneath it to vehicular traffic so as to better protect the structure from the effects of collisions by passing traffic. However, this is not a measure that the OPW can effect as the clo- sure of roads is a matter solely for the elected members of Louth Co. Council to decide, in con- junction with the Executive of the Local Authority. There is also, I understand, in cases such as this where a road closure is proposed, an extensive public consultation process involved.

Officials of the Office of Public Works and the Department of Arts, Heritage and the Gael- tacht have recently been in touch with the Louth Co. Council and have sought a meeting with 155 Questions - Written Answers them in order to put their views directly to them and consult with them in relation to future plans for this Monument.

Flood Relief Schemes

21/06/2016WRP02200321. Deputy Paul Murphy asked the Minister for Public Expenditure and Reform if he will consider the type of flood defence works as carried out by South Dublin County Council in Bohernabreena, Dublin 24, following flooding in 2011 (details supplied) in any new arrange- ments with the insurance industry to provide cover in areas where flood defence works have been carried out, given the difficulties of acquiring insurance; and if he will make a statement on the matter. [16851/16]

21/06/2016WRP02300Minister of State at the Department of Public Expenditure and Reform (Deputy Seán Canney): The Memorandum of Understanding between OPW and Insurance Ireland provides that OPW will provide Insurance Ireland with detailed background and technical information on all completed OPW flood relief schemes, including maps showing the extent of the area protected, and that Insurance Ireland member companies will take into account all information provided by OPW when assessing exposure to flood risk within these areas. The provision of insurance cover and the price at which it is offered is a commercial matter for insurance companies and is based on an assessment of the risks they are willing to accept and adequate provisioning to meet those risks.

Where flood defences are constructed to provide protection to areas which have previously been subject to flooding, the insurance industry has made clear its position that the defences should be designed and constructed to protect against a flood event with an Annual Exceedance Probability (AEP) of 1% (commonly referred to as a 100-year flood event). Currently all OPW flood defence schemes are designed and constructed to provide at least this level of protection.

It is open to any person seeking property insurance to supply insurers or insurance brokers with any relevant information on flood defences which might influence a decision on the provi- sion of cover. Consideration of this information would be a matter for the relevant insurance company.

The subject of insurance against the risk of flooding is one of the issues being considered by the Inter-Departmental Committee which is developing whole-of-Government policies to sup- port the OPW Flood Risk Management Plans. In this context, the Department of Finance has been carrying out research in the area of flood insurance including an analysis of the different approaches to flood insurance taken in other countries. This will then feed into the Committee’s report to Government this summer.

EU Funding

21/06/2016WRP02425322. Deputy Micheál Martin asked the Minister for Public Expenditure and Reform the status of his recent correspondence to the President of the European Commission regarding European Union rules posing a significant threat to the ability to fund major projects in hous- ing. [16952/16]

21/06/2016WRP02500Minister for Public Expenditure and Reform (Deputy Paschal Donohoe): Replies have not yet been received to either the letter sent by the Taoiseach to President Juncker or the letter sent by my predecessor to Commissioner Thyssen, regarding the Government’s concerns at the changing approach being taken by Eurostat to interpreting the agreed ESA 2010 rules govern- 156 21 June 2016 ing the balance sheet classifications of PPPs.

However, I understand from informal contacts at official level with the Commission that a draft reply to An Taoiseach is now with President Juncker and should issue in due course, while the reply to the letter from my predecessor is also expected to issue shortly.

Notwithstanding this position, there has been significant engagement in Brussels on this im- portant matter in recent weeks. Minister Noonan raised the point at the May Eurogroup meet- ing and received support on the issue, while it was also raised at the Economic and Financial Committee in June as well as at meetings of Coreper and of Financial Counsellors, in addition to also being discussed informally on the margins of meetings.

A meeting between Member States and Eurostat representatives, under the auspices of the European PPP Expertise Centre (EPEC), is scheduled for 6th July to discuss the matter. That meeting will review the latest changes to PPP statistical rules in the Eurostat Manual on Gov- ernment Deficit and Debt, issued in March, and will also enable Member States to receive an update on the work that is currently underway between EPEC and Eurostat to produce a guid- ance paper, to be issued jointly by the two organisations, with a view to providing clarity on the statistical classification issue, in view of the confusion that was created by the latest guidance note issued in March.

EU Funding

21/06/2016WRP02600323. Deputy Micheál Martin asked the Minister for Public Expenditure and Reform to clarify his concerns regarding EUROSTAT’s interpretation on public private partnerships in his correspondence to the President of the European Commission. [16953/16]

21/06/2016WRP02650Minister for Public Expenditure and Reform (Deputy Paschal Donohoe): The concerns which were referenced in the letters from the Taoiseach and my predecessor to the European Commission relate to the approach being taken by Eurostat in interpreting the agreed ESA 2010 rules governing the balance sheet classifications of PPPs, which appears to be progressively shifting away from the established and well understood analysis of risk, and how this is shared between the public and private partners, to a new and overly rigorous approach to how rewards are allocated under a PPP contract.

The ESA 2010 rules provide that the majority of risks and rewards are the relevant indica- tors for balance sheet treatment. However, the latest guidance issued by Eurostat in March 2016 has now fundamentally changed the position insofar as rewards are concerned, such that “any part of the profit” accruing to the government unit now appears to be sufficient to result in a PPP being classified as on-balance sheet.

The latest guidance also appears to have changed the ground rules in relation to existing ap- proved projects. Until now, any such existing approved projects only required to be reviewed in the event of a “substantial” change to the contract. However, under the latest guidance, only contract changes which are “not negligible” are permitted without requiring the contract to be reviewed under the new guidance in force at that time, but it is unclear what type of change would meet this “not negligible” test and therefore potentially trigger a requirement for a re- view of the entire contract under the new rules.

The changes introduced in the latest guidance may also require a trade-off between value for money and achieving off-balance sheet status for new PPPs, insofar as changes may have to be made to the manner in which future rewards, for example arising from windfall gains that may accrue to a project in the future, can be shared between the public and private sectors. 157 Questions - Written Answers This changing position of Eurostat has therefore created considerable confusion and un- certainty in relation how PPPs are to be classified in future, which poses a significant threat to the ability of European Governments to continue to pursue PPPs as a means of complementing direct Exchequer investment in much needed infrastructural projects. To this extent, it appears manifestly at odds with the primary objective behind the Juncker Plan, which is to facilitate, and indeed stimulate, increased investment in infrastructure projects across the Union.

A meeting between Member States and Eurostat representatives, under the auspices of the European PPP Expertise Centre (EPEC), is scheduled for 6th July to discuss the matter. That meeting will review the latest changes to PPP statistical rules in the Eurostat Manual on Gov- ernment Deficit and Debt, issued in March, and will also enable Member States to receive an update on the work that is currently underway between EPEC and Eurostat to produce a guid- ance paper, to be issued jointly by the two organisations, with a view to providing clarity on the statistical classification issue, in view of the confusion that was created by the latest guidance note issued in March.

21/06/2016WRP02725Departmental Staff Recruitment

21/06/2016WRP02800324. Deputy James Browne asked the Minister for Public Expenditure and Reform if he is aware of a requirement for temporary staff recruited for Census 2016 to be under 70 years of age; if he will ensure that this age discrimination is not a factor in any recruitment of temporary staff for Census 2021 or any other State data gathering exercises requiring temporary staff; and if he will make a statement on the matter. [16658/16]

21/06/2016WRP02900Minister for Public Expenditure and Reform (Deputy Paschal Donohoe): As the Depu- ty will be aware, the Single Public Service Pension Scheme (Single Scheme) commenced with effect from 1 January 2013. Therefore for new employees who come under the Single Pension scheme they must retire at 70 years of age.

In the civil service, for which I am responsible, the maximum retirement age of 65 years is applicable to civil servants recruited before 1 April 2004 and is provided for in the Civil Service Regulation Act 1956, and also applies to such staff in bodies under the aegis of my Department.

I understand that employers may specify a retirement age for employees provided it is based on objective reasons. You will be aware that in certain High Court cases the State has been suc- cessful in defending the question of existing retirement ages.

Flood Risk Assessments

21/06/2016WRP03000325. Deputy Danny Healy-Rae asked the Minister for Public Expenditure and Reform if he will complete and implement the recommendations of the catchment flood risk assessment and management, CFRAM, study; and if he will make a statement on the matter. [16763/16]

21/06/2016WRP03100Minister of State at the Department of Public Expenditure and Reform (Deputy Seán Canney): Good progress is being made on the Catchment Flood Risk Assessment and Manage- ment (CFRAM) Programme, which is being undertaken by engineering consultants on behalf of the Office of Public Works (OPW) working in partnership with the local authorities. The Programme involves the production of predictive flood mapping for each Area for Further As- sessment (AFA), the development of preliminary flood risk management options and the pro- duction of flood risk management plans. 158 21 June 2016 The draft flood mapping is now being finalised following completion of the national statu- tory public consultation on 23rd December, 2015. Work on the development of preliminary options to address flood risk is underway. Following finalisation of the mapping and the iden- tification of flood risk management options, the final output from this important project will be integrated Flood Risk Management Plans containing specific measures to address in a compre- hensive and sustainable way the significant flood risks identified. The draft Plans are scheduled to be made available for public consultation from mid-2016. Following the public consultation process the finalised Plans will include a prioritised list of measures, both structural and non- structural, to address flood risk in an environmentally sustainable and cost effective manner.

The Government announced increased levels of investment in the area of flood relief as part of the overall Capital Investment Plan 2016-2021 and this investment programme will allow for consideration of measures arising from the Flood Risk Management Plans.

Flood Prevention Measures

21/06/2016WRP03200326. Deputy Danny Healy-Rae asked the Minister for Public Expenditure and Reform his plans to clear blockages from the Gweestin river, County Kerry, to prevent homes from being flooded; and if he will make a statement on the matter. [16787/16]

21/06/2016WRP03300Minister of State at the Department of Public Expenditure and Reform (Deputy Seán Canney): The Gweestin River does not form part of any Arterial Drainage Scheme which would fall under the remit of the Office of Public Works (OPW) under the 1945 Arterial Drain- age Act. The OPW therefore has no responsibility for the maintenance of the channel.

Local flooding issues are a matter, in the first instance, for each Local Authority to inves- tigate and address, and Kerry County Council may carry out flood mitigation works using its own resources.

The Office of Public Works operates a Minor Flood Mitigation Works and Coastal Protec- tion Scheme. This administrative Scheme’s eligibility criteria, including a requirement that any measures are cost beneficial are published on the OPW website at http://www.opw.ie/en/flo- odriskmanagement/operations/minorfloodworkscoastalprotectionscheme/. It is open to Kerry County Council to submit a funding application under the Scheme. Any application received will be considered in accordance with the overall availability of resources for flood risk man- agement and the scheme’s eligibility criteria available on the OPW website.

Public Sector Staff Remuneration

21/06/2016WRP03400327. Deputy David Cullinane asked the Minister for Public Expenditure and Reform the various measures in relation to pay and pensions introduced as part of financial emergency mea- sures in the public interest, FEMPI, legislation 2009 to 2013; and the cost of rescinding each measure if the FEMPI legislation was to lapse, in tabular form. [16910/16]

21/06/2016WRP03500Minister for Public Expenditure and Reform (Deputy Paschal Donohoe): The Financial Emergency Measures in the Public Interests Acts 2009 to 2013 provide for a range of pay and pension reduction measures in respect of serving and retired public servants. The FEMPI leg- islation is available on my Departments website at http://www.per.gov.ie/public-service-pay- policy/.

The Lansdowne Road Agreement begins the process of unwinding the financial emergency

159 Questions - Written Answers measures in a prudent and sustainable fashion thereby reducing the risk to the sustainability of the public finances.

The terms of this agreement are being implemented under the Financial Emergency Mea- sures in the Public Interest Act 2015, with effect from 1 January 2016, at a full year cost of €844m to 2018. Additional provision has also been made for an amelioration of the Public Ser- vice Pension Reduction for public service pensioners at an additional full year cost of €90m in 2018.

FEMPI Measures 2009 to 2013 Remaining to be Restored Following Lans- downe Road Agreement Pay Reduction €692m Pension Related Deduction (PRD) €720m Public Service Pension Reduction €45m

Ministerial Correspondence

21/06/2016WRP03600328. Deputy Niall Collins asked the Minister for Public Expenditure and Reform if he will confirm receipt of correspondence from the general secretary of the Association of Garda Sergeants and Inspectors (details supplied); when he will meet this group; and if he will make a statement on the matter. [16943/16]

21/06/2016WRP03700Minister for Public Expenditure and Reform (Deputy Paschal Donohoe): The corre- spondence referred to by the Deputy has been responded to by my Office. The reply to the As- sociation of Garda Sergeants and Inspectors outlines the circumstances on the appropriate tim- ing for any future meeting to be held between the Association and myself.

Departmental Correspondence

21/06/2016WRP03800329. Deputy Michael Healy-Rae asked the Minister for Public Expenditure and Reform if he will address a matter (details supplied) regarding Parliamentary Question No. 368 of 14 June 2016; and if he will make a statement on the matter. [16957/16]

21/06/2016WRP03900Minister of State at the Department of Public Expenditure and Reform (Deputy Seán Canney): The Office of Public Works has no plans to provide a Guide Service at Carrigafoyle Castle, Co. Kerry.

The Castle is, like many other National Monument locations around the country, freely ac- cessible by the public and the OPW considers that it is already serving visitor needs adequately. In addition, the OPW does not, with the limited funds available to it, have the resources cur- rently to mount a Guide operation at the site.

Flood Risk Assessments

21/06/2016WRP04000330. Deputy Michael Healy-Rae asked the Minister for Public Expenditure and Reform the reason the Office of Public Works deemed an area as a flood area (details supplied), given that it has never experienced floods; and if he will make a statement on the matter. [17030/16]

21/06/2016WRP04100Minister of State at the Department of Public Expenditure and Reform (Deputy Seán Canney): The core strategy for addressing the significant flood risks nationally, in Tralee and

160 21 June 2016 in the Shannon River Basin District generally, is the Office of Public Works’ (OPW) Catchment Flood Risk Assessment and Management (CFRAM) Programme.

In 2011 the OPW completed a national screening exercise, called the Preliminary Flood Risk Assessment (PFRA), which identified 300 areas for further assessment (AFAs), that were potentially, at the most significant risk from flooding. As part of the PFRA a national public consultation process took place.

Tralee, including the area of Ballinorig, is one of 66 locations in the Shannon River Basin District and 300 locations nationwide that are being assessed under the Programme the purpose of which is to implement the EU Floods Directive and national flood policy.

The Programme, which is being undertaken by engineering consultants on behalf of the OPW working in partnership with the local authorities, involves the production of predictive flood mapping for each location, the development of preliminary flood risk management op- tions and the production of flood risk management plans. Under the Shannon CFRAM Study, draft predictive flood maps for Tralee have been produced and were the subject of a Public Consultation Day in Tralee on 24th March 2015. A further Public Consultation Day was held in Tralee on 1st October 2015 to present and discuss preliminary options to manage the significant flood risks.

The draft flood mapping is now being finalised following completion of the national statu- tory public consultation on 23rd December, 2015. Work on the development of preliminary options to address flood risk is underway. Following finalisation of the mapping and the iden- tification of flood risk management options, the final output from this important project will be integrated Flood Risk Management Plans containing specific measures to address in a compre- hensive and sustainable way the significant flood risks identified. The draft Plans are scheduled to be made available for public consultation from mid-2016. Following the public consultation process the finalised Plans will include a prioritised list of measures, both structural and non- structural, to address flood risk in an environmentally sustainable and cost effective manner.

The Government recently announced increased levels of investment in the area of flood relief as part of the overall Capital Investment Plan 2016-2021 and this investment programme will allow for consideration of measures arising from the Flood Risk Management Plans.

Flood Relief Schemes

21/06/2016WRP04200331. Deputy Michael Healy-Rae asked the Minister for Public Expenditure and Reform if he will address the local flooding in Killarney (details supplied) and potential dredging solu- tions to prevent future flooding; and if he will make a statement on the matter. [17059/16]

21/06/2016WRP04300Minister of State at the Department of Public Expenditure and Reform (Deputy Seán Canney): Local flooding issues are, in the first instance, a matter for each local authority to investigate and address. They may carry out flood alleviation works from their own resources or apply to the Office of Public Works (OPW) for funding under the Minor Flood Mitigation Works and Coastal Protection Scheme. This purpose of this scheme is to provide funding to lo- cal authorities to undertake minor flood mitigation works or studies to address localised flood- ing and coastal protection problems within their administrative areas.

The scheme generally applies to relatively straightforward cases where a solution can be readily identified and achieved in a short time frame. Any application received will be consid- ered in accordance with the scheme eligibility criteria, which comprise economic, social and environmental criteria including a requirement that any measures are cost beneficial, and hav- 161 Questions - Written Answers ing regard to the overall availability of resources for flood risk management.

The core strategy for addressing the significant flood risks nationally is the Office of Public Works’ (OPW) Catchment Flood Risk Assessment and Management (CFRAM) Programme. Killarney is one of 27 locations in the South West River Basin District and 300 locations nation- wide that are being assessed under the Programme the purpose of which is to implement the EU Floods Directive and national flood policy. The Lake Hotel, Lake Shore, Muckross Road, is not within the Killarney AFA boundary whilst Glenflesk is an AFA under the CFRAM Programme.

The CFRAM Programme, which is being undertaken by engineering consultants on behalf of the OPW working in partnership with the local authorities, involves the production of predic- tive flood mapping for each location, the development of preliminary flood risk management options and the production of flood risk management plans. Under the South West CFRAM Study, draft predictive flood maps for Killarney have been produced and were the subject of a Public Consultation Day in Killarney on 23rd October, 2015. A further Public Consultation Day was held in Killarney on 8th December, 2015 to present and discuss preliminary options to manage the significant flood risks.

The draft flood mapping is now being finalised following completion of the national statu- tory public consultation on 23rd December, 2015. Work on the development of preliminary options to address flood risk is underway. Following finalisation of the mapping and the iden- tification of flood risk management options, the final output from this important project will be integrated Flood Risk Management Plans containing specific measures to address in a compre- hensive and sustainable way the significant flood risks identified. The draft Plans are scheduled to be made available for public consultation from mid-2016. Following the public consultation process the finalised Plans will include a prioritised list of measures, both structural and non- structural, to address flood risk in an environmentally sustainable and cost effective manner.

The Government recently announced increased levels of investment in the area of flood relief as part of the overall Capital Investment Plan 2016-2021 and this investment programme will allow for consideration of measures arising from the Flood Risk Management Plans.

Departmental Expenditure

21/06/2016WRP04400332. Deputy Sean Sherlock asked the Minister for Public Expenditure and Reform if he will allocate moneys for the remediation works at Haulbowline Island in Cobh, County Cork of which €40 million is required to fund the next intensive phase of the planned programme of works for the island, which is of enormous concern to persons resident in the lower harbour. [17177/16]

21/06/2016WRP04500Minister for Public Expenditure and Reform (Deputy Paschal Donohoe): The Minister for Agriculture, Food and the Marine has Ministerial responsibility for the remediation of Haul- bowline Island and Cork County Council is acting as his agent to deliver the project. A total of €8m has been provided in his Department’s estimate for 2016. The amount to be included for the project in his Department’s Vote for 2017, and future years, is a matter to be determined by him as part of the annual estimates process, in consultation with my Department.

21/06/2016WRQ00150Coastal Erosion

21/06/2016WRQ00200333. Deputy Dara Calleary asked the Minister for Public Expenditure and Reform to set out the funding he sanctioned for coastal erosion works at a location (details supplied) in Coun- 162 21 June 2016 ty Mayo, and the works that were to be carried out; to provide a breakdown of the works that have been carried out and the works that remain outstanding; and if he will make a statement on the matter. [17191/16]

21/06/2016WRQ00300Minister of State at the Department of Public Expenditure and Reform (Deputy Seán Canney): I am advised that it is a matter for Mayo County Council (MCC) in the first instance to investigate and address priority coastal protection issues in the county.

MCC was allocated €4.211 million by the Government in February 2014 to carry out repair works to coastal protection infrastructure damaged in the severe storms which affected the country in late 2013 and early 2014. A further allocation of €6,000 for this purpose was made to MCC in October 2015. Projects for Rinroe, Co. Mayo were included by MCC in its submitted and agreed programme of storm damage repair works. All funds have now been disbursed to MCC in relation to this programme of works. It is a matter for MCC to prioritise and progress works and the OPW has no involvement in the carrying out of the works.

It is also open to MCC to carry out further works at this location using its own resources or to submit an application for further funding if necessary under the Minor Flood Mitigation Works & Coastal Protection Scheme, details of which are on the OPW website www.opw.ie.

21/06/2016WRQ00350Public Sector Pensions Data

21/06/2016WRQ00400334. Deputy David Cullinane asked the Minister for Public Expenditure and Reform to set out the net effect of the imposition of the public service pension reduction which commenced on 1 January 2011, the legislation, as amended, from 1 January 2012 and the additional changes that were part of the Financial Emergency Measures in the Public Interest Act 2013; the net effects of the PSPR amelioration applicable on 1 January 2016, 1 January 2017 and 1 January 2018; the difference between the net loss as a consequence of the FEMPI legislation indicated and the amelioration measures of 2016 to 2018; if this can be broken down in pension bands (details supplied); and if he will make a statement on the matter. [17210/16]

21/06/2016WRQ00500Minister for Public Expenditure and Reform (Deputy Paschal Donohoe): Since its in- troduction in 2011, the Public Service Pension Reduction (PSPR) has applied more significant cuts in both absolute and proportionate terms to public service pensions of relatively higher values while at 2015 year-end, an estimated 33% of public service pensions of relatively lower values were completely exempt from PSPR.

Scheduled over three stages between 2016 and 2018, the initial measures for restoring pub- lic service pensions as provided for under the Financial Emergency Measures in the Public Interest (FEMPI) 2015 Act are based principally on increases to the thresholds before PSPR is applied. On that basis these restoration measures are effective in removing PSPR entirely, that is to say restoring full payment rates, in the cases of those lower value pensions previously af- fected by PSPR.

At 2015 year-end, all public service pensions with annual values up to €12,000 were exempt from PSPR. In 2016, all pensions up to €18,700 are exempt from PSPR. From 1 January 2017, all pensions below €26,000 will be exempt from PSPR, and from 1 January 2018 all pensions up to €34,132 per year will be exempt from PSPR. The increases in exemption thresholds also provide a boost to public service pensions above the applicable thresholds, in most cases de- livering a cumulative pension restoration of €400 per year in 2016, €500 per year in 2017 and €780 per year in 2018.

Based on available data and making assumptions for future retirements and mortality rates, 163 Questions - Written Answers the estimated reduction in public service pension costs delivered by PSPR is as follows: €100 million in 2011, €100 million in 2012, €120 million in 2013, €140 million in 2014, €135 mil- lion in 2015, €105 million in 2016, €75 million in 2017 and €45 million in 2018. These figures indicate the much reduced saving to the public finances arising from the FEMPI 2015 pension restoration measures.

Details of the estimated PSPR impacts on individual pensions and the associated ameliora- tion impacts provided for under the FEMPI 2015 pension restoration measures by reference to the pension income bands included in the Deputy’s question are set out in the following tables.

Table A shows the PSPR amelioration (restoration effect) on most PSPR-impacted pensions, being the pensions paid in respect of persons who retired before 1 March 2012.

Table A:

Pre-PSPR 2015 PSPR 2016 PSPR 2017 PSPR 2018 PSPR Total Pension Pension (€) (€) (€) (€) Restored (€) (€) 14,000 120 0 0 0 120 16,000 240 0 0 0 240 18,000 360 0 0 0 360 20,000 480 78 0 0 480 25,000 810 408 0 0 810 30,000 1,260 858 360 0 1,260 32,000 1,440 1,038 540 0 1,440 35,000 2,280 1,880 1,380 600 1,680 40,000 2,880 2,480 1,980 1,200 1,680 50,000 4,080 3,680 3,180 2,400 1,680 60,000 5,280 4,880 4,380 3,600 1,680 70,000 6,980 6,580 6,080 5,300 1,680 80,000 8,680 8,280 7,280 6,900 1,680 100,000 12,080 11,680 11,180 10,400 1,680

Table B shows the PSPR amelioration for the much less numerous group of pensions award- ed in respect of retirements on or after 1 March 2012.

Table B:

Pre-PSPR 2015 PSPR 2016 PSPR 2017 PSPR 2018 PSPR Total Pension Pension (€) (€) (€) (€) Restored (€) (€) 14,000 0 0 0 0 0 16,000 0 0 0 0 0 18,000 0 0 0 0 0 20,000 0 0 0 0 0 25,000 0 0 0 0 0 30,000 0 0 0 0 0 32,000 0 0 0 0 0 35,000 570 171 0 0 570 40,000 720 321 20 0 720 164 21 June 2016 Pre-PSPR 2015 PSPR 2016 PSPR 2017 PSPR 2018 PSPR Total Pension Pension (€) (€) (€) (€) Restored (€) (€) 50,000 1,020 621 220 0 1,020 60,000 1,320 921 420 0 1,320 70,000 1,820 1,421 920 500 1,320 80,000 2,320 1,921 1,420 1,000 1,320 100,000 3,320 2,921 2,420 2,000 1,320

21/06/2016WRQ00550Architectural Heritage

21/06/2016WRQ00600335. Deputy Charlie McConalogue asked the Minister for Arts, Heritage and the Gael- tacht to outline the grants available from her Department for renovating a thatched roof (details supplied); and if she will make a statement on the matter. [16962/16]

21/06/2016WRQ00700Minister for Arts, Heritage and the Gaeltacht (Deputy Heather Humphreys): My role, as Minister for Arts Heritage and the Gaeltacht, with regard to the protection and management of our architectural heritage, is set out in the provisions of relevant legislation, as are the roles of local authorities and the responsibilities of owners as regards heritage assets.

The architectural conservation officer in the local authority will generally be able to advise of funding available for conservation works to thatched buildings in the area in question and it is advisable for interested parties to remain in contact with their local authority on an on-going basis.

A number of State financial supports provide for the conservation and preservation of heri- tage structures in general.

Financial support is being provided by my Department through a number of structured schemes for the conservation and protection of heritage buildings. My Department itself op- erates a Structures at Risk Fund to enable conservation works to heritage structures, in both private and public ownership, that are protected under the Planning and Development Acts and are deemed to be at significant risk of deterioration. This fund, which has an allocation of over €900,000 in 2016, is administered through the local authorities and seeks to encourage the regeneration and reuse of heritage properties and to help to secure the preservation of protected structures which might otherwise be lost. Applications for this scheme have now closed for 2016, and recommended projects have been recently announced.

I launched a new €2 million scheme - the Built Heritage Investment Scheme - for the repair and conservation of protected structures on 21 October 2015. This scheme will operate in 2016, via the local authorities, on the same model as the very successful Built Heritage Jobs Leverage Scheme, which ran in 2014. It is expected to support a significant number of projects across the country and to create employment in the conservation and construction industries, while help- ing to regenerate urban and rural areas. The scheme for this year is now fully allocated.

The Heritage Council, which my Department funds, also provides grants for the protec- tion and preservation of the built heritage. For 2016, the Council is administering a commu- nity based heritage grants scheme available for projects that contributed to particular heritage themes, of which further details can be found at www.heritagecouncil.ie.

21/06/2016WRQ00750Departmental Properties

165 Questions - Written Answers

21/06/2016WRQ00800336. Deputy Charlie McConalogue asked the Minister for Arts, Heritage and the Gael- tacht if there is a first-refusal option to purchase in the event of land being placed for sale on a lease entered into by her Department (details supplied); if this option is currently being pursued and if she will exercise this option; and if she will make a statement on the matter. [16687/16]

21/06/2016WRQ00900Minister for Arts, Heritage and the Gaeltacht (Deputy Heather Humphreys): I refer the Deputy to my answer to Parliamentary Question 392 on 14 June 2016.

The additional information now sought by the Deputy regarding the legal instrument in question is commercially sensitive to my Department.

21/06/2016WRQ00950National Parks Projects

21/06/2016WRQ01000337. Deputy John Brady asked the Minister for Arts, Heritage and the Gaeltacht if the State will acquire land (details supplied) as part of an expanded Wicklow Mountains National Park. [16697/16]

21/06/2016WRQ01100Minister for Arts, Heritage and the Gaeltacht (Deputy Heather Humphreys): My De- partment is aware of the matter referred to and will keep it under review within the context of expenditure proposals competing for funding resources available for the management of Na- tional Parks and Nature Reserves across the country.

21/06/2016WRQ01150Wildlife Protection

21/06/2016WRQ01200338. Deputy Robert Troy asked the Minister for Arts, Heritage and the Gaeltacht to clarify the position regarding the issue of hedge cutting and burning; if she will maintain the current closed period and the action she will take do to ensure that this closed period is properly en- forced; and if she will make a statement on the matter. [16699/16]

21/06/2016WRQ01300Minister for Arts, Heritage and the Gaeltacht (Deputy Heather Humphreys): Section 40 of the Wildlife Acts 1976, as amended, prohibits the cutting, grubbing, burning or destruc- tion of vegetation, with certain strict exemptions, from 1 March to 31 August.

Following a review of Section 40, which involved, inter alia, consideration of submissions from interested parties, I announced proposals in December 2015 to introduce legislation to al- low for managed hedge cutting and burning at certain times within the existing closed period on a pilot two year basis. The legislation required to allow for these pilot measures was included in the Heritage Bill 2016, which was published in January 2016 and subsequently completed Second Stage in Seanad Éireann.

Steps are in hand to reintroduce the Heritage Bill into Seanad Éireann at the earliest oppor- tunity. In the meantime, the existing provisions relating to Section 40 of the Wildlife Acts are still in operation.

21/06/2016WRQ01350Special Areas of Conservation

21/06/2016WRQ01400339. Deputy Michael Healy-Rae asked the Minister for Arts, Heritage and the Gaeltacht to outline his views on the National Parks and Wildlife Service statements to the Office of Public Works relating to recent storm damage (details supplied); and if she will make a statement on the matter. [16776/16]

166 21 June 2016

21/06/2016WRQ01500Minister for Arts, Heritage and the Gaeltacht (Deputy Heather Humphreys): The Of- fice of Public Works (OPW) and local authorities are the principal response agencies with re- gard to the management of flooding episodes. Ireland, like all EU Member States, is bound by the requirements of the EU Habitats and Birds Directives. I am advised that my Department is not holding up the case in question.

Many of the rivers and low-lying lands affected by the flooding earlier this year are within or near sites that have been designated as SACs or SPAs. These sites fall within the ambit of the European Birds and Habitats Directives, compliance with which is a matter for all public authorities, agencies and landowners. A key protection mechanism is the requirement to con- sider the possible implications of any plan or project on SACs and SPAs before any decision is made to allow that plan or project to proceed.

21/06/2016WRQ01550Arts Funding

21/06/2016WRQ01600340. Deputy Michael Collins asked the Minister for Arts, Heritage and the Gaeltacht to outline her views on the necessity to ensure that funding for the arts is distributed equally across the regions and whether specific areas (details supplied) are not receiving an adequate or equal supply of such funding. [17163/16]

21/06/2016WRQ01700Minister for Arts, Heritage and the Gaeltacht (Deputy Heather Humphreys): The is- sue of the geographical spread of funding for the Arts is a relevant one in term of access and participation. There are many factions which impinge on this including population density, the location of major venues, the headquarters of national artistic bodies and the role of local authorities and other local factors.

My Department has a role in providing capital funding and in this regard I recently an- nounced a new €9 million investment scheme for arts and cultural centres to run over the next three years focussed on upgrading the existing stock of arts and cultural centres around the country. My Department is finalising the guidelines and application forms in relation to the scheme which I intend to open for applications very shortly. These guidelines will provide that the issue of geographical spread may be considered in terms of grant approvals.

Most of Government’s funding for the arts is allocated to the Arts Council to which I al- located a total of €60 million in 2016. The Council is statutorily independent in its funding decisions. Information on the Arts Council’s funding schemes, funding decisions and reasons for those decisions is available for each county through its decisions database at the following link:- http://www.artscouncil.ie/Who we funded/.

The Arts Council is aware of the issue of geographical distribution of support and in its lat- est Strategy Document - Making Great Art Work - it incorporates a Spatial and Demographic Goal under which includes actions regarding:

- The provision of advice to Government about the spatial and demographic considerations which should inform national cultural policy, including capital development;

- the development of the Arts Council’s strategic partnership with local government and with individual local authorities;

- the inclusion of spatial and demographic measures within the ‘public engagement’ criteria that inform the Council’s investment strategy and funding decisions; and

- working with others to ensure a robust evidence base for arts planning and investment,

167 Questions - Written Answers drawing on spatial and demographic knowledge and analysis.

21/06/2016WRQ01750Údarás na Gaeltachta Funding

21/06/2016WRQ01800341. Deputy Brendan Griffin asked the Minister for Arts, Heritage and the Gaeltacht to outline the funding and supports in place for a person (details supplied) who wishes to establish a business in the Gaeltacht area of County Kerry; and if she will make a statement on the mat- ter. [17237/16]

21/06/2016WRQ01900Minister of State at the Department of Arts, Heritage and the Gaeltacht (Deputy Seán Kyne): I have been informed by Údarás na Gaeltachta that they can offer qualifying businesses and companies from various sectors a range of incentives and supports to start up, develop, ex- pand or locate in a Gaeltacht region. Qualifying business sectors include Internationally Traded Services, Audio-visual and Digital Media, Life Sciences, Manufacturing and Engineering, ICT (Information Communications Technology), Aquaculture, Language-based Enterprise, Food, Tourism, Arts and Crafts, Renewable Energy and Island Enterprise. (In general Retail, Personal Services, Construction & Local Building and Professional Services are regarded as ineligible sectors).

Funding eligibility and amounts are dependent on the type of business proposed, geographi- cal location, skills level required and the potential employment level of the project. All busi- ness proposals, which are assessed on an individual basis, must demonstrate their viability, funding and market potential, and must show that the necessary management and technical expertise is available to establish the new industry or service on a stable foundation. A detailed business plan must also be submitted for assessment. Guidelines on preparing a business plan are available on www.udaras.ie. Applications for all grants must be made and approval must be obtained in writing from Udarás na Gaeltachta before starting the relevant project (or any part of it) for which a grant may be requested. The decision lies with An tÚdarás as to whether grant aid is approved for a particular project.

The level of grant aid available for projects is governed by European Union State Aid Guide- lines. All Údarás na Gaeltachta supported businesses must agree to promote and develop the use of Irish in their activities.

The following is a list of incentives and supports available from An tÚdarás (subject to EU State Aid Guidelines and De Minimis Guidelines):

1. Development of Microenterprises

2. Capital Grant

3. Employment Grant

4. Training Grant

5. Marketing Grant

6. Consultancy Services Grant

7. Research and Development Grant

8. Feasibility Study Grant

9. Funding Grant for R&D&I

168 21 June 2016 10. Rent Subsidy

Further details in respect of these initiatives are available at http://www.udaras.ie/enforbairt- fiontraiochta/cunamh-airgid/ or by contacting Údarás na Gaeltachta directly on 091-503100.

In the interest of completeness, I should also say that a number of Enterprise Ireland Schemes are available to Údarás na Gaeltachta client companies. Further information on these schemes is available at: http://www.enterprise-ireland.com.

Finally, I would strong encourage the person referred to by the Deputy to contact the Kerry Regional Office of Údarás na Gaeltachta directly on 066-9150100.

21/06/2016WRQ01950Departmental Agencies Staff Remuneration

21/06/2016WRQ02000342. Deputy Peadar Tóibín asked the Minister for Arts, Heritage and the Gaeltacht to set out the number of persons who are paid a salary from her Department’s budget, either directly or indirectly, through funding agencies such as the Arts Council; and the level of salary to the nearest €10,000 (details supplied). [17254/16]

21/06/2016WRQ02100Minister for Arts, Heritage and the Gaeltacht (Deputy Heather Humphreys): One of the primary support mechanisms for the arts and heritage sectors and for the Gaeltacht is fund- ing provided by my Department through the Arts Council, the Heritage Council, Údarás na Gaeltachta and other statutory agencies. It is primarily a matter for these agencies to decide how their funding should be allocated across the range of programmes supported in 2016. I am advised that in certain cases this funding may indirectly support salaries in the sector. My Department does not, however, hold comprehensive statistics, such as requested by the Deputy, of these cases.

For completeness, the following tables show the breakdown, by salary band, of the numbers of persons paid a salary directly by my Department or by the bodies under the aegis of my De- partment, as of March 2016.

Agencies

Salary Band < 25 25-35 35-45 45-55 55-65 65-75 75 -85 85-95 95-100 100- 100- €000 125 125 Arts Coun- 0 4 8 16 12 0 4 1 0 0 0 cil Irish Film 2 1 4 4 7 0 1 0 1 0 0 Board Chester Be- 0 11 6 2 5 1 1 1 0 0 0 atty Library Irish 6 4 48 5 5 3 1 1 0 0 0 Museum of Modern Art National 32 8 7 8 7 4 0 0 0 1 0 Concert Hall National 10 30 17 8 20 8 2 0 1 0 0 Library of Ireland National 12 79 17 6 17 9 4 2 1 0 0 Museum of Ireland Crawford 8 6 1 3 0 1 0 0 0 0 0 Gallery 169 Questions - Written Answers

Salary Band < 25 25-35 35-45 45-55 55-65 65-75 75 -85 85-95 95-100 100- 100- €000 125 125 Heritage 0 2 2 1 6 2 0 0 0 1 0 Council Tha Boord 2 3 7 1 1 0 0 0 0 0 0 o Ulstèr- Scotch* Foras na 3 8 15 11 9 6 3 2 0 1 0 Gaeilge * Údarás na 0 1 8 24 15 18 1 5 3 3 1 Gaeltachta Waterways 17 179 44 21 20 11 5 2 1 0 0 Ireland** *Foras na Gaeilge and Tha Boord o Ulstèr-Scotch are co-funded by the Department for Communities in Northern Ireland.

**Waterways Ireland is co-funded by the Department for Infrastructure in Northern Ireland.

Department of Arts, Heritage and the Gaeltacht

Salary band < 10 10 - 20 - 30 - 40 - 50 60 - 70 - 80 - 90 - 100 - 110 - 130 - 170 - €000 20 30 40 50 -60 70 80 90 100 110 120 140 180 Department 3 18 48 177 134 60 78 32 7 7 1 1 3 1 of Arts Heritage and the Gaeltacht

21/06/2016WRQ02150Disability Services Provision

21/06/2016WRQ02200343. Deputy Clare Daly asked the Minister for Health to outline the steps he will take to ensure the vocational training department in a school (details supplied) will continue; and if he will make a statement on the matter. [16684/16]

21/06/2016WRQ02300Minister of State at the Department of Health (Deputy Finian McGrath): The Govern- ment is committed to providing services and supports for people with disabilities which will empower them to live independent lives, provide greater independence in accessing the servic- es they choose, and enhance their ability to tailor the supports required to meet their needs and plan their lives. This commitment is outlined in the Programme for Partnership Government, which is guided by two principles: equality of opportunity and improving the quality of life for people with disabilities.

The particular issue raised by the Deputy is a service matter for the HSE. Accordingly I have arranged for the question to be referred to the Health Service Executive (HSE) for direct reply to the Deputy. If the Deputy has not received a reply from the HSE within 15 working days, he can contact my Private Office and they will follow up the matter with the HSE.

21/06/2016WRQ02350Motorised Transport Grant Closure

21/06/2016WRQ02400344. Deputy John Brady asked the Minister for Health if he will reinstate the motorised transport grant and, if so, the timeframe for its reinstatement. [16688/16]

21/06/2016WRQ02500432. Deputy Michael Ring asked the Minister for Health to outline his plans to introduce a scheme to replace the motorised transport grant scheme, which closed to applicants in February

170 21 June 2016 2013; and if he will make a statement on the matter. [17193/16]

21/06/2016WRQ02600Minister of State at the Department of Health (Deputy Finian McGrath): I propose to take Questions Nos. 344 and 432 together.

Conscious of the reports of the Ombudsman regarding the legal status of both the Mobility Allowance and Motorised Transport Grant Schemes in the context of the Equal Status Acts, the Government decided to close both schemes. Both schemes remain closed.

However the Government also decided that monthly payments will continue to be made by the Health Service Executive to 4,700 people who were in receipt of the Mobility Allowance at the time the scheme closed, on the basis that this would prevent hardship and, on an interim basis, alleviate stress, anxiety and uncertainty among a vulnerable group in society.

The Government decided that the detailed preparatory work required for a new Transport Support Scheme and associated statutory provisions should be progressed by the Minister for Health. The Department is seeking a solution which would best meet the aim of supporting people with severe disabilities who require additional income to contribute towards the cost of their mobility needs, while remaining within the available budget and satisfying all legal and equality concerns.

Work is ongoing on the policy proposals to be brought to Government for the drafting of primary legislation for a new scheme. The proposals seek to ensure that:

- There is a firm statutory basis to the Scheme’s operation;

- There is transparency and equity in the eligibility criteria attaching to the Scheme;

- Resources are targeted at those with the greatest needs; and

- The Scheme is capable of being costed and it is affordable on its introduction and on an ongoing basis.

I am pleased to inform the Deputies that the Programme for Partnership Government ac- knowledges the ongoing drafting of primary legislation for a new Transport Support Scheme.

21/06/2016WRQ02650Substance Misuse

21/06/2016WRQ02700345. Deputy John Brady asked the Minister for Health if he is aware of a survey carried out by an inter-agency group to identify gaps in services for persons under 18 years of age experiencing substance abuse in east Wicklow (details supplied); the measures he is taking to address this; and if he will make a statement on the matter. [16913/16]

21/06/2016WRQ02800Minister of State at the Department of Health (Deputy Catherine Byrne): Drugs Task Forces play a key role in assessing the extent and nature of the drug problem in their areas and co-ordinating action at local level so that there is a targeted response to the drug problem in local communities. I am informed that the Interagency Group, who carried out the survey, are presenting their findings to the East Coast Regional Task Force at the next Task Force meeting.

The Department of Health is developing a new National Drugs Strategy, which will take effect in 2017, when the current policy expires. A Steering Committee has been established to oversee the process and to advise me on how the new Strategy should address problem drug use, including the structures through which this should be done and incorporating key perfor- mance indicators to measure the future effectiveness of the new policy.

171 Questions - Written Answers I will shortly be announcing details of the consultation process on the new Strategy, which I intend to be as broad, comprehensive and inclusive as possible. The Interagency Group may wish to avail of the opportunity to submit the findings of their report under the consultation process.

21/06/2016WRQ02850Suicide Prevention

21/06/2016WRQ02900346. Deputy Pat Deering asked the Minister for Health further to a Dáil Éireann adjourn- ment debate on 31 May 2016 (details supplied), the amount of this funding that the services in County Carlow received, given that it is the third highest county by national area and by resi- dence of suicide; and if he will make a statement on the matter. [16979/16]

21/06/2016WRQ03000Minister of State at the Department of Health (Deputy Helen McEntee): As this is a ser- vice issue, this question has been referred to the HSE for direct reply. If you have not received a reply within 15 working days, please contact my Private Office and they will follow up the matter with them.

21/06/2016WRQ03050Ministerial Meetings

21/06/2016WRQ03100347. Deputy Louise O’Reilly asked the Minister for Health if and when he will meet a group (details supplied); and if he will make a statement on the matter. [17282/16]

21/06/2016WRQ03200Minister for Health (Deputy Simon Harris): As the Deputy might be aware I very re- cently received a request to meet with this group and the Deputy may wish to know I will be arranging a meeting with them shortly at a date and time to be confirmed.

21/06/2016WRQ03250Nursing Homes Support Scheme

21/06/2016WRQ03300348. Deputy Michael Healy-Rae asked the Minister for Health to outline his views on correspondence (details supplied) regarding the nursing homes support scheme; and if he will make a statement on the matter. [17289/16]

21/06/2016WRQ03400Minister of State at the Department of Health (Deputy Helen McEntee): The legislation underpinning the Nursing Homes Support Scheme (NHSS) requires each private nursing home to negotiate and agree a price for long-term residential care services with the National Treat- ment Purchase Fund (NTPF) should they wish to be an approved nursing home for the purposes of the Scheme.

In negotiating with nursing homes the NTPF has regard to:

- costs reasonably and prudently incurred by the nursing home and evidence of value for money;

- price(s) previously charged;

- local market price, and

- budgetary constraints and the obligation on the State to use available resources in the most beneficial, effective and efficient manner to improve, promote and protect the health and wel- fare of the public.

172 21 June 2016 The NTPF has statutory independence in the performance of its function, and negotiates with each nursing home on an individual basis. The NTPF may examine the records and ac- counts of nursing homes as part of the process with the objective of setting a fair price which delivers value for money to the individual and the State. Pricing contracts are purely a matter for agreement between the NTPF and the nursing home in question. The Department of Health has no role in such individual negotiations.

The Nursing Homes Support Scheme covers the cost of the standard components of long- term residential care which are:

- Nursing and personal care appropriate to the level of care needs of the person;

- Bed and board;

- Basic aids and appliances necessary to assist a person with the activities of everyday liv- ing; and

- Laundry service.

Additional charges may not be levied in respect of the above components of care. A person’s eligibility for other schemes, such as the medical card scheme or the drugs payment scheme, is unaffected by participation in the NHSS or residence in a nursing home. Part 7 of the Health Act 2007 (Care and Welfare of Residents in Designated Centres for Older People) Regulations 2013 stipulates that the registered provider of the nursing home must agree a contract in writing with each resident on their admission to the nursing home. This contract must include details of the services to be provided to that resident and the fees to be charged. Residents should never be charged fees which are not set out in this contract. The Department of Health are not a party to such contracts which are concluded between each resident and their nursing home.

Registered providers of nursing home care are obliged to provide an accessible and effective complaints procedure. Concerns about additional charges should in the first instance be taken up with the nursing home provider. The Office of the Ombudsman can examine complaints about the actions of a range of public bodies and, from 24 August 2015, complaints relating to the administrative actions of private nursing homes. The Office of the Ombudsman normally only deals with a complaint once the individual has already gone through the complaints proce- dure of the private nursing home concerned.

21/06/2016WRQ03450Hospital Waiting Lists

21/06/2016WRQ03500349. Deputy Michael Healy-Rae asked the Minister for Health to outline the status of an operation for a person (details supplied); and if he will make a statement on the matter. [16654/16]

21/06/2016WRQ03600Minister for Health (Deputy Simon Harris): Under the Health Act 2004, the Health Ser- vice Executive (HSE) is required to manage and deliver, or arrange to be delivered on its be- half, health and personal social services. Section 6 of the HSE Governance Act 2013 bars the Minister for Health from directing the HSE to provide a treatment or a personal service to any individual or to confer eligibility on any individual.

The National Waiting List Management Policy, A standardised approach to managing scheduled care treatment for in-patient, day case and planned procedures, January 2014, has been developed to ensure that all administrative, managerial and clinical staff follow an agreed national minimum standard for the management and administration of waiting lists for sched-

173 Questions - Written Answers uled care. This policy, which has been adopted by the HSE, sets out the processes that hospitals are to implement to manage waiting lists.

In relation to the particular query raised, as this is a service matter, I have asked the HSE to respond to you directly. If you have not received a reply from the HSE within 15 working days please contact my Private Office and my officials will follow the matter up.

Hospital Appointments Status

21/06/2016WRQ03700350. Deputy Pat Deering asked the Minister for Health further to Parliamentary Question No. 899 of 17 May 2016 why a person (details supplied) did not receive a reply from the Health Service Executive. [16663/16]

21/06/2016WRQ03800Minister for Health (Deputy Simon Harris): Under the Health Act 2004, the Health Ser- vice Executive (HSE) is required to manage and deliver, or arrange to be delivered on its be- half, health and personal social services. Section 6 of the HSE Governance Act 2013 bars the Minister for Health from directing the HSE to provide a treatment or a personal service to any individual or to confer eligibility on any individual.

In relation to your further query on this issue, I have asked the HSE to respond to you di- rectly. If you have not received a reply from the HSE within 15 working days please contact my Private Office and my officials will follow the matter up.

Hospital Staff Recruitment

21/06/2016WRQ03900351. Deputy Billy Kelleher asked the Minister for Health when he will lift the embargo on midwives; and if he will make a statement on the matter. [16672/16]

21/06/2016WRQ04000Minister for Health (Deputy Simon Harris): Arising from Budget 2015, the Minister for Public Expenditure and Reform announced an easing of restrictions on the employment of addi- tional staff. The overriding principle now is one of affordability and compliance with allocated pay budgets which managers must forecast and monitor over the course of each year with the completion of a Pay and Numbers Strategy.

The HSE are currently working with my Department on the preparation of their 2016 Pay and Numbers Strategy. While these plans are being developed, interim recruitment measures have been put in place by the HSE, which require that pay budgets are complied with.

These measures do not impact on a hospital’s ability to recruit where funding exists to facili- tate that recruitment for example in the case of a replacement posts or where funding has been allocated for particular roles under the HSE service plan.

All posts are being actively monitored at Group level and National level to ensure that clinical risks are appropriately managed. Specific exemptions have been determined for areas where funding has been agreed, and in areas of critical care and emergency services.

21/06/2016WRQ04050Medicinal Products Reimbursement

21/06/2016WRQ04100352. Deputy Fiona O’Loughlin asked the Minister for Health to outline his plans to restore an allowance for diet supplements to persons diagnosed with a medical condition; and if he will

174 21 June 2016 make a statement on the matter. [16674/16]

21/06/2016WRQ04200Minister for Health (Deputy Simon Harris): The HSE has statutory responsibility for de- cisions on pricing and reimbursement of medicinal products under the community drug schemes in accordance with the provisions of the Health (Pricing and Supply of Medical Goods) Act 2013.

The HSE faces a continuing challenge to deliver services in a way that will minimise any adverse impact on patients and continue to protect, as far as possible, the most vulnerable citi- zens. Unfortunately, as a result it has become necessary for the HSE to suspend certain prod- ucts from its list of items reimbursable under the GMS and other community drug schemes. This includes gluten-free products.

Gluten-free products have become more widely available in supermarkets in recent years and tend to be significantly cheaper than products sold through community pharmacies. Co- eliac patients can also claim medical expenses tax relief on expenditure incurred on gluten-free products manufactured specifically for coeliacs. There are no plans to reinstate these products on the list of reimbursable items under the GMS and other community drug schemes.

21/06/2016WRQ04250Health Services Provision

21/06/2016WRQ04300353. Deputy Michael McGrath asked the Minister for Health the intervention services available for a person (details supplied) who has been diagnosed with autism spectrum dis- order; in particular, to confirm whether any speech and language, and occupational therapy is available for them; and if he will make a statement on the matter. [16676/16]

21/06/2016WRQ04400Minister of State at the Department of Health (Deputy Finian McGrath): The Govern- ment is committed to providing services and supports for people with disabilities which will empower them to live independent lives, provide greater independence in accessing the servic- es they choose, and enhance their ability to tailor the supports required to meet their needs and plan their lives. This commitment is outlined in the Programme for Partnership Government, which is guided by two principles: equality of opportunity and improving the quality of life for people with disabilities.

As the Deputy’s question relates to an individual case, I have arranged for the question to be referred to the Health Service Executive (HSE) for direct reply to the Deputy. If the Deputy has not received a reply from the HSE within 15 working days, he can contact my Private Office and they will follow the matter up with the HSE.

21/06/2016WRQ04450Medical Products

21/06/2016WRQ04500354. Deputy Aengus Ó Snodaigh asked the Minister for Health to set out the status of funding for a medical device for a person (details supplied). [16679/16]

21/06/2016WRQ04600Minister for Health (Deputy Simon Harris): As the particular issue raised relates to an individual case, I have arranged for the question to be referred to the HSE for direct reply to you. If you have not received a reply from the HSE within 15 working days, please contact my Private Office and they will follow up the matter with them.

HSE Agency Staff Data

175 Questions - Written Answers

21/06/2016WRR00200355. Deputy Louise O’Reilly asked the Minister for Health the progress to date in the con- version of agency staff posts to permanent positions; the number of posts converted in the past six, 12 and 18 months; the approximate savings to the Exchequer of converting these positions; and if he will make a statement on the matter. [16683/16]

21/06/2016WRR00300Minister for Health (Deputy Simon Harris): I have asked the HSE to respond to the Deputy directly on this matter. If you have not received a reply from the HSE within 15 work- ing days please contact my Private Office and they will follow up the matter with them.

Hospital Services

21/06/2016WRR00400356. Deputy Pat Buckley asked the Minister for Health if he is aware that MRI scans are not currently carried out at Cork University Hospital on weekends; the rationale for this; and if he will make a statement on the matter. [16701/16]

21/06/2016WRR00500Minister for Health (Deputy Simon Harris): As this is a service matter, I have asked the HSE to respond to you directly. If you have not received a reply from the HSE within 15 work- ing days please contact my Private Office and my officials will follow the matter up.

Hospital Waiting Lists

21/06/2016WRR00600357. Deputy Gerry Adams asked the Minister for Health the waiting time and the number of persons on the waiting list for computed tomography, CT, scans in the Louth hospital in Dundalk, County Louth; and if he will make a statement on the matter. [16731/16]

21/06/2016WRR00700Minister for Health (Deputy Simon Harris): As this is a service matter, I have asked the HSE to respond to you directly. If you have not received a reply from the HSE within 15 work- ing days please contact my Private Office and my officials will follow the matter up.

Health Services

21/06/2016WRR00800358. Deputy John Brassil asked the Minister for Health to expedite the physiology assess- ment for a person (details supplied); and if he will make a statement on the matter. [16735/16]

21/06/2016WRR00900Minister of State at the Department of Health (Deputy Finian McGrath): The Govern- ment is committed to providing services and supports for people with disabilities which will empower them to live independent lives, provide greater independence in accessing the servic- es they choose, and enhance their ability to tailor the supports required to meet their needs and plan their lives. This commitment is outlined in the Programme for Partnership Government, which is guided by two principles: equality of opportunity and improving the quality of life for people with disabilities.

As the Deputy’s question relates to an individual case, I have arranged for the question to be referred to the Health Service Executive (HSE) for direct reply to the Deputy. If the Deputy has not received a reply from the HSE within 15 working days, he can contact my Private Office and they will follow the matter up with the HSE.

Hospital Services

176 21 June 2016

21/06/2016WRR01000359. Deputy John Brassil asked the Minister for Health to expedite an operation for a per- son (details supplied); and if he will make a statement on the matter. [16752/16]

21/06/2016WRR01100Minister for Health (Deputy Simon Harris): Under the Health Act 2004, the Health Ser- vice Executive (HSE) is required to manage and deliver, or arrange to be delivered on its be- half, health and personal social services. Section 6 of the HSE Governance Act 2013 bars the Minister for Health from directing the HSE to provide a treatment or a personal service to any individual or to confer eligibility on any individual.

In relation to the specific case raised, I have asked the HSE to respond to you directly. If you have not received a reply from the HSE within 15 working days please contact my Private Office and my officials will follow the matter up.

Nursing Home Services

21/06/2016WRR01200360. Deputy Peter Burke asked the Minister for Health why a person (details supplied) is not allowed to return home where there are qualified persons to provide adequate care. [16753/16]

21/06/2016WRR01300Minister of State at the Department of Health (Deputy Helen McEntee): The Health Information and Quality Authority is the independent authority established under the Health Act 2007 to drive continuous improvement and to monitor safety and quality in Ireland’s health and personal social care services. Since 2009 all nursing homes - public, voluntary and private have been registered and inspected by HIQA.

The functions of the Minister for Health in relation to HIQA, and the Chief Inspector of Social Services are prescribed in the Act, and in this respect the Chief Inspector is independent in the exercise of these functions.

It would not be possible to comment on any individual case without knowing the full cir- cumstances of that case.

However, the Health Act 2007 (Care and Welfare of Residents in Designated Centres for Older People) Regulations 2013, as amended, include provisions governing the contract for provision of services, including a requirement that a discharge shall be discussed, planned for and agreed with a resident and, where appropriate, with their family or carer, and in accordance with the terms and conditions of the contract.

Registered providers of nursing home care are obliged to provide an accessible and effective complaints procedure. Concerns should in the first instance be taken up with the nursing home provider. The Office of the Ombudsman can examine complaints about the actions of a range of public bodies and, from 24 August 2015, complaints relating to the administrative actions of private nursing homes. The Office of the Ombudsman normally only deals with a complaint once the individual has already gone through the complaints procedure of the private nursing home concerned.

Ambulance Service

21/06/2016WRR01400361. Deputy Danny Healy-Rae asked the Minister for Health if he will provide adequate funding for the ambulance service in County Kerry as it is clearly evident that emergency time targets are not being met; and if he will make a statement on the matter. [16756/16]

177 Questions - Written Answers

21/06/2016WRR01500Minister for Health (Deputy Simon Harris): As this is a service matter, I have asked the HSE to respond to you directly. If you have not received a reply from the HSE within 15 work- ing days please contact my Private Office and my officials will follow the matter up.

Health Services

21/06/2016WRR01600362. Deputy David Cullinane asked the Minister for Health when patients with type 1 diabetes will be able to avail of dose adjustment for normal eating courses or any carbohydrate counting courses in Waterford city and county either at University Hospital Waterford or else- where; and if he will make a statement on the matter. [16771/16]

21/06/2016WRR01700Minister of State at the Department of Health (Deputy Marcella Corcoran Kennedy): As the Deputy’s question relates to service matters, I have arranged for the question to be re- ferred to the Health Service Executive for direct reply to the Deputy. If you have not received a reply from the HSE within 15 working days please contact my Private Office and they will follow up the matter with them.

Health Services Data

21/06/2016WRR01800363. Deputy David Cullinane asked the Minister for Health the number of diabetes adult dieticians employed at University Hospital Waterford; his plans to make new posts available; and if he will make a statement on the matter. [16772/16]

21/06/2016WRR01900364. Deputy David Cullinane asked the Minister for Health the waiting times and number of patients in the bands zero to three months, three months to six months, six months to one year, one year to two years, two years and above to see an endocrinologist at University Hospi- tal Waterford by type 1 and type 2 diabetics in each of the years 2013 to 2016 to date; and if he will make a statement on the matter. [16773/16]

21/06/2016WRR02000365. Deputy David Cullinane asked the Minister for Health the number of cardiac patients in University Hospital Waterford who are diabetics awaiting an appointment with an endocri- nologist; and if he will make a statement on the matter. [16774/16]

21/06/2016WRR02100Minister for Health (Deputy Simon Harris): I propose to take Questions Nos. 363 to 365, inclusive, together.

As these are service matters, I have asked the HSE to respond to you directly. If you have not received a reply from the HSE within 15 working days please contact my Private Office and my officials will follow the matter up.

Mental Health Services

21/06/2016WRR02200366. Deputy James Browne asked the Minister for Health if a nurse is permitted to dis- charge a person from psychiatric care; and if he will make a statement on the matter. [16779/16]

21/06/2016WRR02300Minister of State at the Department of Health (Deputy Helen McEntee): As this is a ser- vice issue, this question has been referred to the HSE for direct reply. If you have not received a reply within 15 working days, please contact my Private Office and they will follow up the matter with them.

178 21 June 2016 Hospital Appointments Status

21/06/2016WRR02400367. Deputy Pearse Doherty asked the Minister for Health when a person (details supplied) will receive a hospital appointment; and if he will make a statement on the matter. [16791/16]

21/06/2016WRR02500Minister for Health (Deputy Simon Harris): Under the Health Act 2004, the Health Ser- vice Executive (HSE) is required to manage and deliver, or arrange to be delivered on its be- half, health and personal social services. Section 6 of the HSE Governance Act 2013 bars the Minister for Health from directing the HSE to provide a treatment or a personal service to any individual or to confer eligibility on any individual.

In relation to the specific case raised, I have asked the HSE to respond to you directly. If you have not received a reply from the HSE within 15 working days please contact my Private Office and my officials will follow the matter up.

HSE Reports

21/06/2016WRR02600368. Deputy Róisín Shortall asked the Minister for Health why there has been a delay in publishing the report on the ophthalmic clinical development programme; his plans for same and his views in respect of this programme. [16795/16]

21/06/2016WRR02700Minister for Health (Deputy Simon Harris): I assume the Deputy is referring to the HSE’s National Clinical Programme for Ophthalmology’s model of care. I am informed by the HSE that this is a work in progress and has not been delayed. The aim of the National Clinical Pro- gramme for Ophthalmology is to ensure equitable and high quality care for ophthalmic patients and to minimise the incidence of needless blindness. The Programme has devised a high level strategy for the delivery of eye care and is of the view that a community based model is most appropriate. This is supportive of the aims of the Primary Care Eye Services Review Group, which was established by the HSE’s Primary Care Division. This Review Group is currently preparing an implementation plan to support its Review. The overall aim of the Review Group is to shift the management of many eye conditions from acute care services to primary care and community services, as appropriate. The Government is committed to updating the national eye care plan, including evaluation of the Sligo model for cataract surgery. These are matters which I intend to consider in the context of the implementation of the Health commitments in the Programme for Government and having regard to the funding available to facilitate such measures.

Health Products Regulatory Authority

21/06/2016WRR02800369. Deputy Clare Daly asked the Minister for Health based on the Health Products Regu- latory Authority adverse reactions database, the ten worst drugs for suicide related reactions, that is, suicidal ideation, suicide attempt or self-harm and completed suicide based on the num- ber of reported suicide related cases expressed as a percentage of the total number of all re- ported adverse reaction cases for each drug listed; and to provide the actual percentage for each of the drugs. [16799/16]

21/06/2016WRR02900370. Deputy Clare Daly asked the Minister for Health based on the Health Products Regu- latory Authority adverse reactions database, the ten worst drugs for completed suicide based on the number of reported completed suicides expressed as a percentage of the total number of all reported adverse reaction cases for each drug listed; and to provide the actual percentage for

179 Questions - Written Answers each of the drugs. [16800/16]

21/06/2016WRR03000Minister for Health (Deputy Simon Harris): I propose to take Questions Nos. 369 and 370 together.

The Health Products Regulatory Authority (HPRA) has informed me that initial searches have indicated that this is a large piece of work. The HPRA will collect the data and revert di- rectly to the Deputy. It will take 2 to 3 weeks to retrieve and review the information requested by the Deputy.

Health Services

21/06/2016WRR03100371. Deputy Billy Kelleher asked the Minister for Health his plans to set up a dedicated clinic for diagnosis and treatment of myalgic encephalomyelitis, ME, chronic fatigue syndrome, CFS; and if he will make a statement on the matter. [16801/16]

21/06/2016WRR03200Minister of State at the Department of Health (Deputy Marcella Corcoran Kennedy): As the Deputy’s question relates to service matters, I have arranged for the question to be re- ferred to the Health Service Executive for direct reply to the Deputy. If you have not received a reply from the HSE within 15 working days please contact my Private Office and they will follow up the matter with them.

Health Services Expenditure

21/06/2016WRR03300372. Deputy Billy Kelleher asked the Minister for Health the amount he invests in the research of myalgic encephalomyelitis-chronic fatigue syndrome; his plans to provide an in- creased investment in research; if so, when and the amount; and if he will make a statement on the matter. [16802/16]

21/06/2016WRR03400Minister of State at the Department of Health (Deputy Marcella Corcoran Kennedy): The Health Research Board is the statutory agency under the aegis of my Department respon- sible for supporting and funding health research, information and evidence. It funds research in all areas including clinical, population health and health services research. All applications received are peer reviewed by international experts and recommendations are made for funding based on a number of parameters; these include excellence, originality, track record of research team, relevance and potential impact.

The Health Research Board has advised my department that it has no record in the last num- ber of years of funding research of the type referred to by the deputy; the one application related to Chronic Fatigue Syndrome that it received in this time was not successful in obtaining fund- ing following peer review. The board has also emphasised that the provenance of any proposal for a research topic is the researcher concerned; and that it would welcome applications from any disease area - including Chronic Fatigue Syndrome.

Special Educational Needs Service Provision

21/06/2016WRR03500373. Deputy Billy Kelleher asked the Minister for Health why he has not ensured the provi- sion of physiotherapy, occupational therapy as well as speech and language therapy for students regarding a school (details supplied); if he is aware that this purpose built school is under-re- sourced compared to mainstream schools and the huge pressure that this places on the school to 180 21 June 2016 provide the basics for these children; and if he will make a statement on the matter. [16812/16]

21/06/2016WRR03600Minister of State at the Department of Health (Deputy Finian McGrath): The Govern- ment is committed to providing services and supports for people with disabilities which will empower them to live independent lives, provide greater independence in accessing the servic- es they choose, and enhance their ability to tailor the supports required to meet their needs and plan their lives. This commitment is outlined in the Programme for Partnership Government, which is guided by two principles: equality of opportunity and improving the quality of life for people with disabilities.

The particular issue raised by the Deputy is a service matter for the HSE. Accordingly I have arranged for the question to be referred to the Health Service Executive (HSE) for direct reply to the Deputy. If the Deputy has not received a reply from the HSE within 15 working days, he can contact my Private Office and they will follow up the matter with the HSE.

Hospital Waiting Lists

21/06/2016WRR03700374. Deputy Kevin O’Keeffe asked the Minister for Health to arrange an urgent appoint- ment for a person (details supplied). [16823/16]

21/06/2016WRR03800Minister for Health (Deputy Simon Harris): Under the Health Act 2004, the Health Ser- vice Executive (HSE) is required to manage and deliver, or arrange to be delivered on its be- half, health and personal social services. Section 6 of the HSE Governance Act 2013 bars the Minister for Health from directing the HSE to provide a treatment or a personal service to any individual or to confer eligibility on any individual.

The National Waiting List Management Policy, A standardised approach to managing scheduled care treatment for in-patient, day case and planned procedures, January 2014, has been developed to ensure that all administrative, managerial and clinical staff follow an agreed national minimum standard for the management and administration of waiting lists for sched- uled care. This policy, which has been adopted by the HSE, sets out the processes that hospitals are to implement to manage waiting lists.

In relation to the particular query raised, as this is a service matter, I have asked the HSE to respond to you directly. If you have not received a reply from the HSE within 15 working days please contact my Private Office and my officials will follow the matter up.

Prescriptions Charges

21/06/2016WRR03900375. Deputy Brendan Griffin asked the Minister for Health the financial exemptions avail- able to a person (details supplied); and if he will make a statement on the matter. [16848/16]

21/06/2016WRR04000Minister for Health (Deputy Simon Harris): Medical card holders are entitled to a range of health services free of charge. These services include:

- a range of family doctor or GP services from a local doctor;

- the supply of prescribed approved medicines, subject to a prescription charge;

- certain medical aids and appliances;

- all in-patient services in public wards in public hospitals, including public consultant

181 Questions - Written Answers services;

- all out-patient services in public hospitals, including public consultant services;

- certain Dental, Ophthalmic, and Aural health services;

- some personal and social care services; for example, public health nursing, social work services and other community care services based on client need.

Medical card holders are required to pay a prescription charge of €2.50 per item for medi- cines and other prescription items supplied to them by community pharmacists under the Gen- eral Medical Services scheme. This charge is subject to a monthly cap of €25 for each person or family.

The prescription charge is a matter which I intend to consider in the context of the imple- mentation of the Health commitments in the Programme for Government and having regard to the funding available to facilitate such measures.

Hospitals Building Programme

21/06/2016WRR04100376. Deputy Louise O’Reilly asked the Minister for Health the rationale for the location of the new children’s hospital; the criteria his Department used in the decision process; the evi- dence and criteria it used to specifically demonstrate that adult co-location was necessary and optimal; the evidence of improved clinical outcomes in respect of adult co-location it used; and if he will make a statement on the matter. [16868/16]

21/06/2016WRR04200Minister for Health (Deputy Simon Harris): Planning permission was granted on 28 April last by An Bord Pleanala for the new children’s hospital development at the St James’s campus, for satellite centres of the hospital to provide urgent and outpatient care on the cam- puses of Tallaght and Connolly hospitals, and for a Children’s Research and Innovation Centre at the St James’s campus and a family accommodation unit adjacent to the children’s hospital. It is intended that the first phase of building works will commence in the coming weeks.

The Government decision that the new children’s hospital should be co-located with St James’s on its campus in Dublin 8 was clinically led. In 2006, the McKinsey report, Children’s Health First, recommended that the population of Ireland and projected demand could support only one world-class tertiary paediatric centre, that this should be in Dublin and that it should ideally be co-located with a leading adult academic hospital, to ensure relevant sub-specialty and academic linkages. The McKinsey report recognised the importance for quality of health- care of having a critical mass of sub-specialist skills in a tertiary centre and stated this could be achieved firstly, by serving a large enough population to support a full complement of sub- specialists and secondly, by co-locating with an adult teaching hospital thus enabling access to specialties that encompass both adult and paediatric patients, facilitating clinical and academic “cross fertilisation”, and attracting the top staff.

Reviews since 2006 have reaffirmed the importance of co-location with a major adult aca- demic teaching hospital. In 2011, Minister Reilly established an Independent Review Group to examine the project at the Mater. The clinical aspect of this review was carried out by a team of four Chief Executive Officers drawn from the National Association of Children’s Hospitals (NACHRI) and the Children’s Hospitals International Executive Forum (CHIEF). Their report stated that “co-locating with tertiary adult and maternity hospitals is essential to the develop- ment of an excellent paediatric service. This has become best practice internationally and was recognised in the McKinsey report.” 182 21 June 2016 In 2012, following the refusal of planning permission for the project at the Mater campus, Minister Reilly established the Dolphin Review Group to advise on next steps. Their report stated that “Having reviewed the reports already prepared and considered their analysis and opinions, and having also consulted with a wide range of professionals in the field, we are also of the view that co-location is essential and tri-location optimal. We recommend that the Minis- ter remains on this path.” It identified St James’s as the most suitable adult partner for the new children’s hospital, from a clinical and research perspective.

The Government decision to co-locate the hospital with St James’s was announced on 6 November 2012. In identifying the location, the Government carefully considered the report of the Dolphin Group, established earlier that year to advise on next steps following refusal of planning permission for the hospital at its previous location. The Government also consid- ered detailed supplementary information on cost, time and planning which was subsequently sought from those members of the Group with the relevant technical expertise. The decision was clinically led, and was made in the best interests of children, with clinical considerations paramount in the decision. As announced in June 2015, the Coombe Women and Infants Uni- versity Hospital will relocate to the campus in time, achieving tri-location of adult, paediatric and maternity services. Tri-location has benefits for children, adolescents, newborns and moth- ers. In all cases, the benefits of tri-location are maximised where the adult hospital provides the broadest possible range of clinical sub-specialties and expertise, which are readily accessible for paediatric and maternity patients on the shared campus. Also, tri-location that delivers the most significant breadth and depth of clinical and academic research on site will enhance the potential of research to drive best clinical outcomes. St James’s Hospital has the broadest range of national specialties of all acute hospitals, as well as strong and well established research and education infrastructure, making it the hospital that best meets the criteria to be the adult co- location partner.

21/06/2016WRS00150Hospitals Building Programme

21/06/2016WRS00200377. Deputy Louise O’Reilly asked the Minister for Health if he supports the tri-location model; why he has not submitted planning permission for the building of the maternity hospital at the St. James’s Hospital site; and if he will make a statement on the matter. [16870/16]

21/06/2016WRS00300Minister for Health (Deputy Simon Harris): Tri-location of adult, paediatric and mater- nity services provides mothers and babies with access to a full range of medical and support services should the need arise, including immediate access on-site to paediatric services when foetal or neonatal surgery and other interventions are required. The availability of these ser- vices helps ensure the delivery of an optimum, safe service, particularly for high risk mothers and babies.

Planning permission was obtained in April this year for the new children’s hospital co-lo- cated with St. James’s Hospital on its campus at Dublin 8. In line with Government policy, the Coombe Women & Infants University Hospital will also relocate to the campus in time, achiev- ing tri-location of adult, paediatric and maternity services on the one campus. Accordingly, in submitting its planning application for the children’s hospital, the National Paediatric Hospital Development Board, as a matter of good planning practice, provided information on all known future developments for the St James’s Hospital campus, including the future maternity hospi- tal. The design for the children’s hospital provides for consideration of future maternity hospi- tal requirements in certain shared service areas.

The decision to relocate the Coombe Women & Infants University Hospital to the St James’s Hospital campus was announced on 30th June last year. This project is, as yet, in the very early 183 Questions - Written Answers stages of development, and significant work, including the preparation of a design brief and design development, will be required before any planning application can be made.

21/06/2016WRS00350National Children’s Hospital

21/06/2016WRS00400378. Deputy Louise O’Reilly asked the Minister for Health the consultation his Depart- ment undertook with parents’ groups, patient advocacy groups, staff and other stakeholders in respect of the location of the new children’s hospital; when it undertook this; and if he will make a statement on the matter. [16871/16]

21/06/2016WRS00500Minister for Health (Deputy Simon Harris): The new children’s hospital will be co- located with St James’s Hospital on its campus at Dublin 8, and ultimately tri-located with the Coombe Women and Infants University Hospital, which will re-locate to the campus in due course. Satellite centres of the hospital, providing urgent and outpatient care, are to be built on the campuses of Connolly and Tallaght Hospitals. The decision of An Bord Pleanála on April 28 last to grant planning permission for the hospital, satellite centres and related buildings is a huge and very welcome milestone for the project, and means that construction can now com- mence on all three sites in the coming weeks and months.

At every stage of this project over the years, through the various reports and the design pro- cess, there has been extensive consultation with numerous stakeholders and interested parties.

In 2012, following the refusal by An Bord Pleanála of planning permission for the hospital at the Mater campus, Minister Reilly established the Dolphin Group to advise on next steps. The Group met with a range of stakeholder groups and interested parties, including advocacy groups such as the Jack and Jill foundation, Children in Hospital Ireland and the Youth Advi- sory Council, CEOs, clinicians, nurses and health and social care professionals from the exist- ing hospitals, and representatives of children’s research organisations.

Previous reports and reviews on the location also involved stakeholder consultation. The 2006 Joint HSE/Department of Health Task Force met with advocacy groups including Heart Children Ireland, Cystic Fibrosis Ireland, the Irish Patients Association and the New Crumlin Hospital Action Group, as well as with the existing hospitals and maternity hospitals. In 2011, an Independent Review Group established by Minister Reilly to review the project at the Mater campus also included stakeholder consultation as part of its work, consulting with parents and care givers as well as the hospitals themselves and the Faculty of Paediatrics.

During the design process for the hospital, there have also been extensive consultations with families, young people and children who are former or current users of the service, with staff of the existing hospitals, with clinical leads and with local residents. This process has led to the development of a world-class building which has been designed to enable staff to deliver the best possible clinical care for children and young people, while also seeking to provide a pleas- ant environment for staff and families.

21/06/2016WRS00550Ambulance Service Response Times

21/06/2016WRS00600379. Deputy Jack Chambers asked the Minister for Health to examine the case of a person (details supplied) who was left to wait almost two hours for an ambulance in extreme pain and discomfort after suffering a fall; the reason for the delay; and if he will make a statement on the matter. [16873/16]

184 21 June 2016

21/06/2016WRS00700Minister for Health (Deputy Simon Harris): As this is a service matter, I have asked the HSE to respond to you directly. If you have not received a reply from the HSE within 15 work- ing days please contact my Private Office and my officials will follow the matter up.

21/06/2016WRS00750Hospital Waiting Lists

21/06/2016WRS00800380. Deputy Jack Chambers asked the Minister for Health why a person (details supplied) was not placed on a waiting list for a hip replacement procedure until 13 May 2016; and if he will make a statement on the matter. [16874/16]

21/06/2016WRS00900420. Deputy Jack Chambers asked the Minister for Health the status of a hospital appoint- ment for a person (details supplied); and if he will make a statement on the matter. [17121/16]

21/06/2016WRS01000Minister for Health (Deputy Simon Harris): I propose to take Questions Nos. 380 and 420 together.

Under the Health Act 2004, the Health Service Executive (HSE) is required to manage and deliver, or arrange to be delivered on its behalf, health and personal social services. Section 6 of the HSE Governance Act 2013 bars the Minister for Health from directing the HSE to provide a treatment or a personal service to any individual or to confer eligibility on any individual.

In relation to the specific queries raised, I have asked the HSE to respond to you directly. If you have not received a reply from the HSE within 15 working days please contact my Private Office and my officials will follow the matter up.

21/06/2016WRS01050Prescriptions Data

21/06/2016WRS01100381. Deputy Billy Kelleher asked the Minister for Health his views that the figures released indicating that 16% of medical card holders are being prescribed anti-depressants are in line with international norms; if he has concerns over the rate of prescriptions; and if he will make a statement on the matter. [16876/16]

21/06/2016WRS01200Minister for Health (Deputy Simon Harris): Under the Health (Pricing and Supply of Medical Goods) Act 2013, the Health Service Executive (HSE) has statutory responsibility for the administration of the primary care schemes; therefore, the matter has been referred to the HSE for attention and direct reply to you.

If you have not received a reply from the HSE within 15 working days, please contact my Private Office and they will follow up the matter with them.

21/06/2016WRS01250Health Strategies

21/06/2016WRS01300382. Deputy David Cullinane asked the Minister for Health if he is aware that 80% of blindness incidence is preventable and that five persons per week became blind here in the years 2013 to 2016; if he agrees that investment in cost effective interventions can reduce economic costs; if he is committed to the development and full implementation of a comprehensive na- tional vision strategy; and if he will make a statement on the matter. [16904/16]

21/06/2016WRS01400389. Deputy Seán Barrett asked the Minister for Health if he has received correspon- dence from an organisation (details supplied) which is seeking the development of a strategic

185 Questions - Written Answers approach to the planning and provision of vision health services here; if he will consider the development and implementation of a comprehensive national vision strategy for Ireland; his plans to address these issues at an early date; and if he will make a statement on the matter. [16961/16]

21/06/2016WRS01500426. Deputy Thomas Pringle asked the Minister for Health his plans to develop and imple- ment a comprehensive national vision strategy as advocated by the National Vision Coalition; and if he will make a statement on the matter. [17182/16]

21/06/2016WRS01600Minister of State at the Department of Health (Deputy Finian McGrath): I propose to take Questions Nos. 382, 389 and 426 together.

The Government is committed to providing and developing vision services and supports through health prevention, screening and intervention policies and programmes, all of which contribute to address the priority goal of the World Health Organisation’s Vision 2020, namely to eliminate avoidable blindness. The health service is addressing the prevention of eye disease and blindness across the broad range of health services including acute, primary and commu- nity based services.

The aim of the National Clinical Programme for Ophthalmology is to ensure equitable and high quality care for ophthalmic patients and to minimise the incidence of needless blindness. The Programme has devised a high level strategy for the delivery of eye care and is of the view that a community based model is most appropriate. This is supportive of the aims of the Prima- ry Care Eye Services Review Group, which was established by the Health Service Executive’s Primary Care Division. This Review Group is currently preparing an implementation plan to support its Review. The overall aim of the Review Group is to shift the management of many eye conditions from acute care services to primary care and community services, as appropri- ate. The Programme for Partnership Government is committed to updating the national eye care plan, including evaluation of the Sligo model for cataract surgery.

People with disabilities, including blind and visually-impaired people, can access special- ist disability services, which are provided in a variety of community and residential settings. These may be provided directly by the HSE or in partnership with voluntary service providers such as the National Council for the Blind and the Irish Guide Dogs for the Blind.

I have received correspondence from Fighting Blindness in relation to these matters, which I intend to consider in the context of the implementation of the Health commitments in the Programme for Partnership Government and having regard to the funding available to facilitate such measures.

21/06/2016WRS01650Home Help Service Provision

21/06/2016WRS01700383. Deputy Aengus Ó Snodaigh asked the Minister for Health to release the required funding to the Health Service Executive for extra home help for a person (details supplied). [16908/16]

21/06/2016WRS01800Minister of State at the Department of Health (Deputy Helen McEntee): As this is a service matter it has been referred to the Health Service Executive for direct reply. If you have not received a reply from the HSE within 15 working days please contact my Private Office and they will follow up the matter with them.

21/06/2016WRS01850Health Services Provision 186 21 June 2016

21/06/2016WRS01900384. Deputy Billy Kelleher asked the Minister for Health the completion date of the review of the primary eye care services review group; when he will publicly comment on the outcome of the review; if he will act on the recommendations; and if he will make a statement on the matter. [16912/16]

21/06/2016WRS02000440. Deputy Thomas Pringle asked the Minister for Health the reason for delays in imple- menting the national eye care plan, which advocates for a shift from a hospital-led model of care, including an increased role for community based optometry; and if he will make a state- ment on the matter. [17206/16]

21/06/2016WRS02100Minister for Health (Deputy Simon Harris): I propose to take Questions Nos. 384 and 440 together.

The Primary Care Eye Services Review Group, which was established by the HSE, is cur- rently preparing an implementation plan to support its Review. The overall aim of the Review Group is to shift the management of many eye conditions from acute care services to primary care and community services, as appropriate. The Government is committed to updating the national eye care plan, including evaluation of the Sligo model for cataract surgery. These are matters which I intend to consider in the context of the implementation of the Health commit- ments in the Programme for Government and having regard to the funding available to facilitate such measures.

21/06/2016WRS02150Hospital Facilities

21/06/2016WRS02200385. Deputy Stephen S. Donnelly asked the Minister for Health if broadband Internet ac- cess is available to patients for each public hospital, in tabular form; if this service is available to patients on a majority of wards; and if he will make a statement on the matter. [16935/16]

21/06/2016WRS02300Minister for Health (Deputy Simon Harris): As this is a service matter, I have asked the Health Service Executive to respond to the Deputy directly. If you have not received a reply from the HSE within 15 working days, please contact my Private Office and they will follow up the matter with them.

21/06/2016WRS02350Hospital Waiting Lists

21/06/2016WRS02400386. Deputy Michael Healy-Rae asked the Minister for Health the status of an operation for a person (details supplied); and if he will make a statement on the matter. [16945/16]

21/06/2016WRS02500Minister for Health (Deputy Simon Harris): Under the Health Act 2004, the Health Ser- vice Executive (HSE) is required to manage and deliver, or arrange to be delivered on its be- half, health and personal social services. Section 6 of the HSE Governance Act 2013 bars the Minister for Health from directing the HSE to provide a treatment or a personal service to any individual or to confer eligibility on any individual.

The National Waiting List Management Policy, A standardised approach to managing scheduled care treatment for in-patient, day case and planned procedures, January 2014, has been developed to ensure that all administrative, managerial and clinical staff follow an agreed national minimum standard for the management and administration of waiting lists for sched- uled care. This policy, which has been adopted by the HSE, sets out the processes that hospitals are to implement to manage waiting lists.

In relation to the particular query raised, as this is a service matter, I have asked the HSE to 187 Questions - Written Answers respond to you directly. If you have not received a reply from the HSE within 15 working days please contact my Private Office and my officials will follow the matter up.

21/06/2016WRS02550Hospital Waiting Lists

21/06/2016WRS02600387. Deputy Michael Healy-Rae asked the Minister for Health the status of an operation for a person (details supplied); and if he will make a statement on the matter. [16946/16]

21/06/2016WRS02700Minister for Health (Deputy Simon Harris): Under the Health Act 2004, the Health Ser- vice Executive (HSE) is required to manage and deliver, or arrange to be delivered on its be- half, health and personal social services. Section 6 of the HSE Governance Act 2013 bars the Minister for Health from directing the HSE to provide a treatment or a personal service to any individual or to confer eligibility on any individual.

The National Waiting List Management Policy, A standardised approach to managing scheduled care treatment for in-patient, day case and planned procedures, January 2014, has been developed to ensure that all administrative, managerial and clinical staff follow an agreed national minimum standard for the management and administration of waiting lists for sched- uled care. This policy, which has been adopted by the HSE, sets out the processes that hospitals are to implement to manage waiting lists.

In relation to the particular query raised, as this is a service matter, I have asked the HSE to respond to you directly. If you have not received a reply from the HSE within 15 working days please contact my Private Office and my officials will follow the matter up.

21/06/2016WRS02750Hospital Waiting Lists

21/06/2016WRS02800388. Deputy Michael Healy-Rae asked the Minister for Health the status of an operation for a person (details supplied); and if he will make a statement on the matter. [16948/16]

21/06/2016WRS02900Minister for Health (Deputy Simon Harris): Under the Health Act 2004, the Health Ser- vice Executive (HSE) is required to manage and deliver, or arrange to be delivered on its be- half, health and personal social services. Section 6 of the HSE Governance Act 2013 bars the Minister for Health from directing the HSE to provide a treatment or a personal service to any individual or to confer eligibility on any individual.

The National Waiting List Management Policy, A standardised approach to managing scheduled care treatment for in-patient, day case and planned procedures, January 2014, has been developed to ensure that all administrative, managerial and clinical staff follow an agreed national minimum standard for the management and administration of waiting lists for sched- uled care. This policy, which has been adopted by the HSE, sets out the processes that hospitals are to implement to manage waiting lists.

In relation to the particular query raised, as this is a service matter, I have asked the HSE to respond to you directly. If you have not received a reply from the HSE within 15 working days please contact my Private Office and my officials will follow the matter up.

Question No. 389 answered with Question No. 382.

21/06/2016WRS03050Medical Aids and Appliances Provision

188 21 June 2016

21/06/2016WRS03100390. Deputy Kevin O’Keeffe asked the Minister for Health if he will provide a manual wheelchair to a person (details supplied). [16967/16]

21/06/2016WRS03200Minister for Health (Deputy Simon Harris): As the particular issue raised relates to an individual case, I have arranged for the question to be referred to the HSE for direct reply to you. If you have not received a reply from the HSE within 15 working days, please contact my Private Office and they will follow up the matter with them.

21/06/2016WRS03250HSE Staff

21/06/2016WRS03300391. Deputy Darragh O’Brien asked the Minister for Health if there are any work place- ment or internship schemes available within the Health Service Executive for recently qualified psychologists to assist them in attaining the 450 hours practical experience to enable them to become fully accredited; and if he will make a statement on the matter. [16971/16]

21/06/2016WRS03400Minister for Health (Deputy Simon Harris): I have asked the HSE to respond to the Deputy directly on this matter. If you have not received a reply from the HSE within 15 work- ing days please contact my Private Office and they will follow up the matter with them.

21/06/2016WRS03450Health Services Data

21/06/2016WRS03500392. Deputy Joe Carey asked the Minister for Health if he will address a matter (details supplied) regarding previous parliamentary questions; and if he will make a statement on the matter. [16975/16]

21/06/2016WRS03600Minister for Health (Deputy Simon Harris): With regard to the specific queries raised by the Deputy, as these are service matters, I have asked the HSE to respond to you directly. If you have not received a reply from the HSE within 15 working days please contact my Private Office and my officials will follow the matter up.

21/06/2016WRS03650Nursing Homes Support Scheme Applications

21/06/2016WRS03700393. Deputy John Brassil asked the Minister for Health to expedite an application by a person (details supplied) under the fair deal scheme; and if he will make a statement on the matter. [16976/16]

21/06/2016WRS03800Minister of State at the Department of Health (Deputy Helen McEntee): As this is a service matter it has been referred to the Health Service Executive for direct reply. If you have not received a reply from the HSE within 15 working days please contact my Private Office and they will follow up the matter with them.

21/06/2016WRS03850Hospital Accommodation Provision

21/06/2016WRS03900394. Deputy Fergus O’Dowd asked the Minister for Health to restore and ensure that there are appropriate emergency beds available in St. Mary’s, Drumcar, for service users who would otherwise need to leave St. Mary’s for one week every month and who are then forced to use private nursing home care which is not always appropriate for their needs. [16981/16]

21/06/2016WRS04000Minister of State at the Department of Health (Deputy Finian McGrath): As the Dep- 189 Questions - Written Answers uty’s question relates to service matters, I have arranged for the question to be referred to the Health Service Executive (HSE) for direct reply to the Deputy. If the Deputy has not received a reply from the HSE within 15 working days, the Deputy can contact my Private Office and they will follow the matter up with the HSE.

21/06/2016WRS04050Nursing Home Services

21/06/2016WRS04100395. Deputy James Lawless asked the Minister for Health the number of staff working in an institution (details supplied); if this is the full complement of staff required for the number of residents in this unit; if there are plans to add additional staff to this unit; and if he will make a statement on the matter. [16982/16]

21/06/2016WRS04200396. Deputy James Lawless asked the Minister for Health the transport arrangements for residents in an institution (details supplied); the number of vehicles available to this institution; if this unit has a specific allocation of specialised vehicles or is in a share arrangement for use of these vehicles; and if he will make a statement on the matter. [16983/16]

21/06/2016WRS04300Minister of State at the Department of Health (Deputy Finian McGrath): I propose to take Questions Nos. 395 and 396 together.

As the Deputy’s questions relate to service matters, I have arranged for the questions to be referred to the Health Service Executive (HSE) for direct reply to the Deputy. If the Deputy has not received replies from the HSE within 15 working days, the Deputy can contact my Private Office and they will follow the matter up with the HSE.

21/06/2016WRS04350Patient Transport Provision

21/06/2016WRS04400397. Deputy James Lawless asked the Minister for Health the policy of the Health Service Executive on receiving transport vehicles (details supplied) that have been sponsored by chari- table organisations; and if he will make a statement on the matter. [16984/16]

21/06/2016WRS04500Minister of State at the Department of Health (Deputy Finian McGrath): As the Dep- uty’s question relates to service matters, I have arranged for the question to be referred to the Health Service Executive (HSE) for direct reply to the Deputy. If the Deputy has not received a reply from the HSE within 15 working days, he can contact my Private Office and they will follow the matter up with the HSE.

Hospital Staff Data

21/06/2016WRT00300398. Deputy Billy Kelleher asked the Minister for Health the number of specialist nurses he will assign to support the newly appointed paediatric urologist and he will hire; the number which currently exist in the system and their starting date; and if he will make a statement on the matter. [16985/16]

21/06/2016WRT00400Minister for Health (Deputy Simon Harris): I have asked the HSE to respond to the Deputy directly on this matter. If you have not received a reply from the HSE within 15 work- ing days please contact my Private Office and they will follow up the matter with them.

Hospital Staff Recruitment 190 21 June 2016

21/06/2016WRT00600399. Deputy Billy Kelleher asked the Minister for Health the number and hospital loca- tions of the specialist paediatric urology nurses that are to be appointed and the date of com- mencement of their contracts, in tabular form; and if he will make a statement on the matter. [16986/16]

21/06/2016WRT00700Minister for Health (Deputy Simon Harris): I have asked the HSE to respond to the Deputy directly on this matter. If you have not received a reply from the HSE within 15 work- ing days please contact my Private Office and they will follow up the matter with them.

21/06/2016WRT00800Vaccination Programme

21/06/2016WRT00900400. Deputy Martin Ferris asked the Minister for Health to address the concerns of par- ents whose children, they believe, developed side effects from the human papilloma virus, HPV, vaccine, Gardasil. [16988/16]

21/06/2016WRT01000Minister for Health (Deputy Simon Harris): The immunisation programme in Ireland is based on the advice of the National Immunisation Advisory Committee (NIAC). NIAC is a committee of the Royal College of Physicians of Ireland comprising of experts in a num- ber of specialties including infectious diseases, paediatrics and public health. The commit- tee’s recommendations are informed by public health advice and international best practice. In 2009, NIAC recommended HPV vaccination for all 12 year old girls, and in September 2010, the HPV vaccination programme was introduced for all girls in the first year of second level schools. The HPV vaccine protects girls from developing cervical cancer when they are adults. It is available free of charge from the HSE for all girls in 1st year of secondary school and over 200,000 girls have received the vaccine since its introduction.

In Ireland, the Health Products Regulatory Authority (HPRA) is the regulatory authority for medicines in Ireland and is the appropriate authority to which possible adverse reactions to medicinal products should be reported. In November 2015, the European Medicines Agency completed a detailed scientific review of the HPV vaccine. The review which the HPRA par- ticipated in, specifically focused on rare reports of two conditions, complex regional pain syn- drome and postural orthostatic tachycardia syndrome associated with heart rate increase. The outcome of the review which was carried out by the EMA Pharmacovigilance and Risk Assess- ment Committee (PRAC) found no evidence of a causal link between the vaccine and the two conditions examined. The European Medicines Agency has advised healthcare professionals that available data does not warrant any change to the use of HPV vaccines as there are no new safety concerns regarding HPV vaccines. Healthcare professionals should therefore continue using the vaccines in accordance with the current product information. On 12 January 2016, the European Commission endorsed the conclusion of the European Medicines Agency stating that there is no need to change the way HPV vaccines are used or to amend the product information. This final outcome by the Commission is now binding in all member states.

However, I am aware of claims of an association between HPV vaccination and a number of symptoms experienced by a group of young women. It appears that some girls first suffered symptoms around the time they received the HPV vaccine, and understandably some parents have connected the vaccine to their daughter’s condition. I want to provide assurances that, as well as being eligible to seek medical attention and to access appropriate health and social care services, irrespective of the cause of their symptoms, the HSE will be in a position to facilitate access to specialist services as required due to the individual nature of the needs of some chil- dren.

191 Questions - Written Answers

21/06/2016WRT01100Air Pollution

21/06/2016WRT01200401. Deputy James Browne asked the Minister for Health if he is aware of a recent report in a publication (details supplied) that has linked air pollution to increased mental illness in children; and if he will make a statement on the matter. [16995/16]

21/06/2016WRT01300Minister of State at the Department of Health (Deputy Helen McEntee): My Depart- ment is aware of the report referred to by the Deputy, which will be taken into account by the HSE and other relevant statutory agencies, in the context of developing mental health policies and services for those under age 18, in the future.

21/06/2016WRT01400Hospital Appointments Status

21/06/2016WRT01500402. Deputy Mary Butler asked the Minister for Health the status of an operation for a person (details supplied) in County Waterford; and if he will make a statement on the matter. [16998/16]

21/06/2016WRT01600Minister for Health (Deputy Simon Harris): Under the Health Act 2004, the Health Ser- vice Executive (HSE) is required to manage and deliver, or arrange to be delivered on its be- half, health and personal social services. Section 6 of the HSE Governance Act 2013 bars the Minister for Health from directing the HSE to provide a treatment or a personal service to any individual or to confer eligibility on any individual.

The National Waiting List Management Policy, A standardised approach to managing scheduled care treatment for in-patient, day case and planned procedures, January 2014, has been developed to ensure that all administrative, managerial and clinical staff follow an agreed national minimum standard for the management and administration of waiting lists for sched- uled care. This policy, which has been adopted by the HSE, sets out the processes that hospitals are to implement to manage waiting lists.

In relation to the particular query raised, as this is a service matter, I have asked the HSE to respond to you directly. If you have not received a reply from the HSE within 15 working days please contact my Private Office and my officials will follow the matter up.

21/06/2016WRT01700Services for People with Disabilities

21/06/2016WRT01800403. Deputy Jim O’Callaghan asked the Minister for Health if he will reverse huge cuts to services made by the Health Service Executive, which will affect a school (details supplied) in County Dublin as there is clearly no reduction in demand for these services; and if he will make a statement on the matter. [17005/16]

21/06/2016WRT01900Minister of State at the Department of Health (Deputy Finian McGrath): The Govern- ment is committed to providing services and supports for people with disabilities which will empower them to live independent lives, provide greater independence in accessing the servic- es they choose, and enhance their ability to tailor the supports required to meet their needs and plan their lives. This commitment is outlined in the Programme for Partnership Government, which is guided by two principles: equality of opportunity and improving the quality of life for people with disabilities.

The particular issue raised by the Deputy is a service matter for the HSE. Accordingly I have arranged for the question to be referred to the Health Service Executive (HSE) for direct 192 21 June 2016 reply to the Deputy. If the Deputy has not received a reply from the HSE within 15 working days, he can contact my Private Office and they will follow up the matter with the HSE.

21/06/2016WRT02000Home Care Packages Provision

21/06/2016WRT02100404. Deputy John Curran asked the Minister for Health when he will allocate a home care package to a person (details supplied) in order to facilitate a return home; and if he will make a statement on the matter. [17018/16]

21/06/2016WRT02200Minister of State at the Department of Health (Deputy Helen McEntee): As this is a service matter it has been referred to the Health Service Executive for direct reply. If you have not received a reply from the HSE within 15 working days please contact my Private Office and they will follow up the matter with them.

21/06/2016WRT02300Hospital Appointments Status

21/06/2016WRT02400405. Deputy Peter Fitzpatrick asked the Minister for Health when a person (details sup- plied) will receive an appointment; and if he will make a statement on the matter. [17031/16]

21/06/2016WRT02500Minister for Health (Deputy Simon Harris): Under the Health Act 2004, the Health Ser- vice Executive (HSE) is required to manage and deliver, or arrange to be delivered on its be- half, health and personal social services. Section 6 of the HSE Governance Act 2013 bars the Minister for Health from directing the HSE to provide a treatment or a personal service to any individual or to confer eligibility on any individual.

The National Waiting List Management Policy, A standardised approach to managing scheduled care treatment for in-patient, day case and planned procedures, January 2014, has been developed to ensure that all administrative, managerial and clinical staff follow an agreed national minimum standard for the management and administration of waiting lists for sched- uled care. This policy, which has been adopted by the HSE, sets out the processes that hospitals are to implement to manage waiting lists.

In relation to the particular query raised, as this is a service matter, I have asked the HSE to respond to you directly. If you have not received a reply from the HSE within 15 working days please contact my Private Office and my officials will follow the matter up.

21/06/2016WRT02600Hospital Groups

21/06/2016WRT02700406. Deputy Sean Sherlock asked the Minister for Health when he proposes to establish a board for the south-south west group of hospitals. [17043/16]

21/06/2016WRT02800Minister for Health (Deputy Simon Harris): Pending the enactment of legislation, the Hospital Groups are being set up in line with the Government’s programme for reform of the health service initially on a non-statutory administrative basis, with Hospital Group Boards op- erating within the existing legislative framework. Hospital Group Chairpersons are already in place (appointed on an administrative basis) for all Hospital Groups, including the South/South West Hospital Group, and Board members in place in three out of seven hospital groups (Saolta University Healthcare, UL and Children’s Hospital Group). The appointment of members to the remaining Hospital Group Boards, including the South/South West Board, is expected to take place over the next months. 193 Questions - Written Answers

21/06/2016WRT02900Hospital Appointments Status

21/06/2016WRT03000407. Deputy Michael Healy-Rae asked the Minister for Health the status of an operation for a person (details supplied); and if he will make a statement on the matter. [17045/16]

21/06/2016WRT03100Minister for Health (Deputy Simon Harris): Under the Health Act 2004, the Health Ser- vice Executive (HSE) is required to manage and deliver, or arrange to be delivered on its be- half, health and personal social services. Section 6 of the HSE Governance Act 2013 bars the Minister for Health from directing the HSE to provide a treatment or a personal service to any individual or to confer eligibility on any individual.

The National Waiting List Management Policy, A standardised approach to managing scheduled care treatment for in-patient, day case and planned procedures, January 2014, has been developed to ensure that all administrative, managerial and clinical staff follow an agreed national minimum standard for the management and administration of waiting lists for sched- uled care. This policy, which has been adopted by the HSE, sets out the processes that hospitals are to implement to manage waiting lists.

In relation to the particular query raised, as this is a service matter, I have asked the HSE to respond to you directly. If you have not received a reply from the HSE within 15 working days please contact my Private Office and my officials will follow the matter up.

21/06/2016WRT03200Hospital Appointments Status

21/06/2016WRT03300408. Deputy Michael Healy-Rae asked the Minister for Health the status of an appoint- ment for an operation for a person (details supplied); and if he will make a statement on the matter. [17046/16]

21/06/2016WRT03400Minister for Health (Deputy Simon Harris): Under the Health Act 2004, the Health Ser- vice Executive (HSE) is required to manage and deliver, or arrange to be delivered on its be- half, health and personal social services. Section 6 of the HSE Governance Act 2013 bars the Minister for Health from directing the HSE to provide a treatment or a personal service to any individual or to confer eligibility on any individual.

The National Waiting List Management Policy, A standardised approach to managing scheduled care treatment for in-patient, day case and planned procedures, January 2014, has been developed to ensure that all administrative, managerial and clinical staff follow an agreed national minimum standard for the management and administration of waiting lists for sched- uled care. This policy, which has been adopted by the HSE, sets out the processes that hospitals are to implement to manage waiting lists.

In relation to the particular query raised, as this is a service matter, I have asked the HSE to respond to you directly. If you have not received a reply from the HSE within 15 working days please contact my Private Office and my officials will follow the matter up.

21/06/2016WRT03500Medicinal Products Prices

21/06/2016WRT03600409. Deputy Robert Troy asked the Minister for Health why medical card patients are charged an individual charge for differing strengths of warfarin, that is, if a person is on two warfarin tablets of differing strengths they are charged €5 instead of €2.50. [17047/16]

194 21 June 2016

21/06/2016WRT03700Minister for Health (Deputy Simon Harris): Medical card holders are required to pay a €2.50 charge per item for medicines and other prescription items supplied to them by commu- nity pharmacists, subject to a cap of €25 per month for each person or family.

Where a drug or medicine is required to be dispensed in different strengths and different containers, a prescription charge applies to each claimed strength of the product.

The administration of the prescription charge is an operational matter for the HSE, and I have asked the Executive to respond directly to you on this matter. If you have not received a reply from the HSE within 15 working days, please contact my Private Office and they will follow up the matter with them.

21/06/2016WRT03800Medical Card Applications

21/06/2016WRT03900410. Deputy Bernard J. Durkan asked the Minister for Health the status of an application for a medical card by a person (details supplied); and if he will make a statement on the matter. [17050/16]

21/06/2016WRT04000Minister for Health (Deputy Simon Harris): The Health Service Executive has been asked to examine this matter and to reply to the Deputy as soon as possible.

The Health Service Executive operates the General Medical Services scheme, which in- cludes medical cards and GP visit cards, under the Health Act 1970, as amended. It has es- tablished a dedicated contact service for members of the Oireachtas specifically for queries relating to medical cards and GP visit cards, which the Deputy may wish to use for an earlier response. Contact information has recently issued to Oireachtas members.

If the Deputy has not received a reply from the HSE within 15 working days, please contact my Private Office who will follow up the matter with them.

21/06/2016WRT04100Hospital Admissions

21/06/2016WRT04200411. Deputy Kevin O’Keeffe asked the Minister for Health if he will arrange an admission date for a person (details supplied). [17057/16]

21/06/2016WRT04300Minister for Health (Deputy Simon Harris): Under the Health Act 2004, the Health Ser- vice Executive (HSE) is required to manage and deliver, or arrange to be delivered on its be- half, health and personal social services. Section 6 of the HSE Governance Act 2013 bars the Minister for Health from directing the HSE to provide a treatment or a personal service to any individual or to confer eligibility on any individual.

The National Waiting List Management Policy, A standardised approach to managing scheduled care treatment for in-patient, day case and planned procedures, January 2014, has been developed to ensure that all administrative, managerial and clinical staff follow an agreed national minimum standard for the management and administration of waiting lists for sched- uled care. This policy, which has been adopted by the HSE, sets out the processes that hospitals are to implement to manage waiting lists.

In relation to the particular query raised, as this is a service matter, I have asked the HSE to respond to you directly. If you have not received a reply from the HSE within 15 working days please contact my Private Office and my officials will follow the matter up.

195 Questions - Written Answers Health Services

21/06/2016WRT04500412. Deputy Louise O’Reilly asked the Minister for Health if he is aware of the plan to build 24 rapid-build homes adjacent to Pinewood Estate in Balbriggan, County Dublin, and the plan to locate a small community facility on this site; his plans to locate a social worker on this site; if there is a social worker located specifically here, why he deems this necessary; why this service is not being located in the new primary care centre on the main street in Balbriggan; and if he will make a statement on the matter. [17062/16]

21/06/2016WRT04600Minister for Health (Deputy Simon Harris): I am advised by the HSE that the project and associated issues referred to by the Deputy concerning the building of Rapid Houses in Balbrig- gan is a matter proper to Fingal County Council and is not under the remit of the HSE.

Primary Care Centres Data

21/06/2016WRT04800413. Deputy Louise O’Reilly asked the Minister for Health the number of staff, by grade, that will be employed in the new primary care centre in Balbriggan, and the services they will provide; and if he will make a statement on the matter. [17063/16]

21/06/2016WRT04900Minister for Health (Deputy Simon Harris): I have asked the HSE to respond to the Deputy directly on this matter. If you have not received a reply from the HSE within 15 work- ing days please contact my Private Office and they will follow up the matter with them.

21/06/2016WRT05000Primary Care Centres Provision

21/06/2016WRT05100414. Deputy Louise O’Reilly asked the Minister for Health his plans to build a primary care centre in Swords, County Dublin; the timeline for this; and if he will make a statement on the matter. [17064/16]

21/06/2016WRT05200Minister for Health (Deputy Simon Harris): As this is a service issue, this question has been referred to the Health Service Executive for direct reply to the Deputy. If you have not received a reply from the HSE within 15 working days, please contact my Private Office and they will follow up the matter with them.

21/06/2016WRT05300Voluntary Sector Funding

21/06/2016WRT05400415. Deputy Dara Calleary asked the Minister for Health to outline all engagements be- tween the Health Service Executive and an organisation (details supplied) since March 2008; if the HSE received a sum of money in May 2009 on behalf of the organisation via a second organisation; to confirm the sum as €300,000 and to outline, in tabular form, how that money was spent; why a memorandum of association was used instead of the agreed service level agreement; to outline any funding from within his Department or within the HSE paid to this organisation since 2008; if he is aware that this organisation has been declined funding under the Pobal SSNO scheme; and if he will agree that this organisation is delivering an essential service to the families of ill children. [17068/16]

21/06/2016WRT05500Minister for Health (Deputy Simon Harris): CD Helping Hands received €78,000 in Lot- tery Funding from my Department in 2008 in order to set up a new office. Leukaemia Trust/ Cancer Care West Ltd. received €50,000 Lottery Funding from my Department in 2009 to go

196 21 June 2016 towards a new purpose built support centre.

The Pobal SSNO scheme is a matter for the Minister for Housing, Planning and Local Gov- ernment.

The remainder of the Deputy’s question relates to service delivery matters and accordingly I have asked the HSE to respond directly to him.

If you have not received a reply from the HSE within 15 working days please contact my Private Office and my officials will follow up the matter with them.

21/06/2016WRT05600Voluntary Sector Funding

21/06/2016WRT05700416. Deputy Niall Collins asked the Minister for Health the process a charity which pro- vides counselling and support to adults must follow in applying for annual funding under sec- tion 39; and if he will make a statement on the matter. [17077/16]

21/06/2016WRT05800Minister for Health (Deputy Simon Harris): Your question has been referred to the Health Service Executive for direct reply.

If you have not received a reply from the HSE within 15 working days please contact my Private Office and my officials will follow the matter up.

21/06/2016WRT05900Hospital Appointments Status

21/06/2016WRT06000417. Deputy Éamon Ó Cuív asked the Minister for Health when he will provide an opera- tion for a person (details supplied); the reason for the delay; and if he will make a statement on the matter. [17079/16]

21/06/2016WRT06100Minister for Health (Deputy Simon Harris): Under the Health Act 2004, the Health Ser- vice Executive (HSE) is required to manage and deliver, or arrange to be delivered on its be- half, health and personal social services. Section 6 of the HSE Governance Act 2013 bars the Minister for Health from directing the HSE to provide a treatment or a personal service to any individual or to confer eligibility on any individual.

The National Waiting List Management Policy, A standardised approach to managing scheduled care treatment for in-patient, day case and planned procedures, January 2014, has been developed to ensure that all administrative, managerial and clinical staff follow an agreed national minimum standard for the management and administration of waiting lists for sched- uled care. This policy, which has been adopted by the HSE, sets out the processes that hospitals are to implement to manage waiting lists.

In relation to the particular query raised, as this is a service matter, I have asked the HSE to respond to you directly. If you have not received a reply from the HSE within 15 working days please contact my Private Office and my officials will follow the matter up.

21/06/2016WRT06200Medical Card Applications Data

21/06/2016WRT06300418. Deputy Michael Healy-Rae asked the Minister for Health the number of medical card applications he has granted and refused to persons in County Kerry in 2015 and 2016 to date; and if he will make a statement on the matter. [17083/16] 197 Questions - Written Answers

21/06/2016WRT06400Minister for Health (Deputy Simon Harris): The Health Service Executive has been asked to examine this matter and to reply to the Deputy as soon as possible.

The Health Service Executive operates the General Medical Services scheme, which in- cludes medical cards and GP visit cards, under the Health Act 1970, as amended. It has es- tablished a dedicated contact service for members of the Oireachtas specifically for queries relating to medical cards and GP visit cards, which the Deputy may wish to use for an earlier response. Contact information has recently issued to Oireachtas members.

If the Deputy has not received a reply from the HSE within 15 working days, please contact my Private Office who will follow up the matter with them.

21/06/2016WRU00150Medical Card Data

21/06/2016WRU00200419. Deputy Michael Healy-Rae asked the Minister for Health the processing time of medical card applications and the number of applications being dealt with at present; and if he will make a statement on the matter. [17084/16]

21/06/2016WRU00300Minister for Health (Deputy Simon Harris): The Health Service Executive has been asked to examine this matter and to reply to the Deputy as soon as possible. The Health Service Executive operates the General Medical Services scheme, which includes medical cards and GP visit cards, under the Health Act 1970, as amended. It has established a dedicated contact service for members of the Oireachtas specifically for queries relating to medical cards and GP visit cards, which the Deputy may wish to use for an earlier response. Contact information has recently issued to Oireachtas members. If the Deputy has not received a reply from the HSE within 15 working days, please contact my Private Office who will follow up the matter with them.

Question No. 420 answered with Question No. 380.

Medicinal Products Availability

21/06/2016WRU00500421. Deputy Gino Kenny asked the Minister for Health to clarify the availability of the anti-epilepsy drug Epidiolex here; the arrangements for facilitating access by children with Dravet syndrome and Lennox-Gastaut syndrome; and if he will intervene to improve access. [17166/16]

21/06/2016WRU00600Minister for Health (Deputy Simon Harris): Epidiolex is an investigational medicinal product currently undergoing clinical trials in a number of EU Member States. I have no au- thority to compel pharmaceutical companies to conduct such trials in Ireland.

Under European and Irish legislation, before a medicine can be placed on the market the manufacturer must seek an authorisation from the Health Products Regulatory Authority (HPRA), or in the case of certain medicines, the European Medicines Agency. To my knowl- edge, no application for a marketing authorisation for Epidiolex for the treatment of Dravet Syndrome or Lennox-Gastaut Syndrome has been submitted.

Where a medicine is not authorised in Ireland, a licensed wholesaler may import it if it has been prescribed by a doctor for a patient under his/her care, on his/her direct responsibility and in order to meet the special needs of a patient. The process for this is described in the ‘Guide to the Notification System for Exempt Medicinal Products’ which is available on the website of the HPRA, which administers the scheme. For this notification scheme, there are a number of 198 21 June 2016 specialist wholesalers which deal with the importation of ‘exempt medicines’ into Ireland, via pharmacies and subject to a prescription.

The HPRA is aware that Epidiolex has been prescribed and imported into Ireland under the exempt medicinal product route in response to a bona fide order from a registered medical practitioner.

National Drugs Strategy

21/06/2016WRU00700422. Deputy Aengus Ó Snodaigh asked the Minister for Health when he will complete and publish the new national drugs strategy and the reason for the delay. [17170/16]

21/06/2016WRU00800Minister of State at the Department of Health (Deputy Catherine Byrne): The Depart- ment of Health is developing a new National Drugs Strategy, which will take effect in 2017, when the current policy expires. A Steering Committee has been established to oversee the process and to advise me on how the new Strategy should address problem drug use, including the structures through which this should be done and incorporating key performance indicators to measure the future effectiveness of the new policy.

I will shortly be announcing details of the consultation process on the new Strategy, which I intend to be as broad, comprehensive and inclusive as possible. I am expecting the final report of the Steering Committee by the end of the year.

Disability Support Services Provision

21/06/2016WRU00900423. Deputy Joan Burton asked the Minister for Health when he will make available sup- port services from an organisation to a person (details supplied); and if he will make a statement on the matter. [17171/16]

21/06/2016WRU01000Minister of State at the Department of Health (Deputy Finian McGrath): The Govern- ment is committed to providing services and supports for people with disabilities which will empower them to live independent lives, provide greater independence in accessing the servic- es they choose, and enhance their ability to tailor the supports required to meet their needs and plan their lives. This commitment is outlined in the Programme for Partnership Government, which is guided by two principles: equality of opportunity and improving the quality of life for people with disabilities.

As the Deputy’s question relates to an individual case, I have arranged for the question to be referred to the Health Service Executive (HSE) for direct reply to the Deputy. If the Deputy has not received a reply from the HSE within 15 working days, she can contact my Private Office and they will follow the matter up with the HSE.

Hospital Waiting Lists

21/06/2016WRU01100424. Deputy Michael Healy-Rae asked the Minister for Health when he will allocate a date for an operation for a person (details supplied); and if he will make a statement on the matter. [17178/16]

21/06/2016WRU01200Minister for Health (Deputy Simon Harris): Under the Health Act 2004, the Health Ser- vice Executive (HSE) is required to manage and deliver, or arrange to be delivered on its be-

199 Questions - Written Answers half, health and personal social services. Section 6 of the HSE Governance Act 2013 bars the Minister for Health from directing the HSE to provide a treatment or a personal service to any individual or to confer eligibility on any individual.

The National Waiting List Management Policy, A standardised approach to managing scheduled care treatment for in-patient, day case and planned procedures, January 2014, has been developed to ensure that all administrative, managerial and clinical staff follow an agreed national minimum standard for the management and administration of waiting lists for sched- uled care. This policy, which has been adopted by the HSE, sets out the processes that hospitals are to implement to manage waiting lists.

In relation to the particular query raised, as this is a service matter, I have asked the HSE to respond to you directly. If you have not received a reply from the HSE within 15 working days please contact my Private Office and my officials will follow the matter up.

Mental Health Services Provision

21/06/2016WRU01300425. Deputy Michael Healy-Rae asked the Minister for Health the status of the allocation of psychiatric services (details supplied) in County Kerry; and if he will make a statement on the matter. [17181/16]

21/06/2016WRU01400Minister of State at the Department of Health (Deputy Helen McEntee): As this is a ser- vice issue this question has been referred to the HSE for direct reply. If you have not received a reply within 15 working days, please contact my Private Office and they will follow up the matter with them.

Question No. 426 answered with Question No. 382.

Commencement of Legislation

21/06/2016WRU01600427. Deputy Margaret Murphy O’Mahony asked the Minister for Health if Part 2 of the 2005 Disability Act has been fully commenced. [17183/16]

21/06/2016WRU01700Minister of State at the Department of Health (Deputy Finian McGrath): All Parts of the Disability Act 2005 have been commenced except for Part 2, relating to the Assessment of Need, which has been partly commenced. In 2008, the then Government decided, in the light of financial circumstances, to defer further commencement of Part 2 of the Disability Act 2005 and the Education for Persons with Special Educational Needs Act (EPSEN) 2004.

The Assessment of Need process, which at present encompasses younger children, provides for the needs of eligible applicants occasioned by their disability to be assessed. The Cross Sec- toral Team comprising, inter alia, representatives of the Department of Health, the Department of Education and Skills, the Department of Children and Youth Affairs and the HSE, which was established to provide for a joint approach to the implementation of the Disability and EPSEN Acts, continues to meet to address issues of mutual concern for children with disabilities.

Hospital Appointments Status

21/06/2016WRU01800428. Deputy Barry Cowen asked the Minister for Health when a person (details supplied) will receive an appointment. [17184/16]

200 21 June 2016

21/06/2016WRU01900Minister for Health (Deputy Simon Harris): Under the Health Act 2004, the Health Ser- vice Executive (HSE) is required to manage and deliver, or arrange to be delivered on its be- half, health and personal social services. Section 6 of the HSE Governance Act 2013 bars the Minister for Health from directing the HSE to provide a treatment or a personal service to any individual or to confer eligibility on any individual.

The National Waiting List Management Policy, A standardised approach to managing scheduled care treatment for in-patient, day case and planned procedures, January 2014, has been developed to ensure that all administrative, managerial and clinical staff follow an agreed national minimum standard for the management and administration of waiting lists for sched- uled care. This policy, which has been adopted by the HSE, sets out the processes that hospitals are to implement to manage waiting lists.

In relation to the particular query raised, as this is a service matter, I have asked the HSE to respond to you directly. If you have not received a reply from the HSE within 15 working days please contact my Private Office and my officials will follow the matter up.

Hospital Appointments Status

21/06/2016WRU02000429. Deputy Barry Cowen asked the Minister for Health when a person (details supplied) will receive an appointment. [17185/16]

21/06/2016WRU02100Minister for Health (Deputy Simon Harris): As this is a service matter, it has been re- ferred to the Health Service Executive for direct reply to the Deputy. If you have not received a reply from the HSE within 15 working days please contact my Private Office and they will follow up the matter with them.

Hospital Appointments Status

21/06/2016WRU02200430. Deputy Barry Cowen asked the Minister for Health when a person (details supplied) will receive an appointment. [17186/16]

21/06/2016WRU02300Minister for Health (Deputy Simon Harris): Under the Health Act 2004, the Health Ser- vice Executive (HSE) is required to manage and deliver, or arrange to be delivered on its be- half, health and personal social services. Section 6 of the HSE Governance Act 2013 bars the Minister for Health from directing the HSE to provide a treatment or a personal service to any individual or to confer eligibility on any individual.

The scheduling of appointments for patients is a matter for the hospital to which the patient has been referred. Should a patient’s general practitioner consider that the patient’s condition warrants an earlier appointment, he or she should take the matter up with the consultant and the hospital involved. In relation to the specific case raised, I have asked the HSE to respond to you directly. If you have not received a reply from the HSE within 15 working days please contact my Private Office and my officials will follow the matter up.

Hospital Appointments Status

21/06/2016WRU02400431. Deputy Michael Healy-Rae asked the Minister for Health the status of the case of a person (details supplied) awaiting surgery; and if he will make a statement on the matter. [17192/16] 201 Questions - Written Answers

21/06/2016WRU02500Minister for Health (Deputy Simon Harris): Under the Health Act 2004, the Health Ser- vice Executive (HSE) is required to manage and deliver, or arrange to be delivered on its be- half, health and personal social services. Section 6 of the HSE Governance Act 2013 bars the Minister for Health from directing the HSE to provide a treatment or a personal service to any individual or to confer eligibility on any individual.

In relation to the specific case raised, I have asked the HSE to respond to you directly. If you have not received a reply from the HSE within 15 working days please contact my Private Office and my officials will follow the matter up.

Question No. 432 answered with Question No. 344.

Human Rights

21/06/2016WRU02700433. Deputy Louise O’Reilly asked the Minister for Health his plans to provide a person (details supplied) with adequate compensation and to make available any needed psychologi- cal treatment, as per the recommendation of the United Nations Human Rights Committee; the costs associated with this; and if he will make a statement on the matter. [17194/16]

21/06/2016WRU02800Minister for Health (Deputy Simon Harris): The United Nations Human Rights Com- mittee published its findings on Thursday 19th of June on complaints brought by AM against the State. The Human Rights Committee found that Ireland breached its obligations under a number of Articles of the International Covenant on Civil and Political Rights. The UN Human Rights Committee is the body of 18 independent experts that monitors implementation of the International Covenant on Civil and Political Rights by state parties.

We are currently studying the findings of the Committee and considering what measures should be taken. We are also consulting with the Office of the Attorney General and with the Department of Foreign Affairs and Trade as they act as the Agent for the Government before the United Nations Human Rights Committee.

Counselling Services Provision

21/06/2016WRU02900434. Deputy Louise O’Reilly asked the Minister for Health his plans to ensure that coun- selling is available for all women who terminate pregnancies, irrespective of reason, cause or circumstance; the costs associated with extending counselling services; and if he will make a statement on the matter. [17195/16]

21/06/2016WRU03000Minister for Health (Deputy Simon Harris): The provision of counselling services for women who terminate pregnancies is the responsibility of the Health Service Executive (HSE) and as such, I have referred the question to the HSE for attention and direct reply. If the deputy has not received a reply from the HSE within 15 working days, please contact my Private Office who will follow up the matter with them.

Services for People with Disabilities

21/06/2016WRU03100435. Deputy Robert Troy asked the Minister for Health if he has secured additional fund- ing for more residential care units; if so, the regions that will benefit from the additional units; and to supply a comparative list of number of persons with intellectual disabilities who are waiting for residential care placements, by county. [17196/16] 202 21 June 2016

21/06/2016WRU03200Minister of State at the Department of Health (Deputy Finian McGrath): The HSE’s report “Time to Move on from Congregated Settings – A Strategy for Community Inclusion”, (2011) proposes a new model of support in the community by moving people from institutional settings to the community. The plan is being rolled out at a regional and local level and involves full consultation with stakeholders.

The Programme for Government contains a commitment to continue to move people with disabilities out of congregated settings, to enable them to live independently and to be included in the community. Currently, 2725 people live in congregated settings and our objective is to reduce this figure by one-third by 2021 and ultimately, to eliminate all congregated settings.

The HSE has established a subgroup, under ‘Transforming Lives’, the Programme to imple- ment the recommendations of the Value for Money and Policy Review of Disability Services, which is developing an implementation plan for moving people from institutions. I welcome the fact that the needs of people moving from congregated settings will be fully taken into ac- count during this process as the model of care for individuals will be based on a person centred plan.

In terms of housing, the Department of Health and the Department of the Environment, Community and Local Government are working in collaboration to support the transition of people with a disability from institutions to social housing in the community under the Govern- ment’s National Housing Strategy for People with Disability 2011 to 2016.

The HSE’s 2016 National Service Plan has set a target of 165 people to move from institu- tions in 2016 into suitable accommodation. This is being supported by €20 million in capital funding from the Department of Health in respect of acquiring and renovating properties in priority institutions identified by the HSE.

In addition, I am pleased to note that the Department of the Environment will provide €10 million under the Capital Assistance Scheme to provide suitable accommodation for people transitioning from institutions in 2016. The HSE estimate that a further 100 people could ben- efit from this initiative. €1 million in ring-fenced leasing funding is also being made available by the Department of the Environment in 2016 to support people moving from institutions into suitable social housing in the community. This demonstrates the joined up commitment of both Departments to support the de-congregation programme.

The HSE has developed a three strand approach to accelerate transitions from institutions in the period 2016-2021, with a target of 900 people to move to more suitable accommodation in this period.

- Strand 1- is focussed on large institutional settings at high risk of not meeting HIQA Standards;

- Strand 2- is focussed on moving people into suitable social housing in the community through the Department of Environment schemes; and

- Strand 3- is focussed on remaining service users in congregated settings who could move to suitable accommodation.

Furthermore, underlining this Government’s commitment to the disability sector, I recently announced the provision of additional funding of €3m in 2016 for new initiatives, including an additional provision for services to meet the need of school leavers with disabilities and the anticipated cost of a number of emergency residential placements arising this year.

These emergency residential placements are intended to support individuals and families in 203 Questions - Written Answers crisis situations where their existing accommodation is no longer suitable.

As the Deputy’s question relates to service matters, I have arranged for the question to be referred to the Health Service Executive for direct reply to the Deputy. If the Deputy has not received a reply from the HSE within 15 working days, he can contact my Private Office and they will follow up the matter with the HSE.

Addiction Treatment Services

21/06/2016WRU03300436. Deputy Pat The Cope Gallagher asked the Minister for Health if he is aware that addiction services for persons under 18 years of age are not available in County Donegal; the arrangements he has put in place to deal with those availing of such services; when full-time services will be reinstated; and if he will make a statement on the matter. [17197/16]

21/06/2016WRU03400459. Deputy Pearse Doherty asked the Minister for Health if he is aware that there is currently no addiction counselling service available to persons under 18 years of age who are resident in County Donegal following the vacating of a counselling post; if he acknowledges that such an absence will have a detrimental impact on the health and well being of service us- ers in the county; the action he is taking to expedite the recruitment of a replacement counsellor in order to reinstate the service; the details of the provisional measures which he will introduce in order to ensure that minors in County Donegal may avail of addiction services during the interim period; and if he will make a statement on the matter. [17335/16]

21/06/2016WRU03500Minister of State at the Department of Health (Deputy Catherine Byrne): I propose to take Questions Nos. 436 and 459 together.

As this is a service matter, it has been referred to the Health Service Executive for attention and direct reply to the Deputies.

If the Deputies have not received a reply from the HSE within 15 working days they should contact my Private Office who will follow up the matter with them.

Hospital Appointments Delays

21/06/2016WRU03600437. Deputy Éamon Ó Cuív asked the Minister for Health when he will provide treatment for a person (details supplied) in County Galway; the reason for the delay; and if he will make a statement on the matter. [17198/16]

21/06/2016WRU03700Minister for Health (Deputy Simon Harris): Under the Health Act 2004, the Health Ser- vice Executive (HSE) is required to manage and deliver, or arrange to be delivered on its be- half, health and personal social services. Section 6 of the HSE Governance Act 2013 bars the Minister for Health from directing the HSE to provide a treatment or a personal service to any individual or to confer eligibility on any individual.

In relation to the specific case raised, I have asked the HSE to respond to you directly. If you have not received a reply from the HSE within 15 working days please contact my Private Office and my officials will follow the matter up.

Hospital Waiting Lists

204 21 June 2016

21/06/2016WRU03800438. Deputy Robert Troy asked the Minister for Health if he will schedule an operation date for a person (details supplied). [17199/16]

21/06/2016WRU03900Minister for Health (Deputy Simon Harris): Under the Health Act 2004, the Health Ser- vice Executive (HSE) is required to manage and deliver, or arrange to be delivered on its be- half, health and personal social services. Section 6 of the HSE Governance Act 2013 bars the Minister for Health from directing the HSE to provide a treatment or a personal service to any individual or to confer eligibility on any individual.

The National Waiting List Management Policy, A standardised approach to managing scheduled care treatment for in-patient, day case and planned procedures, January 2014, has been developed to ensure that all administrative, managerial and clinical staff follow an agreed national minimum standard for the management and administration of waiting lists for sched- uled care. This policy, which has been adopted by the HSE, sets out the processes that hospitals are to implement to manage waiting lists.

In relation to the particular query raised, as this is a service matter, I have asked the HSE to respond to you directly. If you have not received a reply from the HSE within 15 working days please contact my Private Office and my officials will follow the matter up.

Health Services Provision

21/06/2016WRU04000439. Deputy Michael Healy-Rae asked the Minister for Health the status of an application for an injury grant by a person (details supplied); and if he will make a statement on the matter. [17200/16]

21/06/2016WRU04100Minister for Health (Deputy Simon Harris): I have asked the HSE to respond to the Deputy directly on this matter. If you have not received a reply from the HSE within 15 work- ing days please contact my Private Office and they will follow up the matter with them.

Question No. 440 answered with Question No. 384.

21/06/2016WRV00300Primary Care Centres Expenditure

21/06/2016WRV00400441. Deputy Louise O’Reilly asked the Minister for Health further to Parliamentary Ques- tion No. 249 of 25 May 2016, when he will request the Health Service Executive to consider the most appropriate locations for further primary care centre developments, taking into account service needs and other relevant factors; if he will publish the criteria; where he has agreed primary care centres will be located; and if he will make a statement on the matter. [17211/16]

21/06/2016WRV00500Minister for Health (Deputy Simon Harris): The development of primary care is central to the Government’s objective to deliver a high quality, integrated and cost effective health care system. Supporting infrastructure, procured through a combination of public and private in- vestment, will facilitate the delivery of multi-disciplinary primary health care. The mechanism and timescale for delivery of primary care infrastructure is dependent on a number of factors. The additional primary care facilities across the country will be provided through a combina- tion of direct build, through Public Private Partnership (PPP) and by way of operational leasing.

Regardless of the delivery mechanism, all potential primary care infrastructure is subject to suitable locations being offered/provided/available, to successful planning processes and GP commitment to sharing accommodation and delivering health care services with HSE staff. In addition the operational lease mechanism is subject to market pressures such as the develop-

205 Questions - Written Answers ers’ access to adequate financing. Delivery of primary care infrastructure is a dynamic pro- cess, constantly evolving to take account of changing circumstances including the feasibility of implementation.

In addition to other locations being developed, in November 2014 the HSE advertised an additional 73 locations for delivery of Primary Care Centres by operational lease. The HSE has been reviewing all Expressions of Interest to determine which are suitable for more detailed consideration and progression to the next stage of the process.

Assessment/evaluation criteria employed by the HSE in respect of proposed locations for primary care centres fall under three principal headings: 1. Accommodation needs; 2. Service priority; and 3. deprivation level.

I intend to consider future infrastructure in the context of the implementation of the Health commitments in the Programme for Government and having regard to the funding available to facilitate such measures.

21/06/2016WRV00600Primary Care Centres Data

21/06/2016WRV00700442. Deputy Louise O’Reilly asked the Minister for Health the number and location of primary care centres, in tabular form; and if he will make a statement on the matter. [17212/16]

21/06/2016WRV00800Minister for Health (Deputy Simon Harris): As this is a service issue, this question has been referred to the Health Service Executive for direct reply to the Deputy. If you have not received a reply from the HSE within 15 working days, please contact my Private Office and they will follow up the matter with them.

21/06/2016WRV00900Primary Care Centres Data

21/06/2016WRV01000443. Deputy Louise O’Reilly asked the Minister for Health the services provided in each primary care centre by centre basis, in tabular form; and if he will make a statement on the mat- ter. [17213/16]

21/06/2016WRV01100Minister for Health (Deputy Simon Harris): As this is a service issue, this question has been referred to the Health Service Executive for direct reply to the Deputy. If the Deputy has not received a reply from the HSE within 15 working days, she should contact my Private Of- fice and they will follow up the matter with the HSE.

21/06/2016WRV01200Primary Care Centres Data

21/06/2016WRV01300444. Deputy Louise O’Reilly asked the Minister for Health the number of staff working in primary care centres in total, by occupation and by grade, in tabular form; and if he will make a statement on the matter. [17214/16]

21/06/2016WRV01400Minister for Health (Deputy Simon Harris): I have asked the HSE to respond to the Deputy directly on this matter. If the Deputy has not received a reply from the HSE within 15 working days she should contact my Private Office and they will follow up the matter with the HSE.

206 21 June 2016

21/06/2016WRV01500Mental Health Services Data

21/06/2016WRV01600445. Deputy Louise O’Reilly asked the Minister for Health the number of children and young persons waiting for appointments in the child and adolescent mental health services by Health Service Executive region, by those waiting for six, 12, 18, 24 and 24 plus months, in tabular form; and if he will make a statement on the matter. [17216/16]

21/06/2016WRV01700Minister of State at the Department of Health (Deputy Helen McEntee): As this is a service issue this question has been referred to the HSE for direct reply. If the Deputy has not received a reply within 15 working days, she should contact my Private Office and they will follow up the matter with the HSE.

21/06/2016WRV01800Services for People with Disabilities

21/06/2016WRV01900446. Deputy Louise O’Reilly asked the Minister for Health the waiting times for children’s disability services, by service, including physiotherapy, speech and language therapy and oc- cupational therapy, by Health Service Executive region, by those waiting for six, 12, 18, 24 and 24 plus months, in tabular form; and if he will make a statement on the matter. [17217/16]

21/06/2016WRV02000Minister of State at the Department of Health (Deputy Finian McGrath): The Govern- ment is committed to providing services and supports for people with disabilities which will empower them to live independent lives, provide greater independence in accessing the servic- es they choose, and enhance their ability to tailor the supports required to meet their needs and plan their lives. This commitment is outlined in the Programme for Partnership Government, which is guided by two principles: equality of opportunity and improving the quality of life for people with disabilities.

As the Deputy’s question relates to service matters, I have arranged for the question to be referred to the Health Service Executive for direct reply to the Deputy. If the Deputy has not received a reply from the HSE within 15 working days, she can contact my Private Office and they will follow up the matter with the HSE.

21/06/2016WRV02100Medical Card Data

21/06/2016WRV02200447. Deputy Louise O’Reilly asked the Minister for Health the number of medical card holders, by county, in tabular form; and if he will make a statement on the matter. [17218/16]

21/06/2016WRV02300Minister for Health (Deputy Simon Harris): The Health Service Executive has been asked to examine this matter and to reply to the Deputy as soon as possible.

The Health Service Executive operates the General Medical Services scheme, which in- cludes medical cards and GP visit cards, under the Health Act 1970, as amended. It has es- tablished a dedicated contact service for members of the Oireachtas specifically for queries relating to medical cards and GP visit cards, which the Deputy may wish to use for an earlier response. Contact information has recently issued to Oireachtas members.

If the Deputy has not received a reply from the HSE within 15 working days, she should contact my Private Office which will follow up the matter with the HSE.

21/06/2016WRV02400Medical Card Data

207 Questions - Written Answers

21/06/2016WRV02500448. Deputy Louise O’Reilly asked the Minister for Health the number of persons covered under the medical card scheme, the general practitioner visit card scheme and the discretionary medical card scheme, by Health Service Executive region, in tabular form; and if he will make a statement on the matter. [17219/16]

21/06/2016WRV02600Minister for Health (Deputy Simon Harris): The Health Service Executive has been asked to examine this matter and to reply to the Deputy as soon as possible.

The Health Service Executive operates the General Medical Services scheme, which in- cludes medical cards and GP visit cards, under the Health Act 1970, as amended. It has es- tablished a dedicated contact service for members of the Oireachtas specifically for queries relating to medical cards and GP visit cards, which the Deputy may wish to use for an earlier response. Contact information has recently issued to Oireachtas members.

If the Deputy has not received a reply from the HSE within 15 working days, she should contact my Private Office which will follow up the matter with the HSE.

21/06/2016WRV02700Dental Services Waiting Lists

21/06/2016WRV02800449. Deputy Louise O’Reilly asked the Minister for Health the status of the provision of primary school dental services by Health Service Executive region, with particular reference to the waiting times and availability of dentists visiting primary schools, in tabular form; and if he will make a statement on the matter. [17220/16]

21/06/2016WRV02900Minister for Health (Deputy Simon Harris): As this is a service matter, it has been re- ferred to the Health Service Executive for direct reply to the Deputy. If the Deputy has not received a reply from the HSE within 15 working days she should contact my Private Office and it will follow up the matter with the HSE.

Dental Services Waiting Lists

21/06/2016WRV03100450. Deputy Louise O’Reilly asked the Minister for Health the waiting lists for dental checks for children of six, nine and 12 years of age, by county, in tabular form; the areas where there is a delay in children receiving these checks; what he is doing to ensure they receive checks in a timely manner; and if he will make a statement on the matter. [17221/16]

21/06/2016WRV03200Minister for Health (Deputy Simon Harris): As this is a service matter, it has been re- ferred to the Health Service Executive for direct reply to the Deputy. If the Deputy has not received a reply from the HSE within 15 working days she should contact my Private Office and it will follow up the matter with the HSE.

21/06/2016WRV03300Long-Term Illness Scheme Coverage

21/06/2016WRV03400451. Deputy Michael McGrath asked the Minister for Health his plans to include a certain condition (details supplied) on the list of conditions qualifying under the long-term illness card scheme; and if he will make a statement on the matter. [17264/16]

21/06/2016WRV03500Minister for Health (Deputy Simon Harris): The Long Term Illness (LTI) Scheme was established under Section 59(3) of the Health Act, 1970 (as amended). Regulations were made in 1971, 1973 and 1975 specifying the conditions covered by the LTI Scheme, which are as fol-

208 21 June 2016 lows: acute leukaemia; mental handicap; cerebral palsy; mental illness (in a person under 16); cystic fibrosis; multiple sclerosis; diabetes insipidus; muscular dystrophies; diabetes mellitus; parkinsonism; epilepsy; phenylketonuria; haemophilia; spina bifida; hydrocephalus; and condi- tions arising from the use of Thalidomide. There are no plans to extend the list of conditions covered by the LTI Scheme.

Under the Drugs Payment Scheme, no individual or family pays more than €144 per cal- endar month towards the cost of approved prescribed medicines. The scheme significantly re- duces the cost burden for families and individuals incurring ongoing expenditure on medicines.

Under the provisions of the Health Acts, medical cards are provided to persons who are un- able, without undue hardship, to arrange health services for themselves and their dependants. In the assessment process, the HSE can take into account medical costs incurred by an indi- vidual or a family.

21/06/2016WRV03600Hospital Waiting Lists

21/06/2016WRV03700452. Deputy Aengus Ó Snodaigh asked the Minister for Health why a person (details supplied) has been in St. Vincent’s Hospital for eight months waiting for a bed in the National Rehabilitation Hospital; and when the person is likely to be admitted to the NRH. [17268/16]

21/06/2016WRV03800Minister of State at the Department of Health (Deputy Finian McGrath): The Govern- ment is committed to providing services and supports to people with disabilities which will em- power them to live independent lives, provide greater independence in accessing the services they choose and enhance their ability to tailor the supports required to meet their needs and plan their lives. This commitment is outlined in the Programme for Partnership Government, which is guided by two principles: equality of opportunity and improving the quality of life for people with disabilities.

As the Deputy’s question relates to services matters, I have arranged for the question to be referred to the Health Service Executive (HSE) for direct reply to the Deputy. If the Deputy has not received a reply from the HSE within 15 working days, he can contact my Private Office and they will follow the matter up with the HSE.

21/06/2016WRV03900Medical Aids and Appliances Applications

21/06/2016WRV04000453. Deputy Brendan Griffin asked the Minister for Health if a new battery for a power- chair will be provided to a person (details supplied). [17270/16]

21/06/2016WRV04100Minister for Health (Deputy Simon Harris): As the particular issue raised relates to an individual case, I have arranged for the question to be referred to the HSE for direct reply to the Deputy. If the Deputy has not received a reply from the HSE within 15 working days, please contact my Private Office and they will follow up the matter with the HSE.

21/06/2016WRV04200Medical Conditions

21/06/2016WRV04300454. Deputy Brendan Griffin asked the Minister for Health the number of cases of uniden- tified adolescent syndrome from 1980 to 2015, inclusive; and if he will make a statement on the matter. [17271/16]

209 Questions - Written Answers

21/06/2016WRV04400Minister for Health (Deputy Simon Harris): I understand that this question relates to My- algic Encephalomyelitis (ME), which is commonly referred to as Chronic Fatigue Syndrome (CFS). As there is no national ME register in Ireland, it is not possible to give accurate num- bers of cases. However, estimates suggest that ME may affect approximately 10,000-12,000 individuals in Ireland. This represents a prevalence rate of 0.2% - 0.4% which correlates to the reported prevalence rate in other European countries.

21/06/2016WRV04500Medical Conditions

21/06/2016WRV04600455. Deputy Louise O’Reilly asked the Minister for Health if funding is available for projects looking into the benefits of complementary and holistic therapies in the treatment of Alzheimer’s disease; and if he will make a statement on the matter. [17281/16]

21/06/2016WRV04700Minister of State at the Department of Health (Deputy Helen McEntee): As this is a ser- vice matter it has been referred to the Health Service Executive for direct reply. If the Deputy has not received a reply from the HSE within 15 working days she should contact my Private Office and they will follow up the matter with the HSE.

21/06/2016WRV04800Ministerial Meetings

21/06/2016WRV04900456. Deputy Louise O’Reilly asked the Minister for Health if he will meet with a group (details supplied); if he will provide details of the date and time of this meeting; and if he will make a statement on the matter. [17283/16]

21/06/2016WRV05000Minister for Health (Deputy Simon Harris): I am aware of claims of an association be- tween HPV vaccination and a number of conditions experienced by a group of young women.

It appears that some girls first suffered symptoms around the time they received the HPV vaccine, and understandably some parents have connected the vaccine to their daughter’s con- dition.

As you are aware in November 2015 the European Medicines Agency completed a detailed scientific review of the HPV vaccine which found no evidence of a link between the vaccine and the two conditions examined. On 12 January 2016 the European Commission endorsed the conclusion of the European Medicines Agency that there is no need to change the way HPV vaccines are used or to amend the product information.

However, this does not get away from the fact that these young women are unwell. I want to provide assurances that, as well as being eligible to seek medical attention and to access ap- propriate health and social care services, irrespective of the cause of their symptoms, the HSE will be in a position to facilitate access to specialist services as required due to the individual nature of the needs of some children.

As Minister I consider meeting any organisation that requests to do so. However due to busy parliamentary and other business it is not always possible to meet all representative groups. I have been informed that the HSE has met with members of REGRET concerning this issue and I will ask them for an update.

Hospital Waiting Lists

210 21 June 2016

21/06/2016WRV05200457. Deputy Pearse Doherty asked the Minister for Health when a person (details sup- plied) will receive an appointment for an operation; and if he will make a statement on the matter. [17308/16]

21/06/2016WRV05300Minister for Health (Deputy Simon Harris): Under the Health Act 2004, the Health Ser- vice Executive (HSE) is required to manage and deliver, or arrange to be delivered on its be- half, health and personal social services. Section 6 of the HSE Governance Act 2013 bars the Minister for Health from directing the HSE to provide a treatment or a personal service to any individual or to confer eligibility on any individual.

The National Waiting List Management Policy, A standardised approach to managing scheduled care treatment for in-patient, day case and planned procedures, January 2014, has been developed to ensure that all administrative, managerial and clinical staff follow an agreed national minimum standard for the management and administration of waiting lists for sched- uled care. This policy, which has been adopted by the HSE, sets out the processes that hospitals are to implement to manage waiting lists.

In relation to the particular query raised, as this is a service matter, I have asked the HSE to respond to the Deputy directly. If the Deputy has not received a reply from the HSE within 15 working days he should contact my Private Office and my officials will follow the matter up.

21/06/2016WRV05400Home Help Service Provision

21/06/2016WRV05500458. Deputy Brendan Griffin asked the Minister for Health if he will provide adequate home help and a home care package to a person (details supplied). [17314/16]

21/06/2016WRV05600Minister of State at the Department of Health (Deputy Helen McEntee): As this is a ser- vice matter it has been referred to the Health Service Executive for direct reply. If the Deputy has not received a reply from the HSE within 15 working days he should contact my Private Office and they will follow up the matter with the HSE.

Question No. 459 answered with Question No. 436.

21/06/2016WRV05800Food Harvest 2020 Strategy

21/06/2016WRV05900460. Deputy Mattie McGrath asked the Minister for Agriculture, Food and the Marine his role in the implementation of Food Harvest 2020; and if he will make a statement on the matter. [16970/16]

21/06/2016WRV06000Minister for Agriculture, Food and the Marine (Deputy Michael Creed): Food Wise 2025, the new ten year strategy for the agrifood sector published in July last year, replaced Food Harvest 2020. It identifies the opportunities and challenges facing the sector and provides an enabling strategy that will allow the sector to grow and prosper. Food Wise includes more than 400 specific recommendations, spread across the cross-cutting themes of sustainability, in- novation, human capital, market development and competitiveness, as well as specific sectoral recommendations.

The implementation process for any strategy is vital for its success. I chair the Food Wise High Level Implementation Committee (HLIC), with high level representatives from all the relevant Departments and State agencies. The committee reviews progress on detailed actions on a quarterly basis, in order to identify and solve problems quickly. Stakeholders regularly present to the committee meetings on their priorities for particular sectors or themes and by the 211 Questions - Written Answers end of this year, the HLIC will have reviewed in detail progress on the five cross-cutting themes and the twelve individual sectors outlined in Food Wise 2025. So it is very much a live and continuously updated process.

I will be publishing a first year progress report on Food Wise 2025 next month.

21/06/2016WRW00150Agriculture Scheme Payments

21/06/2016WRW00200461. Deputy Brendan Griffin asked the Minister for Agriculture, Food and the Marine when he will issue payment for the Scottish derogation to a person (details supplied); and if he will make a statement on the matter. [16736/16]

21/06/2016WRW00300Minister for Agriculture, Food and the Marine (Deputy Michael Creed): My Depart- ment received a Scottish Derogation application from the persons named on 14th May 2015.

In order to qualify for ‘Scottish Derogation’ applicants must submit verifiable evidence that they were engaged in agricultural production in 2013. An official from my Department has been in contact with the second named person to request this documentary evidence. On receipt of this information the application will be processed and the person named will be notified of the outcome of his application shortly thereafter.

21/06/2016WRW00350GLAS Applications

21/06/2016WRW00400462. Deputy Michael Healy-Rae asked the Minister for Agriculture, Food and the Marine the status of a payment to a person (details supplied) under the green low-carbon agri-environ- ment scheme; and if he will make a statement on the matter. [16738/16]

21/06/2016WRW00500Minister for Agriculture, Food and the Marine (Deputy Michael Creed): The Sheep Technology Adoption Programme was a three year programme for 2013, 2014 and 2015. The person named submitted a valid application and was subsequently paid in full for 2013 and 2014. In the absence of confirmation that the person named had completed all required tasks for 2015, payment was not authorised. The required confirmation has since been received and payment issued to the above named person on 19th April, 2016.

The person named has been approved under Tranche 1 of GLAS with a contract start date of 1 October 2015. GLAS applicants with a start date of 1 October 2015 are entitled to a 2015 part-year payment in respect of 2015 eligible actions, covering the three month period 1 Octo- ber 2015 - 31 December 2015.

My Department received a request to withdraw a parcel approved for the Low Input Perma- nent Pasture action from the approved GLAS application and this has delayed the processing of the payment. In light of the changed situation the application must be re-assessed and payment will issue when this re-assessment is complete.

21/06/2016WRW00550Agri-Environment Options Scheme Payments

21/06/2016WRW00600463. Deputy Michael Healy-Rae asked the Minister for Agriculture, Food and the Ma- rine the status of a payment to a person (details supplied) under the agri-environment options scheme; and if he will make a statement on the matter. [16741/16]

212 21 June 2016

21/06/2016WRW00700Minister for Agriculture, Food and the Marine (Deputy Michael Creed): My Depart- ment has no record of receiving an AEOS application for the person named.

An application under Tranche 1 of GLAS was received in my Department on the 20 May 2015 and the person named was approved into the GLAS 1 with effect from 1 October 2015. There are a number of applications, including that of the person named, remaining which have not passed all of the Department’s pre-payment validation checks for the 2015 part-year pay- ment. The Department is continuing to investigate these issues including the correction of any IT or data issues with a view to all these remaining cases meeting the payment validation checks so that they will be paid the full amount of their 2015 part-year payment at the same time the 2015 GLAS balancing payment is due to issue.

21/06/2016WRW00750Agri-Environment Options Scheme Payments

21/06/2016WRW00800464. Deputy Michael Healy-Rae asked the Minister for Agriculture, Food and the Marine the status of a payment to a person (details supplied) under the green low-carbon agri-environ- ment scheme; and if he will make a statement on the matter. [16742/16]

21/06/2016WRW00900Minister for Agriculture, Food and the Marine (Deputy Michael Creed): An applica- tion under Tranche 1 of GLAS was received in my Department on the 21 May 2015 and the per- son named was approved into the GLAS 1 with effect from 1 October 2015. There are a number of applications, including that of the person named, remaining which have not passed all of the Department’s pre-payment validation checks for the 2015 part-year payment. The Department is continuing to investigate these issues including the correction of any IT or data issues with a view to all these remaining cases meeting the payment validation checks so that they will be paid the full amount of their 2015 part-year payment at the same time the 2015 GLAS balancing payment is due to issue.

TAMS Applications

21/06/2016WRW01000465. Deputy Michael Healy-Rae asked the Minister for Agriculture, Food and the Marine the status of a payment to a person (details supplied) under the targeted agricultural modernisa- tion scheme 2; and if he will make a statement on the matter. [16743/16]

21/06/2016WRW01100Minister for Agriculture, Food and the Marine (Deputy Michael Creed): The applicant in question has submitted three separate applications to the TAMS II Scheme.

Approval issued on the 15th of June 2016 in relation to his application for a bulk tank under the Dairy Equipment Scheme. A query issued to the applicant on the 13th of June in relation to his application for a feed system and meal bin and when responded to the matter will be pro- cessed further. The application for Animal Housing and Tank under the Animal Welfare, Safety and Nutrient Storage Scheme has passed the administrative checks and will be sent to the local office in a matter of days for final technical checks.

21/06/2016WRW01150Agriculture Scheme Appeals

21/06/2016WRW01200466. Deputy Michael Healy-Rae asked the Minister for Agriculture, Food and the Marine the status of an appeal by a person (details supplied); and if he will make a statement on the matter. [16744/16]

213 Questions - Written Answers

21/06/2016WRW01300Minister for Agriculture, Food and the Marine (Deputy Michael Creed): The person named submitted an application under the ‘Old Young Farmer’ category of the 2015 National Reserve to my Department. This application was not successful as the applicant was deemed to have commenced farming prior to 2008. On this basis the person named did not meet the eligibility criteria under the National Reserve ‘Old Young Farmer’ category. The person named was notified of this decision in writing and was offered the opportunity to submit an appeal.

My Department received an appeal from the person named in relation to his National Re- serve application. This appeal has been considered by my Department and a letter issued to the applicant on 16th June 2016 informing him of a successful outcome to the appeal. Any outstanding monies due on foot of this decision will issue shortly.

Agri-Environment Options Scheme Payments

21/06/2016WRW01400467. Deputy Michael Healy-Rae asked the Minister for Agriculture, Food and the Marine the status of a payment to a person (details supplied) under the farm payment scheme; and if he will make a statement on the matter. [16745/16]

21/06/2016WRW01500Minister for Agriculture, Food and the Marine (Deputy Michael Creed): The person named commenced their AEOS contract on 1 September 2011. Payment issued in respect of the 2011-2013 Scheme years. Issues arose which resulted in a delay in the processing of the 2014 Scheme year payment. These issues have now been resolved and payment in respect of the 2014 scheme year issued on 2nd June 2016. It is expected that payment in respect of the 2015 Scheme year will issue shortly.

21/06/2016WRW01550GLAS Applications

21/06/2016WRW01600468. Deputy Michael Healy-Rae asked the Minister for Agriculture, Food and the Marine the status of an application by a person (details supplied) under the green low-carbon agri- environment scheme; and if he will make a statement on the matter. [16746/16]

21/06/2016WRW01700Minister for Agriculture, Food and the Marine (Deputy Michael Creed): An applica- tion under Tranche 2 of GLAS was received in my Department on the 8 December 2015 and the person named was approved into the GLAS 2 with effect from 1 January 2016. The first pay- ment due to the person named is in respect of the 2016 scheme year and this will be processed in the final quarter of the year.

An application was also received from the person named under the GLAS Traditional Farm Building Scheme, which supports the conservation of traditional farm buildings on GLAS- approved farms. This scheme is jointly funded by my Department and the European Union, and is administered by the Heritage Council on behalf of my Department.

I am delighted to say that over 500 applications were received for the Scheme under the first tranche and the Heritage Council is currently validating and evaluating all of these, rank- ing each application in accordance with the selection criteria published as part of the Scheme documentation. When that process is complete, they will notify all applicants of the outcome of their own application in due course. The Scheme remains open for the whole RDP period and anybody unsuccessful in one tranche is perfectly free to apply again under the next.

21/06/2016WRW01750Agriculture Scheme Applications 214 21 June 2016

21/06/2016WRW01800469. Deputy Michael Healy-Rae asked the Minister for Agriculture, Food and the Ma- rine the status of an application by a person (details supplied) under the Scottish derrogation scheme; and if he will make a statement on the matter. [16747/16]

21/06/2016WRW01900Minister for Agriculture, Food and the Marine (Deputy Michael Creed): My Depart- ment received a Scottish Derogation application from the person named on 8th May 2015.

In order to qualify for ‘Scottish Derogation’ applicants must submit verifiable evidence that they were engaged in agricultural production in 2013. No such evidence was included with the application from the person named and an official from my Department has been in contact with the person named to request this information.

On receipt of this information the application will be processed further and the person named will be notified of the outcome of his application shortly thereafter.

21/06/2016WRW01950TAMS Applications

21/06/2016WRW02000470. Deputy Michael Healy-Rae asked the Minister for Agriculture, Food and the Marine the status of an application by a person (details supplied) under the targeted agricultural mod- ernisation scheme 2; and if he will make a statement on the matter. [16748/16]

21/06/2016WRW02100Minister for Agriculture, Food and the Marine (Deputy Michael Creed): The applicant in this case applied under the Young Farmers Capital Investment Scheme of TAMS II. Ap- proval issued on the 14th of June.

21/06/2016WRW02150Basic Payment Scheme Applications

21/06/2016WRW02200471. Deputy Michael Healy-Rae asked the Minister for Agriculture, Food and the Marine the status of an appeal by a person (details supplied); and if he will make a statement on the matter. [16749/16]

21/06/2016WRW02300Minister for Agriculture, Food and the Marine (Deputy Michael Creed): The person named submitted a 2015 Basic Payment/Areas of Natural Constraint Scheme application on 29 April 2015. EU Regulations governing the administration of these schemes require that full and comprehensive administrative checks, including in some cases on-farm inspections, be com- pleted before any payments issue.

The application of the person named was selected for a Ground Eligibility inspection. The inspection identified discrepancies between the areas declared and the area found resulting in an over-declaration in area of greater than 50%. Based on the terms and conditions of the schemes this resulted in no payments being due under the 2015 Direct Payments Scheme and the appli- cation of an administrative fine to be offset against any future EU payments.

The person named was notified of the inspection outcome on 16 November 2015 and ap- pealed this decision to the District Inspector. The outcome of this appeal was to uphold the inspection findings.

The person named has appealed to the independent Agriculture Appeals Office (AAO) and that process is currently ongoing. The person named will be directly notified in writing of the outcome of this appeal by the AAO. At the end of this process if the person named feels that they have been treated unfairly they can raise the matter with the Office of the Ombudsman.

215 Questions - Written Answers

21/06/2016WRW02350TAMS Administration

21/06/2016WRW02400472. Deputy Charlie McConalogue asked the Minister for Agriculture, Food and the Ma- rine the lead time for inspections for applications for new shed grants under the targeted ag- ricultural modernisation scheme; the measures he will take to ensure that inspections happen promptly so that construction works can be completed before winter; and if he will make a statement on the matter. [16750/16]

21/06/2016WRW02500Minister for Agriculture, Food and the Marine (Deputy Michael Creed): Inspec- tions will be carried out on a small percentage of the applications received under the TAMS II schemes. My Department is ensuring that all inspections required will be carried out as speedily as possible.

21/06/2016WRW02550Single Payment Scheme Payments

21/06/2016WRW02600473. Deputy Charlie McConalogue asked the Minister for Agriculture, Food and the Ma- rine when he will make a payment to a person (details supplied) under the single farm payment scheme; and if he will make a statement on the matter. [16777/16]

21/06/2016WRW02700Minister for Agriculture, Food and the Marine (Deputy Michael Creed): The person named submitted an application for the 2013 Single Payment Scheme on 6th May 2013. Fol- lowing the identification of an overclaim found as part of the Land Parcel Identification System review reduced payments issued to the nominated bank account of the person named in October and December 2013, respectively.

This decision was successfully appealed by the person named and additional payments due on foot of this decision will shortly issue to the nominated bank account of the person named.

21/06/2016WRW02750Basic Payment Scheme Eligibility

21/06/2016WRW02800474. Deputy Martin Ferris asked the Minister for Agriculture, Food and the Marine if he is in the process of revising the regulations surrounding land eligibility under the basic payment scheme. [16810/16]

21/06/2016WRW02900Minister for Agriculture, Food and the Marine (Deputy Michael Creed): EU regula- tions governing the Direct Payment Schemes and Rural Development Programme Measures in relation to all aspects of the Basic Payment Scheme including land eligibility are compiled by the European Council, Parliament and Commission, in conjunction with all Member States. Any revision to these regulations is not a matter for an individual Member State.

My Department and I have been and will continue to be fully involved in the process for the compilation and revision of EU regulations.

21/06/2016WRW02950Basic Payment Scheme Administration

21/06/2016WRW03000475. Deputy Martin Ferris asked the Minister for Agriculture, Food and the Marine if there are internal discussions ongoing regarding recategorising certain types of land which were deemed to be scrub in the past. [16811/16]

21/06/2016WRW03100Minister for Agriculture, Food and the Marine (Deputy Michael Creed): Under the 216 21 June 2016 provisions of the Basic Payment Scheme, farmers need to declare an eligible hectare for each payment entitlement held in order to benefit from payment. The hectares declared must be eli- gible and must have an agricultural activity carried out on them.

The governing EU regulations define an agricultural activity as the production, rearing or growing of agricultural products, including harvesting, milking, breeding animals, and keeping animals for farming purposes and maintaining an agricultural area in a state which makes it suitable for grazing or cultivation. An applicant is free to choose an activity that best suits their farming enterprise and if s/he so wishes can choose to just top the land.

Where an officially approved environmental management plan under the Birds or Habitats Directives requires certain management practices to be conducted on agricultural areas, such management practices will take precedence in determining the level of agricultural activity to be conducted on agricultural areas declared for payment.

Article 32(2)(b)(i) of Regulation (EU) 1307/2013 provides that an eligible hectare is any area which gave a right to payments in 2008 under the single payment scheme and which no longer complies with the definition of ‘eligible hectare’ as a result of the implementation of Directive 92/43/EEC, Directive 2000/60/EC and Directive 2009/147/EC.

In that regard, the 2016 BPS Terms and Conditions set out details with regard to land eligi- bility and payments. Section 1.5 refers to land eligibility and states that in order to draw down payment in respect of BPS entitlements, applicants must have an “eligible hectare” to accom- pany each entitlement. An eligible hectare is land that is used for an agricultural activity.

Section 1.5 of the Terms and Conditions further makes reference to The Birds and Habitats Directives and provides that land that no longer complies with the definition of eligible hectare as a result of the implementation of Council Directive 79/409/EEC on the conservation of wild birds or Council Directive 92/43/EEC on the conservation of natural habitats and of wild fauna and flora is eligible for payment where the following conditions are met:

(i) The land must have been claimed under the 2008 Single Payment Scheme;

(ii) The land must have been eligible for payment under the 2008 Single Payment Scheme;

(iii) Any increase in the ineligible area should be directly linked to the management require- ments for the habitat.

Where an applicant is claiming on land that is otherwise considered ineligible, for example scrub, such land will be deemed eligible for payment where the applicant satisfies the require- ments as set out above.

21/06/2016WRW03150Aquaculture Licence Applications

21/06/2016WRW03200476. Deputy Clare Daly asked the Minister for Agriculture, Food and the Marine further to Parliamentary Question No. 183 of 14 April 2016, if he will appoint an independent person from outside his Department to undertake the review of his Department’s aquaculture licensing system in order that non-industry stakeholders and the general public can have confidence that the review is balanced and genuine. [16824/16]

21/06/2016WRW03300Minister for Agriculture, Food and the Marine (Deputy Michael Creed): Ireland’s first National Strategic Plan for Sustainable Aquaculture Development was published by my De- partment in December 2015. That Plan proposes 24 actions to drive the sustainable develop-

217 Questions - Written Answers ment of the aquaculture sector and sustainably grow production in the sector by 45,000 tonnes.

Action number 22 of the National Strategic Plan for Sustainable Aquaculture Development commits my Department to a review and revision of the aquaculture licence process, including the applicable legal framework. This Review is echoed by the recommendations of FoodWise 2025.

Aquaculture licences are determined by my Department having regard to the requirements of the Fisheries (Amendment) Act 1997, the Foreshore Acts 1993-2011, the EU Habitats and Birds Directives, the Aarhus Convention, requirements relating to underwater archaeology and certain other regulatory requirements.

Thus, the process around aquaculture licensing is extremely complex and demanding. Nev- ertheless, since 2012 my Department has made 471 aquaculture licence determinations. Some 53 licence determinations have been made so far this year and in the order of 150 - 200 licence determinations are expected for the year as a whole.

The National Strategic Plan for Sustainable Aquaculture Development recognised the com- plexity of the present process and related legal framework. It committed to a review of the licensing process, including the legal framework. I am presently considering this commitment and the many other actions proposed in the National Strategic Plan for Sustainable Aquaculture Development and expect to progress the issue in 2016.

21/06/2016WRW03350Aquaculture Licences

21/06/2016WRW03400477. Deputy Clare Daly asked the Minister for Agriculture, Food and the Marine further to Parliamentary Questions Nos. 97 of 27 January 2016 and 181 of 14 April 2016, when he will complete the examination of overstocking at the sites; if he will publish the findings; and if he will ensure that the requirement to comply with European Union environmental legislation on aquaculture will not be ignored by his Department in the handling of these cases. [16825/16]

21/06/2016WRW03500Minister for Agriculture, Food and the Marine (Deputy Michael Creed): As the Deputy has been advised, my Department is currently examining possible overstocking in respect of three sites.

Every effort is being made to complete the process, having regard to the technical and leg- islative complexities involved.

My Department administers the licensing of aquaculture in accordance with all applicable national and EU legislation.

As the examination by my Department of possible overstocking remains ongoing as part of a statutory process it would not be appropriate for me to comment further at this time.

21/06/2016WRW03550Aquaculture Licences

21/06/2016WRW03600478. Deputy Clare Daly asked the Minister for Agriculture, Food and the Marine if he will suspend the granting of aquaculture licences until the proposed review of his Department’s aquaculture licensing regime, as promised in Food Wise 2025, is completed. [16826/16]

21/06/2016WRW03700Minister for Agriculture, Food and the Marine (Deputy Michael Creed): Applications for aquaculture licences are considered by my Department in accordance with the 1997 Fisher-

218 21 June 2016 ies (Amendment) Act, the 1933 Foreshore Act and applicable EU legislation.

The legislation does not provide for the suspension of the licence assessment process.

21/06/2016WRW03750Milk Supply

21/06/2016WRW03800479. Deputy Carol Nolan asked the Minister for Agriculture, Food and the Marine his plans to put in place a scheme under delegated regulation C(2016)2122 via the urgency procedure au- thorising agreements and decisions of co-operatives and other forms of producer organisations in the milk and milk products sector on the planning of production; his further plans for any alternatives to alleviate the impact of the dairy crisis on Irish farmers, given that the European Commissioner has stated that he does not have the tools to suspend the superlevy; if he will provide a breakdown of the allocation of the €13.7 million under the aid package for European Union farmers since 2015; the amount of this that has been drawn down; the timeframe left to allocate moneys from the package; the sectors that have benefited; his plans to allocate any re- maining moneys under the package; and if he will make a statement on the matter. [16914/16]

21/06/2016WRW03900Minister for Agriculture, Food and the Marine (Deputy Michael Creed): The proposal referred to by the Deputy permits co-operatives or producer organisations in the dairy sector to engage in concerted voluntary supply control among their members. Ireland has supported these measures provided they are voluntary, temporary and financed only at individual Member State level. I do not believe that there is demand from the co-operative sector in Ireland for a supply control measure.

The EU targeted aid package referred to by the Deputy was matched by Exchequer funding, giving a total amount of €27.4m. Some €27m has been paid to date, with a small number of outstanding cases still being examined. In total some 17,600 dairy farmers received a payment of €1,395, with c. 1,500 of these getting an additional €1,000 under a Young Farmer top up. Just under 300 pig producers have also received c.€3,300 each and my Department is consider- ing allocating some funding under this package to the Irish Pig Society to further their work in representing smaller scale producers. Payments must be made by 30th June 2016.

A number of other market support measures have been put in place at EU level, including an increase in the ceiling for SMP intervention to 350,000 tonnes, which was sought by Ireland. In terms of input costs at farm level, I have called on the Commission to consider looking at temporary suspension of EU import tariffs on fertilisers to reduce input costs for Irish and EU farmers. I have also called for a further targeted aid package for farmers and for a facility to advance 75% of the Basic Payment Scheme to farmers in 2016 to assist with cash flow.

At a National level, there have been a number of imaginative initiatives at co-op level, in- cluding fixed price and margin contracts, and flexible financing instruments. I would like to see more of this from industry. Developing new and existing markets is also part of the solution. In this regard I intend to lead a trade mission to the Far East in the early Autumn, and dairy will be a significant feature. Work is also continuing through Teagasc, ICBF and AHI to increase farm efficiency, reduce the costs of production and increase margins. Making farmers more resilient to price volatility is key.

I have also been talking to the main banks to exercise forbearance with farmers temporarily under financial pressure, and a number of other options to improve the availability of liquidity are being explored. At today’s dairy forum I launched a new cash flow tool which will assist farmers in managing their finances and help them in engaging with banks.

I am acutely conscious of the pressure farmers are under at present, and I can assure the 219 Questions - Written Answers Deputy that I am committed to exploring all of the appropriate options for providing assistance to the farm sector through this temporary price trough. I met the dairy forum today and dis- cussed these issues with the key stakeholders.

21/06/2016WRW03950Agri-Environment Options Scheme Payments

21/06/2016WRW04000480. Deputy Michael Ring asked the Minister for Agriculture, Food and the Marine when a person (details supplied) will receive a final year payment under the agri-environment options scheme. [16958/16]

21/06/2016WRW04100Minister for Agriculture, Food and the Marine (Deputy Michael Creed): The person named commenced their AEOS contract on 1st September 2010. Payment issued in respect of the 2010-2014 Scheme years.

Under the EU Regulations governing the Scheme and other area-based payment schemes, a comprehensive administrative check, including cross-checks with the Land Parcel Identifica- tion System must take place. As all AEOS 1 participants will be receiving their final payments under the scheme, re-checks on payments made for all scheme years must be completed before final payment can be processed. Officials are working through these files and payment will be made in all cases as soon as possible.

21/06/2016WRW04150Basic Payment Scheme Payments

21/06/2016WRW04200481. Deputy Brendan Griffin asked the Minister for Agriculture, Food and the Marine when he will issue a payment for 2015 to a person (details supplied) under the basic payment scheme; and if he will make a statement on the matter. [16980/16]

21/06/2016WRW04300Minister for Agriculture, Food and the Marine (Deputy Michael Creed): The person named submitted an application for the 2015 Basic Payment Scheme on 9 th May 2015. An issue arose in relation to the eligibility of one parcel of land. An official from my department has been in direct contact with the person named and the matter has now been satisfactorily resolved. All outstanding payments will shortly issue to the nominated bank account of the person named.

21/06/2016WRX00200Greyhound Industry

21/06/2016WRX00300482. Deputy Mick Wallace asked the Minister for Agriculture, Food and the Marine when he will publish the Morris report on greyhound racing; and if he will make a statement on the matter. [16990/16]

21/06/2016WRX00400Minister for Agriculture, Food and the Marine (Deputy Michael Creed): The Morris Report was commissioned by Bord na gCon and its publication is a matter for that organisation. I understand that Bord na gCon expects to publish the report in the coming weeks.

21/06/2016WRX00500Afforestation Programme

21/06/2016WRX00600483. Deputy Billy Kelleher asked the Minister for Agriculture, Food and the Marine if he will immediately review the case of a person (details supplied); and if he will make a statement

220 21 June 2016 on the matter. [17022/16]

21/06/2016WRX00700Minister for Agriculture, Food and the Marine (Deputy Michael Creed): The person in question planted 3.79 hectares of forestry in March 1998 under the Department’s Afforestation Grant and Premium Scheme.

In November 2015, a Department Forestry Inspector carried out an inspection and found that part of the forest is very under-stocked, with few trees (less than 2-3% stocking). The area involved amounts to approximately 1.30 hectares. The Department wrote to the applicant on 10th February 2016 informing her of the remedial works required to comply with Forest Ser- vice standards under the Scheme. Pending completion of the remedial works, the applicant’s forestry premiums were suspended with effect from 2016.

In light of the information provided by the Deputy, I have asked my officials to carry out a review of this case. I will let the Deputy know the outcome of the review in due course.

21/06/2016WRX00800Agri-Environment Options Scheme Conditions

21/06/2016WRX00900484. Deputy Martin Kenny asked the Minister for Agriculture, Food and the Marine if he is aware that contracts are being terminated where soil sample analyses are late even by a few days, even though soil sample analysis can take ten days to complete and weather conditions can cause delay under the agri-environment options scheme. [17024/16]

21/06/2016WRX01000Minister for Agriculture, Food and the Marine (Deputy Michael Creed): AEOS is a voluntary scheme and to remain in the scheme all participants must abide by the terms of their contracts.

Soil Samples are an integral part of AEOS contracts. The Terms and Conditions state that in the case of AEOS 2 applicants whose contracts commenced in 2011, soil samples must be taken between 1st January 2010 and 31st March 2012. This was subsequently extended to 31st March 2013. AEOS 3 applicants whose contracts commenced in 2013 must have taken soil samples between 1st January 2011 and 31st March 2014.

Applicants were then required to submit a declaration to the Department stating that the soil samples were taken within the permitted time frame and in accordance with the Terms and Conditions of the scheme before payment in respect of the first full contract year would be processed.

Termination of contracts have occurred where either the soil samples were not taken, or where a declaration stating that the soil samples were taken in accordance with the Terms and Conditions of the scheme was submitted, but subsequently found to be untrue.

There is an appeals process in place which can be accessed by any person who feels that they have been unfairly treated by my Department.

21/06/2016WRX01100Agri-Environment Options Scheme Conditions

21/06/2016WRX01200485. Deputy Martin Kenny asked the Minister for Agriculture, Food and the Marine if he will apply a period of grace on the deadline for submission of soil analyses under the agri- environment options scheme or under the basic payment scheme. [17025/16]

21/06/2016WRX01300Minister for Agriculture, Food and the Marine (Deputy Michael Creed): AEOS is a

221 Questions - Written Answers voluntary scheme and to remain in the scheme all participants must abide by the terms of their contract.

Soil samples are an integral part of AEOS contracts. The Terms and Conditions state that in the case of AEOS 2 applicants whose contracts commenced in 2011, soil samples must be taken between 1st January 2010 and 31st March 2012. This was subsequently extended to 31st March 2013. AEOS 3 applicants whose contracts commenced in 2013 must have taken soil samples between 1st January 2011 and 31st March 2014.

I do not envisage making any further changes to the timelines set out above. There is no requirement to undertake soil analysis under the Basic Payments Scheme.

21/06/2016WRX01400Agriculture Scheme Payments

21/06/2016WRX01500486. Deputy Kevin O’Keeffe asked the Minister for Agriculture, Food and the Marine when he will issue payment under a specific scheme for 2014 and 2015 to a person (details sup- plied). [17042/16]

21/06/2016WRX01600Minister for Agriculture, Food and the Marine (Deputy Michael Creed): While the person named was issued with an application for 2014 BVD Compensation Programme on 17th November 2014 there is no record of the completed application having being returned to my Department. The Terms and Conditions of the programme advised applicants to obtain proof of postage in the form of a Swift post or a Registered post receipt when submitting their appli- cations. As no proof of postage has been submitted payment under 2014 BVD Compensation Programme cannot be made.

Payment in respect of the 2015 BVD Compensation Programme has been processed and payment is expected to the nominated bank account of the person named shortly.

21/06/2016WRX01700Beef Data Programme

21/06/2016WRX01800487. Deputy Kevin O’Keeffe asked the Minister for Agriculture, Food and the Marine when he will issue payment under a specific scheme to a person (details supplied). [17044/16]

21/06/2016WRX01900Minister for Agriculture, Food and the Marine (Deputy Michael Creed): The person named submitted an application under the Beef Data and Genomics Programme (BDGP) 2015 – 2020 on the 5th June 2015.

As the person named has not submitted sufficient survey data 2015 payment has not yet is- sued. Upon receipt of the outstanding survey data outstanding payment will issue to the person named. An official from my Department has been in contact with the person named to explain the position.

21/06/2016WRX02000TAMS Applications

21/06/2016WRX02100488. Deputy Brendan Griffin asked the Minister for Agriculture, Food and the Marine if he will give full approval under TAMS II for a person (details supplied); and if he will make a statement on the matter. [17066/16]

21/06/2016WRX02200Minister for Agriculture, Food and the Marine (Deputy Michael Creed): The applicant

222 21 June 2016 in question applied under the Dairy Equipment Scheme of TAMS II. Part approval issued to the applicant on the 17th December 2015 to allow him to go ahead with urgently required equip- ment. A full approval issued to the applicant on the 13th June 2016.

21/06/2016WRX02300Areas of Natural Constraint Scheme Payments

21/06/2016WRX02400489. Deputy Pat Breen asked the Minister for Agriculture, Food and the Marine further to Parliamentary Question No. 203 of 14 April 2016, when he will issue payment to a person (details supplied); and if he will make a statement on the matter. [17067/16]

21/06/2016WRX02500Minister for Agriculture, Food and the Marine (Deputy Michael Creed): Under EU Regulatory requirements, only farmers who were entitled to receive a direct payment in 2013 (Single Payment, Grassland Sheep Scheme, Burren Life Scheme, Beef Data Scheme) have an automatic allocation right under the Basic Payment Scheme in 2015. The European Commis- sion has further stated that applications submitted 25 days after the Single Payment Scheme closing date are deemed inadmissible unless there are force majeure or extenuating circum- stances pertaining to the late submission of this application. As the person named submitted his 2013 Single Payment Scheme application form on 28th June 2013, this application is deemed inadmissible and therefore he does not qualify for an automatic allocation right under the new Basic Payment Scheme.

The person named was previously advised that where extenuating circumstances had result- ed in the late submission of the 2013 Single Payment application he should submit a Review of Entitlements application form to my Department outlining the extenuating circumstances per- taining to his case. No such request for Review has been received by my Department to date.

21/06/2016WRX02600Agriculture Scheme Eligibility

21/06/2016WRX02700490. Deputy James Browne asked the Minister for Agriculture, Food and the Marine the position regarding the land designation assigned to agricultural lands that have a commissioned solar farm installation under the land parcel identification system, and the pro rata availability of land, with a commissioned solar farm, under the basic payment system-greening payment and other area aid schemes; and if he will make a statement on the matter. [17072/16]

21/06/2016WRX02800Minister for Agriculture, Food and the Marine (Deputy Michael Creed): Support under the Basic Payment Scheme (BPS) is payable upon activation of an eligible hectare per payment entitlement. In general terms for both BPS and Greening and other area based schemes an eli- gible hectare is one that is used for an agricultural activity or, where the area is also used for a non-agricultural activity, is predominantly used for such activities. An area is predominantly used for agricultural activity if that activity can be exercised without being significantly ham- pered by the intensity, nature, duration and timing of the non-agricultural activity.

While cases involving solar panels will be examined on an individual basis, the area covered by the solar panels will be deemed ineligible for the purposes of claiming BPS, in line with advice from the EU Commission. Furthermore, in line with the Department’s current approach on land eligibility, where the area of a parcel covered by solar panels is 70% or greater of the overall parcel, that parcel will be wholly ineligible. If less than 70% is covered by solar panels and the agricultural activity is not hampered by the presence of the solar panels, the area not covered by solar panels may be eligible.

Finally it is important to note that all agricultural land used to claim BPS must be owned/ 223 Questions - Written Answers leased, used and managed by the applicant. If the applicant is not the owner of the land, they must have a lease or rental agreement in place with the owner of the land. There must be inde- pendent and suitable access for animals and/or machinery for the farming enterprise being con- ducted. Independent access means access by public or private roadway or by a defined right of way. Access over adjoining landowners land, or over land which is subject to a lease or rental agreement to another person, is not acceptable.

Basic Payment Scheme Eligibility

21/06/2016WRX03000491. Deputy Michael Healy-Rae asked the Minister for Agriculture, Food and the Marine if he will examine the case of a person (details supplied); and if he will make a statement on the matter. [17189/16]

21/06/2016WRX03100Minister for Agriculture, Food and the Marine (Deputy Michael Creed): Under EU Regulations governing the Basic Payment Scheme, entitlements may be allocated to farmers from the National Reserve under certain circumstances as defined by the Regulations. Priority access must be given to applicants who meet the definition of ‘young farmer’ or ‘new entrant to farming’. A young farmer is defined as a farmer aged no more than 40 years of age in the year when s/he first submits an application under the Basic Payment Scheme and who commenced their farming activity no more than five years prior to submitting that application. A new en- trant is defined as a farmer who commenced their agricultural activity since 1 January 2013.

The Regulations also set down that an eligible applicant must have submitted an application under the Basic Payment Scheme in the year of application to the National Reserve. Records held by my Department show the person named did not apply to the Basic Payment Scheme for 2015. On this basis the person named does not meet the eligibility criteria under the National Reserve. Records held by my Department also indicate that the person named did not submit an application to the National Reserve in 2015.

As all of the National Reserve Funding was utilised in 2015, there is no funding available to cover the operation of a 2016 National Reserve. From 2017 onwards the National Reserve will be replenished from unused entitlements, as EU Regulations governing the scheme provide that entitlements that are unused for 2 consecutive years are surrendered to the National Reserve. Decisions in relation to the Reserve for 2017 will be considered once the position on potential funding arising from unused entitlements and clawback from the sale of entitlements without land has been established.

21/06/2016WRX03200Forestry Grants

21/06/2016WRX03300492. Deputy Brendan Griffin asked the Minister for Agriculture, Food and the Marine if he will provide a forestry reinstatement grant to persons (details supplied) whose forestry was damaged in a storm in February 2014; and if he will make a statement on the matter. [17247/16]

21/06/2016WRX03400Minister for Agriculture, Food and the Marine (Deputy Michael Creed): Approval to fund a reconstitution scheme for forests affected by the winter storms of 2013/2014 was sought by my Department from the Department of Public Expenditure and Reform. The Department of Public Expenditure and Reform has sought further information on the impacts of such a scheme on the insurance market for forestry and also on other issues of concern. My Depart- ment is currently examining these requests.

224 21 June 2016 TAMS Applications Data

21/06/2016WRX03600493. Deputy Aindrias Moynihan asked the Minister for Agriculture, Food and the Ma- rine the number of approved, part-approved and unapproved applications, and when the part- approved applicants will be paid, under the targeted agricultural modernisation scheme II. [17256/16]

21/06/2016WRX03700Minister for Agriculture, Food and the Marine (Deputy Michael Creed): 1,107 part approvals issued in respect of urgently required Dairy Equipment and Low Emission Slur- ry Spreading Equipment. 1,182 full approvals have now issued across all of the TAMS II Schemes. The processing is continuing for eligible applications that have not yet received approval. The IT payment claim system is being finalised in order that applicants who have completed their work can submit their claim application online. This will be in place in July with payments to follow.

21/06/2016WRX03800Agri-Environment Options Scheme Payments

21/06/2016WRX03900494. Deputy Charlie McConalogue asked the Minister for Agriculture, Food and the Ma- rine why there has been a delay in payment being issued to a person (details supplied) under the agri-environment options scheme; and if he will make a statement on the matter. [17269/16]

21/06/2016WRX04000Minister for Agriculture, Food and the Marine (Deputy Michael Creed): The person named commenced their AEOS contract on 1st November 2010. Payment issued in respect of the 2010-2014 scheme years.

Under the EU Regulations governing the Scheme and other area-based payment schemes, a comprehensive administrative check, including cross-checks with the Land Parcel Identifica- tion System must take place. As all AEOS 1 participants will be receiving their final payments under the scheme, re-checks on payments made for all scheme years must be completed before final payment can be processed. Officials are working through these files and payment will be made in all cases as soon as possible.

Aquaculture Licences

21/06/2016WRX04200495. Deputy Margaret Murphy O’Mahony asked the Minister for Agriculture, Food and the Marine if aquaculturalists who are outside the remit of Natura 2000 areas and are fully compliant and awaiting his determination on a renewal licence may receive interim licences in circumstances where resources are not in place to provide licences within a timely manner; and if he will make a statement on the matter. [17272/16]

21/06/2016WRX04300Minister for Agriculture, Food and the Marine (Deputy Michael Creed): Applications for aquaculture licences are considered by my Department in accordance with the 1997 Fisher- ies (Amendment) Act, the 1933 Foreshore Act and applicable EU legislation.

The legislation does not provide for the issue of interim licences.

In the case of operators who are currently in the process of renewing their licences, pending a decision on the renewal, operators may lawfully continue their aquaculture activity under the provisions of Section 19A(4) of the 1997 Fisheries (Amendment) Act.

225 Questions - Written Answers

21/06/2016WRX04400Aquaculture Licences

21/06/2016WRX04500496. Deputy Margaret Murphy O’Mahony asked the Minister for Agriculture, Food and the Marine if he has considered the report entitled The National Economic and Social Council on Sustainable Development of Aquaculture in Ireland, dated June 2016, in relation to the posi- tion pertaining to aquaculture business operators who are currently in the process of renewing their licences; and if he will make a statement on the matter. [17273/16]

21/06/2016WRX04600Minister for Agriculture, Food and the Marine (Deputy Michael Creed): The report referred to by the Deputy has been noted by my Department.

Applications for aquaculture licences are considered by my Department in accordance with the 1997 Fisheries (Amendment) Act, the 1933 Foreshore Act and applicable EU legislation.

Pending a decision on the renewal of licences, operators may lawfully continue their aqua- culture activity under the provisions of Section 19A(4) of the 1997 Fisheries (Amendment) Act.

Departmental Contracts

21/06/2016WRX04800497. Deputy David Cullinane asked the Minister for Communications, Energy and Natural Resources if he is aware that the company awarded a public contract for delivering the emer- gency call answering service is not a living wage employer and that staff are paid less than the living wage; and if he will make a statement on the matter. [16911/16]

21/06/2016WRX04900Minister for Communications, Energy and Natural Resources (Deputy Denis Naught- en): The contract to operate the Emergency Call Answering Service was awarded to BT Ireland in 2009, following a public procurement process. Under the terms of the Concession contract, BT Ireland and its sub-contractor, Conduit Global, are required to comply with all statutory employment legislation.

Operational issues are a matter for Conduit Global in the first instance and it would not be appropriate for me to comment further on the matter.

21/06/2016WRX05000Inland Fisheries

21/06/2016WRX05100498. Deputy Michael Collins asked the Minister for Communications, Energy and Natural Resources his views on an issue (details supplied) regarding fishing. [17160/16]

21/06/2016WRX05200Minister of State at the Department of Communications, Energy and Natural Re- sources (Deputy Seán Kyne): Inland Fisheries Ireland (IFI) is the state agency responsible for the protection, management and conservation of Ireland’s inland fisheries and sea angling resources. In that role, IFI provides management advice to my Department regarding the most up-to-date status of species within its brief.

IFI manages salmon stocks on an individual river basis as each of Ireland’s 143 salmon rivers has its own genetically unique stock of salmon. IFI is supported in its management of salmon stocks by a statutorily independent Standing Scientific Committee (SSC) comprising scientists from IFI, Bord Iascaigh Mhara (BIM), the Loughs Agency, the National Parks and Wildlife Service, the Marine Institute, the Agri-Food and Biosciences Institute (AFBI - North- ern Ireland), and other State bodies and third level institutions. The SSC’s assessment of each individual stock is carried out every year.

226 21 June 2016 The offshore and inshore salmon and sea trout commercial fisheries were closed by a Gov- ernment decision from 2007 in order to comply with the EU Habitats Directive. A Hardship Scheme totalling €30m was established for fishermen and fishing communities. The scheme was administered by BIM and I am advised that the average payment under the scheme was almost €23,000, with the highest payment at over €195,000. Conditions of the scheme included that nets had to be verifiably decommissioned and those availing of the Scheme would not be entitled to a licence in the future.

Commercial harvest fisheries are currently permitted on individual river stocks which are shown to be meeting their conservation limit. Fisheries in estuaries may also be permitted where the stocks from individual rivers entering the estuaries are each meeting their individual conservation limits. The current management approach allows Ireland to meet its international and national obligations in terms of salmon conservation and operate sustainable harvest fisher- ies (commercial and recreational) where this is feasible.

I am advised by IFI that where there is a mix of individual river stocks at sea, from both Irish rivers and the rivers of other countries, it is not possible to disaggregate fish in a mixed stock area and the conservation imperative is to protect vulnerable individual stocks from in- discriminate fishing and avoid exploitation of those stocks within the mix that are below their conservation limit.

21/06/2016WRX05300Post Office Network

21/06/2016WRX05400499. Deputy Thomas Pringle asked the Minister for Communications, Energy and Natural Resources the number of times the post office service hubs working group has met since it was set up earlier in 2016; when pilot projects due to be in place by summer 2016 will be estab- lished; and if he will make a statement on the matter. [16659/16]

21/06/2016WRX05500500. Deputy Thomas Pringle asked the Minister for Communications, Energy and Natural Resources the number of times the network renewal working group has met since the general election; if he will publish a report on its progress in time for the July 2016 deadline; and if he will make a statement on the matter. [16660/16]

21/06/2016WRX05600Minister for Communications, Energy and Natural Resources (Deputy Denis Naugh- ten): I propose to take Questions Nos. 499 and 500 together.

The Network Renewal Implementation Group, which is being independently Chaired by Mr Bobby Kerr, was established in January and has met at least once a month. This Group is exam- ining the number and spatial distribution of branches, branch modernisation, the streamlining of products and services, postmaster payments and contracts, and training and qualifications for post office employees. An Post and the Irish Postmasters Union (IPU) are members of the Group. Good progress has been made in a collaborative environment. Work is ongoing in the group and will continue for the foreseeable future.

A second group has been established to look at financial services and is making good prog- ress, while a further Group has recently been established to look at the more complicated issues of contract renewal. The Group looking at developing Post Offices as ‘Hubs’ has not yet been convened.

Responsibility for implementation of the recommendations in the Final Report will shortly transfer to the Minister for Regional Development, Rural Affairs, Arts and the Gaeltacht. It has been agreed that experienced senior officials from my Department, who have knowledge of the Report and its recommendations, will move to the Department of Regional Development, Rural 227 Questions - Written Answers Affairs, Arts and the Gaeltacht to work on the implementation of the various recommendations.

21/06/2016WRX05700Public Service Obligation Levy Increase

21/06/2016WRX05800501. Deputy Hildegarde Naughton asked the Minister for Communications, Energy and Natural Resources if he is aware of the plan to increase the public service obligation levy by nearly 36% in each of the years 2016 and 2017; if so, the estimated knock-on effect of this on the price of electricity; and if he will make a statement on the matter. [16784/16]

21/06/2016WRX05900Minister for Communications, Energy and Natural Resources (Deputy Denis Naugh- ten): The Public Service Obligation (PSO) Levy has been in place since 2001. The PSO levy consists of a set of support schemes designed to facilitate the Government’s electricity policy objectives for renewables, indigenous fuels and security of supply. It also supports employment in the renewables industry and in the peat industry.

The PSO has supported the connection of over 2,000 MW of renewable energy to the electricity grid and will continue to support further renewable energy connection so that the 40% target for renewable electricity penetration by 2020 is met.

Recent international studies have shown that Ireland has had one of the lowest per unit sup- ports for renewable energy in the European Union.

The calculation of the level of the PSO is a matter for the Commission on Energy Regula- tion (CER). The CER has published a consultation on a proposed levy of €441 million for 2016-2017, equating to less than €7 per month for domestic consumers and representing an annual increase for domestic consumers of €19.32 on last year. The biggest drivers for the proposed levy increase are the lower predicted wholesale market electricity price and increased deployment of renewables.

I am very conscious of the impacts of energy costs on industry and households alike. As the wholesale electricity price is falling, this should be passed onto domestic electricity customers and more than outweigh the proposed levy increase in the current period. In this regard I am pleased to see that some electricity suppliers have recently reduced their prices. Customers can also avail of discounts available, either by contacting their supplier directly or by using the price comparison websites accredited by CER to switch to discounted tariffs. A customer consuming the average amount of electricity could save over €150 by switching suppliers.

21/06/2016WRX06000Warmer Homes Scheme

21/06/2016WRX06100502. Deputy David Cullinane asked the Minister for Communications, Energy and Natural Resources the amount of the €5 million pilot scheme under the warmer homes scheme whereby persons who have a medical illness can have their homes upgraded by Waterford City and County Council; if he is aware that the Waterford Energy Bureau is aware of at least 50 private homes that would benefit under the scheme; and if he will make a statement on the matter. [16785/16]

21/06/2016WRX06200Minister for Communications, Energy and Natural Resources (Deputy Denis Naugh- ten): The Warmth and Wellbeing pilot scheme is an initiative under the Government’s Strategy to Combat Energy Poverty and the Healthy Ireland Framework. The scheme is one of a number of measures outlined in the Strategy which aims to alleviate energy poverty among our most vulnerable citizens. Its objective is to better understand how we can improve health and wellbe-

228 21 June 2016 ing by making homes warmer and more energy efficient. The scheme has been developed by my Department in close cooperation with the Department of Health and is being operated by the Sustainable Energy Authority of Ireland (SEAI) and the HSE.

€20m has been ear-marked by my Department for the scheme in the period 2016 to 2018, of which €4m will be spent this year. As it is a pilot scheme, eligibility is necessarily limited. In 2016, the scheme is targeting people over 55, living with chronic obstructive pulmonary disease (COPD) or asthma, in the Dublin 12 and Dublin 24 areas, living in either an owner-occupied, local authority or approved housing association house and who are in receipt of or eligible for the Fuel Allowance. The geographical area and clinical criteria for the scheme were identified in close cooperation with the HSE. Once the HSE has approved an application, a range of mea- sures will be carried out by SEAI, including wall and attic insulation, heating system upgrades, installation of heating controls and other measures deemed appropriate to bring the house to a high level of energy efficiency. The number of participants will depend on the condition of the homes of those participating and the resulting cost per home.

While the focus this year is on alleviating the chronic conditions in older people, the aim is to expand the scope of the scheme in 2017 to children with asthma who are also at risk of energy poverty to establish how energy efficiency can also be used to help prevent these conditions developing in the first place. For the longer term, to determine the scheme’s effectiveness, an important part of the scheme is a research and evaluation project which will gather the evidence necessary to inform decisions on a future national roll-out.

I welcome the Deputy’s engagement on this important initiative and his understanding of the need to support those at risk of energy poverty in his constituency. Ongoing engagement with stakeholders beyond the immediate reach of this particular pilot scheme will be a very important part of the process to develop the case for wider action, and I look forward to keeping Deputies informed of progress. While the Warmth & Wellbeing project is limited in scope at present, free energy efficiency upgrades are available to those at risk of energy poverty under the Better Energy Warmer Homes scheme, which is also run by SEAI.

21/06/2016WRY00200Energy Prices

21/06/2016WRY00300503. Deputy Seán Haughey asked the Minister for Communications, Energy and Natural Resources the efforts he is making to bring about a reduction of electricity prices, having regard to the decreasing price of oil and gas internationally; and if he will make a statement on the matter. [16903/16]

21/06/2016WRY00400Minister for Communications, Energy and Natural Resources (Deputy Denis Naugh- ten): It is important to note that the electricity market is commercial, liberalised, and competi- tive and operates within national and European regulatory regimes. I have no statutory function in the setting or review of electricity prices. Price setting by all electricity suppliers is a com- mercial and operational matter for those companies.

21/06/2016WRY00500Broadband Service Provision

21/06/2016WRY00600504. Deputy Stephen S. Donnelly asked the Minister for Communications, Energy and Natural Resources to conduct a review of public access to broadband within State-owned pub- lic hospitals, primary care centres, public libraries, Intreo centres, Garda stations and county council offices; and if he will make a statement on the matter. [16937/16]

229 Questions - Written Answers

21/06/2016WRY00700Minister for Communications, Energy and Natural Resources (Deputy Denis Naugh- ten): The National Broadband Plan (NBP) aims to deliver high speed services to every city, town, village and individual premises in Ireland. The Programme for a Partnership Government commits to the delivery of the NBP as a matter of priority. This is being achieved through pri- vate investment by commercial telecommunications companies and through a State interven- tion in areas where commercial investment is not forthcoming.

The Government’s NBP intervention will ensure the availability of quality future-proofed broadband services to all other parts of the country. Over 750,000 premises are the focus of the procurement process, which formally commenced in December 2015 with the publication of the Pre-Qualification Questionnaire (‘PQQ’) and Project Information Memorandum.

The second stage in the procurement process will be a formal invitation to Participate in Di- alogue (ITPD) to qualifying bidders, following the assessment of responses to the PQQ stage. I expect that my Department will move to this stage in the next month. The third stage of the pro- curement process involves the issue of final tender documentation which follows the dialogue process. Following the submission of final tenders by bidders, a winning bidder(s) will be se- lected for the contract(s) which will comprise one or two lots as set out in the NBP Intervention Strategy. The Department will then enter into formal contract negotiations with the winning bidder(s). It is expected that contract(s) will be awarded by June 2017.

As part of the competitive process, the Department will engage with winning bidder(s) on the best roll-out strategy, in order to target areas of particularly poor service, business needs and / or high demand. This will need to be balanced with the most efficient network roll-out plan. A prioritisation programme will be put in place in this regard, in consultation with the Minister for Regional Development, Rural Affairs, Arts and the Gaeltacht, Deputy Heather Humphreys. A detailed roll-out plan for the network will be published once contract(s) are in place.

My Department’s current priority in this area is to complete all elements of the procurement phase of the NBP market intervention in order to ensure all property owners or occupiers who wish to access high-speed broadband can do so as soon as possible. It is a matter for the indi- vidual State-owned agencies and centres to decide whether they wish to provide public access to broadband within their respective buildings.

The Minister, Deputy Humphreys is leading on the establishment of county or regional broadband action groups, working with Local Authorities, Local Enterprise Offices, LEADER Groups and other relevant agencies to help accelerate the broadband network build in rural Ireland, once a contract(s) has been awarded. It is open to Local Authorities and other entities developing these strategies to consider whether broadband services should be made available in public offices as part of their wider Regional Digital Strategies.

21/06/2016WRY00800Broadband Service Provision

21/06/2016WRY00900505. Deputy Mattie McGrath asked the Minister for Communications, Energy and Nat- ural Resources the number of households and businesses that have no access to high-speed broadband in County Tipperary; the measures he is taking to address this; and if he will make a statement on the matter. [16968/16]

21/06/2016WRY01000Minister for Communications, Energy and Natural Resources (Deputy Denis Naugh- ten): The National Broadband Plan (NBP) aims to deliver high speed services to every city, town, village and individual premises in Ireland. The Programme for a Partnership Govern- ment commits to the delivery of the NBP as a matter of priority. This is being achieved through

230 21 June 2016 private investment by commercial telecommunications companies and through a State inter- vention in areas where commercial investment is not forthcoming.

The High Speed Broadband Map, which is available at www.broadband.gov.ie, shows the extent of the State Intervention area, which is the subject of procurement. The areas marked BLUE represent those areas where commercial providers are either currently delivering or have previously indicated plans to deliver high speed broadband services. The Department continues to monitor the commercial deployment plans in the BLUE area to ensure that those services are delivered. The areas marked AMBER on the High Speed Broadband Map represent the target areas for the State Intervention.

The map provides information on a county by county basis with a breakdown of coverage across the townlands in every county including Tipperary. Individuals can check whether their premises is in a BLUE or an AMBER area by scrolling through the map online or entering their Eircode. The Department also has a dedicated mailbox for queries in relation to the map - [email protected].

The county breakdown for the National Broadband Plan Intervention in Tipperary is set out in the table below, with the percentage of premises to be covered by the Intervention and through commercial investment also listed.

County Name Total Number of Number of % Premises % Premises County Town- Premises Cov- within the NBP within Commer- lands ered by National Intervention cial Operator’s Broadband Plan Area Area Areas Area Tipperary 3,167 41,207 49% 51% Over 750,000 premises are the focus for the procurement process, which formally com- menced in December 2015 with the publication of the Pre-Qualification Questionnaire (‘PQQ’) and Project Information Memorandum. Five responses were received from prospective bidders to this stage of the competitive procurement process by the deadline of 31 March 2016. The second stage in the procurement process will be a formal invitation to Participate in Dialogue (ITPD) to qualifying bidders. I expect that my Department will move to this stage in the next month. The third stage of the procurement process involves the issue of final tender documen- tation which follows the dialogue process. Following the submission of final tenders by bid- ders, a winning bidder(s) will be selected for the contract which will comprise one or two lots as set out in the NBP Intervention Strategy. The Department will then enter into formal contract negotiations with the winning bidder(s). It is expected that contract(s) will be awarded by June 2017.

As part of the competitive process, the Department will engage with winning bidder(s) on the best roll-lout strategy, in order to target areas of particularly poor service, business needs and / or high demand. This will need to be balanced with the most efficient network roll-out plan. A prioritisation programme will be put in place in this regard, in consultation with the Minister for Regional Development, Rural Affairs, Arts and the Gaeltacht, Deputy Heather Humphreys. A detailed roll-out plan for the network will be published once contract(s) are in place.

The Programme for a Partnership Government commits also to measures to assist in the rollout of the network once a contract is awarded. In this regard, the Minister, Deputy Hum- phreys is leading on the establishment of county or regional broadband taskforces, working with Local Authorities, Local Enterprise Offices, LEADER Groups and other relevant agencies to help accelerate the broadband network build in rural Ireland, once a contract(s) has been awarded.

231 Questions - Written Answers

21/06/2016WRY01100Energy Infrastructure

21/06/2016WRY01200506. Deputy Mattie McGrath asked the Minister for Communications, Energy and Natu- ral Resources the status of the support for the roll-out of EirGrid’s policy for the south-east region Grid Link project; and if he will make a statement on the matter. [16969/16]

21/06/2016WRY01300Minister for Communications, Energy and Natural Resources (Deputy Denis Naugh- ten): EirGrid is the designated Transmission System Operator and its role includes the op- eration, maintenance and development of the electricity transmission network in Ireland. As detailed in the Government Policy Statement on the Strategic Importance of Transmission and Other Energy Infrastructure, the Government does not seek to direct EirGrid in the develop- ment of energy infrastructure to particular sites or routes or technologies.

The Grid Link project was originally proposed as a new 400kV overhead line from Knock- raha (near Cork City) to Great Island (near Waterford City) and on to Dunstown (near Naas). The March 2015 EirGrid strategy “Your Grid, Your Views, Your Tomorrow ”, confirmed the need for the Grid Link project, but due to changing demand forecasts, upgrades to existing lines and advances in technology made new options possible.

EirGrid examined various technical solutions and in October 2015 announced that it in- tended to move forward with the Grid Link project using the Regional Option. The Regional Option uses series compensation which is an advanced, smart grid technology that will enable more power to flow through existing lines.

This means there is now no requirement to proceed with the previously proposed Grid Link 400 kV overhead line.

To complete the Grid Link project using the Regional Solution, an underwater cable across the Shannon estuary is required in addition to some upgrade works to existing transmission lines.

21/06/2016WRY01400Renewable Energy Feed in Tariff Scheme

21/06/2016WRY01500507. Deputy James Browne asked the Minister for Communications, Energy and Natural Resources the position regarding the total, per month and per county, in megawatt and connec- tion values, of solar farm developments looking for grid connection since 1 January 2015; the notional recommended maximum of solar capacity on the Irish grid; the timescales for the in- troduction of a solar renewable energy feed in tariff, REFIT, and if a future solar REFIT will be open to participation by all households with solar panels on their rooftops, as there is currently no solar REFIT; and if he will make a statement on the matter. [17071/16]

21/06/2016WRY01600Minister for Communications, Energy and Natural Resources (Deputy Denis Naugh- ten): The White Paper on Energy Policy includes a commitment to introduce a new Renewable Electricity Support Scheme (RESS) designed to encourage the development of Ireland’s abun- dant, diverse and indigenous renewable energy resources, including solar photovoltaics (PV) technology. The Programme for Government also commits to facilitating the development of solar energy projects.

It is widely recognised that the deployment of solar PV in Ireland has the potential to increase energy security, contribute to our renewable energy targets, and support economic growth and jobs. In addition, solar PV can be deployed in roof-mounted or ground-mounted installations, and in this way it can empower Irish citizens and communities to take control of the production

232 21 June 2016 and consumption of energy.

While I do not have a breakdown per month or per county of solar applications for connec- tion to the grid, I understand from ESB Networks that there have been 469 solar applications (amounting to 2.9 GW approx. of solar capacity) for connection to the distribution grid in the last 18 months. As part of the connection process to the electricity grid, ESB Network carries out a technical assessment for each project. This technical analysis ensures that a robust, secure electricity grid is maintained for the benefit of all electricity consumers.

In July 2015, the Department launched the first public consultation regarding the introduc- tion of a proposed Renewable Electricity Support Scheme (RESS) in Ireland. In-depth eco- nomic analysis is now underway to inform the cost of a new scheme and, while no decision has been taken on the precise renewable technologies to be supported, the cost and technical viability of solar photovoltaic (PV), both roof-top and utility-scale, is being examined as part of the assessment process.

Once the detailed economic analysis is complete, there will be an additional public con- sultation phase on the design of the new scheme. The details of this will be advertised on the Department’s website www.dcenr.gov.ie.

The introduction of any new scheme - including the overall costs and technologies to be supported - will be subject to Government approval and State aid clearance from the European Commission. It is expected that a new scheme will become available in 2017.

21/06/2016WRY01700Radio Spectrum Management

21/06/2016WRY01800508. Deputy Mattie McGrath asked the Minister for Communications, Energy and Natu- ral Resources his views that it is important for Raidió Teilifís Éireann to retain its longwave radio service to maintain contacts between Ireland and the Irish diaspora, many of whom are elderly and may not be in a position to avail of the Internet (details supplied); and if he will make a statement on the matter. [17215/16]

21/06/2016WRY01900Minister for Communications, Energy and Natural Resources (Deputy Denis Naught- en): RTÉ is an independent national public service broadcaster whose remit and obligations are set out in the Broadcasting Act 2009. Section 114(1) of the Act states the principal objects and associated powers of RTÉ and Section 98 provides that it shall be independent in the pursuance of these objects, subject to the requirements of the Act. As such I, as Minister, have no function in RTÉ’s decision making in regard to general day to day matters, including the provision of the 252 longwave radio service.

As announced by RTÉ on 19 December 2014, the service is to be retained until 2017. This decision extends the life of the longwave service and gives the broadcaster time to engage with those listeners likely to be affected by its closure about alternative ways of accessing RTÉ radio. It also provides the company with the opportunity to explore the availability of other platforms for delivering the service that might help meet the needs of these individual listeners and com- munities.

21/06/2016WRY02000Broadband Service Provision

21/06/2016WRY02100509. Deputy Michael Healy-Rae asked the Minister for Communications, Energy and Nat- ural Resources the status of broadband access in an area (details supplied); and if he will make

233 Questions - Written Answers a statement on the matter. [17235/16]

21/06/2016WRY02200Minister for Communications, Energy and Natural Resources (Deputy Denis Naugh- ten): The National Broadband Plan (NBP) aims to deliver high speed services to every city, town, village and individual premises in Ireland. The Programme for a Partnership Govern- ment commits to the delivery of the NBP as a matter of priority. This is being achieved through private investment by commercial telecommunications companies and through a State inter- vention in areas where commercial investment is not forthcoming.

The High Speed Broadband Map, is available at www.broadband.gov.ie and shows the ex- tent of the State Intervention area, which is the subject of procurement. The areas marked BLUE represent those areas where commercial providers are either currently delivering or have previously indicated plans to deliver high speed broadband services. The Department continues to monitor the commercial deployment plans in the BLUE area to ensure that those services are delivered. The areas marked AMBER on the High Speed Broadband Map represent the target areas for the State Intervention and includes the townlands of Cangullia and Scartaglin, County Kerry.

The map provides information on a county by county basis with a breakdown of coverage across the townlands in every county including Kerry. Individuals can check whether their premises is in a BLUE or an AMBER area by scrolling through the map online or entering their Eircode. The Department also has a dedicated mailbox for queries in relation to the map - [email protected].

Over 750,000 premises are the focus for the procurement process, which formally com- menced in December 2015 with the publication of the Pre-Qualification Questionnaire (‘PQQ’) and Project Information Memorandum. Five responses were received from prospective bidders to this stage of the competitive procurement process by the deadline of 31 March 2016. The second stage in the procurement process will be a formal invitation to Participate in Dialogue (ITPD) to qualifying bidders. I expect that my Department will move to this stage in the next month. The third stage of the procurement process involves issuing final tender documentation which follows the dialogue process. Following the submission of final tenders by bidders, a winning bidder(s) will be selected for the contract which will comprise one or two lots as set out in the NBP Intervention Strategy. The Department will then enter into formal contract negotia- tions with the winning bidder(s). It is expected that contract(s) will be awarded by June 2017.

As part of the competitive process, the Department will engage with winning bidder(s) on the best roll-out strategy, in order to target areas of particularly poor service, business needs and/or high demand. This will need to be balanced with the most efficient network roll-out plan. A prioritisation programme will be put in place in this regard, in consultation with the Minister for Regional Development, Rural Affairs, Arts and the Gaeltacht. A detailed roll-out plan for the network will be published once contract(s) are in place.

The Programme for a Partnership Government commits also to measures to assist in the rollout of the network once a contract is awarded. In this regard, the Minister, Deputy Hum- phreys is leading on the establishment of county or regional broadband taskforces, working with Local Authorities, Local Enterprise Offices, LEADER Groups and other relevant agencies to help accelerate the broadband network build in rural Ireland, once a contract(s) has been awarded.

21/06/2016WRY02300National Broadband Plan Implementation

234 21 June 2016

21/06/2016WRY02400510. Deputy Brendan Griffin asked the Minister for Communications, Energy and Natural Resources the status of the national broadband plan with reference to Derrymore East, Tralee, County Kerry (details supplied). [17239/16]

21/06/2016WRY02500Minister for Communications, Energy and Natural Resources (Deputy Denis Naugh- ten): The National Broadband Plan (NBP) aims to deliver high speed services to every city, town, village and individual premises in Ireland. The Programme for a Partnership Govern- ment commits to the delivery of the NBP as a matter of priority. This is being achieved through private investment by commercial telecommunications companies and through a State inter- vention in areas where commercial investment is not forthcoming.

The High Speed Broadband Map is available at www.broadband.gov.ie. It shows the extent of the State Intervention area, which is the subject of procurement. The areas marked BLUE represent those areas where commercial providers are either currently delivering or have previ- ously indicated plans to deliver high speed broadband services. The Department continues to monitor the commercial deployment plans in the BLUE area to ensure that those services are delivered. The areas marked AMBER on the High Speed Broadband Map represent the target areas for the State Intervention and includes the townland of Derrymore East, County Kerry.

The map provides information on a county by county basis with a breakdown of coverage across the townlands in every county including County Kerry. Individuals can check whether their premises is in a BLUE or an AMBER area by scrolling through the map online or entering their Eircode. The Department also has a dedicated mailbox for queries in relation to the map - [email protected].

Over 750,000 premises are the focus for the procurement process, which formally com- menced in December 2015 with the publication of the Pre-Qualification Questionnaire (‘PQQ’) and Project Information Memorandum. Five responses were received from prospective bidders to this stage of the competitive procurement process by the deadline of 31 March 2016. The second stage in the procurement process will be a formal invitation to Participate in Dialogue (ITPD) to qualifying bidders. I expect that my Department will move to this stage in the next month. The third stage of the procurement process involves the issue of final tender documen- tation which follows the dialogue process. Following the submission of final tenders by bid- ders, a winning bidder(s) will be selected for the contract which will comprise one or two lots as set out in the NBP Intervention Strategy. The Department will then enter into formal contract negotiations with the winning bidder(s). It is expected that contract(s) will be awarded by June 2017.

As part of the competitive process, the Department will engage with winning bidder(s) on the best roll-out strategy, in order to target areas of particularly poor service, business needs and/or high demand. This will need to be balanced with the most efficient network roll-out plan. A prioritisation programme will be put in place in this regard, in consultation with the Minister for Regional Development, Rural Affairs, Arts and the Gaeltacht. A detailed roll-out plan for the network will be published once contract(s) are in place.

The Programme for a Partnership Government commits also to measures to assist in the roll- out of the network once a contract is awarded. In this regard, Minister Humphreys is leading on the establishment of county or regional broadband taskforces, working with Local Authorities, Local Enterprise Offices, LEADER Groups and other relevant agencies to help accelerate the broadband network build in rural Ireland, once a contract(s) has been awarded.

Better Energy Communities Programme

235 Questions - Written Answers

21/06/2016WRY02700511. Deputy David Cullinane asked the Minister for Communications, Energy and Natural Resources if he is aware that Waterford City and County Council has made an application to the Sustainable Energy Authority of Ireland for funding under the better energy communities scheme and that its initial application was declined; if he is providing additional funding under the scheme; if he will consider the application in this context; and if he will make a statement on the matter. [17246/16]

21/06/2016WRY02800Minister for Communications, Energy and Natural Resources (Deputy Denis Naugh- ten): The Sustainable Energy Authority of Ireland (SEAI) administers the Better Energy Com- munities scheme on behalf of my Department. The scheme aims to support and encourage community based partnerships to improve the energy efficiency of homes, businesses and com- munity facilities in a local area. To date the scheme has supported the upgrade of more than 12,000 homes and several hundred shared community facilities, from sports clubs to commu- nity centres and childcare facilities.

My Department agreed a number of high level principles for the operation of the Better Energy Communities scheme in 2016 with SEAI. This led to the creation of a new category of projects with a value under €50,000 that can be progressed faster than normal applications. It also capped funding for any single project at €1,000,000 to ensure more projects and commu- nities can receive funding and made €500,000 available for the creation of a support network dedicated to helping community organisations that want to develop a sustainable energy system in their local area.

However, all applications to the Better Energy Communities Scheme are assessed in a com- petitive process operated by SEAI and neither I, nor my Department, have any function in rela- tion to the evaluation or selection of projects. This competitive process is fully transparent and I would urge any unsuccessful applicants to contact SEAI directly for feedback in relation to their application, which may assist in any future applications for support.

21/06/2016WRY02900Postal Codes

21/06/2016WRY03000512. Deputy Aindrias Moynihan asked the Minister for Communications, Energy and Natural Resources the process in place to allow householders to correct the description on their Eircode postcode, where it carries misspellings or fictitious townlands. [17257/16]

21/06/2016WRY03100Minister for Communications, Energy and Natural Resources (Deputy Denis Naugh- ten): The most comprehensive address database available in the country is the postal address database, which is owned by An Post GeoDirectory, a subsidiary company of An Post and Ordnance Survey Ireland. Eircode uses the addresses from this database in the Eircode Finder. Eircode does not have the right to change postal address data on behalf of GeoDirectory.

If a householder has any queries in relation to the postal address for their property, they should contact Customer Service division for An Post who can be contacted at ph: 01 705 8242 / E: [email protected].

21/06/2016WRY03200Broadband Service Provision

21/06/2016WRY03300513. Deputy Declan Breathnach asked the Minister for Communications, Energy and Natural Resources the additional efforts and focus being provided to ensure the speedy roll-out of high-speed broadband to businesses, homes and schools that are at serious disadvantage in this imbalance of broadband delivery, and to publish a detailed plan for same, given the recent 236 21 June 2016 survey of broadband usage stating that 25% of small to medium businesses here do not have an online presence and-or no broadband service. [17305/16]

21/06/2016WRY03400Minister for Communications, Energy and Natural Resources (Deputy Denis Naugh- ten): I am very conscious of the need for SMEs and micro-enterprises to develop an online trad- ing presence. Data from the EU Commission’s digital economic and social index shows that 32% of Irish SMEs are trading online, which ranks Ireland first among 28 EU Member States where the average is 16%. Research commissioned by my Department shows however that small businesses tend to be significantly less engaged in online activity.

Supporting more micro-enterprises to develop an online presence is a key priority and my Department’s Trading Online Voucher Scheme is yielding strong results. Small and micro-en- terprises that have developed an online trading presence through this scheme are experiencing growth of 21% with 3 in 5 beginning to export, and the potential for 1.4 new jobs per company. The scheme continues to operate through the Local Enterprise Office Network, and offers a financial incentive of up to €2,500 per business to develop a trading online presence.

Delivering high speed broadband to all businesses in Ireland is another key support for SMEs and micro-enterprises. The National Broadband Plan (NBP) aims to deliver high speed services to every city, town, village and individual premises in Ireland. The Programme for a Partnership Government commits to the delivery of the NBP as a matter of priority. This is being achieved through private investment by commercial telecommunications companies and through a State intervention in areas where commercial investment is not forthcoming.

The High Speed Broadband Map, which is available at www.broadband.gov.ie, shows the extent of the State Intervention area, which is the subject of procurement. The areas marked BLUE represent those areas where commercial providers are either currently delivering or have previously indicated plans to deliver high speed broadband services. The Department continues to monitor the commercial deployment plans in the BLUE area to ensure that those services are delivered. The areas marked AMBER on the High Speed Broadband Map represent the target areas for the State Intervention.

The map provides information on a county by county basis with a breakdown of coverage across the townlands in every county. Individuals can check whether their premises is in a BLUE or an AMBER area by scrolling through the map online or entering their Eircode. The Department also has a dedicated mailbox for queries in relation to the map - broadband@dcenr. gov.ie.

The Department conducted extensive consultation and analysis in developing the updated Intervention Strategy, published in December 2015. The Intervention Strategy is available at:

http://www.dcenr.gov.ie/communications/en-ie/Pages/Publication/Ireland’s-Broadband- Intervention-Strategy-update.aspx.

It sets out a detailed service specification including a requirement that the State-funded network must be capable of delivering high-quality, high speed broadband of at least 30Mbps download and 6Mbps upload, available 99.95% of the time. Together with other detailed tech- nical specifications, this will ensure that a reliable fast service is available to consumers. In addition to these requirements, bidder(s) will be asked to put forward minimum speeds for businesses which can be built into the service standards of the winning bidder(s) and proposals to future-proof the network to meet ongoing demand for bandwidth. To this end, the winning bidder(s) will be subject to periodic reviews on a 3-5 year basis.

Over 750,000 premises are the focus for the procurement process, which formally com-

237 Questions - Written Answers menced in December 2015 with the publication of the Pre-Qualification Questionnaire ( ‘PQQ’) and Project Information Memorandum. Five responses were received from prospective bidders to this stage of the competitive procurement process by the deadline of 31 March 2016. The second stage in the procurement process will be a formal invitation to Participate in Dialogue (ITPD) to qualifying bidders. I expect that my Department will move to this stage in the next month. The third stage of the procurement process involves the issue of final tender documen- tation which follows the dialogue process. Following the submission of final tenders by bid- ders, a winning bidder(s) will be selected for the contract which will comprise one or two lots as set out in the NBP Intervention Strategy. The Department will then enter into formal contract negotiations with the winning bidder(s). It is expected that contract(s) will be awarded by June 2017.

As part of the competitive process, the Department will engage with winning bidder(s) on the best rollout strategy, in order to target areas of particularly poor service, business needs and / or high demand. This will need to be balanced with the most efficient network rollout plan. A prioritisation programme will be put in place in this regard, in consultation with the Minister for Regional Development, Rural Affairs, Arts and the Gaeltacht. A detailed rollout plan for the network will be published once contract(s) are in place.

The Programme for a Partnership Government commits also to measures to assist in the rollout of the network once a contract is awarded. In this regard, Minister Humphreys is leading on the establishment of county or regional broadband taskforces, working with Local Authori- ties, Local Enterprise Offices, LEADER Groups and other relevant agencies to help accelerate the broadband network build in rural Ireland, once a contract(s) has been awarded.

In the meantime, my Department continues to liaise closely with industry and relevant other Departments and agencies to assist in the commercial deployment of telecommunications net- works. The commercial telecommunications sector has invested over €2bn in upgrading and modernising telecoms networks and services. These investments are further improving the coverage and quality of broadband and mobile voice and data services throughout the country.

The Programme for a Partnership Government also commits to the establishment of a mo- bile phone and broadband taskforce which will consider further measures to address telecom- munications deficits in rural Ireland. My Department is engaging with the Department of Arts, Heritage and the Gaeltacht, in relation to the terms of reference for the taskforce to ensure de- livery of the Programme for Government commitment within the timelines set out. It is likely that the taskforce will involve a number of Government Departments and agencies, as well as engagement with telecommunications operators. My Department has already conducted a number of meetings with vendors and mobile operators to help inform the process and identify what tangible measures can be taken.

21/06/2016WRY03500Driver Licensing Exchange Agreements

21/06/2016WRY03600514. Deputy Eugene Murphy asked the Minister for Transport, Tourism and Sport if there is a protocol in place for British driving licences to be recognised in the State or if they will have to be replaced with European Union driving licences for the purpose of British citizens living and working here who still drive on their British driving licence and in the event of Britain leav- ing the EU; and if he will make a statement on the matter. [16690/16]

21/06/2016WRY03700Minister for Transport, Tourism and Sport (Deputy Shane Ross): ​The EU sets common standards for driver licensing across all Member States. This means that all Member States ob- serve the same standards for obtaining a driving licence, and issue licences in the same standard 238 21 June 2016 format. Under EU law, a person is required to obtain their driving licence from the Member State in which they are normally resident. They may drive on their national licence in other Member States. If they move from one Member State to another, they should exchange their licence for the equivalent licence in their new State of residence. UK driving licence holders may therefore drive in Ireland on their licences, and may exchange their licences for the Irish equivalent if taking up residence here.

Were the UK to leave the EU, any future arrangements would be a matter for negotiation between the EU and the UK in the first instance, and I cannot speculate on what might emerge in such a hypothetical scenario.

The Deputy may wish to know, however, that under international road traffic conventions, holders of foreign licences may drive in Ireland for up to a year on a visitor basis.

21/06/2016WRY03800Roads Maintenance Funding

21/06/2016WRY03900515. Deputy Carol Nolan asked the Minister for Transport, Tourism and Sport the fund- ing he has allocated to Tipperary County Council for road upgrades in each of the years 2006 to 2016 to date; if he will increase the allocation to the council given the poor condition of the roads in certain parts of the county, specifically in the parish of Aglish, Roscrea, County Tipper- ary; and if he will make a statement on the matter. [17032/16]

21/06/2016WRY04000Minister for Transport, Tourism and Sport (Deputy Shane Ross): Details of the regional and local road grant payments in the period 2006 to 2015 to County Councils including Tipper- ary County Council are outlined in the regional and local road grant payment booklets which are available in the Dáil Library. Details of the 2016 regional and local road grant allocations to each County Council are also available in the Dáil library. In addition to the initial 2016 al- location as outlined in the allocation booklet, €5.35 million in severe weather funding has also been allocated to Tipperary County Council this year.

All funding for regional and local road programmes for 2016 has now been allocated.

With regard to national roads, details of payments are a matter for Transport Infrastructure Ireland (TII) (formerly known as the NRA).

Noting the above position, I have referred the Deputy’s question to TII for direct reply. I ask the Deputy to please advise my private office if she does not receive a reply within 10 working days.

Bus Éireann Staff

21/06/2016WRY04200516. Deputy Danny Healy-Rae asked the Minister for Transport, Tourism and Sport why an inspector (details supplied) is not going to be replaced; and if he will make a statement on the matter. [16661/16]

21/06/2016WRY04300Minister for Transport, Tourism and Sport (Deputy Shane Ross): ​The issue raised is a matter for Bus Éireann and I have forwarded the Deputy’s question to the company for direct reply. Please advise my private office if you do not receive a response within ten working days.

21/06/2016WRY04400Regional Airports

239 Questions - Written Answers

21/06/2016WRY04500517. Deputy John Paul Phelan asked the Minister for Transport, Tourism and Sport if he will ensure the continuance of passenger flight operations to and from Waterford Airport fol- lowing the recent announcement that a company (details supplied) is to cease operating from Waterford Airport; and if he will make a statement on the matter. [16691/16]

21/06/2016WRY04600539. Deputy David Cullinane asked the Minister for Transport, Tourism and Sport if he will meet the board of Waterford Airport given the discontinuation of services by a company (details supplied) recently at the airport; and if he will make a statement on the matter. [16973/16]

21/06/2016WRY04700540. Deputy David Cullinane asked the Minister for Transport, Tourism and Sport the increased funding he will make available to Waterford Airport following commitments in the programme for Government; the restrictions that exist in terms of European Union law and EU directives in providing funding to regional airports; and if he will make an investment in extending the runway at the airport.

[16974/16]

21/06/2016WRY04800Minister for Transport, Tourism and Sport (Deputy Shane Ross): I propose to take Questions Nos. 517, 539 and 540 together.

I was disappointed to learn a fortnight ago that VLM Airlines ceased its operations on the Luton service out of Waterford Airport. VLM also withdrew its Birmingham route at the airport last month.

The Deputy will appreciate however that securing routes and negotiating deals with airlines are matters for the airports themselves, be they State airports or regional airports. These are commercial matters for the airports and airlines.

I understand that the authorities in Waterford Airport are in detailed discussions with a num- ber of airlines with a view to securing a replacement carrier.

Waterford Airport receives substantial Exchequer support under the Regional Airports Programme that is administered by my Department. This Programme also supports the other three regional airports in Counties Donegal, Mayo and Kerry with funding but only in relation to the implementation of necessary safety and security related projects and activities. The extra €10 million in capital funding that is provided in the Programme for Government will enhance my Department’s ability to further support the four airports, including Waterford.

Development projects at the airports which are designed to expand capacity are a com- mercial matter for the airports themselves. In the case of the proposed runway extension at Waterford, I understand that the airport authorities are pursuing local interests for funding in relation to that project.

Over the past five years, Waterford Airport has received €9.6 million in operational and investment support from my Department. In April of this year, a further €157,500 in grant aid was approved for a number of security related projects, including new X-ray equipment, at the airport.

The current Regional Airports Programme covers the period 2015 to 2019 and this Pro- gramme was approved in August last year by the EU Commission as being in conformity with the Commission’s 2014 Guidelines on State aid to airports and airlines. Those EU Guidelines may be accessed here:

http://eur-lex.europa.eu/legal-content/EN/TXT/?uri=uriserv:OJ.C_.2014.099.01.0003.01. ENG .

240 21 June 2016 Some of the salient points arising from these Guidelines are contained in the current Regional Airports Programme which is published on my Department’s website: http://www.dttas.ie/sites/ default/files/publications/aviation/english/irelands-regional-airports-programme-2015-2019/ irelands-regional-airports-programme-2015-2019.pdf .

It is my intention to visit Waterford Airport and meet management there when time permits.

21/06/2016WRY04900Electric Vehicles

21/06/2016WRY05000518. Deputy John Paul Phelan asked the Minister for Transport, Tourism and Sport his plans to make electric vehicle car usage easier by providing free parking, access to bus lanes, removal of purchase taxes, lower road tax rates or removal of value added tax on leasing; and if he will make a statement on the matter. [16692/16]

21/06/2016WRY05100Minister for Transport, Tourism and Sport (Deputy Shane Ross): Policy regarding the promotion of electric vehicles lies within the Department of Communications, Climate Change and Natural Resources (DCCNR). Good progress has been made in creating the best environ- ment for promoting the penetration of EVs into the market in Ireland. A grant support scheme for EV consumers has been in place since 2011 whereby the consumer receives up to €5,000 off the cost price of the vehicle. In addition, EVs are also treated favourably under our motor tax system and qualify for VRT reliefs of up to €5,000.

In line with the Programme for Government commitment that Ireland become a leader in the take-up of electric vehicles, DCCNR have been tasked with establishing a taskforce involv- ing relevant government departments, agencies, industry and representative groups, to work on this goal and to set ambitious and achievable targets. The Task Force will examine various measures including, I expect, those proposed by the Deputy to make electric car use easier and more attractive. My Department anticipates that it will co-chair this task force and continue to work closely with DCCNR to promote electric car use.

Road Projects

21/06/2016WRY05300519. Deputy Robert Troy asked the Minister for Transport, Tourism and Sport his plans to upgrade the N4 road between Mullingar, County Westmeath and County Longford; and if he will make a statement on the matter. [16696/16]

21/06/2016WRY05400Minister for Transport, Tourism and Sport (Deputy Shane Ross): ​As Minister for Trans- port, Tourism and Sport, I have responsibility for overall policy and funding in relation to the national roads programme. The planning, design and implementation of individual national road projects (such as the N4 Mullingar to Longford) is a matter for Transport Infrastructure Ireland (formerly known as the NRA) under the Roads Acts 1993-2015 in conjunction with the local authorities concerned.

Noting the above position, I have referred the Deputy’s question regarding the N4 to TII for direct reply. I ask the Deputy to please advise my private office if he does not receive a reply within 10 working days.

21/06/2016WRY05500Regional Road Network

21/06/2016WRY05600520. Deputy Danny Healy-Rae asked the Minister for Transport, Tourism and Sport to 241 Questions - Written Answers make provisions to keep deer off the road infrastructure; and if he will make a statement on the matter. [16716/16]

21/06/2016WRY05700Minister for Transport, Tourism and Sport (Deputy Shane Ross): As Minister for Transport, Tourism and Sport, I have responsibility for overall policy and funding in relation to the national roads programme. The planning, design and maintenance of individual national roads is a matter for Transport Infrastructure Ireland (formerly known as the NRA) under the Roads Acts 1993-2015 in conjunction with the local authorities concerned.

Noting the above position, I have referred the Deputy’s question to TII for direct reply. Please advise my private office if you don’t receive a reply within 10 working days.

Concerning regional and local roads, the improvement and maintenance of these roads is the statutory responsibility of each local authority, in accordance with the provisions of Section 13 of the Roads Act 1993 (as amended). Works on those roads are funded from local authorities’ own resources supplemented by State road grants. The initial selection and prioritisation of works to be funded (including measures to prevent deer from straying) is also a matter for the relevant local authority.

21/06/2016WRY05800Roads Maintenance Funding

21/06/2016WRY05900521. Deputy Danny Healy-Rae asked the Minister for Transport, Tourism and Sport to provide funding for traffic calming and public lighting at Poulgorm junction where the R569 meets the N22; and if he will make a statement on the matter. [16719/16]

21/06/2016WRY06000Minister for Transport, Tourism and Sport (Deputy Shane Ross): ​As Minister for Trans- port, Tourism and Sport, I have responsibility for overall policy and funding in relation to the national roads programme. The planning, design and implementation of individual road proj- ects is a matter for Transport Infrastructure Ireland (TII) (formerly known as the NRA) under the Roads Acts 1993 to 2015 in conjunction with the local authorities concerned.

Within its capital budget, the assessment and prioritisation of individual projects is a matter in the first instance for Transport Infrastructure Ireland (TII) in accordance with Section 19 of the Roads Act.

Noting the above position, I have referred your question to TII for direct reply. I ask the Deputy to please advise my private office if he does not receive a reply within 10 working days.

21/06/2016WRY06100Road Safety

21/06/2016WRY06200522. Deputy Danny Healy-Rae asked the Minister for Transport, Tourism and Sport to reduce the speed limit on the N22 at Glenflesk Village to 50 km/h and from Lissivigeen round- about to the bottom of the Pike Hill; and if he will make a statement on the matter. [16720/16]

21/06/2016WRY06300Minister for Transport, Tourism and Sport (Deputy Shane Ross): A​ny proposed altera- tion of the existing speed limits applying to the stretches of road in question is a matter for Kerry County Council in the first instance.

While the Road Traffic Act 2004 sets default speed limits for national roads (both within and outside built up areas) local authorities, in this case Kerry County Council, may, in consultation with the Garda Commissioner and with the consent of Transport Infrastructure Ireland (TII), make bye-laws to set Special Speed Limits for such roads in their administrative areas. 242 21 June 2016 To assist local authorities in the application of Special Speed Limits, the Department’s up- dated Guidelines for Setting and Managing Speed Limits are available on the Department’s website at www.dttas.ie. These Guidelines reiterate that the making of Special Speed Limit bye-laws is a matter for the elected members of local authorities, subject to following Depart- ment Guidelines and to the consent of TII in the case of national roads.

At the time of the issuing of these Guidelines, local authorities were requested to review and update all speed limits in their administrative areas in accordance with the Guidelines and this is currently on-going. In the case of national roads, local authorities were requested to undertake this review in conjunction with TII.

21/06/2016WRY06400Road Safety

21/06/2016WRY06500523. Deputy Pearse Doherty asked the Minister for Transport, Tourism and Sport if he is aware of the road safety concerns of residents living in the vicinity of the N15 Blackburn Bridge Road project in County Donegal, phase 1 of which is currently under construction; the measures he will put in place to alleviate these concerns and to address the immediate hazards posed by the present condition of the road including poor road alignment, hazardous junctions as well as an uneven road surface; and if he will make a statement on the matter. [16721/16]

21/06/2016WRY06600Minister for Transport, Tourism and Sport (Deputy Shane Ross): ​As Minister for Transport, Tourism and Sport, I have responsibility for overall policy and funding in relation to the national roads programme. The planning, design and implementation of individual road projects (such as the N15 Blackburn Bridge scheme in Donegal) is a matter for the Transport Infrastructure Ireland (formerly known as the NRA) under the Roads Acts 1993-2015 in con- junction with Donegal County Council.

Within its capital budget, the assessment and prioritisation of individual projects is a matter in the first instance for TII in accordance with Section 19 of the Roads Act.

Noting the above position, I have referred the Deputy’s question to TII for direct reply. I ask the Deputy to please advise my private office if he does not receive a reply within 10 working days.

21/06/2016WRZ00200Rail Network Maintenance

21/06/2016WRZ00300524. Deputy Danny Healy-Rae asked the Minister for Transport, Tourism and Sport to provide funding to repair the railway crossing at Minish in Killarney, County Kerry; and if he will make a statement on the matter. [16722/16]

21/06/2016WRZ00400Minister for Transport, Tourism and Sport (Deputy Shane Ross): ​As Minister for Trans- port, Tourism & Sport, I have responsibility for policy and overall funding in relation to public transport. Matters relating to the repair of railway crossings are a day-to-day operational matter for Iarnród Éireann and I have no function in such matters.

Noting this I have referred the Deputy’s question to Iarnród Éireann for direct reply. Please advise my private office if you don’t receive a reply within 10 working days.

21/06/2016WRZ00500Local Improvement Scheme Funding

243 Questions - Written Answers

21/06/2016WRZ00600525. Deputy Danny Healy-Rae asked the Minister for Transport, Tourism and Sport to im- mediately provide funding for the commencement of the local improvement schemes in County Kerry (details supplied); and if he will make a statement on the matter. [16724/16]

21/06/2016WRZ00700Minister for Transport, Tourism and Sport (Deputy Shane Ross): ​The Local Improve- ment Scheme (LIS) remains in place this year. This Scheme provides funding for private roads and laneways, the maintenance and improvement of which is, in the first instance, a matter for the relevant landowner. While the LIS does not have a ring-fenced allocation in 2016, local authorities may use a proportion of State grant funding (15% of the Discretionary Grant which is €506,390 in the case of Kerry County Council) for LIS should they wish to do so. In addition Councils can also provide monies from their own resources towards LIS.

The Programme for a Partnership Government provides that, as the economy recovers, the Government will promote increased funding for Local Improvement Schemes, on an annual ba- sis. A major objective for the new Government will be to prioritise new investment in local and regional road maintenance and improvements and LIS funding will be promoted in that context.

21/06/2016WRZ00800Bus Services

21/06/2016WRZ00900526. Deputy Brendan Griffin asked the Minister for Transport, Tourism and Sport if he will request Bus Éireann to have the Limerick city to County Kerry bus service wait for the Galway city connection to arrive, given that the buses to County Kerry often leave seconds be- fore the Galway city bus arrives in Limerick city; and if he will make a statement on the matter. [16732/16]

21/06/2016WRZ01000Minister for Transport, Tourism and Sport (Deputy Shane Ross): ​The issue raised is a matter for the National Transport Authority (NTA) in conjunction with Bus Éireann and I have forwarded the Deputy’s question to the NTA for direct reply. Please advise my private office if you do not receive a response within ten working days.

21/06/2016WRZ01100Local Improvement Scheme Funding

21/06/2016WRZ01200527. Deputy Joe Carey asked the Minister for Transport, Tourism and Sport to consider the design of a scheme to provide funding for the ongoing maintenance of roads that have been reconstituted under the construction industry scheme or local improvement scheme; and if he will make a statement on the matter. [16754/16]

21/06/2016WRZ01300Minister for Transport, Tourism and Sport (Deputy Shane Ross): ​There is a Local Improvement Scheme (LIS) in place. This Scheme provides funding for private roads and laneways, the maintenance and improvement of which is, in the first instance, a matter for the relevant landowner. While the LIS does not have a ring-fenced allocation in 2016, local authorities may use a proportion of State grant funding (15% of the Discretionary Grant) for LIS should they wish to do so. In addition Councils can also provide monies from their own resources towards LIS.

There is also a Community Involvement Scheme (CIS) in place whereby local authorities may set aside 7.5% of their Restoration Improvement and Restoration Maintenance grants for CIS schemes if they so wish.

The Programme for a Partnership Government provides that, as the economy recovers, the Government will promote increased funding for Community Improvement Schemes and Local

244 21 June 2016 Improvement Schemes, on an annual basis. A major objective for the new Government will be to prioritise new investment in local and regional road maintenance and improvements, and CIS and LIS funding will be promoted in that context.

21/06/2016WRZ01400Road Projects Status

21/06/2016WRZ01500528. Deputy Danny Healy-Rae asked the Minister for Transport, Tourism and Sport the year of the six year investment programme that he will provide funding for the proposed Mac- room bypass; and if he will make a statement on the matter. [16757/16]

21/06/2016WRZ01600Minister for Transport, Tourism and Sport (Deputy Shane Ross): ​As Minister for Transport, Tourism & Sport, I have responsibility for overall policy and funding in relation to the national roads programme. The planning, design and implementation of individual road projects (including the Macroom Bypass) is a matter for Transport Infrastructure Ireland (TII) (formerly known as the NRA) under the Roads Acts 1993 to 2015 in conjunction with the local authorities concerned.

Within its capital budget, the assessment and prioritisation of individual projects is a matter in the first instance for Transport Infrastructure Ireland (TII) in accordance with Section 19 of the Roads Act.

Noting the above position, I have referred your question to TII for direct reply. Please ad- vise my private office if you don’t receive a reply within 10 working days.

21/06/2016WRZ01700Road Tolls

21/06/2016WRZ01800529. Deputy Alan Kelly asked the Minister for Transport, Tourism and Sport the amount of variable operational payments paid by the National Roads Authority and Transport Infrastruc- ture Ireland in respect of under usage of the N18 Limerick tunnel in each year to date in 2016; the future cost for the duration of the public private partnership based on projected future usage levels; and if he will make a statement on the matter. [16760/16]

21/06/2016WRZ01900Minister for Transport, Tourism and Sport (Deputy Shane Ross): As Minister for Trans- port, Tourism and Sport, I have responsibility for overall policy and funding in relation to the national roads programme. The implementation of individual national road schemes, including the N18 Limerick Tunnel is a matter for Transport Infrastructure Ireland (TII) (formerly known as the NRA) under the Roads Acts 1993-2015.

In addition, the statutory power to levy tolls on national roads, to make toll bye-laws and to enter into toll agremeents with private investors in respect of national roads is vested in TII under part V of the Roads Act (as amended).

The contracts for the privately operated toll schemes (such as the Limerick Tunnel) are com- mercial agreements between TII and the PPP concessionaires concerned. Noting this position, I have referred the Deputy’s question to TII for further reply. Please advise my office if you do not receive a reply within 10 working days.

21/06/2016WRZ02000Road Improvement Schemes

21/06/2016WRZ02100530. Deputy Danny Healy-Rae asked the Minister for Transport, Tourism and Sport to pro-

245 Questions - Written Answers vide funding for the national secondary road from Blackwater Bridge to Sneem as two medium or large vehicles cannot pass on many sections of this road; and if he will make a statement on the matter. [16767/16]

21/06/2016WRZ02200Minister for Transport, Tourism and Sport (Deputy Shane Ross): I understand the road in question is the N70 national road. As Minister for Transport, Tourism and Sport, I have responsibility for overall policy and funding in relation to the national roads programme. The planning, design and implementation of individual road projects (including the N70) is a mat- ter for the Transport Infrastructure Ireland (formerly known as the NRA) under the Roads Acts 1993-2015 in conjunction with the local authorities concerned.

Within its capital budget, the assessment and prioritisation of individual projects is a matter in the first instance for TII in accordance with Section 19 of the Roads Act.

Noting the above position, I have referred the Deputy’s question to TII for direct reply. I ask the Deputy to please advise my private office if he does not receive a reply within 10 working days.

21/06/2016WRZ02300Road Projects

21/06/2016WRZ02400531. Deputy Danny Healy-Rae asked the Minister for Transport, Tourism and Sport to provide proper funding for the Ring of Kerry road; and if he will make a statement on the mat- ter. [16769/16]

21/06/2016WRZ02500Minister for Transport, Tourism and Sport (Deputy Shane Ross): ​As Minister for Trans- port, Tourism & Sport, I have responsibility for overall policy and funding in relation to the national roads programme. The planning, design and implementation of individual national road projects in Kerry is a matter for the Transport Infrastructure Ireland (formerly known as the NRA) under the Roads Acts 1993-2015 in conjunction with Kerry County Council.

Within its capital budget, the assessment and prioritisation of individual projects is a matter in the first instance for TII in accordance with Section 19 of the Roads Act.

Noting the above position, I have referred the Deputy’s question to TII for direct reply. I ask the Deputy to please advise my private office if he does not receive a reply within 10 working days.

21/06/2016WRZ02600Tourism Project Funding

21/06/2016WRZ02700532. Deputy Pat Deering asked the Minister for Transport, Tourism and Sport the status of an application for funding for works at Altamont House and Gardens, in County Carlow, fol- lowing a submission made in February 2015 seeking funding for the conservation of the roof and restoration of tea rooms. [16770/16]

21/06/2016WRZ02800Minister of State at the Department of Transport, Tourism and Sport (Deputy Pat- rick O’Donovan): While my Department provides capital funding for investment in tourism initiatives to Failte Ireland it is not directly involved in the management of these funding pro- grammes. This is a matter for the Board and Management of Fáilte Ireland.

I have referred the Deputy’s question to Fáilte Ireland for consideration and direct reply. I ask the Deputy to please advise my private office if he does not receive a reply within ten work- ing days. 246 21 June 2016

21/06/2016WRZ02900Tourism Industry

21/06/2016WRZ03000533. Deputy Robert Troy asked the Minister for Transport, Tourism and Sport the an- nual spend for training in the hospitality sector since the Council for Education, Recruitment and Training was merged into Fáilte Ireland; and if he will make a statement on the matter. [16778/16]

21/06/2016WRZ03100Minister of State at the Department of Transport, Tourism and Sport (Deputy Pat- rick O’Donovan): The issue raised by the Deputy is an operational matter for the Board and Management of Fáilte Ireland. I have referred the Deputy’s question to Fáilte Ireland for direct reply. Please contact my private office if a response is not received within ten working days.

21/06/2016WRZ03200Road Projects

21/06/2016WRZ03300534. Deputy Imelda Munster asked the Minister for Transport, Tourism and Sport to con- firm that the funds promised in the capital spend programme 2015 for the urgently needed Laytown-Bettystown link road (R150) are guaranteed for the project; and when this funding will be released. [16780/16]

21/06/2016WRZ03400Minister for Transport, Tourism and Sport (Deputy Shane Ross): ​As the Deputy is aware, the improvement and maintenance of regional and local roads in Co. Meath is the statu- tory responsibility of Meath County Council, in accordance with the provisions of Section 13 of the Roads Act 1993. Works on those roads are funded from the Council’s own resources supplemented by State road grants.

The Capital Plan 2016-2021 provides that a number of schemes targeting bottlenecks in the road network will be progressed subject to necessary approvals. The R150 Laytown to Bettys- town Spine road is one of the projects included in the Capital Plan, subject to necessary approv- als including planning. I understand that Meath County Council is liaising with my officials in relation to the project, including the timescale for implementation.

21/06/2016WRZ03500Tourism Promotion

21/06/2016WRZ03600535. Deputy Éamon Ó Cuív asked the Minister for Transport, Tourism and Sport why only 130 post offices out of the 180 post offices along the Wild Atlantic Way received a franchise to sell Wild Atlantic Way passports and stamps; if he will extend the scheme to the other post offices along the Wild Atlantic Way; and if he will make a statement on the matter. [16781/16]

21/06/2016WRZ03700Minister of State at the Department of Transport, Tourism and Sport (Deputy Pat- rick O’Donovan): The matter raised by the Deputy is an operational matter for the Board and Management of Fáilte Ireland. I have referred the Deputy’s question to Fáilte Ireland for direct reply. Please contact my private office if a response is not received within ten working days.

21/06/2016WRZ03800Road Projects

21/06/2016WRZ03900536. Deputy Danny Healy-Rae asked the Minister for Transport, Tourism and Sport his plans to widen Carah Bridge, Ballinahulla Bridge and Kells Bridge at the viaduct on the Ring of Kerry Road (details suplied); and if he will make a statement on the matter. [16794/16]

21/06/2016WRZ04000Minister for Transport, Tourism and Sport (Deputy Shane Ross): As Minister for Trans- 247 Questions - Written Answers port, Tourism & Sport, I have responsibility for overall policy and funding in relation to the na- tional roads programme. The planning, design and implementation of individual national road projects is a matter for Transport Infrastructure Ireland (formerly known as the NRA) under the Roads Acts 1993-2015 in conjunction with the local authorities concerned.

Noting the above position, I have referred the Deputy’s question to TII for direct reply. Please advise my private office if a response is not received within 10 working days.

21/06/2016WRZ04100Traffic Management

21/06/2016WRZ04200537. Deputy Imelda Munster asked the Minister for Transport, Tourism and Sport his plans to restrict the access of higher goods vehicles and other large vehicles from St. Laurence’s Gate in Drogheda, County Louth, given that these vehicles are causing damage to the structure. [16834/16]

21/06/2016WRZ04300Minister for Transport, Tourism and Sport (Deputy Shane Ross): While the Depart- ment of Transport, Tourism and Sport (DTTAS) is responsible for general legislation in regard to traffic matters, DTTAS has no direct role in relation to local traffic. The implementation of local traffic management measures is a matter for the relevant local authority, which in this in- stance is Louth County Council. Concerns have already been brought to my attention by local representatives and I am aware of the issues involved.

21/06/2016WRZ04400Tourism Promotion

21/06/2016WRZ04500538. Deputy Imelda Munster asked the Minister for Transport, Tourism and Sport if he has considered the inclusion of St. Laurence’s Gate, Drogheda, County Louth in Ireland’s Ancient East; and if he will identify it as a gateway to Ireland’s Ancient East. [16836/16]

21/06/2016WRZ04600Minister of State at the Department of Transport, Tourism and Sport (Deputy Patrick O’Donovan): As Minister of State for Tourism and Sport, my responsibility in relation to tour- ism lies primarily in the area of national policy. The development of Ireland’s Ancient East is an operational matter for the Board and Management of Fáilte Ireland. My Department is not directly involved in its development or promotion.

Accordingly, I have asked Fáilte Ireland to reply to the Deputy directly. Please contact my private office if you do not hear from them within ten working days.

Questions Nos. 539 and 540 answered with Question No. 517.

21/06/2016WRZ04800Air Traffic Control Services

21/06/2016WRZ04900541. Deputy Robert Troy asked the Minister for Transport, Tourism and Sport his position on the Single European Sky, SES, initiative at the European Union; the level of engagement he has had with European officials and industry stakeholders on this issue; his views on the ob- stacles to the creation of a Single European Sky; and if he will make a statement on the matter. [16989/16]

21/06/2016WRZ05000Minister for Transport, Tourism and Sport (Deputy Shane Ross): ​The Single European Sky (SES) is a policy initiative agreed in 2004 with the central objective of reforming and im- proving the performance of air traffic management in Europe. This objective continues to be

248 21 June 2016 valid and I accordingly fully support the SES initiative.

While I have not yet had an opportunity to discuss this initiative with my Europe- an Union counterparts at Transport Council, or to engage with industry stakeholders, officials from my Department are engaged in SES-related discussions on an ongoing basis at EU level. These discussions are aimed at progressing various technical elements of the SES and are de- livering positive results.

Insofar as obstacles to the creation of a Single European Sky are concerned, I do not believe that there is a particular obstacle per se. The SES is a highly complex project that will take time to fully deliver. It is important to emphasise that considerable progress has already been achieved over the past decade or so, particularly in relation to the establishment of Functional Airspace Blocks and also the agreement reached between all Member States on the develop- ment of a performance scheme. This multi-annual scheme requires the achievement of de- manding performance targets in relation to safety, capacity, environment and cost efficiency, which are delivering improvements. While further steps clearly need to be taken to overcome the inefficiencies associated with fragmented European airspace, I believe that with persistence and commitment by all parties the key goals of the Single European Sky will be progressively achieved over the coming years.

21/06/2016WRZ05100Sports Events

21/06/2016WRZ05200542. Deputy Clare Daly asked the Minister for Transport, Tourism and Sport to outline the remit of the National Horse Arena in Abbotstown and to whom it is accountable, with particular reference to where income from the equestrian activities that it hosts goes. [16992/16]

21/06/2016WRZ05300543. Deputy Clare Daly asked the Minister for Transport, Tourism and Sport if he is aware that activities and shows being run by the National Horse Arena are impacting upon local busi- nesses which traditionally provided Dressage Ireland shows, and other riding club qualifiers, with particular reference to the fact that such activities are being organised with the support of taxpayers’ money, which went into establishing the facility, thus giving an unfair advantage. [16993/16]

21/06/2016WRZ05400Minister of State at the Department of Transport, Tourism and Sport (Deputy Patrick O’Donovan): I propose to take Questions Nos. 542 and 543 together.

Sport Ireland, which is funded by my Department, is the statutory body with responsibil- ity for the development and operation of the National Sports Campus at Abbotstown. This includes responsibility for the operation of facilities such as the National Horse Sport Arena.

As this question is a matter for Sport Ireland, I have referred it to Sport Ireland for direct reply. I would ask the Deputy to inform my office if a reply is not received within 10 days.

21/06/2016WRZ05500Sports Capital Programme Applications

21/06/2016WRZ05600544. Deputy Michael Healy-Rae asked the Minister for Transport, Tourism and Sport when he will accept applications for sports capital grants; the opening date and the deadline; and if he will make a statement on the matter. [17082/16]

21/06/2016WRZ05700548. Deputy Brendan Smith asked the Minister for Transport, Tourism and Sport his pro- posals to launch a new sports capital programme 2016, if he has provided for this in his Esti- mates for 2016; and if he will make a statement on the matter. [17240/16] 249 Questions - Written Answers

21/06/2016WRZ05800549. Deputy Pat Deering asked the Minister for Transport, Tourism and Sport his plans for sports capital funding in 2016; when he will open the expression of interest period for this; and if he will make a statement on the matter. [17243/16]

21/06/2016WRZ05900Minister of State at the Department of Transport, Tourism and Sport (Deputy Patrick O’Donovan): I propose to take Questions Nos. 544, 548 and 549 together.

No decision has been made on the timing or any other aspects of any future rounds of the Sports Capital Programme.

No provision was made in the Estimates for a new round of allocations in 2016. In any case, allocations are not generally paid in the year in which they are allocated.

Any interested group should register online now at www.sportscapitalprogramme.ie as this portal will be the only way to apply under any future rounds of the Programme.

21/06/2016WRAA00200Penalty Points System

21/06/2016WRAA00300545. Deputy Seán Haughey asked the Minister for Transport, Tourism and Sport his plans to reform the legislation which allows for the imposition of penalty points; his views on wheth- er it is reasonable to double the penalty points for a motorist who loses an appeal to the courts; if he will introduce a system whereby both fine and penalty points are relinquished in the case of motorists who undertake a driver awareness course, as is the case in the UK and Northern Ireland; and if he will make a statement on the matter. [17164/16]

21/06/2016WRAA00400Minister for Transport, Tourism and Sport (Deputy Shane Ross): ​One of the main fea- tures of the penalty points and fixed charge system is the opportunity for a person to pay a charge where they have been detected committing certain offences under the Road Traffic Acts. The payment of the charge is an administrative provision that acts as an alternative to going to court.

When a driver pays the charge, they are accepting responsibility for the commission of the offence, and are allocated a reduced number of penalty points. The purpose of this is to avoid court time being taken up unnecessarily due to cases where there is no disputing the guilt of the individual in question. This frees up the courts to deal with more serious matters, as well as reducing the administrative and time burden on members of the Garda Síochána in prosecuting such cases. Where people feel that they did not commit the offence, they can go to court.

In relation to your query on a driver awareness course, the Road Safety Strategy 2013 – 2020 includes an action to “legislate for, subject to legal advice, and implement rehabilitation and driving awareness courses as court-based sentencing options for repeat offenders”.

While the Road Safety Authority recently submitted a position paper in relation to this ac- tion, it is being recalled by the Authority for some further work. I look forward to the results of its work.

21/06/2016WRAA00500Olympic Games Qualification

21/06/2016WRAA00600546. Deputy Róisín Shortall asked the Minister for Transport, Tourism and Sport if he is aware of the case of a person (details supplied) who has been excluded from the Irish Olympic team; and if he will make a statement on the matter. [17180/16]

250 21 June 2016

21/06/2016WRAA00700Minister of State at the Department of Transport, Tourism and Sport (Deputy Patrick O’Donovan): ​The selection of team members to represent Ireland in athletics events at the Olympic Games is a matter for Athletics Ireland as the National Governing Body for Athletics. All National Governing Bodies of Sport are independent, autonomous bodies and are respon- sible for their own governance procedures and competition rules for their sports. I have no role in relation to team selection by Athletics Ireland or any of the sporting bodies and it would be inappropriate for me to intervene in this matter.

21/06/2016WRAA00800Road Traffic Legislation

21/06/2016WRAA00900547. Deputy Charlie McConalogue asked the Minister for Transport, Tourism and Sport if a person (details supplied) in County Donegal can obtain an exemption from section 87 of the Road Traffic Act in order to complete level 3 of the emergency services driving standard train- ing course; and if he will make a statement on the matter. [17208/16]

21/06/2016WRAA01000Minister for Transport, Tourism and Sport (Deputy Shane Ross): ​I note that the Deputy asked my predecessor about this case on 26 January last. The facts of the matter have not changed since.

Section 87 of the Road Traffic Act 2010, as amended, provides exemptions from certain provisions of the Road Traffic Acts for members of the emergency services, including An Garda Síochána, in the course of their duties. I have no proposals to extend the exemptions already provided to any organisation or individual outside of those already provided for.

Questions Nos. 548 and 549 answered with Question No. 544.

21/06/2016WRAA01200Bus Services

21/06/2016WRAA01300550. Deputy Alan Farrell asked the Minister for Transport, Tourism and Sport if he will engage with Dublin Bus and Fingal County Council to ensure residents in Knocksedan estate in Swords, County Dublin, are provided with an adequate bus service to meet their needs; the action he will take in this regard; and if he will make a statement on the matter. [17250/16]

21/06/2016WRAA01400Minister for Transport, Tourism and Sport (Deputy Shane Ross): ​The issue raised is a matter for the National Transport Authority (NTA) and I have forwarded the Deputy’s question to the NTA for direct reply. Please advise my private office if you do not receive a response within ten working days.

21/06/2016WRAA01500Employment Rights

21/06/2016WRAA01600551. Deputy Michael Lowry asked the Minister for Jobs, Enterprise and Innovation the current status of au pairs here; their responsibilities associated with this work; the full suite of employment legislation including minimum wage legislation that applies; the duties and responsibilities of families who hire au pairs; her plans for future development and protection; and if she will make a statement on the matter. [16797/16]

21/06/2016WRAA01700Minister for Jobs, Enterprise and Innovation (Deputy Mary Mitchell O’Connor): There is no separate legal definition of the term “au pair” in Irish legislation, and individuals described as “au pairs”, “nannies” or “child-minders” are not exempted or treated as separate categories of workers under Irish employment law. Ireland’s body of employment rights legis- 251 Questions - Written Answers lation protects all employees who are legally employed on an employer-employee basis, regard- less of what title is given to them. Therefore, once it is clear that a person is working under a contract of employment (written or verbal), on a full-time or part-time basis, that person has the same protection under employment law as other employees, including entitlement to the national minimum wage.

All employers, including those in private homes, carry the same obligations in relation to compliance with employment law. Where the Workplace Relations Commission (WRC), which is responsible for securing compliance with employment legislation, receives a com- plaint involving somebody described as an au pair, the WRC will investigate with a view to establishing whether a person has statutory entitlements under employment law. Complaints involving au pairs are considered on a case-by-case basis, in the light of the facts of each case.

The recent WRC decision regarding an individual working as an Au Pair does not constitute any change to existing employment law and any persons working under a contract of employ- ment must continue to have the full protection of employment law. I have no plans to review the status of au pairs with a view to deeming persons determined to be employees to be outside the scope of employment law. My Department’s remit in this area is primarily concerned with ensuring that those people found to be employees can enforce their rights, as provided for under employment rights legislation.

21/06/2016WRAA01800EU Directives

21/06/2016WRAA01900552. Deputy Alan Kelly asked the Minister for Jobs, Enterprise and Innovation if she in- tends to transpose the European Union audit directive by the deadline of 17 June 2016; and the practical consequences of not having it transposed by the deadline. [16853/16]

21/06/2016WRAA02000Minister for Jobs, Enterprise and Innovation (Deputy Mary Mitchell O’Connor): I signed the European Union (Statutory Audits) (Directive 2006/43/EC, as amended by Directive 2014/56/EU, and Regulation (EU) No 537/2014) Regulations 2016 (S. I. No. 312 of 2016) on 15 June 2016 and they took effect on 17 June 2016. This Statutory Instrument gives effect to Ireland’s obligations under the EU audit reform package, comprising Directive (2014/56/EU) which amends Directive 2006/43/EC and a Regulation (EU) No 537/2014. Accordingly, the EU transposition deadline has been met.

The Directive applies to all statutory audits while the Regulation adds more stringent rules for the audits of entities such as financial and credit institutions, insurers, funds and listed companies, known as ‘public interest entities’. Both the Directive and the Regulation contain options for Member States to consider.

Had Ireland not transposed the EU reforms on time, there could have followed a number of negative consequences. Those include the fact that the Irish Auditing and Accounting Supervi- sory Authority (IAASA) would not have been designated as the competent authority and would not have had the appropriate powers to implement and enforce the new enhanced oversight regime for audit. New rules on ethics and audit reporting for auditors would not have the nec- essary legislative underpinning. Auditors and other stakeholders would not have clarity as to how Ireland intended to exercise the options in the EU Directive and Regulation. National law also gives clarity on how to comply with some of the provisions in the EU Regulation, which is directly applicable in Ireland. Furthermore, Ireland could have faced infringement proceedings by the European Commission and fines.

There were some options in the EU Directive and Regulation that were not essential to

252 21 June 2016 transposition and could not be accommodated in the Statutory Instrument. As some of these are considered to be beneficial, it is my intention to introduce a Statutory Audits Bill in the Oireachtas in due course to avail of those options.

21/06/2016WRAA02100Job Creation

21/06/2016WRAA02200553. Deputy Maurice Quinlivan asked the Minister for Jobs, Enterprise and Innovation her strategy for job creation on the Inishowen Peninsula in County Donegal; how Inishowen can benefit from the designation of Letterkenny as a gateway town and from the Letterkenny- Derry Gateway initiative; and if she will make a statement on the matter. [16854/16]

21/06/2016WRAA02300554. Deputy Maurice Quinlivan asked the Minister for Jobs, Enterprise and Innovation the number of visits to County Donegal by potential investors organised by the Industrial De- velopment Agency Ireland in each of the years 2009 to 2016 to date. [16855/16]

21/06/2016WRAA02400556. Deputy Maurice Quinlivan asked the Minister for Jobs, Enterprise and Innovation her role in working in partnership with her Northern counterpart in marketing the north west cross-Border region of counties Donegal, Sligo, Leitrim, Derry, Tyrone and Fermanagh for in- ward investment and job creation. [16857/16]

21/06/2016WRAA02450Minister for Jobs, Enterprise and Innovation (Deputy Mary Mitchell O’Connor): I propose to take Questions Nos. 553, 554 and 556 together.

Official employment figures maintained by the Central Statistics Office show that the border region, including Donegal, has seen an increase in employment of 8,300 from the start of 2011 to end of quarter 1 2016. Donegal has performed strongly as well in terms of IDA Ireland- supported inward investment since 2011. The following table sets out IDA Ireland-sponsored site visits to Donegal since 2009.

The concrete evidence of winning further investment for the county can be seen in the Agency’s results. In the past five years, employment in IDA Ireland client companies there has grown to 2,530 from a base of 1,877 – an increase of 35%. In addition, there are now 3,567 people directly employed in Enterprise Ireland client companies in Donegal.

This all shows that significant strides have already been made when it comes to- secur ing new employment and investment projects for Donegal and the region. The Government remains focused, however, on uncovering new opportunities for the Inishowen peninsula, the county and the wider area in the time ahead.

The North East/North West Action Plan for Jobs, launched last November, aims to deliver 28,000 extra jobs in the counties of Sligo, Leitrim, Donegal, Louth, Monaghan, and Cavan by 2020. Key targets in service of this overall goal are to achieve an increase of at least 25% in the number of start-ups in the region; a 25% improvement in the survival rate of new businesses; an increase in the number of IDA investments in the region by 30-40% up to 2019; and an increase of 300 to 500 in the numbers of Údarás na Gaeltachta-supported jobs in the North West.

My Department works closely with its counterpart in Northern Ireland on enterprise devel- opment issues which are mutually beneficial. We also strongly support the work of InterTrade Ireland, the North-South Body, which operates a range of business development activities and is co-funded by both administrations. In addition, our two Departments co-fund a strand of the EU’s INTERREG Programme, which supports a range of research and innovation initiatives. I look forward to meeting my counterpart, Minister Simon Hamilton MLA, at a meeting of the North-South Ministerial Council in due course. 253 Questions - Written Answers Last November, the Minister for Foreign Affairs and Trade, Charlie Flanagan T.D, an- nounced the establishment of a North West Development Fund to support the North West Gate- way Initiative work of Donegal County Council and Derry City and Strabane District Council. Through the Department of Foreign Affairs and Trade, the Government has allocated €2.5 mil- lion towards the establishment of the Fund. The Northern Ireland Executive is working on the modalities of its matching funding.

IDA Ireland-sponsored Site Visits to Donegal 2009 - 2015, and Q1 2016

County 2009 2010 2011 2012 2013 2014 2015 Q1 2016 Donegal 3 4 2 1 7 6 5 1

21/06/2016WRAA02600IDA Site Visits

21/06/2016WRAA02700555. Deputy Maurice Quinlivan asked the Minister for Jobs, Enterprise and Innovation the number of visits organised by the Industrial Development Agency Ireland for potential in- vestors, by county, in each of the years 2009 to 2016 to date. [16856/16]

21/06/2016WRAA02800Minister for Jobs, Enterprise and Innovation (Deputy Mary Mitchell O’Connor): IDA Ireland maintains statistics of site visits by potential investors on a county-by-county basis only. From 2009 to the end of quarter one in 2016, there were a total of 2,777 IDA Ireland sponsored site visits by potential investors to various locations throughout the country. The following sets out, on a county-by-county basis, the number of IDA Ireland sponsored site visits during this period.

The Table shows the number of IDA Ireland sponsored site visits by potential investors by county from 2009 to the end of quarter one in 2016.

County 2009 2010 2011 2012 2013 2014 2015 2016Q1 Carlow 1 3 2 4 1 2 1 0 Cavan 0 3 0 3 2 1 0 0 Clare 9 7 15 14 5 9 12 3 Cork 29 44 27 38 31 30 48 11 Donegal 3 4 2 1 7 6 5 1 Dublin 90 197 150 196 180 205 242 57 Galway 22 41 35 18 15 19 41 10 Kerry 3 2 2 1 1 3 6 1 Kildare 1 2 3 1 1 1 7 5 Kilkenny 2 0 0 3 3 4 10 4 Laois 1 0 2 0 2 0 4 0 Leitrim 0 0 0 0 1 2 8 1 Limerick 18 38 40 30 23 22 40 9 Longford 0 0 0 0 0 0 2 1 Louth 28 25 26 12 4 10 20 5 Mayo 1 1 0 1 3 4 3 2 Meath 2 0 2 0 1 2 7 2 Monaghan 0 0 1 0 1 0 2 0 Offaly 6 7 1 3 1 1 8 1 Roscom- 0 0 0 0 4 0 2 0 mon

254 21 June 2016

County 2009 2010 2011 2012 2013 2014 2015 2016Q1 Sligo 5 12 3 6 10 7 15 4 Tipperary 1 1 1 5 4 3 12 4 Waterford 8 11 11 26 14 11 31 4 Westmeath 14 22 15 7 9 12 28 9 Wexford 0 1 0 3 2 1 4 1 Wicklow 2 5 3 6 1 4 7 1 TOTAL 246 426 341 378 326 359 565 136

Question No. 556 answered with Question No. 553.

21/06/2016WRAA03000EU Directives

21/06/2016WRAA03100557. Deputy Clare Daly asked the Minister for Jobs, Enterprise and Innovation the steps she will take to implement the European Union Directive 97/81/EC to protect part-time workers here. [16877/16]

21/06/2016WRAA03200Minister for Jobs, Enterprise and Innovation (Deputy Mary Mitchell O’Connor): The Protection of Employees (Part-Time Work) Act 2001 (the 2001 Act) implemented EU Council Directive 97/81/EC into Irish law. The purpose of the Directive was to implement the Frame- work Agreement on part-time work concluded by the European cross-industry organisations UNICE, CEEP and the ETUC.

The purpose of the Framework Agreement, which was annexed to the Directive, was to eliminate discrimination against part-time employees and to improve the quality of part-time work. It also aimed to facilitate the development of part-time work on a voluntary basis and to contribute to the flexible organisation of working time in a manner which takes into account the needs of employers and employees. Accordingly, the 2001 Act provides a wide degree of protection for part-time employees, including the general protection that a part-time employee shall not be treated in a less favourable manner in respect of his/her conditions of employment than a full time employee.

In addition, Section 13(5) of the 2001 Act contained provisions in relation to the preparation and publishing of a Code of Practice by the Labour Relations Commission in relation to the steps that could be taken by employers for the purposes of Clause 5.3 of the Framework Agree- ment. Clause 5.3 of the Framework Agreement provided that, as far as possible, an employer should give consideration to a request by workers to transfer from full-time work to part-time and vice-versa. The Labour Relations Commission prepared this Code of Practice, based on the provisions in Section 13 of the 2001 Act, following consultation with the social partners. It was deemed to be a Code of Practice and implemented, in 2006, by the Industrial Relations Act 1990 (Code of Practice on Access to Part-Time Working) (Declaration) Order 2006 (S.I. No. 8 of 2006). Under Section 42(4) of the Industrial Relations Act 1990, the Code is admissible in evidence in any proceedings before a Court, the Labour Court or an Adjudication Officer of the Workplace Relations Commission. Since 1st October 2015, the Workplace Relations Commis- sion has taken over the role and functions of the Labour Relations Commission (including the Rights Commissioners’ Service of the Labour Relations Commission) as well as the functions of the Equality Tribunal and the National Employment Rights Authority (NERA) and the first- instance (complaints and referrals) functions of the Employment Appeals Tribunal.

I understand that IBEC and ICTU were involved in the process of drafting the Directive’s

255 Questions - Written Answers Framework Agreement at European level through the European social partner organisations. At national level, a tripartite group involving this Department, ICTU, IBEC and other relevant Government Departments was established to consider the measures necessary for implement- ing the Directive in Ireland prior to publication of the Protection of Employees (Part-Time Work) Bill 2000.

I am satisfied that Ireland has met its responsibilities in transposing EU Council Directive 97/81/EC into national law. This position is confirmed by a 2003 report on the implementation of the Directive undertaken by the European Commission.

21/06/2016WRAA03300Industrial Disputes

21/06/2016WRAA03400558. Deputy Bríd Smith asked the Minister for Jobs, Enterprise and Innovation if he is aware of the actions of management at an organisation (details supplied). [16977/16]

21/06/2016WRAA03500559. Deputy Bríd Smith asked the Minister for Jobs, Enterprise and Innovation if he will intervene in the ongoing strike at an organisation (details supplied) to make clear that this Gov- ernment is opposed to employers forcibly making staff redundant in the selective manner that management has tried to do here and that Government funding will not be dispensed to employ- ers who treat their workers in this manner. [16978/16]

21/06/2016WRAA03600560. Deputy Bríd Smith asked the Minister for Jobs, Enterprise and Innovation if she aware of the actions of an organisation (details supplied). [17167/16]

21/06/2016WRAA03700Minister for Jobs, Enterprise and Innovation (Deputy Mary Mitchell O’Connor): I propose to take Questions Nos. 558 to 560, inclusive, together.

There is a robust suite of employment rights legislation in place providing protection for employees, including legislation governing working time and pay. This includes the Redun- dancy Payments Acts 1967 to 2014, the National Minimum Wage Act 2000, the Payment of Wages Act 1991, the Organisation of Working Time Act 1997, the Minimum Notice and Terms of Employment Acts 1973 to 2001, the Protection of Employees (Part-Time Work) Act 2001, the Protection of Employees (Fixed-Term Work) Act 2003 and the Protection of Employees (Temporary Agency Work) Act 2012.

I understand the dispute at issue relates to the proposed redundancy of a number of staff members due to restructuring of the organisation. I would like to assure the Deputy that the State’s industrial relations machinery continues to be available to assist in the resolution of this dispute and I would urge the parties to engage with them in order to resolve their differences in this instance. Even what often appears to be the most intractable of disputes is capable of resolution where both sides engage constructively and in good faith in this voluntary process. The principle of good faith implies that both sides in a dispute make every effort to reach an agreement and endeavour, through genuine and constructive negotiations, to arrive at a conclu- sion that is satisfactory to all concerned.

Ireland’s system of industrial relations is, essentially, voluntary in nature and responsibility for the resolution of industrial disputes between employers and workers, whether in redundancy or other collective disputes, rests with the employer, the workers and their representatives. The State provides the industrial relations dispute settlement mechanisms to support parties in their efforts to resolve their differences.

21/06/2016WRAA03800Industrial Disputes 256 21 June 2016

21/06/2016WRAA03900561. Deputy Maurice Quinlivan asked the Minister for Jobs, Enterprise and Innovation if she is aware of the dispute at the Garda Síochána Training College in Templemore, County Tipperary, where contract cleaners have been brought in without consultation with the Man- date trade union which represents permanent workers employed by the Office of Public Works and without the latter being given the opportunity to work overtime; that numerous health and safety concerns have been raised with management to no avail; and if she will make a statement on the matter. [17249/16]

21/06/2016WRAA04000Minister of State at the Department of Jobs, Enterprise and Innovation (Deputy Pat Breen): Issues relating to employees of the Office of Public Works and their terms and condi- tions of employment are a matter for my colleague, Pascal Donohoe TD, Minister for Public Expenditure and Reform. Issues relating to the tendering of public contracts are also outside my remit and should be addressed to the Office of Public Procurement which operates as an office of the Department of Public Expenditure and Reform.

The Lansdowne Road Agreement provides for issues that may arise around out sourcing. That agreement provides (par 3.3.5) that where any dispute arises the parties shall seek to re- solve any matter through direct dialogue and if this fails the dispute resolution mechanisms set out in paragraph 4 and 6 of the Agreement will be used.

As regards any health and safety concerns in a workplace, a complaint may be made to the HSA Workplace Contact Unit, Health and Safety Authority, Metropolitan Building, James Joyce Street, Dublin 1 (email: [email protected], Telephone: 1890 289 389).

I would urge any parties to a dispute to avail of the dispute resolution mechanisms available to them.

21/06/2016WRAA04100Departmental Funding

21/06/2016WRAA04200562. Deputy John Lahart asked the Minister for Children and Youth Affairs if she will restore funding to the Women’s Aid organisation and to the extension of its national free- fone helpline to a 24 hour, seven day service; and if she will make a statement on the matter. [17034/16]

21/06/2016WRAA04300Minister for Children and Youth Affairs (Deputy Katherine Zappone): Tusla, the Child and Family Agency, is in receipt of funding of €20.6m in 2016, to support for frontline Domes- tic, Sexual and Gender Based services. This includes additional funding of €200,000 to sup- port Tusla in implementing the provisions of the Council of Europe Convention on Preventing and Combating Violence against Women and Domestic Violence (the Istanbul Convention). The overall level of funding provided to Tusla is supporting some 60 services throughout the country:

- 20 Crisis Refuges and support services,

- 16 Rape Crisis Centres, and

- 24 Domestic Violence Support Services.

There is an obligation under the Istanbul Convention to have dedicated free 24 hour national helplines in place for contact around issues of domestic violence and sexual violence. Helpline provision will be a specific theme within Tusla’s commissioning processes into the future.

Tusla recognises the substantive work undertaken by Women’s Aid in the current provision

257 Questions - Written Answers of national helpline services for domestic violence and also the many other specialist domestic violence service provider organisations that operate helpline contact.

The Agency is working with these organisations and other stakeholders as part of its helpline commissioning process. The intention is to support the availability of co-ordinated, accessible national helpline services that can provide both initial contact points and facilitate integrated responses to victims of domestic violence. The issue of costs as well as service specifications, will be considered as part of the helpline commissioning process.

Tusla is currently engaged with Women’s Aid around service and funding arrangements for 2016, including provision of its helpline services.

At all times the Agency’s key priority will be to ensure that the needs of victims and survi- vors of domestic, sexual and gender based violence are being met in the best way possible and to address issues of equity in access to and outcomes from services.

Domestic Violence Services Funding

21/06/2016WRAA04500563. Deputy John Lahart asked the Minister for Children and Youth Affairs if she will re- store and increase funding to domestic violence services so they can meet demand and increase refuge provision to meet international standards; if she will provide dedicated funding for train- ing of relevant State agencies; and if she will make a statement on the matter. [17035/16]

21/06/2016WRAA04600Minister for Children and Youth Affairs (Deputy Katherine Zappone): The informa- tion sought by the Deputy has been requested from the Child and Family Agency. I will re- spond directly to the Deputy when the information is received.

21/06/2016WRAA04700Child and Family Agency Services

21/06/2016WRAA04800564. Deputy John Lahart asked the Minister for Children and Youth Affairs her plans to increase the number of women’s refuges of which Ireland has 21, with places for 141 families, which represents only 31% of what is needed; and if she will make a statement on the matter. [17039/16]

21/06/2016WRAA04900Minister for Children and Youth Affairs (Deputy Katherine Zappone): The informa- tion sought by the Deputy has been requested from the Child and Family Agency. I will re- spond directly to the Deputy when the information is received.

21/06/2016WRAA05000Foster Care Policy

21/06/2016WRAA05100565. Deputy Mattie McGrath asked the Minister for Children and Youth Affairs to direct Tusla, the Child and Family Agency, to review the application of its policy which states there must be no more than 40 years in age between a foster parent and foster child; the number of foster children in placements where this age gap is exceeded; and if she will make a statement on the matter. [16695/16]

21/06/2016WRAA05200Minister for Children and Youth Affairs (Deputy Katherine Zappone): The age of the carers is never the sole reason for deciding on the suitability of a placement for a child in State care. It must be emphasised that the 40 year age difference between a foster carer, or relative foster carer, and foster child is only a guideline, not a policy, and does not determine a decision

258 21 June 2016 regarding suitability. It is more important that the relative carer has the ability to meet the needs of the particular child or sibling group. The potential carer’s age is just one of many factors that is taken into account, including health, level of co-operation and communication with social service and the ability to meet all the needs, including any specialised need, of the child.

Placement endings are made following careful consideration of the best interest of the child and, where appropriate, oversight by the courts. The Child and Family Agency, Tusla, does not collect data at a national level on the ages of relative foster carers and the children placed with them. At the end of April 2016, 28% of all children in care were in a foster care placement with relatives, many of whom are grandparents.

Tusla is currently in the process of planning a review of the criteria used by foster care com- mittees in decision-making.

21/06/2016WRBB00150Domestic Violence Refuges Provision

21/06/2016WRBB00200566. Deputy Eoin Ó Broin asked the Minister for Children and Youth Affairs how much it costs per year and per unit, for each of the years from 2012 to 2016 to date, to fund the 141 family accommodation units available in refuges, for those experiencing domestic violence, in tabular form. [17021/16]

21/06/2016WRBB00300Minister for Children and Youth Affairs (Deputy Katherine Zappone): The informa- tion sought by the Deputy has been requested from the Child and Family Agency. I will re- spond directly to the Deputy when the information is received.

21/06/2016WRBB00350Child Care Services Provision

21/06/2016WRBB00400567. Deputy James Browne asked the Minister for Children and Youth Affairs the schemes she provides for subsidised child care; and if she will make a statement on the matter. [17074/16]

21/06/2016WRBB00500Minister for Children and Youth Affairs (Deputy Katherine Zappone): My Depart- ment administers a number of childcare support programmes. These are the Early Childhood Care and Education (ECCE) programme, the Community Childcare Subvention (CCS) pro- gramme, the Community Childcare Subvention Private (CCSP) Programme and the Training and Employment Childcare (TEC) programmes. These programmes primarily provide support for childcare and early years education in formal settings such as preschools and creches, some of which are run by community/not-for profit organisations, and some by private providers.

With a budget of €233m for 2016, the largest scheme is the Early Childhood Care and Edu- cation (ECCE) programme, which from September 2016, will make free pre-school available for 15 hours per week for 38 weeks per annum to all children from the time they turn three, until they go to school, entering at three points in the year: September, January and April. On average, from September 2016, children will benefit from 61 weeks, up from the current provi- sion of 38 weeks. Access to the average 61 weeks of ECCE can reduce the cost of childcare by €4000 per child. The current ECCE programme is accommodating 67,000 children; this figure will increase to 127,000 next year.

The Community Childcare Subvention (CCS) programme provides funding to childcare services to enable them to provide quality childcare, including after-school care, at reduced rates to disadvantaged and low income working parents. Parents qualify as disadvantaged or low income on the basis of means-tested entitlements. With a budget of €45m it is anticipated

259 Questions - Written Answers that over 25,000 children will benefit from this programme this year in over 900 community (not for profit) childcare services. In the case of full day care, parents qualifying for the higher rate of subvention under the CCS programme can have up to €95 per week deducted from the overall charge for childcare in the participating childcare facility.

As part of Budget 2016, the Department announced an expansion of the CCS programme to allow private services to offer the programme. This expansion provided for a further 3,200 (full-time equivalent) childcare places on the programme, or approximately 8,000 children based on average uptake. The expansion also saw the inclusion of private childcare providers in the CCS Programme for the first time. The Community Childcare Subvention Private (CCSP) Programme launched in March 2016. The 2016 budget for CCSP programme is €16m.

The following three programmes ASCC, CEC and CETS are known collectively as the Training and Employment Childcare (‘TEC’) programmes.

My Department administers two programmes, ASCC and CEC, on behalf of the Depart- ment of Social Protection. ASCC provides subsidised childcare to support low-income and unemployed people who have primary school children in returning to work or to increase their days of employment. CEC supports people engaged in Community Employment Schemes who have preschool children and/or primary school children. Eligibility for these programmes is the responsibility of the Department of Social Protection.

- The After-School Childcare (ASCC) programme has a budget of €1.3m and has capacity to provide 300 whole-time equivalent after-school places annually. Each eligible parent is given a maximum allowance of 52 weeks of ASCC which does not have to be used consecutively, but when it is exhausted, the parent’s eligibility ceases. ASCC childcare places are subject to availability and are allocated on a first-come first served basis. The programme pays childcare services €40 per week for an after-school place or €80 per week in situations where the child- care service provides a pick-up service that collects or brings the child to and from school. The programme also provides a full day care rate of €105 per week, for a maximum of 10 weeks, to cater for school holiday periods. In all cases, the maximum fee payable by parents is €15 per week per child.

- The Community Employment Childcare (CEC) programme, with a budget of €6.7m, has capacity to provide 1,200 preschool places and 800 afterschool places. It is targeted specifi- cally at participants in the Community Employment (CE) schemes. Each eligible parent can avail of a maximum of 50 weeks childcare per academic year. Eligible parents must reapply for childcare each September. CEC childcare places are subject to availability and are allocated on a first-come first served basis. Under this programme, childcare services are paid €80 per week for pre-school places for children up to the age of 5 and €40 per week for after-school places for primary school children up to the age of 13, with a set charge of €15 per week to the parent in either case. The programme also provides a part time day care rate of €80 per week, for a maximum of 10 weeks, to cater for school holiday periods.

My Department administers the Childcare Education and Training Support programme on behalf of the Department of Education and Skills. With a budget of €17m, the Childcare Edu- cation and Training Support (CETS) programme provides 2,500 childcare places, including part-time and after-school places, to qualifying Solas or Education and Training Boards (ETB) trainees or students for the duration of their courses. Eligibility for the CETS programme is determined by the Department of Education and Skills.

In late 2017 my Department plans to replace ASCC, CEC, CETS and CCS/CCSP; with a single and simplified programme that will be available through community (not for profit) and private childcare providers. This programme, when developed, will provide an infrastructure 260 21 June 2016 through which any further investment in the subsidisation of childcare can be delivered.

21/06/2016WRBB00550Adoption Data

21/06/2016WRBB00600568. Deputy Sean Fleming asked the Minister for Children and Youth Affairs the number of adoptions into Ireland from China, for each of the past ten years; when adoptions will re- sume; and if she will make a statement on the matter. [17309/16]

21/06/2016WRBB00700Minister for Children and Youth Affairs (Deputy Katherine Zappone): The Adoption Authority of Ireland has provided details of intercountry adoptions from China 2006 to 2015 as follows:

Number of adoptions effected in China between 2006 and 2015 where the adopters were ha- bitually resident in Ireland and held a valid Declaration of Eligibility and Suitability to Adopt

Year Nos. 2006 33 2007 31 2008 19 2009 10 2010 9 2011 6 2012 1 2013 3 2014 1 2015 15 Total 128 Helping Hands Adoption Mediation Agency was accredited on 16/4/2014 to facilitate adop- tions from China under the Chinese Special Needs programme and adoptions from China are ongoing.

21/06/2016WRBB00750Defence Forces Personnel Data

21/06/2016WRBB00800569. Deputy Aengus Ó Snodaigh asked the Taoiseach and Minister for Defence why he has not published the Dunbar report into suicide in the Defence Forces; and if he will issue a copy to me. [16670/16]

21/06/2016WRBB00900Taoiseach and Minister for Defence (Deputy Enda Kenny): In August 1999 the Minis- ter for Defence established an inter-Departmental Group under the Chairmanship of Mr. Liam Dunbar to conduct a review of the provision of medical services to members of the Defence Forces. The Group carried out a detailed study of the role and operations of the Medical Corps. The Group completed its report in June 2000. The report did not relate to suicide in the Defence Forces.

I trust that this clarifies the matter for you.

21/06/2016WRBB00950Defence Forces Operations 261 Questions - Written Answers

21/06/2016WRBB01000570. Deputy Bríd Smith asked the Taoiseach and Minister for Defence if he shares the sen- timents of his counterpart in Finland, the Finnish Defence Minister, Jussi Niinistö, who recently stated that he supports reform of Finnish law to provide a legal mandate to expand the Finnish Defence Forces ability to participate in European wide international crisis management; and if those who opposed Ireland’s participation in the United Kingdom led battle group which also had Finnish participation would be concerned with this statement in view of general concerns regarding the use and purpose of such battle groups. [16782/16]

21/06/2016WRBB01100Taoiseach and Minister for Defence (Deputy Enda Kenny): I am given to understand that the changes proposed relate solely to matters where bi-lateral assistance is requested of Finland or where Finland may wish to offer such bi-lateral assistance in specific circumstances. The intention is to expand the Finnish Defence Forces’ legal mandate to include providing and accepting international assistance.

As the Deputy will appreciate it is a matter for individual States including Finland and Ire- land to determine their particular Security and Defence policies and arrangements. Ireland’s Defence Policy is clearly set out in the White Paper on Defence which was published last July. At this time there are no plans or identified requirements for legislative change to give effect to Ireland’s international security and defence policy as provided for in the White Paper. In addi- tion, developments in relation to Finland’s security and defence policy have no impact on our participation in EU Battlegroups, nor on the role or operational deployment of EU Battlegroups.

21/06/2016WRBB01150Defence Forces Operations

21/06/2016WRBB01200571. Deputy Bríd Smith asked the Taoiseach and Minister for Defence if he shares the views of his predecessor, who stated he wanted Ireland to become a testing zone for advanced military and weapons guidance systems (details supplied); and if he will assure Dáil Éireann that he does not wish Ireland to become a haven for firms trading or researching such weapons of mass destruction. [16783/16]

21/06/2016WRBB01300Taoiseach and Minister for Defence (Deputy Enda Kenny): I support my predecessor’s view that there is an ongoing requirement to examine new and innovative means of improving capabilities in the security and the defence domain, so that the Defence Forces are in a position to undertake the roles assigned by Government.

The Government approved arrangements in July 2011, whereby Enterprise Ireland (EI) would support the Department of Defence and Defence Forces capability development through engagement with Irish-based enterprise and research institutes on the use and application of innovative civilian technologies in the military sphere. The capabilities being researched and developed by the Defence Forces in cooperation with research companies and institutions are wide ranging. They cover capabilities such as Maritime Surveillance; Energy Conservation; Chemical Biological Radiological and Nuclear (CBRN) research including CBRN Protection, and Improvised Explosives Devices (IED) detection and destruction. The Defence Forces are a military force. Given the nature of their role on robust overseas operations, they need access to the latest communications and surveillance technologies and weapons systems in order to carry out their duties safely and effectively. This means having weapons guidance systems, simula- tors, surveillance systems, communications, robots and drones along with all the other elements of force protection.

While there is not an armaments industry in Ireland and it is not the intention of this Gov- ernment to establish one, there are however opportunities to develop new technologies and to further develop and enhance existing systems and capabilities which will have dual military 262 21 June 2016 and civil application. This can be achieved through continuous engagement between Irish en- terprise and research centres and the Defence Forces. The aim is to maximise synergies across both the civil and military domains and to develop leading edge technology and systems here in Ireland. Successful exploitation of current research and the knowledge which the Defence Forces can provide in this area also has the potential to contribute to the creation and retention of jobs in innovation and research in Irish Enterprises and third level institutes.

Projects proposed by institutes and companies, where they are seeking to partner with or involve Defence Forces participation, are submitted to the Defence Enterprise Committee for consideration. The Defence Enterprise Committee consists of personnel from the Department of Defence, the Defence Forces and Enterprise Ireland. All capabilities to be researched are vet- ted and agreed at the highest level in the Defence Organisation to ensure compatibility with the roles assigned to the Defence Forces by the Government.

21/06/2016WRBB01350Ministerial Responsibilities

21/06/2016WRBB01400572. Deputy Micheál Martin asked the Taoiseach and Minister for Defence the status of the responsibilities of the Minister of State at the Department of Defence who is based in the Department of the Taoiseach; and if he will make a statement on the matter. [11434/16]

21/06/2016WRBB01500Taoiseach and Minister for Defence (Deputy Enda Kenny): Deputy Paul Kehoe has been appointed Minister of State attending Government and Minister of State at the Department of the Taoiseach and the Department of Defence with special responsibility for the Department of Defence.

Following this appointment, the Government have made the Defence (Delegation of Min- isterial Functions) Order 2016 under the Minister and Secretaries (Amendment) (No. 2) Act 1977 to delegate ministerial functions to Minister Kehoe. Under this Order, Minister Kehoe now has statutory responsibility for a wide range of Defence matters as specified in the Order. The principal powers delegated include the powers to make regulations under the Defence Acts 1954 to 2015 covering a wide range of Defence matters and personnel-related issues affecting members of the Defence Forces. In addition, Ministerial powers arising under other legislation relating to the Department of Defence have been delegated to Minister Kehoe.

On an overall level, primary responsibility for the Department of Defence will rest with me as Minister for Defence and I remain responsible to Dáil Éireann and as a member of the Gov- ernment for the exercise and performance of the statutory powers and duties delegated. Every power or duty delegated to Minister Kehoe continues to be vested in me as Minister for Defence concurrently with the Minister of State and may be exercised or performed by either.

21/06/2016WRBB01550Livestock Issues

21/06/2016WRBB01600573. Deputy Fiona O’Loughlin asked the Taoiseach and Minister for Defence the mea- sures the Curragh forum is taking to ensure the safety of livestock legally grazing on the Cur- ragh plains in County Kildare; and if he will make a statement on the matter. [14450/16]

21/06/2016WRBB01700Taoiseach and Minister for Defence (Deputy Enda Kenny): As you are aware the man- agement and protection of the Curragh Plains, one of the most open and accessible areas in the Country, is an ongoing process.

On 8 March 2016 the first meeting of the newly established Curragh Plains Representative

263 Questions - Written Answers Forum was held. The meeting was chaired by the Department of Defence and was attended by invited representatives from the Defence Forces, an Garda Síochána, Curragh Racecourse, Cur- ragh Racehorse Trainers and Kildare County Council. The purpose of the Forum is to provide an opportunity for the consideration/progress of a wide range of issues regarding the use and management of the Curragh Plains. Issues to be considered include the development and pro- tection of the Curragh Plains and how best to deal with the many day to day issues arising from the management of the Curragh including but not limited to illegal parking, illegal encamp- ments, dumping and environmental protection.

The 1870 Curragh of Kildare Act awarded the right to graze sheep free of charge on the Cur- ragh on the basis of common of pasture, in respect of lands adjacent to the Curragh. The current bye-laws which were made in 1964 deal mainly with the management of sheep and depasturing of sheep. Animals other than sheep are not permitted to graze on the Curragh. Sheep grazing rights are not transferable apart from the occupation of the lands in respect of which they were awarded. There are currently 6,013 rights, which are owned by approximately 150 landowners and utilised by about 30 flock owners. These rights may be exercised over all lands other than those occupied by the military at the Curragh Camp (The Brown Lands) and the area enclosed at the Racecourse.

The owners of the sheep are responsible for the welfare of their animals while grazing them on the Curragh and must also comply with the provisions of the 1964 bye-laws.

21/06/2016WRBB01750Defence Forces Remuneration

21/06/2016WRBB01800574. Deputy Richard Boyd Barrett asked the Taoiseach and Minister for Defence if he will respond to allegations that some members of the Defence Forces serving on missions, such as Operation Pontus in the Mediterranean, are being paid less than the minimum wage; and if he will make a statement on the matter. [17052/16]

21/06/2016WRBB01900Taoiseach and Minister for Defence (Deputy Enda Kenny): The pay structures and con- ditions of service of members of the Permanent Defence Force contain professional and tech- nical remuneration elements in addition to allowances in the nature of pay which reflect the unique aspects of military life both at home and abroad. As such, the non-standard time and attendance patterns of military personnel and the rates of remuneration associated with a wide spectrum of duties undertaken by such military personnel can vary across the different branches of the Defence Forces.

Defence Forces personnel engaged in overseas peace support missions are paid their basic pay along with the Overseas Peace Support Allowance (Armed or unarmed depending on the mandate of the mission). Members of the Naval Service engaged in the Maritime Humanitar- ian Search and Rescue Operation in the Mediterranean are being paid the rate for “Unarmed Peace Support Allowance” in addition to their usual remuneration. This allowance is paid for the duration of the mission including from the date of departure to the date of return to base.

This is the same allowance as has been paid in the past to Army personnel deployed on hu- manitarian operations such as in Sierra Leone, Haiti etc. The Naval Service operation is not an armed mission in the sense of armed missions in UNIFIL (Lebanon), UNDOF (Golan Heights), Chad etc.

This Unarmed Peace Support Allowance is a daily allowance and is currently exempt from Income Tax and PRD deductions.

The following table sets out the weekly rate of the unarmed peace support allowance: 264 21 June 2016 ALLOWANCES - ENLISTED PERSON- Weekly Rate from 01/07/2013 NEL Overseas Peace Support Allowance – OPSA Sergeant and higher €413.49 Corporal €399.70 Private €385.98 Rates of remuneration and conditions of employment in the Irish public sector have tradi- tionally been set by reference to relative levels of pay across the various sectors of the Irish public sector. The Financial Emergency Measures in the Public Interest Acts of 2009-2015 and the Public Service Stability Agreement 2013-2018 (Lansdowne Road Agreement) define cur- rent pay policy.

Permanent Defence Forces Other Ranks Representative Association (PDFORRA) has made a claim for the application of the Minimum Wage to PDFORRA members of the Naval Service. This claim is currently being examined by the Departmental Side through the Conciliation and Arbitration (C&A) Scheme for Members of the Permanent Defence Force. As matters under consideration through the C&A Scheme are confidential, it would not be appropriate for me as Taoiseach to comment further at this time.

Overseas Missions

21/06/2016WRBB02000575. Deputy Richard Boyd Barrett asked the Taoiseach and Minister for Defence the number of operations the Defence Forces are involved in abroad; and if he will make a state- ment on the matter. [17053/16]

21/06/2016WRBB02050589. Deputy Brendan Ryan asked the Taoiseach and Minister for Defence for a report on all overseas missions that the Defence Forces are currently participating in; the number of Defence Forces personnel currently participating in overseas missions; and if he will make a statement on the matter. [17298/16]

21/06/2016WRBB02075596. Deputy Bernard J. Durkan asked the Taoiseach and Minister for Defence the level of deployment and location of Irish troops overseas on United Nations or European Union related missions; the extent to which further missions are planned or required; the extent to which the need for any upgrading of equipment has been noted, arising from such deployment; and if he will make a statement on the matter. [17317/16]

21/06/2016WRBB02100Taoiseach and Minister for Defence (Deputy Enda Kenny): I propose to take Questions Nos 575, 589 and 596 together.

As of 1 June 2016, Ireland is contributing 474 Defence Forces personnel to 11 overseas mis- sions throughout the world. Full details of all personnel currently serving overseas are listed in the following tabular statement.

The main overseas missions in which Defence Forces personnel are currently deployed are the United Nations Interim Force in Lebanon (UNIFIL) with 199 personnel, the United Nations Disengagement Observer Force (UNDOF) in Syria with 136 personnel and the Naval Service humanitarian mission in the Mediterranean with 57 personnel.

Irish soldiers deployed to UNIFIL serve alongside Finnish Armed Forces as part of a joint Battalion which is currently under Finnish command. Ireland takes over command of the Bat- talion from Finland in November 2016 at which time an additional Company of some 150

265 Questions - Written Answers personnel will be deployed to UNIFIL bringing Ireland’s contribution to this mission to 340 personnel approximately. In addition, Major General Michael Beary will take over the presti- gious role of Force Commander and Head of Misssion in UNIFIL shortly. A headquarters team and close protection support team amounting in total to approximately 20 personnel will deploy with him. Apart from these additional deployments to UNIFIL, no new missions are envisaged at this time.

The Department of Defence constantly reviews the deployment of Defence Forces person- nel overseas. With regard to any future deployments of Defence Forces personnel overseas, Ireland receives requests, from time to time, in relation to participation in various missions and these are considered on a case-by-case basis. When considering any particular request, the existence of realistic objectives and a clear mandate, which has the potential to contribute to a political solution, consideration of how the mission relates to the priorities of Irish foreign policy and the degree of risk involved are amongst the factors considered.

Defence Forces personnel serving on all overseas missions are equipped with the most mod- ern and effective equipment. This equipment enables troops to carry out the mission assigned, as well as providing the required protection specific to the mission. Ongoing threat assessments are carried out in mission areas and we continually review both equipment and force assets to ensure that Defence Forces personnel are appropriately equipped to fulfill their roles. I am satisfied that all appropriate security measures are in place to ensure the safety of all Defence Forces personnel serving overseas.

The current contribution of some 474 Defence Forces personnel to overseas missions re- flects the Government’s continued commitment to our responsibilities in the area of interna- tional peace and security.

Members of the Permanent Defence Force serving oversease as at 1st January 2016

1. UN MISSIONS

(i) UNIFIL (United Nations Interim Force in Lebanon) HQ 9

UNIFIL 53rd Infantry Group 186

UNIFIL Sector West HQ 4

(ii) UNTSO (United Nations Truce Supervision Organisation) Israel & Syria 10

(iii) MINURSO (United Nations Mission for the Referendum in Western Sahara) 3

(iv) MONUSCO (United Nations Stabilisation Mission in the Democratic 4

Republic of the Congo

(v) UNOCI (United Nations Mission in Ivory Coast) 2

(vi) UNDOF (United Nations Disengagement Observer Force) HQ, 7

Golan Heights, Syria

UNDOF 52nd Infantry Group 129

TOTAL 354

UN MANDATED MISSIONS

266 21 June 2016 (vii) EUFOR (EU-led Operation in Bosnia and Herzegovina) 7

(viii) EUTM Mali (EU-Led Training Mission) 13

(ix) KFOR (International Security Presence in Kosovo ) HQ 12

2. OTHER DEPLOYMENTS

Naval Service Humanitarian Mission in Mediterranean L.É. Róisín 57

TOTAL NUMBER OF PERSONNEL SERVING WITH UN MISSIONS 443

3. ORGANISATION FOR SECURITY AND CO-OPERATION IN EUROPE (OSCE)

(i) OSCE Mission to Bosnia & Herzegovina 1

(ii) Staff Officer, High Level Planning Group, Vienna 1

TOTAL NUMBER OF PERSONNEL SERVING WITH OSCE 2

4. EU MILITARY STAFF

Brussels 4

5. EU BATTLE GROUP

German-led Battle Group 2016 - HQ, STRASBOURG 10

UK-led Battle Group 2016, UK 5

6. MILITARY REPRESENTATIVES/ADVISERS/STAFF

(i) Military Adviser, Permanent Mission to UN, New York 1

(ii) Military Adviser, Irish Delegation to OSCE, Vienna 1

(iii) Military Representative to EU (Brussels) 4

(iv) Liaison Officer of Ireland, NATO /PfP (Brussels) 2

(v) EU OHQ Operation Althea, Mons, Belgium 1

(vi) Irish Liaison Officer to SHAPE & Military Co-Op Division, Mons, Belgium 1

TOTAL NUMBER OF DEFENCE FORCES PERSONNEL SERVING OVERSEAS 474

Defence Forces Medicinal Products

21/06/2016WRBB02200576. Deputy Richard Boyd Barrett asked the Taoiseach and Minister for Defence why the Defence Forces use the anti-malaria drug Larium, despite its known side effects; and if he will make a statement on the matter. [17054/16]

21/06/2016WRBB02250577. Deputy Richard Boyd Barrett asked the Taoiseach and Minister for Defence if he will justify the enforcement of the use of the anti-malaria drug, Larium, which has serious side effects, by the Defence Forces abroad; and if he will make a statement on the matter. [17055/16]

21/06/2016WRBB02300Taoiseach and Minister for Defence (Deputy Enda Kenny): I propose to take Questions Nos. 576 and 577 together. 267 Questions - Written Answers There are three anti-malarial drugs in use by the Defence Forces. They are Lariam (meflo- quine), Malarone and Doxycycline.

The health and welfare of the men and women of the Irish Defence Forces is a priority for me. The choice of medication for overseas deployment, including the use of Lariam, is a medical decision made by Medical Officers in the Defence Forces, having regard to the specific circumstances of the mission and the individual member of the Irish Defence Forces.

A Malaria Chemoprophylaxis working group was re-convened in August 2015. The purpose of the group is to review inter alia issues arising in relation to the use of Lariam, particularly in the context of the current and potential litigation; to review the Defence Forces approach in relation to the use of malaria chemoprophylaxis in the Defence Forces; and to ensure that the procedures in relation thereto continue to be appropriate and in accordance with best medical practice as promulgated by the relevant medical authorities. In addition, the Group was tasked with reviewing the use of the drug Primaquine as part of the overall medical treatment process for those deployed to malarious areas.

Malaria is a serious disease that killed approximately 438,000 people in 2015 as reported by the World Health Organization, with 90% of deaths occurring in sub-Saharan Africa. It is a serious threat to any military force operating in the area. Anti-malarial medications, including Lariam, remain in the formulary of medications prescribed by the Medical Corps for Defence Forces personnel on appropriate overseas missions, to ensure that our military personnel can have effective protection from the very serious risks posed by this highly dangerous disease.

21/06/2016WRBB02550Defence Forces Expenditure

21/06/2016WRBB02600578. Deputy Mick Wallace asked the Taoiseach and Minister for Defence the cost of the presence of the Defence Forces at Shannon Airport, protecting civilian troop carriers and mili- tary planes of the United States of America, by month in 2016 to date; and if he will make a statement on the matter. [17202/16]

21/06/2016WRBB02700Taoiseach and Minister for Defence (Deputy Enda Kenny): The Department of Justice and Equality and An Garda Síochána have primary responsibility for the internal security of the State. Among the roles assigned to the Defence Forces in the White Paper on Defence is the provision of Aid to the Civil Power (ATCP) which, in practice, means to assist An Garda Síochána when requested to do so.

Since 5 February 2003, the Gardaí have requested support from the Defence Forces at Shan- non Airport on occasion. The cost of the presence of Defence Forces at Shannon Airport, pro- tecting civilian troop carriers and military planes of the United States of America, by month in 2016 to date is set out in the following table.

Month Costs January €15,457.23 February €13,896.72 March €15,492.39 April €15,101.14 May €15,422.07 June (to date) €14,796.42 I am satisfied that there is ongoing and close liaison between both An Garda Síochána and the Defence Forces, and between my Department and the Department of Justice and Equality regarding security matters generally, including the Defence Forces ATCP roles. 268 21 June 2016 Overseas Missions

21/06/2016WRBB02800579. Deputy Mick Wallace asked the Taoiseach and Minister for Defence if he had dis- cussions regarding discontinuing the Irish participation in the United Nations Disengagement Observer Force mission in the Golan Heights; and if he will make a statement on the matter. [17203/16]

21/06/2016WRBB02900Taoiseach and Minister for Defence (Deputy Enda Kenny): A contingent of the Perma- nent Defence Force has been deployed to the United Nations Disengagement Observer Force (UNDOF) on the Golan Heights since 2013. The current Irish contingent, the 52nd Infantry Group, comprising 129 personnel, is based in UNDOF Headquarters in Camp Ziouani. It oper- ates in the role of a Quick Reaction Force, which is on standby to assist with on-going opera- tions within the UNDOF Area of Responsibility. Seven (7) other Defence Forces personnel are also deployed in UNDOF Headquarters.

The Department of Defence constantly reviews the deployment of Defence Forces person- nel overseas and these missions are reviewed on a case-by-case basis. On the margins of an informal meeting of Defence Ministers in Luxembourg on the 3rd September 2015, the then Minister for Defence, Mr. Simon Coveney, had discussions regarding the Defence Forces par- ticipation in UN peacekeeping operations including the UNDOF mission, with the UN Under Secretary-General, Mr Hervé Ladsous. He confirmed Ireland’s commitment to UN operations, and in relation to UNDOF, he pointed out that if Ireland was not required in UNDOF in the future, then consideration would be given to other options for deployments on behalf of the UN. At this point in time, however, the presence of the UNDOF mission remains an important element in ensuring stability on the Golan Heights and in the Middle East region and, today, the Government approved continued participation by the Defence Forces in UNDOF for a further twelve months.

21/06/2016WRBB02950Military Neutrality

21/06/2016WRBB03000580. Deputy Mick Wallace asked the Taoiseach and Minister for Defence with regard to the Government’s White Paper on Defence, if he will consider a change to the State’s policy and definition of military neutrality; if he will consider the use of Shannon Airport by the military of the United States of America to be a breach of that military neutrality and of other policies under the Defence portfolio; and if he will make a statement on the matter. [17204/16]

21/06/2016WRBB03100Taoiseach and Minister for Defence (Deputy Enda Kenny): As the Deputy was previous- ly informed in the House on the 8th of October 2015, the White Paper on Defence, which was published last summer, following a broad and extensive consultative process, did not change Ireland’s policy of military neutrality.

The Government’s recent review of foreign policy and the Defence White Paper confirmed that Ireland will continue to maintain its traditional policy of military neutrality which means non-membership of military alliances and non-participation in common or mutual defence ar- rangements. Ireland’s policy of military neutrality was formed in an era when inter-State con- flict was the key issue of national security for most States. The State’s policy of remaining out- side of military alliances has remained in place ever since. Thankfully, the threat of inter-State war in Europe is much diminished, but there will continue to be new and emerging threats in the defence and security environment. The reality remains that the world continues to evolve to such an extent that no one country alone can respond adequately to the threats in the defence and security environment and inevitably there will be threats and challenges that have not yet

269 Questions - Written Answers been anticipated. Such security challenges will require an enhanced collective and comprehen- sive approach, and there will be an increasing emphasis on security co-operation. The issue of overflights by foreign military aircraft and the use of Shannon by foreign military aircraft are the responsibility of the Minister for Foreign Affairs and Trade. However, the Deputy will be aware that successive Governments have made overflight and landing facilities available at Shannon Airport to the United States for well over 50 years. These arrangements do not amount to any form of military alliance with the US and are governed by strict conditions, applied to ensure compatibility with our traditional policy of military neutrality. Finally, it goes without saying that all our policy responses must realistically reflect current and future security chal- lenges and should be able to accommodate the necessary responses, both national and collective without prejudice to our policy of military neutrality.

21/06/2016WRBB03150Defence Forces Recruitment

21/06/2016WRBB03200581. Deputy Brendan Smith asked the Taoiseach and Minister for Defence when the next recruitment campaign for the Permanent Defence Force will be announced; and if he will make a statement on the matter. [17209/16]

21/06/2016WRBB03300Taoiseach and Minister for Defence (Deputy Enda Kenny): The Government is commit- ted to maintaining the stabilised strength of the Permanent Defence Force at 9,500 personnel, comprising of 7,520 Army, 886 Air Corps and 1,094 Naval Service as stated in the 2015 White Paper on Defence. The manpower requirement of the Defence Forces is monitored on an on- going basis in accordance with the operational requirements of each of the three services, the establishment of the Permanent Defence Force and available resources. As there is significant turnover of personnel in the Permanent Defence Force targeted recruitment takes place so as to maintain personnel numbers at or near the agreed strength levels as set out above in accordance with available training capacity.

A General Service recruitment campaign that was launched on 13 April 2016, and the 2016 Cadet Competition launched on 16 March 2016, are both now closed and applications are being processed. It is envisaged that there will be an intake of 600 general services recruits in 2016 and a further 850 in 2017. This recruitment will be phased over the 2016 and 2017 period in a manner designed to increase personnel numbers towards the established strength and plans are being put in place to this end. Most of the new recruits this year will be drawn from the existing General Service panel. A new panel will be established from the current competition. Recruitment from this new panel is expected to commence towards the end of 2016 and will continue through 2017 and into 2018. It is too early at this stage to predict whether and when a 2017 competition may be required, as it will depend on the numbers who are successful in the current competition and are placed on the panel and the level of turnover in the Defence Forces.

21/06/2016WRBB03350Employment Support Services

21/06/2016WRBB03400582. Deputy Brendan Ryan asked the Taoiseach and Minister for Defence the consulta- tion which took place with the Defence Forces on the commitment in the new programme for Government for a pilot employment support scheme that will be run in conjunction with the Defence Forces; when this programme will be rolled-out and the amount it will cost per year; how participants will be selected for this programme; if concerns have been expressed by the Defence Forces on this proposal, given the high level of professionalism of the Defence Forces; and if he will make a statement on the matter. [17291/16]

270 21 June 2016

21/06/2016WRBB03500Taoiseach and Minister for Defence (Deputy Enda Kenny): In line with the Govern- ment’s commitment to a strengthened approach across the economy to social responsibility, the Programme for a Partnership Government includes a commitment to introduce a pilot employ- ment support scheme with the direct involvement of the Defence Forces. This commitment was also reflected in the White Paper on Defence, published in August 2015, which sets out the policy framework for the Defence Organisation for the next ten years and beyond.

The objective of this Scheme is to help develop a path to economic independence for partici- pants, aged between 18 and 24 years, who are at a serious disadvantage owing to their current socio-economic situation. The Scheme is designed with the specific aim of equipping partici- pants with life and other marketable and certifiable skills which will assist them both in their capacity to participate fully in society as well as their ability to enter the workforce and or to pursue further education for this purpose.

The scheme and the programme content have been developed through detailed consultation involving the Department of Defence, the Defence Forces, the Department of Social Protec- tion and the Dublin and Dun-Laoghaire Education and Training Board. A work group which includes representatives from each of the aforementioned organisations has developed the de- tail of the initiative with key stakeholders. Prior to the commencement of the scheme, a meet- ing was held with both Defence Forces Representative Associations, RACO and PDFORRA, where a detailed presentation and briefing was provided on the proposal and the content of the scheme and, at which, any questions raised were addressed

The pilot scheme commenced on Monday 20th June 2016 for 28 participants in Gormanston Camp for 10 weeks. The total estimated cost of running the pilot programme is €240,000. Fol- lowing review of the pilot programme, it will be possible to calculate the actual costs incurred and therefore accurately estimate the cost of the Scheme on an ongoing basis. The success of the pilot scheme will inform the progress of rolling out the scheme. However, it is important to state, that based on current resource constraints, it is anticipated that a maximum of 2 pro- grammes could be run per annum.

The Department of Social Protection identified suitable participants for participation in the pilot scheme. The basic selection criteria applied were that the person must:

- be aged between 18 and 24,

- be unemployed,

- want to take part in the programme, and

- be healthy and physically able to take part in outdoor activities.

Potential candidates were then interviewed by a member of the Defence Forces and an of- ficial from Department of Social Protection. Candidates who were successful at interview were then required to pass security vetting and be medically certified as fit to undergo the programme of training by their own civilian GP, prior to the commencement date of the pilot scheme.

The following revision was received from the Department on 14 July 2016

The written reply stated that there were 28 participants taking part in the pilot scheme. The Department would like to clarify that in fact the number of participants is 25.

21/06/2016WRBB03550Naval Service Operations

271 Questions - Written Answers

21/06/2016WRBB03600583. Deputy Brendan Ryan asked the Taoiseach and Minister for Defence to update Dáil Éireann on the continued role of Irish Naval Service personnel in assisting with the migrant crisis in the Mediterranean; how long the mission will continue; the level of engagement he has had with European Union partners on this vital humanitarian mission; and if he will make a statement on the matter. [17292/16]

21/06/2016WRBB03700Taoiseach and Minister for Defence (Deputy Enda Kenny): Following a Government Decision on 12 May 2015, the Irish Naval Vessel, L.É. Eithne, was deployed to the Mediter- ranean on 16 May, 2015, to assist the Italian Authorities with the migrant crisis. During 2015 two further vessels were deployed, L.É. Niamh and L.É. Samuel Beckett, and the deployment was completed on 29 November 2015, with L.É. Samuel Beckett arriving back in Ireland on 17 December 2015. During that period 8,592 people were rescued in the Mediterranean by Irish Naval Vessels.

The Government, on 6 April 2016, approved the return of an Irish Naval vessel to under- take humanitarian search and rescue tasks in the Mediterranean. This deployment will involve naval vessel rotation. Subject to the operational demands and requirements of the mission, it is intended that there will be two rotations (i.e. 3 Naval Service vessels deployed) with each deployment lasting approximately 12 weeks.

L.É Róisín was deployed to the Mediterranean on 1 May 2016 and commenced patrolling in the area of operations on 12 May 2016. To date, L.É. Róisín has rescued 893 people bringing the overall total rescued by the Naval Service during 2015 and 2016 to 9,485.

The role of the Irish Naval vessel is to provide a search and rescue capability and to un- dertake humanitarian search and rescue operations at sea in the Mediterranean. Assistance to persons in distress at sea will be provided in accordance with the applicable provisions of international conventions governing search and rescue situations. Co-ordination of search and rescue efforts and the provision of humanitarian assistance is being achieved through close co- operation with the relevant Italian authorities.

The despatch of Irish naval vessels represents a tangible and valuable Irish national contri- bution to assisting with the continuing migration crisis in the Mediterranean.

At EU level, the EU naval operation against human smugglers and traffickers in the Medi- terranean, Operation Sophia, was launched at the Foreign Affairs Council meeting on 22 June 2015. Its mission is to identify, capture and dispose of vessels and enabling assets used or sus- pected of being used by migrant smugglers or traffickers. The objectives are not only to disrupt the activities of smugglers and traffickers; but also to prevent further loss of life at sea and to reduce the suffering and exploitation of migrants caused by criminal organisations.

While Ireland is fully supportive of the EU approach to this crisis, there are no plans to deploy naval personnel or vessels to the EU Mission at this time. Any consideration of future participation by the Irish Defence Forces in Operation Sophia will be subject to the applicable National statutory requirements being met.

21/06/2016WRBB03750Defence Forces Operations

21/06/2016WRBB03800584. Deputy Brendan Ryan asked the Taoiseach and Minister for Defence whether the Army Ranger Wing is to double in size due to current concerns regarding the threat of interna- tional terrorism; when this will happen and if additional funding will be allocated specifically for this purpose; if he will indicate what he believes is the optimum level of the Army Ranger Wing; and if he will make a statement on the matter. [17293/16] 272 21 June 2016

21/06/2016WRBB03900Taoiseach and Minister for Defence (Deputy Enda Kenny): Primary responsibility for the internal security of the State rests with the Department of Justice and Equality and An Garda Síochána. Among the roles assigned to the Defence Forces in the White Paper on Defence is the provision of Aid to the Civil Power (ATCP) which, in practice, means to provide assistance and support to An Garda Síochána when requested to do so. There is ongoing and close liai- son between An Garda Síochána and the Defence Forces regarding security matters, including ATCP deployments and a wide variety of military training activities are specifically designed to counter or respond to possible security emergencies.

The Army Ranger Wing is an integral unit of the Defence Forces whose roles include provi- sion of specialist ATCP support to An Garda Síochána. The need for a high level of prepared- ness to deal with any requests for special forces operations is inherent in the unit’s mission. Members are trained to the highest levels of motivation, physical fitness and skill at arms for their specialist role. The ARW is on standby to be called upon to undertake duties in any part of the country.

In the recently published White Paper on Defence, the Government has committed to fur- ther enhance the capabilities of the Army Ranger Wing, in particular with the aim of increasing the strength of the unit considerably. As the Army Ranger Wing is the Special Operations Force of the Defence Forces the exact number of personnel in it or their location is not released into the public domain, for reasons of operational security.

It is my priority as Minister for Defence to ensure that the operational capacity of the De- fence Forces is maintained to the greatest extent possible.

21/06/2016WRBB03950Defence Forces Operations

21/06/2016WRBB04000585. Deputy Brendan Ryan asked the Taoiseach and Minister for Defence if he has liaised with his counterparts across the European Union on the ongoing threat of international terror- ism; if he is satisfied at the level of preparedness here if such an event were to occur; if the Irish Defence Forces have been asked to provide any support, whether of an intelligence nature or otherwise, to support European Union partner states who have been the subject of international terror threats or actions; and if he will make a statement on the matter. [17294/16]

21/06/2016WRBB04100Taoiseach and Minister for Defence (Deputy Enda Kenny): At Ministerial level there is ongoing contact as required with EU colleagues both in the context of European Defence and Security and other matters. In addition to these bilateral contacts, two official meetings of EU Defence Ministers are held during each six month EU Presidency. The most recent meeting of Foreign Affairs and Defence Ministers took place in Luxembourg on 18 April 2016. Discus- sions at the meeting covered a wide range of European Defence and Security issues and associ- ated challenges.

I am also kept fully appraised of issues arising from meetings which officials from my De- partment attend in the area of European Defence and Security. As with the formulation of any policy, best practice and lessons learnt in other jurisdictions will always inform EU policy for- mulation to ensure that it is robust, coherent and meets the needs of Member States. In relation to International Terrorism issues at EU level, this is primarily a matter for Justice and Interior ministers under the Justice and Home Affairs umbrella. Within this State, primary responsibil- ity for the internal security of the State rests with the Department of Justice and Equality and An Garda Síochána. In that regard, among the roles assigned to the Defence Forces in the White Paper on Defence is the provision of Aid to the Civil Power (ATCP) which, in practice, means to provide assistance and support to An Garda Síochána when requested to do so. There is on- 273 Questions - Written Answers going and close liaison between An Garda Síochána and the Defence Forces regarding security matters, including ATCP deployments and a wide variety of military training activities are spe- cifically designed to counter or respond to possible security emergencies. Regular coordination and liaison meetings also take place between the Defence Forces and An Garda Síochána in relation to ATCP issues. It is my priority as Minister for Defence to ensure that the operational capacity of the Defence Forces is maintained to the greatest extent possible.

As the Deputy will be aware, the previous Minister for Defence attended the meeting of For- eign Affairs and Defence Ministers that took place in Brussels on 17 November 2015 at which my French colleague, Defence Minister Jean-Yves Le Drein, invoked Article 42.7 of the Treaty on European Union, following the tragic events in Paris on 13 November last.

Following that invocation there was liaison between officials from my Department and French authorities at which various options for how Ireland could be of assistance were ex- plored. One option explored was examining what support Ireland would be able to offer to UN mandated missions which suffered gaps as a result of France or other nations withdrawing forces to undertake alternative operations. The outcome to the French request for support was the enhancement of our current deployment to the EU Training Mission in Mali, to which 10 members of the Permanent Defence Force were previously deployed. This figure will now be increased to 18. The issue of also providing support to the UN peacekeeping mission MINUS- MA is being kept under review.

Defence Forces Operations

21/06/2016WRBB04200586. Deputy Brendan Ryan asked the Taoiseach and Minister for Defence the purpose of the recent exercises in the Irish Sea conducted by the Defence Forces Special Operations Task Group; if this was a regular exercise or was it linked to ongoing anti-terror training; if other anti-terror exercises have recently taken place; and if he will make a statement on the matter. [17295/16]

21/06/2016WRBB04300Taoiseach and Minister for Defence (Deputy Enda Kenny): The Department of Justice and Equality and An Garda Síochána have primary responsibility for the internal security of the State and, in the event of a terrorist incident, it is An Garda Síochána who would be the first responders. Among the roles assigned to the Defence Forces in the White Paper on Defence is the provision of Aid to the Civil Power (ATCP) which, in practice, means to provide assistance and support to An Garda Síochána when requested to do so. There is ongoing and close liai- son between An Garda Síochána and the Defence Forces regarding security matters, including ATCP deployments and a wide variety of military training activities are specifically designed to counter or respond to possible security emergencies. Regular coordination and liaison meetings also take place between the Defence Forces and An Garda Síochána in relation to ATCP issues.

When not engaged in actual operations, the Defence Forces conduct regular exercises. In this regard, on the 18th of May 2016, Exercise Poseidon was commenced. The purpose of the exercise was to enhance military capability and particularly joint operations involving all three services (Army, Air Corps, Navy). This is a regular exercise which is generally conducted ev- ery two years. The exercise was not linked to any known terror threat.

For security and operational reasons it would not be appropriate to comment further.

EU Battle Groups

274 21 June 2016

21/06/2016WRCC00200587. Deputy Brendan Ryan asked the Taoiseach and Minister for Defence for a report on the recent decision for Ireland to participate in an European Union United Kingdom-led battle group; how the new United Kingdom-led battle group will operate; the training exercises or deployment he expects to occur within this new framework; and if he will make a statement on the matter. [17296/16]

21/06/2016WRCC00300Taoiseach and Minister for Defence (Deputy Enda Kenny): At its meeting of 14th July 2015, the Government approved the participation of five members of the Permanent Defence Force in the UK Battlegroup 2016, which will be on stand-by for 6 months from July 2016.

On 9th June 2016 Dail Éireann passed a Resolution on Ireland’s accession to the Memo- randum of Understanding concerning the establishment and operation of the UK Battlegroup. The UK led EU Battlegroup Memorandum of Understanding is an agreement between the par- ticipants, namely Finland, Ireland, Latvia, Lithuania, Sweden, and the UK, which sets out prin- ciples in relation to the operation, deployment and management of the UK led EU Battlegroup. The MoU which has been laid before the House is broadly structured as follows:

Sections 1 to 4 deal with the definitions and reference documents that are applicable to this MoU. They define the UK as the Framework Nation and broadly outline the purpose of the MoU.

Sections 5 to 8 deal with the consultation process, exercise, training, certification and the operation of the Battlegroup. Most Battlegroup training will take place in the contributing Member States - i.e. Irish troops will mainly be trained in Ireland.

Sections 9 to 13 outline the arrangements in terms of financing, logistics, classified informa- tion, the status of forces and the issue of claims and liabilities.

Sections 14 to 16 deal with the process of participation of additional States in the Battle- group, the timelines for coming into effect and termination of the Memorandum of Understand- ing and the process for dispute resolution.

For the most part, Battlegroup training takes place in the contributing member States i.e. Irish troops are mainly trained in Ireland, Swedish troops in Sweden etc. Training and military exercises which have already taken place over the six months in preparation for the Battlegroup standing up on 01 July 2016 are set out in the following table.

In respect of deployment, it is the case that no Battlegroup has ever deployed to date. Each Battlegroup participant retains the right to deploy or not to deploy its forces, irrespective of an EU decision to launch a Battlegroup operation. Equally, each Battlegroup participant retains the sovereign right to withdraw its contingent at any time. The command of each contingent re- mains under national control, with operational control delegated to the operational commander. Any deployment of the Irish contribution as part of the UK Battlegroup will be subject to a UN mandate and Government approval. The following training exercises were undertaken as part of the preparations for the Battlegroup Standby Period.

UK led EU Battlegroup, 2016-2

Participants: Ireland, UK, Finland, Sweden, Latvia, Lithuania.

Exercise Dates Description Participants Ex CAST 07 Mar 16 – 11 Mar 16 A Force Headquarters 5 Staff Officers deployed to Command and Staff Exer- the Operational and Force cise conducted in Catterick, Headquarters. UK.

275 Questions - Written Answers

Exercise Dates Description Participants Live Ex 02 May 16 – 21 May 16 The Battlegroup Certifica- 5 Staff Officers deployed to tion Exercise involving all the Operational and Force elements of the Battlegroup Headquarters. conducted on Salisbury Plains, UK. POLEX 28 Apr 16 Poliocy/Political decision Defence Policy Director, making exercise held in Deputy Defence Policy London, UK Director, Deputy Chief of Staff (Support), Director of Operations

21/06/2016WRCC00400Defence Forces Medicinal Products

21/06/2016WRCC00500588. Deputy Brendan Ryan asked the Taoiseach and Minister for Defence his views on the comments made by Permanent Defence Force Other Ranks Representative Association, that Irish soldiers should be given a choice of whether to use the anti-malaria drug Lariam, in par- ticular, in respect of criticisms by a British defence committee of the drug; and if he will make a statement on the matter. [17297/16]

21/06/2016WRCC00600Taoiseach and Minister for Defence (Deputy Enda Kenny): I am aware of the British Defence Committee report, its conclusions and the reported comments of the Permanent De- fence Force Other Ranks Representative Association. However, the choice of malaria chemo- prophylaxis for use by other armed forces is an internal matter for those forces. It would be in- appropriate for me to comment on other States’ policy in this regard. The health and welfare of the men and women of the Irish Defence Forces is a priority for me. The choice of medication for overseas deployment, including the use of Lariam, is a medical decision made by Medical Officers in the Defence Forces, having regard to the specific circumstances of the mission and the individual member of the Irish Defence Forces.

The aim of the Defence Forces policy on malaria chemoprophylaxis is to minimise risk to all deployed personnel. Significant precautions are taken by Irish Defence Forces Medical Officers in assessing the medical suitability of members of our Defence Forces to take any of the anti-malarial medications. It is the policy of the Irish Defence Forces that personnel are individually screened for fitness for service overseas and medical suitability. The objective is to ensure that our military personnel can have effective protection from the very serious risks posed by malaria.

Malaria is a serious disease which killed approximately 438,000 people in 2015, with 90% of deaths occurring in sub-Saharan Africa as reported by the World Health Organisation. It is a serious threat to any military force operating in the area. Anti-malarial medications, including Lariam, remain in the formulary of medications prescribed by the Medical Corps for Defence Forces personnel on appropriate overseas missions, to ensure that our military personnel can have effective protection from the very serious risks posed by this highly dangerous disease.

Question No. 589 answered with Question No. 575.

21/06/2016WRCC00850Defence Forces Representative Organisations

21/06/2016WRCC00900590. Deputy Brendan Ryan asked the Taoiseach and Minister for Defence his views on the Labour Party’s proposals to facilitate Irish Congress of Trade Union membership for Defence Forces representative associations, which has been welcomed by the Permanent Defence Force

276 21 June 2016 Other Ranks Representative Association; his plans to progress this policy; the estimated time- frame for implementation; and if he will make a statement on the matter. [17299/16]

21/06/2016WRCC01000Taoiseach and Minister for Defence (Deputy Enda Kenny): There are no plans to al- low PDFORRA to take up associate membership of ICTU. Under the terms of the Defence (Amendment) Act, 1990 the Defence Forces Representative Associations are prohibited from being associated with, or affiliated to any trade unions, or any other body without the consent of the Minister. Accordingly, the Representative Association PDFORRA cannot be affiliated to ICTU at present.

The basis for the prohibition is that it would be inappropriate to apply the provisions of the Industrial Relations Act, 1990 to members of the Defence Forces. The taking of any form of industrial action is irreconcilable with military service. The Defence Forces may be called on to contribute to maintaining vital services in times of industrial action. The potential for serious difficulties and conflicts could arise in these circumstances if the Defence Forces Representa- tive Associations were associated with, or affiliated to ICTU, given ICTU rules in relation to such matters.

A number of mechanisms have been put in place through the Defence Forces Conciliation and Arbitration Scheme to compensate for the prohibition on affiliation to ICTU. These pro- vide the Representative Associations with structures and processes, which enable them to make representations and negotiate on behalf of their members.

In addition, a framework exists which facilitates the Associations engaging with the offi- cial side in talks parallel to those taking place between ICTU and the official side at National level. This parallel process was successfully operated in respect of discussions, facilitated by the Workplace Relations Commission, on an extension to the ‘Public Service Agreement 2010 – 2018’ leading to the Lansdowne Road Agreement.

Following on from a case brought to the European Committee on Social Rights by the Association of Garda Sergeants and Inspectors (AGSI), a complaint has now been made by EUROMIL (a European umbrella body for military associations), on behalf of the Permanent Defence Forces Other Ranks Representative Association (PDFORRA), alleging a breach of Article 5 (the right to organise) and Article 6 (the right to bargain collectively) of the European Social Charter.

Submissions have been lodged to the European Committee on Social Rights setting out Ireland’s position in the matter and a further Response to Submissions was lodged by Ireland on 26th February 2016.

I remain satisfied with the present arrangements in place and have no plans to make any changes.

21/06/2016WRCC01050White Paper on Defence

21/06/2016WRCC01100591. Deputy Brendan Ryan asked the Taoiseach and Minister for Defence for an update on the implementation of the White Paper on Defence; the measures that have been implemented and the timeline for the implementation of all other measures; and if he will make a statement on the matter. [17300/16]

21/06/2016WRCC01200Taoiseach and Minister for Defence (Deputy Enda Kenny): Following the publication of the White Paper on Defence a total of 88 projects were identified to be completed over a ten year period. Implementation of the White Paper projects is included in the Programme for a 277 Questions - Written Answers Partnership Government. The Implementation process is being led and overseen by the Minis- ter, the Management Board and the General Staff assisted by a Joint Civil Military White Paper Implementation Facilitation Team (Joint Team).

As the White Paper sets the policy framework for the coming years implementation is of critical importance and is a significant undertaking for the Defence Organisation. In this con- text, implementation of projects is being phased with the immediate projects to be initiated in the period ending June 2017. Work has already commenced and in some cases is well advanced on a number of projects. These projects include, but are not limited to, the Employment Sup- port Scheme, the Diversity and Inclusion strategy, the formation of the Civil Defence Inter- agency Guidance Team and various equipment procurement projects.

The approach to implementation takes account of the significance of the White Paper, har- nesses the civil and military effort on implementation while also recognising the finite resource envelope available and the operational imperative in Defence.

21/06/2016WRCC01300White Paper on Defence

21/06/2016WRCC01400592. Deputy Brendan Ryan asked the Taoiseach and Minister for Defence the measures he is bringing forward to ensure the enhanced participation of women in the Defence Forces, as was specifically proposed in the White Paper; and if he will make a statement on the matter. [17301/16]

21/06/2016WRCC01500Taoiseach and Minister for Defence (Deputy Enda Kenny): The Government is commit- ted to a policy of equal opportunity for men and women throughout the Defence Forces and to the full participation by women in all aspects of Defence Forces activities.

The Defence Forces have no restrictions with regard to the assignment of men or women to the full range of operational and administrative duties , and all promotions and career courses are open to both sexes on merit.

A key impediment in achieving greater numbers of women serving in the Defence Forces is the lack of women recruitment applicants relative to men. This may be as a result of societal perception and attitudes to female soldiers and officers. There have been a number of recent initiatives to encourage greater female participation, such as:

- The introduction of best practices in recruitment, such as the adjustment of physical stan- dards for female applicants,

- Special consideration is paid to women as a target group for recruitment,

- A balanced composition between men and women on recruitment and selection boards,

- All promotions and career courses are open to both sexes on merit,

- A Gender Advisor has been appointed to promote gender equality policies and training within the Defence Forces and,

- Work is well-progressed on a Defence Forces Equality and Diversity Policy, which will be published in the near future.

Over the course of the White Paper 2015 , further initiatives will be developed to encourage more women to apply for the Defence Forces and to increase female participation at all ranks. This will include a survey to identify any impediments to the advancement of women in the

278 21 June 2016 PDF, as well as the impact of the requirements of career courses and overseas service on female retention and advancement. The development and active management of a Diversity and an Inclusion Strategy in the Defence Forces, building on existing policies in relation to ethnicity, gender, sexual orientation and equality will also be included.

Departmental Budgets

21/06/2016WRCC01700593. Deputy Brendan Ryan asked the Taoiseach and Minister for Defence his views that there should be an increase in defence expenditure in budget 2017, given that the current expen- diture of around 0.5% of gross domestic product is one of the lowest expenditures on defence across the European Union; and if he will make a statement on the matter. [17302/16]

21/06/2016WRCC01800Taoiseach and Minister for Defence (Deputy Enda Kenny): The White Paper on De- fence, published in 2015, sets out the roles that Government has assigned to the Defence Forces and the associated capability requirements. I am fully committed to ensuring that appropriate financial resources are provided to facilitate effective delivery across all of these assigned roles, into the future.

Security environments differ across all EU states and each country pursues a Defence policy that reflects its particular requirements. Consequently, the proportion of overall funding allo- cated to Defence can fluctuate between member states.

The expenditure allocation for Defence for 2017 will not be settled until the Budgetary pro- cess is concluded, later in the year.

21/06/2016WRCC01850Overseas Missions

21/06/2016WRCC01900594. Deputy Brendan Ryan asked the Taoiseach and Minister for Defence to report on the recent appointment of a person (details supplied) as the head of mission and force commander of the United Nations Interim Force in Lebanon, UNIFIL; the length and scope of this mission; and if he will make a statement on the matter. [17303/16]

21/06/2016WRCC02000Taoiseach and Minister for Defence (Deputy Enda Kenny): On 25 May 2016, United Nations Secretary-General Ban Ki-moon announced the appointment of Major General Mi- chael Beary as the Head of Mission and Force Commander of the United Nations Interim Force in Lebanon (UNIFIL). The appointment will be for an initial period of one year.

Major General Beary succeeds Major General Luciano Portolano of Italy, who will com- plete his assignment in mid July 2016.

The Head of Mission/Force Commander exercises “UN Operational Authority in the Field” over all military and civilian UN entities assigned to UNIFIL and commands UNIFIL, exercis- ing operational control over all UNIFIL military elements and military observers.

Major General Beary has a wide range of previous overseas experience including three deployments in South Lebanon with UNIFIL and has served as Company Commander and Staff Officer in UNIFIL HQ Operations Branch. He served as Mission Commander in EUTM Somalia from 2011 to 2013 where he commanded a multinational force from 12 EU Member States who were tasked with training military forces for the Somali Government. He is cur- rently serving as the General Officer Commanding 2 Brigade.

The UNIFIL mission continues to represent Ireland’s largest overseas deployment. Cur- 279 Questions - Written Answers rently, Ireland serves as part of a joint Finnish/Irish Battalion. There are currently 199 Irish personnel serving with UNIFIL. Ireland will take over command of the Battalion from Finland in November 2016 and will deploy an additional Company of approx 150 personnel as part of the Battalion.

21/06/2016WRCC02050Defence Forces Reserve Strength

21/06/2016WRCC02100595. Deputy Brendan Smith asked the Taoiseach and Minister for Defence his proposals to increase numbers in the Reserve Defence Force; and if he will make a statement on the mat- ter. [17316/16]

21/06/2016WRCC02200Taoiseach and Minister for Defence (Deputy Enda Kenny): The White Paper on Defence (2015) set the overall establishment of the Reserve Defence Forces (RDF) at 4,169 personnel. Recruitment campaigns for the RDF are conducted on an annual ongoing basis. The current campaign started in September 2015, and has been paused on an administrative basis as the De- fence Forces are administering the recruitment campaign for candidates to enter the Permanent Defence Force. I am advised that it is intended to recommence the campaign for Reservists in the coming months. The Defence Forces have received 2,733 applications to date in this com- petition which is encouraging .

The 2015 White Paper on Defence confirms that the primary role of the Reserve is to aug- ment the Permanent Defence Force in times of crisis and to participate in ceremonial events. The implementation of measures set out in the White Paper 2015 will provide opportunities for certain members of the RDF to undertake new duties. I am of the view that this, and other initia- tives outlined in the White Paper will be a key motivating factor in encouraging more people to join the RDF as well as making it more attractive for current members to stay.

Question No. 596 answered with Question No. 575.

21/06/2016WRCC02375Defence Forces Training

21/06/2016WRCC02500597. Deputy Bernard J. Durkan asked the Taoiseach and Minister for Defence the extent to which Army, Navy and Air Corps training continues to be in line with modern and interna- tional best practice; if any particular procedures are shown to be in need of updating; and if he will make a statement on the matter. [17318/16]

21/06/2016WRCC02550602. Deputy Bernard J. Durkan asked the Taoiseach and Minister for Defence the degree to which current ongoing training of the Army, Navy and Air Corps including Reserves, is scheduled in the course of the next three years; the extent to which such ongoing training meets international best practice; and if he will make a statement on the matter. [17323/16]

21/06/2016WRCC02600Taoiseach and Minister for Defence (Deputy Enda Kenny): I propose to take Questions Nos. 597 and 602 together.

The primary function of training and education in the Defence Forces is to develop and maintain the capabilities necessary to enable it to fulfil the roles laid down by Government.

The Defence Forces conducts training and education under four broad categories namely; leadership, skills, career and collective training. Leadership training is the medium through which the Defence Forces ensures that personnel are prepared for exercising command au- thority across the full range of military functions both at home and overseas in international operations. Skills training is the medium through which the Defence Forces ensures that its 280 21 June 2016 personnel have the requisite individual, specialist, and crew skills. Career training and educa- tion provides the organisation with the necessary pool of leaders and commanders at all levels. Collective Training allows military personnel to use their individual, crew, specialist and career training together, to develop integrated and coherent combat forces. Emphasis is placed on joint and combined exercises.

The scheduling of training in the Defence Forces, including the Reserve, is underpinned by an analysis of training needed to meet operational output requirements and capability develop- ment needs. A long term strategy is adopted with current planning horizons out to 2021.

The Defence Forces seek to constantly benchmark training across all three arms against best military and academic practice. Military best practice is ensured by implementing a policy of standardisation that is in line with EU and Partnership for Peace partners. Academic best prac- tice is ensured by benchmarking courses through accreditation under the National Framework of Qualifications which is validated by Quality and Qualifications Ireland. The Defence Forces delivers military programmes and modules meeting national and international standards. The Defence Forces also engages with external educational institutions in order to facilitate or- ganisational learning. This engagement with national and international educational institutions, military and civilian, aims to ensure that the Defence Forces retains currency with regards to best international practice.

21/06/2016WRCC02700Defence Forces Expenditure

21/06/2016WRCC02800598. Deputy Bernard J. Durkan asked the Taoiseach and Minister for Defence the ex- tent to which military equipment including transport, communications and procedures in the Army continues to be upgraded and improved; and if he will make a statement on the matter. [17319/16]

21/06/2016WRCC02900599. Deputy Bernard J. Durkan asked the Taoiseach and Minister for Defence the degree to which all sea-going vessels available to the Naval Service are upgraded, improved or refur- bished on a regular basis; and if he will make a statement on the matter. [17320/16]

21/06/2016WRCC03000600. Deputy Bernard J. Durkan asked the Taoiseach and Minister for Defence the degree to which training, equipment and quality and variety of aircraft available to the Air Corps con- tinues to be in line with the best standards internationally; and if he will make a statement on the matter. [17321/16]

21/06/2016WRCC03100Taoiseach and Minister for Defence (Deputy Enda Kenny): I propose to take Questions Nos. 598 to 600, inclusive, together.

The acquisition of new equipment and equipment upgrades for the Defence Forces remains a focus for me as Minister for Defence, and I am satisfied that the Defence Forces have a mod- ern and effective range of equipment which is line with best international standards, as required to fulfil all roles that are assigned to them by Government.

The future equipment priorities for the Army, Air Corps and the Naval Service are being considered in the context of the lifetime of the recently published White Paper, which recog- nises that there are several new and or enhanced platforms to be procured.

In the context of the Air Corps fleet, this includes the replacement of the Cessna fleet with three larger aircraft suitably equipped for Intelligence, Surveillance, Target Acquisition and Re- connaissance tasks. In addition, it is intended that the two CASA 235s Maritime Patrol Aircraft will also be replaced. The existing Pilatus PC9 aircraft are not due for replacement until 2025. 281 Questions - Written Answers The current EC 135 and AW 139 helicopter fleet in the Air Corps will continue to deliver the required Defence Forces support and other support capabilities over the lifetime of the White Paper.

The current Naval Service ship replacement programme will have replaced three Naval Service vessels by July 2016. Last week, agreement was reached with Babcock International for the provision of an additional ship for the Naval Service to be built in their facility in Ap- pledore, Devon. This ship will be the same class as the three newest vessels, LÉ Samuel Beck- ett, LÉ James Joyce and LÉ William Butler Yeats. The agreed contract value for the further ship is €54.3 million (€66.78 million inclusive of VAT) and delivery is expected within two years, which ties in with the project planning process in place under the White Paper on Defence which will determine the Defence Organisation’s maritime capability requirements.

The White Paper also provides for the replacement of the current flagship, LÉ Eithne, by a multi-roled vessel (MRV). LÉ Eithne was built in 1984. In addition, the two existing Coastal Patrol Vessels, LÉ Ciara and LÉ Orla are due for replacement in the coming years (also built in 1984). It is outlined in the White Paper that these ships will be replaced with similar type ves- sels with countermine and counter IED capabilities.

In addition, the White Paper recognises the requirement to maintain and upgrade vessels having regard to emerging operational requirements and changes in technology.

The Army will continue to retain all-arms conventional military capabilities, and the princi- pal aim over the period of the White Paper will be to replace and upgrade, as required, existing capabilities in order to retain a flexible response for a wide range of operational requirements at home and overseas. There will be a focus on the essential force protection provided by ar- moured vehicles and the most cost effective option of upgrading or replacing the current fleet will be identified. A small number of armoured logistical vehicles will be procured for force protection overseas.

There will be continued investment in the development of suitable network enabled com- munications in order to meet the challenges of an increasingly complex operational environ- ment. This is also a priority area for investment if additional funding, beyond that required to maintain existing capabilities, is made available.

I am satisfied that the current provisions, together with the courses of action in the White Paper, will ensure that the Defence Forces are fully equipped and trained to meet the roles placed on them.

21/06/2016WRCC03150Defence Forces Strength

21/06/2016WRCC03200601. Deputy Bernard J. Durkan asked the Taoiseach and Minister for Defence the total strength of the Defence Forces including Reserves, Army, Navy and Air Corps; the number of women in each branch of the services; the likelihood of recruitment and replacements follow- ing retirements, now and over the next five years; and if he will make a statement on the matter. [17322/16]

21/06/2016WRCC03300Taoiseach and Minister for Defence (Deputy Enda Kenny): The following tabular state- ment sets out the total strength of the Defence Forces, as of the 31st May, 2016 (the latest date for which figures are available), broken down as requested by the Deputy.

282 21 June 2016 Reserve Army Naval Service Air Corps Total 2,134 7,310 1,094 733 No. of Females 285 452 58 36 It should be noted that figures for the Reserve (above) are for effective members, while figures for the Army, Naval Service and Air Corps are for Whole Time Equivalent positions.

The Government is committed to maintaining the stabilised strength of the Permanent Defence Force at 9,500 personnel, comprising of 7,520 Army, 886 Air Corps and 1,094 Naval Service as stated in the 2015 White Paper on Defence. The manpower requirement of the De- fence Forces is monitored on an ongoing basis in accordance with the operational requirements of each of the three services. Personnel are posted on the basis of operational needs across the organisation both at home and abroad. The significant turnover of personnel that is the norm in military services requires targeted recruitment so as to maintain personnel numbers at or near the agreed strength levels.

A General Service recruitment campaign was launched on the 13th April, 2016, and the 2016 Cadet Competition launched on the 16th of March, 2016. Applications from these com- petitions are currently being processed. 241 General Service Recruits have been inducted to date in 2016. It is planned to recruit a total of 600 enlisted personnel in the current year and 75 cadets. An apprentice competition for the Air Corps is currently being planned. In addition, it is planned to recruit 850 enlisted personnel in 2017.

Question No. 602 answered with Question No. 597.

21/06/2016WRCC03550Defence Forces Reserve Strength

21/06/2016WRCC03600603. Deputy Bernard J. Durkan asked the Taoiseach and Minister for Defence the extent to which membership of the local Defence Forces continues to be maintained at a required level; the extent to which ongoing training standards and requirements are met. [17324/16]

21/06/2016WRCC03700Taoiseach and Minister for Defence (Deputy Enda Kenny): The White Paper on Defence 2015 provides that the overall establishment of Army Reserve (AR) and the Naval Service Re- serve (NSR) is set at 4,169 personnel, consisting of 3,869 Army Reservists and the expansion of the four Naval Service Reserve Units from 200 to 300 personnel.

Generally, recruitment campaigns for the RDF are conducted on an annual ongoing basis. The current campaign began on the 7th September 2015, and the Defence Forces have received a total of 2,733 applications to date in this competition. The recruitment campaign has been paused on an administrative basis as the Defence Forces are administering the recruitment cam- paign for candidates to enter the Permanent Defence Force. I am advised that it is intended to recommence the campaign for Reservists in the coming months.

The 2015 White Paper on Defence confirms that the primary role of the Reserve is to aug- ment the Permanent Defence Force in times of crisis and to participate in ceremonial events. The implementation of measures set out in the White Paper 2015 will provide opportunities for certain members of the RDF to undertake new duties. I am of the view that this, and other ini- tiatives outlined in the White Paper will be a key motivating factor in encouraging more people to join the RDF as well as making it more attractive for current members to stay.

With regard to RDF training, a key recommendation of the Value for Money Review of the RDF, published in November 2012, was to ensure that all effective members of the Reserve were afforded the opportunity to undertake annual training. In 2016, Subhead A.5 provides for a 283 Questions - Written Answers budget of €2.150 million. of which €2.068 million. provides for a total of 30,000 Paid Training Man-day’s for members of the Reserve. This provision is sufficient having regard to the exist- ing strength of the RDF and the voluntary nature of Reserve training.

Defence Forces Operations

21/06/2016WRCC03900604. Deputy Bernard J. Durkan asked the Taoiseach and Minister for Defence the extent to which the Defence Forces are adequately equipped to deal with emergencies in respect of which they might be called upon; and if he will make a statement on the matter. [17325/16]

21/06/2016WRCC04000Taoiseach and Minister for Defence (Deputy Enda Kenny): My priority as Minister for Defence is to ensure that the operational capacity of the Defence Forces, including the availability of specialised equipment, is maintained to the greatest extent possible to enable the Defence Forces to carry out their roles both at home and overseas. In accordance with the Framework for Major Emergency Management, primary responsibility for responding to emergencies caused by severe weather events, such as the storms and flooding, rests with the three designated principal response agencies, namely, the relevant Local Authority, An Garda Síochána, and the Health Service Executive. The Defence Forces provide the fullest possible assistance to the appropriate Lead Department in the event of a natural disaster or emergency situation in its Aid to the Civil Authority (ATCA) role.

At National level, representation on the Government Task Force on Emergency Planning, by both the Department of Defence and the Defence Forces, ensures the fullest coordination and cooperation in the event of an emergency and that the command structure within the De- fence Forces is compatible with the requirements in this area.

The full spectrum of Defence Forces personnel and equipment, commensurate with op- erational requirements, is available for deployments in response to emergency situations. The most important asset that the Defence Forces can provide is personnel, equipped with personal protective equipment, to operate during extreme weather events and with the capacity to sup- port and sustain those personnel if deployed.

I am satisfied that the Defence Forces have the necessary resources available to them, in- cluding specialised equipment, to fulfil all roles assigned to them by Government including the provision of relevant assistance in emergency situations that may arise.

21/06/2016WRCC04100Defence Forces Operations

21/06/2016WRCC04200605. Deputy Bernard J. Durkan asked the Taoiseach and Minister for Defence the extent to which the Defences Forces can be mobilised in the event of natural disasters, including flood- ing, given the preferability of early deployment and the need for the use of specified equipment in such situations; and if he will make a statement on the matter. [17326/16]

21/06/2016WRCC04300Taoiseach and Minister for Defence (Deputy Enda Kenny): My priority as Minister for Defence is to ensure that the operational capacity of the Defence Forces is maintained to the greatest extent possible.

In relation to non-security related emergencies, whilst the Defence Forces are not a Prin- cipal Response Agency as defined in the Framework for Major Emergency Management, they provide the fullest possible assistance to the appropriate Principle Response Agency in the event of a natural disaster or emergency situation in the Aid to the Civil Authority (ATCA) role.

284 21 June 2016 At National level, representation on the Government Task Force on Emergency Planning, by both my Department and the Defence Forces, ensures the fullest coordination and coopera- tion in the event of an emergency.

To facilitate support to the principle response agencies locally, the Defence Forces are also represented on the eight Regional Working Groups that have been established under the Frame- work and, on an ongoing basis, designated members of the Defence Forces, based around the country, act as Liaison Officers to Local Authorities.

I am satisfied that Defence Forces personnel have a modern and effective range of equip- ment to fulfil all roles that are assigned to them by Government.

21/06/2016WRCC04400Defence Forces Operations

21/06/2016WRCC04500606. Deputy Bernard J. Durkan asked the Taoiseach and Minister for Defence the extent to which the Air Corps and the Naval Service continue to co-operate in air-sea rescue missions; and if he will make a statement on the matter. [17327/16]

21/06/2016WRCC04600Taoiseach and Minister for Defence (Deputy Enda Kenny): My priority as Minister for Defence is to ensure that the operational capacity of the Defence Forces is maintained to the greatest extent possible and that they have the resources to carry out their operational commit- ments. I am satisfied that the Defence Forces, including the Air Corps and Naval Service, have the necessary resources to meet these commitments.

With reference to Search and Rescue, the Irish Coast Guard has overall responsibility for the provision of such services within the Irish search and rescue region. Both the Naval Service and the Air Corps provide support to the Irish Coast Guard in maritime Search and Rescue op- erations and a Service Level Agreement is in place setting out their roles and responsibilities in this regard.

21/06/2016WRCC04650Defence Forces Recruitment

21/06/2016WRCC04700607. Deputy Bernard J. Durkan asked the Taoiseach and Minister for Defence the extent to which young unemployed persons have been able to join the Defence Forces; if a specific programme exists in this regard; and if he will make a statement on the matter. [17328/16]

21/06/2016WRCC04800Taoiseach and Minister for Defence (Deputy Enda Kenny): There is no specific recruit- ment campaign aimed at young unemployed persons. Applications to join the Defence Forces are open to all persons who fulfil certain criteria. The criteria that apply to all applicants for General Service Recruitment Competitions require that they must undergo psychometric tests, a fitness test, a competency based interview, a medical examination and undergo Security Clear- ance. Applicants must be at least 18 years of age and under 25 years of age on the closing date of applications for the Army, and must be at least 18 years of age and under 27 years of age on the closing date of applications for the Naval Service. No formal education qualifications are required to join the Defence Forces as a recruit. However, applicants must satisfy the Interview Board and the Recruiting Officer that they possess a sufficient standard of education for service in the Permanent Defence Force. I am advised that educational standards for those who do not sit exams in the State are governed by the National University of Ireland set equivalent. A General Service recruitment campaign that was launched on 13 April 2016, and the 2016 Ca- det Competition launched on 16 March 2016, are both now closed and applications are being processed. 285 Questions - Written Answers However, in line with the Government’s commitment to a strengthened approach across the economy to social responsibility, the Programme for a Partnership Government includes a com- mitment to introduce a pilot employment support scheme with the direct involvement of the Defence Forces. This commitment was also reflected in the White Paper on Defence, published in August 2015, which sets out the policy framework for the Defence Organisation for the next ten years and beyond.

The objective of this Scheme is to help develop a path to economic independence for partici- pants, aged between 18 and 24 years, who are at a serious disadvantage owing to their current socio-economic situation. The Scheme is designed with the specific aim of equipping partici- pants with life and other marketable and certifiable skills which will assist them both in their capacity to participate fully in society as well as their ability to enter the workforce and or to pursue further education for this purpose.

The scheme and the programme content have been developed through detailed consultation involving the Department of Defence, the Defence Forces, the Department of Social Protection and the Dublin and Dún Laoghaire Education and Training Board. A work group which in- cludes representatives from each of the aforementioned organisations has developed the detail of the initiative with key stakeholders.

286